Download as pdf or txt
Download as pdf or txt
You are on page 1of 580

K

END
RIY
AVI
DYA
LAY
AS A
NGA
THAN
ER
NAKUL
AMREG
ION

S
TUDE
NT
S
UPPORTMA
TERI
AL

T
ERMI
I(
202
1-2
2)

CL
ASSXI
I-
PHYSI
CS
I
N
S
P
Shri
.RSENTHI
LKUMAR
DEPUTYCOMMISSI
ONER
KVSROERNAKULAM

I
R
A
T
Smt.
DEEPTINAIR Shri
.SANTHOSHKUMARN
ASST
.COMMI
SSIONER ASST.
COMMISSI
ONER
KVSROERNAKULAM KVSROERNAKULAM

I
O
N Smt
PRI
.HEMAK
NCIPAL,
NTPCKAY
AMKULAM
3
CONTENT DEVELOPEMENT TEAM
S.NO NAME OF THE NAME OF THE NAME OF THE
CHAPTER/ TOPIC TEACHER KV
Electromagnetic Waves Mr. Binoj Bose KV Pattom Shift 1
1
Ms. Padmaja M G KV Pattom Shift 2
Ray Optics and Optical Mr. Sukesh G KV Pangode
2 Instruments Ms. Chitra G
Mr. Naresh Banoth KV Kavaratti
Wave Optics Mrs. Susmitha Mary Robbins KV Kalpetta
3 KV Malappuram
Ms. P C Radha
Dual Nature of Radiation Mrs Jayasree V R KV Port Trust
4 and Matter Mrs Roja Rani KV Port Trust
Atoms Ms. Gayathri S V KV SAP
5
Ms. Lekshmi Devi S Peroorkada
Nuclei Ms. R Amudha KV Port Trust
6
Mr. Francis Paul KV No 2 Kochi
Semiconductor Electronics: Mr Prasanth Kumar M. KV Payyannur
7 Materials, Devices and
Ms. Sindhu R Menon KV Kannur
Simple circuits
SAMPLE QUESTION PAPER PREPARATION
Ms Sachu S R KV Akkulam
1 Sample Question Paper 1 KV CRPF
Ms. Preetha R
Pallipuram
Ms. A Jyothi KV Thrissur
2 Sample Question Paper 2
Mr. Haridasan M K
Ms. Bindu R KV Ernakulam
3 Sample Question Paper 3
Ms. Mini Panicker
Ms. Beena Kumari KV Ottapalam
4 Sample Question Paper 4
Ms. Meera V
Mr. Vilesh V L KV No. 1 Kasargod
5 Sample Question Paper 5
Ms. Maya G KV No. 2 Kasargod
REVIEW COMMITTEE
1 Ms. K V Sudha KV No 1 Calicut
2 Mr. Ramachandran M KV Thrissur
3 Ms.Mony P KV Palakkad
4 Mr. K Sivadas KV Kanjikode
5 Mr. Prasanth Kumar M KV Payyannur
6 Mr. Sreekanth S KV No 2 Calicut

4
INDEX
S.No Content Page No
1 Syllabus (Term-II) 7-8
2 Chapter-8; Electromagnetic Waves 9 - 20
3 Chapter-9; Ray Optics and Optical Instruments 21 - 47
4 Chapter-10; Wave Optics 48 - 93
5 Chapter-11; Dual Nature of Radiation and Matter 94 - 114
6 Chapter-12; Atoms 115 - 140
7 Chapter-13; Nuclei 141 - 159
Chapter-14; Semiconductor Electronics:
8 160 - 180
Materials, Devices and Simple circuits
9 CBSE sample Question paper with answer key 181 - 191
10 Sample Question paper 1 193 - 197
11 Sample Question paper 2 198 - 201

12 Sample Question paper 3 202 - 205


13 Sample Question paper 4 206 - 209
14 Sample Question paper 5 210 - 213
15 Answer Key ( Sample Question Papers) 214 - 223

❖ Every Chapter Contains:

✓ Gist of Lesson
✓ Important formulas used
✓ Mind map
✓ MCQ Questions
✓ Reason Assertion Questions
✓ Case Study Based Questions
✓ 1 Marks, 2 Marks, 3 Marks & 5 Marks Questions with Answer

5
CLASS-XII Syllabus assigned for Term II (Theory)
Sub: PHYSICS Session 2021-22
Time: 2 Hours Max Marks: 35

Unit Chapter Mar


ks
Unit–V Electromagnetic Waves
Chapter–8: Electromagnetic Waves

Unit– Optics 17
VI
Chapter–9: Ray Optics and Optical Instruments

Chapter–10: Wave Optics


Unit– Dual Nature of Radiation and Matter
VII Chapter–11: Dual Nature of Radiation and
Matter
11
Unit– Atoms and Nuclei
VIII
Chapter–12: Atoms

Chapter–13: Nuclei
Unit– Electronic Devices
IX Chapter–14: Semiconductor -Electronics:
Materials, Devices and Simple Circuits 7

Total 35

6
Detailed syllabus
Unit V: Electromagnetic waves
Chapter–8: Electromagnetic Waves
Electromagnetic waves, their characteristics, their Transverse nature (qualitative ideas
only).Electromagnetic spectrum (radio waves, microwaves, infrared, visible,
ultraviolet, X-rays, gamma rays) including elementary facts about their uses.

Unit VI: Optics


Chapter–9: Ray Optics and Optical Instruments
Ray Optics:
Refraction of light, total internal reflection and its applications, optical fibers, refraction
at spherical surfaces, lenses, thin lens formula, lensmaker's formula, magnification, power
of a lens, combination of thin lenses in contact, refraction of light through a prism.
Optical instruments: Microscopes and astronomical telescopes (reflecting and
refracting) and their magnifying powers.
Chapter–10: Wave Optics
Wave optics:
Wave front and Huygen's principle, reflection and refraction of plane wave at a plane
surface using wave fronts. Proof of laws of reflection and refraction using Huygen's
principle. Interference, Young's double slit experiment and expression for fringe
width, coherent sources and
sustained interference of light, diffraction due to a single slit, width of central maximum

Unit II: Dual Nature of Radiation and Matter


Chapter11: Dual Nature of Radiation and Matter
Dual nature of radiation, Photoelectric effect, Hertz and Lenard's
observations; Einstein's
Photoelectric equation-particle nature of light. Experimental study of photoelectric
effect
Matter waves-wave nature of particles, de-Broglie relation

Unit VIII: Atoms and Nuclei


Chapter–12: Atoms
Alpha-particle scattering experiment; Rutherford's model of atom; Bohr model,
energy levels, hydrogen spectrum.

7
Chapter–13: Nuclei
Composition and size of nucleus Nuclear force Mass-energy relation, mass defect,
nuclear fission, nuclear fusion.

Unit IX: Electronic Devices


Chapter–14: Semiconductor Electronics: Materials, Devices and
Simple Circuits
Energy bands in conductors, semiconductors and insulators (qualitative ideas only)
Semiconductor diode - I-V characteristics in forward and reverse bias, diode as a
rectifier; Special purpose p-n junction diodes: LED, photodiode, solar cell.

8
ELECTROMAGNETIC WAVES
Electromagnetic waves are produced by accelerated charges.

An electromagnetic wave travelling along the z axis can be represented by


E = EX (t) = E0 sin (kz – ωt)
B = By (t) = B0 sin (kz – ωt)

Characteristics of electromagnetic waves


1.Electromagnetic waves consists of mutually perpendicular electric and magnetic
fields and both are perpendicular to the direction of propagation of waves
2.They do not require a medium for their propagation
3.They travel with a speed of 3 × 108 m/s in vacuum.
1
4.The speed of electromagnetic waves in vacuum is
√𝜇0𝜀0
1
5.The speed of electromagnetic waves in a medium is
√𝜇𝜀

6.They obey the principle of superposition of waves


7. They carry energy as they propagate and is equally divided between electric field
and magnetic field
𝐸0
8.In free space c=
𝐵0

9. Electric field is mainly responsible for the optical effects of electromagnetic


waves.
𝑈
10.The momentum of electromagnetic waves is given by p=
𝐶

9
ELECTROMAGNETIC SPECTRUM

RADIO WAVES
Radio waves are produced by the accelerated motion of charges in conducting
wires. They are used in radio and television communication systems. They are
generally in the frequency range from 500 kHz to about 1000 MHz. The AM
(amplitude modulated) band is from 530 kHz to 1710 kHz. Higher frequencies up to
54 MHz are used for short wave bands. TV waves range from 54 MHz to 890 MHz.
The FM (frequency modulated) radio band extends from 88 MHz to 108 MHz.
Cellular phones use radio waves to transmit voice communication in the ultrahigh
frequency (UHF) band.
MICROWAVES
Microwaves (short-wavelength radio waves), with frequencies 1010 Hz to 1012
Hz range, are produced by special vacuum tubes (called klystrons, magnetrons and
Gunn diodes). Due to their short wavelengths, they are suitable for the radar systems
used in aircraft navigation. They are used in Microwave ovens

10
INFRARED WAVES
Infrared waves are produced by hot bodies and molecules. Infrared waves are
known as heat waves. Frequency range is from1012 Hz to 1014 Hz .Infrared lamps are
used in physical therapy. Infrared radiation plays an important role in maintaining the
earth’s warmth or average temperature through the greenhouse effect Infrared
detectors are used in Earth satellites, infrared is widely used in the remote switches of
household electronic systems such as TV sets, video recorders and hi-fi systems

VISIBLE LIGHT
It is the part of the spectrum that is detected by the human eye. It runs from
about 4 × 1014 Hz to about 7 × 1014 Hz or a wavelength range of about 700 – 400 nm.
This is the narrow region of electromagnetic spectrum

ULTRAVIOLET RAYS
It covers wavelengths ranging from about 4 × 10–7 m (400 nm) down to 6 × 10–
10
m (0.6 nm). Ultraviolet (UV) radiation is produced by special lamps and very hot
bodies. Due to its shorter wavelengths, UV radiations can be focussed into very
narrow beams for high precision applications such as LASIK (Laser assisted in situ
keratomileusis) eye surgery. UV lamps are used to kill germs in water purifiers.
Ozone layer in the atmosphere plays a protective role, and hence its depletion by
chlorofluorocarbons (CFCs) gas (such as freon) is a matter of international concern.

X RAYS
X-rays Beyond the UV region of the electromagnetic spectrum lies the X-ray
region. It covers wavelengths from about 10–8 m (10 nm) down to 10–13 m (10–4 nm).
It is generated by bombarding a metal target by high energy electrons. X-rays are
used as a diagnostic tool in medicine and as a treatment for certain forms of cancer.
GAMMA RAYS
They are high frequency electromagnetic waves with wavelengths from about
10–10m to less than 10–14m. This high frequency radiation is produced in nuclear
reactions and also emitted by radioactive nuclei. They are used in medicine to destroy
cancer cells.

11
MCQ

1. In a plane em wave, the electric field oscillates sinusoidally at a frequency of 2.5x


10 10Hz and amplitude 480 V/m.The amplitude of the oscillating magnetic field will
be
A. 1.5x 10 -8 Wb/m2 C. 1.6x10-6 Wb/m2
B. 1.52x 10-7 Wb/m2 D 1.6x10 -7 Wb/m2

2.Which of the following is the infrared wavelength?


A.10 -4 cm. C. 10 -6 cm
B. 10 -5 cm. D. 10 -7 cm

3. There are three wave lengths 10 7 m,10 -10 m,10 -7 m. Find their respective names.
A. Radio waves, X- rays,visible rays. C. Visible rays,gamma rays, X - rays
B. X - rays,gamma rays,visible light.
D. X -rays,gamma rays,visible rays

4. Velocity of electromagnetic wave is parallel to


A. BXE. B. EXB.
C. E. D. B

12
5.The frequencies of X - rays, gamma rays and U V rays are respectively a,band
c,then
A. a<b, b>c.
B. a>b, b>c.
C . a>b, b<c.
D. a<b , b<c.

6. The sun delivers 10 3 W/m 2 of electromagnetic flux to the earth’s surface. The
total power that is incident on a roof dimensions 8 m x 20 m will be
A. 2.56 X 10 4W
B. 6.4X 10 5 W
C .4 X 10 5 W
D. 1.6 X 10 5 W

7. Which of the following is not true for em waves?


A. They transport energy.
B. They travel at different speeds in air depending on their frequency.
C. They have momentum
D. They travel at different speeds in medium.

8. The magnetic field in a plane em wave is given by


By =2 x 10 -7 sin (0.5 x10 3 z + 1.5 x 10 11 t) T. The expression for electric field is
A. Ey =2 x 10 -7 sin (0.5 x10 3 z + 1.5 x 10 11 t) V/m.
B. Ex = 2 x 10 -7 sin (0.5 x10 3 z + 1.5 x 10 11 t) V/m.
C. Ey = 60 sin (0.5 x10 3 z + 1.5 x 10 11 t) V/m.
D . Ex = 60 sin (0.5 x10 3 z + 1.5 x 10 11 t) V/m.
9. Fundamental particle in an em wave is
A. photon.
B. electron.
C. proton.
D.phonon.
10. The magnetic field in a plane em wave is By = 2x 10 -7 sin ( 0.5x 103 x+1.5x 1011
t) .This em wave is
A. visible light.
B. infrared wave.
C. microwave.
D. radiowave

13
ANSWERS AND EXPLANATIONS MCQ
1 .C B0 =E0/c=480/(3x 108)=1.6x 10-6Wb/m2
2. A
3. A
4. B
5. A
6. D (Power=103x8x20=1.6x 105 W)
7. B ( The em waves of all frequencies travel with the same speed in air )
8. D (Since the wave is propagating in z direction and magnetic field is along y
direction, the electric field will be along x direction.
9. A
10. C ( frequency= ω/2π=2.39x 1010 Hz)

ASSERTION(A) REASON(R) QUESTIONS

Given below are two statements labeled as Assertion(A) and Reason(R).


Select the most appropriate answer from the options given below.
(i) Both A and R are true, and R is the correct explanation of A.
(ii) Both A and R are true, and R is not the correct explanation of A.
(iii) A is true but R is false
(iv) A is false and R is also false.
1.A. The frequencies of incident, reflected and refracted b beam of monochromatic light
incident from one medium to another are same.
R. The incident, reflected and refracted rays are coplanar.
2. A. The earth without its atmosphere will be inhospitably cold.
R. All heat would escape in the absence of atmosphere.
3. A. Microwaves are better carrier of signal than optical wave.
R. Microwaves move faster than optical waves.
4. A. Gamma rays are more energetic than x rays.
R. Gamma rays are of nuclear origin but x rays are produced due to sudden
deceleration of high energy electrons while falling in the metal of high atomic
number.
5. A. Electromagnetic waves are transverse in nature.
R. The electric and magnetic fields are perpendicular to each other and perpendicular
to the direction of propagation.

14
ANSWERS
1. Both A and R are true and R is not the correct explanation of A
2. Both A and R are true and R is the correct explanation of A.
3. Both A and R are wrong.
4. A and R are true and R is not correct explanation of A
5. Both A and R are true and R is the correct explanation of A

CASE STUDY
The electromagnetic waves carry energy and momentum. The linear momentum carried
by the portion of the wave having energy U is given by P=U/c. Thus if the wave
incident on a surface is completely absorbed, it delivers energy U and momentum p
=U/c to the surface. If the wave is totally reflected, the momentum delivered is 2xU/c it
follows that em waves incident on a surface exerts a force on the surface.
1. A plane em wave is incident on a surface. The wave delivers energy E and
momentum p. Which is the correct option (a) p=0, E≠0 (b)p≠0 E=0 (c)p≠0 E≠0 (d) p=0
and E=0
2. Which statement is correct for a linearly polarized em wave. (a) Electric and
magnetic fields have equal average values (b) electric and magnetic field energy have
equal average values.
a. both the statements are true
b. both the statements are false
c. (a) is false and (b) is true
d. (b) is false and (a) is true
3. What is the order of energy of x-rays(Ex), radio waves(Er) and micro waves
(a) Ex<Er<Em (b)Ex>Em>Er (c)Em>Er>Ex (d) Em<Er<Ex
Radio waves are produced by the accelerated motion of charges in conducting wires.

Microwaves are produced by special vacuum tubes. Infrared waves are produced by hot

bodies and molecules also known as heat waves. UV rays are produced by special lamps

and very hot bodies like Sun.

15
(1) Earth's atmosphere is richest in

(a) ultraviolet (b) infrared (c) X-rays (d) microwaves

(2) What is the cause of greenhouse effect?

(a) Infrared rays (b) Ultraviolet rays (c) X-rays (d) Radio waves

(3) Biological importance of ozone layer is

(a) it stops ultraviolet rays (b) It layer reduces greenhouse effect

(c) it reflects radio waves (d) none of these

(4) Ozone is found in

(a) stratosphere (b) ionosphere (c) mesosphere (d) troposphere

Case Study 1
1. c
2. a
3. b
Case Study 2
1. b 3. a
2. a 4. a

16
1 MARK QUESTIONS

1. How are microwaves produced?


2. Name the part of em spectrum of wavelength 10 -2 m. Mention its one application.
3. Write the following radiations in ascending order in respect of their frequencies.
X rays, microwaves, uv rays, radio waves, gamma rays.
4. Name the em waves used in Radar.
5. Write two applications of ultra violet radiations.

ANSWERS AND EXPLANATIONS ( 1 MARK QUESTIONS)


1.Microwaves are produced in special vacuum tubes called klystrons due to
oscillating current.
2.Microwaves,microwave ovens,radar system
3.Radiowaves,microwaves,u v rays,x rays,gamma rays.
4.microwaves.
5. To preserve food stuff,to sterilize surgical instruments.

2 MARKS QUESTIONS

1. Write any four properties of em waves.


Ans : Reference, short note
2. Give one use of each of the following.
a. infrared rays. b. gamma rays. c. microwaves. d. u v radiations.
Ans : Ref. short note
3. Identify the part of em spectrum which is
a. suitable for radar system in aircraft navigation.
b. adjacent to the low frequency end of the em spectrum.
c. produced in nuclear reaction.
d. produced by bombarding a metal target by high speed electron.
Ans : a. microwaves . b. radio waves c.gamma rays d.x rays
4. Which of the following, if any, can act as a source of em waves? Give reason.
a. a charge moving with a constant velocity. b.a charge at rest. c.a charge moving in a
circular orbit.
Ans: C. Circular motion is an accelerated motion and accelerated charge produces em
waves.

17
5. Draw a diagram showing the propagation of em waves along x direction, indicating
clearly the directions of electric and magnetic fields associated with it.
Ans:Ref diagram ncert page number 275
6. Find the wavelength of em waves of frequency 4x 109 Hz in free space. Give its two
applications.
Ans: λ=c/ν= 0.075 m. This belongs to microwave region. Used in radar, microwave
oven.
7. Identify the part of em spectrum to which frequency 10 20 Hz belong. Give its one
use.
Ans: gamma rays. used in treatment of tumers
8. Let the wavelengths of em waves used for a. killing germs in household water
purifiers b. remote sensing and c. treatment of cancer are labeled as λ1,λ2 and λ3
.Identify the waves and arrange them in increasing order of wavelength.
Ans: λ1=uv rays,λ2=microwaves,λ3=gamma rays, λ3,λ1,λ2
9. In a plane em wave, the electric field oscillates sinusoidally at a frequency of 2 x 10
10
Hzand amplitude 48 V/m. What is the wavelength of the wave? What is the amplitude
of the oscillating magnetic field?
Ans: λ=c/ν=1.5x 10-2 m B0=E0/c=48/3x 108=1.6x 10-7 T
10. Given below are some famous numbers associated with em radiations in different
contexts in physics. State the part of the em spectrum to which each belong ?
a. 5890 A0 to 5896 A0(double lines of sodium)
b. 2.7 K (temperature associated with the isotropic radiations filling all space)
Ans: a. visible region (yellow)
b. T=2.7 K. λm.T=0.29 cm K .λm=0.29/T=0.11 cm

18
3 MARKS QUESTIONS

1. Write two applications each of a. microwaves b. infrared waves c.radio waves.


2. Write the order of frequency range and one use each of the following em waves.
a. microwaves. b. uv rays and c. gamma rays.
3. a. Name the em waves which are produced during radioactive decay of a nucleus.
Write their frequency range .
Ans: a. Gamma rays,109to 1023 Hz
* Glass goggles protect the eyes from uv radiations produced by welding arcs.
* water molecules present in the material readily absorb the IR rays and gets heated up.
Q. welders wear special glass goggles while working .Why?
Q. Why are infrared waves often called as heat waves? Give their one application?
4.Name the part of em spectrum which is
a. suitable for radar system used in aircraft navigation .
b. used to treat muscular strain.
c. used as diagnostic tool in medicine.
Write in brief how these waves can be produced.
5. Describe an activity to illustrate that em waves carry energy and momentum.
Ans: NCERT P.277
6. The electric field part of an em wave in vacuum is
E= 3.1 N/C cos[ (1.8rad/m) y +( 5.4 x 106 rad/s) t] î.
a. What is the direction of propagation of the wave?
b. What is the wavelength and frequency of the wave?
c. What is the amplitude of the magnetic field part?
d. Write the expression for the magnetic field part.
Ans: 1. - 𝑗̂, 2. 3.5 m, 3. 86MHz,
4. 100 nT, 5. 100 nT cos[ (1.8 rad/m y+5.4x106 rad/s) ]k
7. Which physical quantity is same for x-rays of wavelength 10-10 m, red light of
wavelength 6800 A0 and radio waves of wavelength 500 m?
b. Show that for em waves, the average energy density of electric field equals the
average energy density of magnetic field.

19
Ans: a. speed in vacuum,
1 1
b.UE= ϵ0 E2, UB= B2 ,
2 2𝜇0

using E=c xB and c=1/√μ0ϵ0,


UE=UB
8. The electric field part of an em wave is
Ey =30 sin (2x1011 t + 300πx) V/m.
a. Find the wavelength of the wave. b. Amplitude of magnetic field.
c. Write the epression for the electric field part.
Ans: λ=6.67x10-3m. B0=E0/c =1x10-7T B=1x10-7sin(2x1011t+300πx)T
9.The wavelength range of some constituents of em spectrum is given below
s.no Wavelength range
1 1mm to 700nm
2 400nm to 1nm
3 1mm to 10-3nm
4 <10-3nm
Select the wavelength range and name the waves that are
a. used in remote switches of household electronic devices.
b. produced in nuclear reaction.
Ans: 1. 1mm to 700nm(IR rays), 2. <10-3nm(ϒrays)
10.a. Write the expression for velocity of em waves in a medium of permeability μ
and permittivity ϵ
b. The peak value of oscillating electric field in an em wave is 14 V/m. Calculate the
intensity of the wave and rms value of electric field.
1
Ans: 10. 𝜖0 𝐸0 2 =0.26Wm2 , Erms=E0/√2=9.9V/m
2

20
RAY OPTICS AND OPTICAL INSTRUMENTS
𝑛1
Snell’s Law
𝑛2

Refractive
Index

𝑛1
Apparent depth
𝑛2

Refraction Lateral shift


through
parallel glass
slab

Total Internal
Reflection
𝐶 is the
critical angle

Refraction
through a
spherical
surface

Lens Maker’s
Formula

Thin Lens
Formula

21
Power of a Lens

Combinations
of Lenses
)in contact(

Refraction
through Prism

:Image formed at LDDV

Simple
Microscope
:Normal Adjustment

:Image formed at LDDV

Compound
microscope
Compound
microscope :Normal Adjustment

22
:Image formed at LDDV

Astronomical
Telescope

:Normal Adjustment

Cassegrain
type telescope

23
24
MCQs: Ray Optics and Optical Instruments

1. A lens of refractive index 1.5 is dipped in to a medium of refractive index 1.5. Then

the new refractive index of lens

a) 1.5 b) 1.5x1.5 c) 1 d) 1.8

2. A convex lens has focal length f. It is cut in to two equal parts perpendicular to its

principal axis. The focal length of each part is

a) f/2 b)f c)3/2 f d) 2f

3. The brilliance of diamond is due to

a) Reflection b) Refraction c) TIR d) Scattering

4. The refractive index is greater for

a) light of low frequency b) light of shorter wavelength

c) lighter of greater wavelength d) All the above

5. A ray of light travels from vacuum to a medium of refractive index n. If the angle of

incidence is found to be twice angle of refraction, then angle of incidence is

a) cos-1 (n/2) b) 2 cos-1 (n/2) c) sin-1 (n/2) d) 2 sin-1 (n/2)

6. A convex lens is used to form a real inverted image of an illuminated object. If the
top half of the lens is covered with a black paper then the
a) bottom half of the image will vanish b) top half of the image will vanish
c) brightness of the image will decrease d) magnification of image will decrease
7. When a ray of light incident normally on one refracting surface of an equilateral
prism of refractive index 1.5 then
a) emergent ray is deviated by 300
b) emergent ray is deviated by 450
c) emergent ray just grazes the second refracting surface
d) the ray undergoes TIR at the second refracting surface

25
8. When a ray is refracted ,which of the following does not change

a) frequency b) wavelength c) velocity d) amplitude

9. The focal length of an equiconvex lens in air is equal to either of its radii of

curvature. The refractive index of the material of the lens is

a) 4/3 b) 2.5 c) 1.25 d) 1.5

10. If the critical angle for a material to air is 300 , then the refractive index of the

material will be

a) 1 b) 1.5 c) 2 d) 2.5

11. The angle of the prism is 300 .The rays incident at 600 at one refracting face suffer a

deviation of 300. The angle of emergence is

a) 0 b) 30 c) 60 d) 90

12. Light travelling from a transparent medium to air undergoes TIR at an angle of

incidence 450. Then the refractive index of the medium may be

a) 1.5 b) 1.4 c) 1.1 d) 1/√2

13. A convex lens is made of three different materials as shown in figure. For a point

object on the principal axis, the number of images formed are

a) 5 b) 1 c) 3 d) 4

26
14. A compound microscope has two lenses. The magnifying power of one is 5 and the

combined magnifying power is 100. The magnifying power of the other lens is

a) 10 b) 20 c) 50 d) 25

15. A convex lens of focal length 20 cm produces a real image twice the size of the

object. Then the distance of the object from the lens is

a) 10 cm b) 20 cm c) 30 cm d)60 cm

16. An astronomical telescope has two lenses of powers 0.5 D and 20 D. Then its

magnifying power will be

a) 8 b) 20 c) 30 d)40

17. In a compound microscope the intermediate image is

a) virtual, inverted and magnified b) real, inverted and diminished

c) virtual erect and magnified d) real , inverted and magnified

18. An endoscope is employed by a physician to view the internal parts of a body organ.

It is based on the principle of

a) Refraction b) Reflection c) TIR d) Dispersion

19. A prism of refractive index √2 has a refracting angle of 600. At what angle a ray must

be incident on it so that it suffers a minimum deviation

a) 45 b) 60 c) 90 d) 180

20. A lens acts as converging in air and diverging when immersed in water. The

refractive index of lens is ( nw=1.33 )

a) 1 b) below 1.33 c) greater than 1.33 d) less than uni

27
Answers and Key points
1. c
2. d
3. c
4. b
5. b
by snell’s law, sin i/sin r=n
i=2r
n=sin2r/sinr=2sinr.cosr/sin r= 2cos r
r=cos-1 n/2 , then i= 2 cos-1 n/2
6. c
7. d
8. a
9. d
1/f= (n-1)( 1/R1-1/R2) , For equi convex lens R1=R and R2= -R
1/R= (n-1)2/R
10. c
11. a
d= i1+i2-A
12. b
sin ic= i/n
13. c
14. b
MC= MO* ME
15. c
F=20 cm
v/u =-2, v=-2u
1/f= 1/v -1/u
Sub, u= -30 cm
16. d
M= fo/fe=Pe/Po
17. d
18. c
19. a
at minimum deviation D, 2r=A
r= A/2 =30
n=sin i/sin r
sub, i=45
20. b

28
Assertion Reason type Question: Ray Optics & Optical Instruments

Option A: Both Assertion and reason are right and reason is the correct
explanation of assertion.
Option B: Both Assertion and reason are right and reason is not correct
explanation of assertion.
Option C : Assertion is right but reason is wrong
Option D: Both Assertion and reason are false

1. Assertion: Power of a lens is inversely proportional to its focal length


Reason: focal length is measured in metres whereas power of a lens in diopter
2. Assertion: focal length of lens varies when it is immersed in liquids
Reason: focal length of lens depends on refractive index of medium in which it
is placed
3. Assertion: A glass slab is placed over different colored dots ( violet to red) and
viewed through it, then violet dot appeared to be raised more compared to red
dot.
Reason: Red has least deviation than violet color
4. Assertion: Magnifying power of a telescope depends on its focal lengths of
objective and eyepiece
Reason: In telescope focal length of eyepiece is greater than that of
objective
5. Assertion: A convex can act as divergent lens
Reason: when a convex lens placed in medium of higher refractive index, it
behaves like concave lens

Answers
1. B
2. A
3. A
4. C
5. A

29
CASE BASED QUESTION

1. Optical fibres are used extensively to transmit


a) Optical Signal b) current
c) Sound waves d) None of the above
2. Which of the following statement is not true.
a) Optical fibres is based on the principle of total internal reflection.
b) The refractive index of the material of the core is less than that of the cladding.
c) An optical fibre can be used to act as an optical pipe.
d) There is no appreciable loss in the intensity of the light signal while propagating
through an optical fibre
3. In which of the following the principle of total internal reflection is not used.
a) Endoscope b) Mirage c) Total reflection prism d) Difference
between apparent and real depth of a pond
4. The basic reason for the extraordinary sparkle of suitably cut diamond is that
(a) It has low refractive index (b) It has high transparency
(c)It has high refractive index (d) It is very hard
5. A substance has critical angle 45° for yellow light. What is its refractive index?
a) √2 b) 1/√2 c) 2 d) ½
ANSWERS
1. A) optical signal 4. C)
2. B) 5. A)
3. D)

30
VSA QUESTIONS: Ray Optics & Optical Instruments(1 mark)
1. When a converging and diverging lens of equal focal length are combined what is
the power of the combination?
Ans: P=0, P=1/f1+1/f2
2 How does the angle of minimum deviation of a glass prism vary if the incident
violet light is replaced by red light?
Ans: Decreases. dR<dV
3 For the same angle of incidence, the angle of refraction in two media A and B are
25° and 35° respectively. In which one of the two media is the speed of light
lesser?
Ans: VA<VB
n=sini/sinr=v1/v2, nA/nB=sinrB/sinrA=VB/VA rA<rB , VA<VB
4 The focal length of an equiconvex lens is equal to the radius of curvature of either
face. What is the value of refractive index of the material of the lens?
1 2
Ans: n=1.5 = (𝑛 − 1) , f=R, (n-1)=1/2
𝑓 𝑅

5 A concave lens of refractive index 1.5 is immersed in a medium of refractive index


1.65. What is the nature of the lens?
Ans: Convex lens
1 𝑛𝑔 −2
=( − 1)( ), nm>ng, so fm =+ve
𝑓𝑚 𝑛𝑚 𝑅

6 A short pulse of white light is incident from air to a glass slab at normal incidence.
After travelling through the slab which colour will emerge out first from it?
Ans: Red
7 Does the magnifying power of a microscope depend on the colour of the light
used? Justify your answer.
𝐿 𝐷
Ans: Yes. M= x(1+ ), fαλ
𝑓𝑜 𝑓𝑒

8 An unsymmetrical double convex thin lens forms the image of a point object on its
axis. Will the position of the image change if the lens is reversed?
Ans: No change. Principle of reversibility of light
9 State the condition under which a large magnification can be achieved in an
astronomical telescope.
Ans: focal length of objective must be greater than focal length of eyepiece. M=fo/fe
10 A lens of power of –4.0 D what is its focal length? Identify the nature of the lens.
Ans: f=-25cm, concave

31
SA QUESTIONS ( 2 MARK): Ray optics & Optical instruments

1. Focal length of a convex lens of refractive index 1.5 is 2 cm. What will be the Focal
length of lens, when immersed in a liquid of refractive index of 1.25?
1 2
Ans: =(n-1) ,1/2=(1.5-1)*2/R, R=2cm
𝑓 𝑅
1
=(1.5/1.25-1)*2/R, f=5cm
𝑓

2. A ray of light passes through an equilateral glass prism such that the angle of
incidence is equal to angle of emergence and each of these angles is equal to 3/4 of
angle of prism. What is the value of angle of deviation?
Ans: In prism i + e = A + D = and i= e=3/4A
A+D=3/4A+3/4A, D=A/2=30
3. A spherical convex surface of radius of curvature 20 cm, made of glass (n = 1.5) is
placed in air. Find the position of the image formed, if a point object is placed at 30
cm in front of the convex surface on the principal axis.
𝑛2 𝑛1 𝑛2−𝑛1
Ans: - = 𝑉 = −180𝑐𝑚
𝑣 𝑢 𝑅

4. Calculate the radius of curvature of an equi-concave lens of refractive index 1.5,


when it is kept in a medium of refractive index 1.4, to have a power of –5D
1 2
Ans: =(n-1) , R=2.86cm
𝑓 𝑅

5. An astronomical telescope has an angular magnification of magnitude 5 for distant


objects. The separation between the objective and an eye piece is 36 cm and the
final image is formed at infinity. Calculate the focal length of the objective and the
focal length of the eye piece?
Ans: Magnification m = f0 / fe = 5 f0 = 5 fe
length of the tube, L = f0 +fe=36cm fe=6cm & fo=30cm
6. The refractive index of a material of a concave lens is n1. It is immersed in a
medium of refractive index n2. A parallel beam of light is incident on the lens. Trace
the path of emergent rays when (i) n2 = n1 (ii) n2>n1 (iii) n2< n1.
Ans:

32
7. The radii of curvature of both the surfaces of a lens are equal. If one of the surfaces
is made plane by grinding, then will the focal length of lens change? Will the power
change?
1 2
Ans: =(n-1) , f=R/2(n-1) 𝑓𝑜𝑟 𝑎 𝑝𝑙𝑎𝑛𝑒 𝑠𝑢𝑟𝑓𝑎𝑐𝑒 𝑅 = 𝛼
𝑓 𝑅

F’=R/(n-1)=2f, f is doubled
P=1/f , p’=p/2, power is halved
8. Give two reasons to explain why a reflecting telescope is preferred over a refracting
telescope.
Ans: Reflecting telescope is preferred over refracting telescope because
(a) No chromatic aberration, because mirror is used.
(b) Spherical aberration can be removed by using a parabolic mirror.
(c) Image is bright because no loss of energy due to reflection.
(d) Large mirror can provide easier mechanical support.
9. Calculate the speed of light in a medium whose critical angle is 45°. Does critical
angle for a given pair of media depend on wave length of incident light? Give
reason.
Ans: n=1/sinic=1/sin 45=√2
Yes, critical angle for a pair of media depends on wavelength, because n=a+ b/λ2
where a and b are constants of the media.
10. An equilateral glass prism has a refractive index 1.6 in air. Calculate the
angle of minimum deviation of the prism, when kept in a medium of refractive
index 4√2 /5.
𝐴+𝑑𝑚) 60+𝑑𝑚)
sin ( 𝑛2 1.6𝑥5 sin (
2 2
Ans: n= 𝐴 = = = 60
𝑠𝑖𝑛 𝑛1 4√2 𝑠𝑖𝑛
2 2

Dm=30

33
Ray Optics and Optical Instruments 3 MARKS

1. A lens forms a real image of an object. The distance of the object. From the lens is
U cm and the distance of the image from the lens is cm. The given graph shows
the variation of V and U (3m)
(a) What is the nature of the lens?
(b) Using the graph find the focal
length of the lens?

Answer :
(a) convex lens

b)

, , V= f

In the given graph f = 10 cm.


2. Below given figures A and B show refraction of a ray in air incident at 60° with the
normal to a glass-air and water-air interface, respectively. Predict the angle of
refraction in glass when the angle of incidence in water is 45° with the normal to a
water-glass interface figure C. (3m)

From the figure A 𝜇𝑤a = sin 60° /sin 47°


𝜇𝑎 sin 60° = 𝜇𝑔 sin 35° 𝜇𝑤A = 1.18
𝜇𝑔a = sin 60°/ sin 35° 𝜇𝑔w = 1.51/ 1.18 = 1.27
𝜇𝑔a = 1.51 From figure C
From figure B 𝜇w sin 45° = 𝜇𝑔 sin 𝜃
𝜇𝑎 sin 60° = 𝜇𝑤 sin 47° sin 45° = 𝜇𝑔w sin 𝜃 so 𝜃 ≃ 34°
𝜇𝑤a = sin 60° /sin 47°

34
3. An object of size 3.0 cm is placed 14 cm in front of a concave lens of focal length

21 cm. Describe the image produced by the lens. What happens if the object is

moved further away from the lens? (3M)

ANSWER :

Given, Object distance 𝑢 = −14 cm


Focal length 𝑓 = −21 cm
From the thin lens formula,
1 /𝑣 − 1 /𝑢 = 1 /𝑓

𝑣 = 𝑢f/ 𝑢 + 𝑓
= (−14) × (−21) /−14 − 21 = −8.4 cm
Magnification 𝑚 = 𝑣/𝑢 = −8.4 /−14 = 0.6
size of the image = 0.6 × 3.0 = 1.8 cm
Image is formed at a distance 8.4 cm left to the lens. The image formed is virtual,

erect, diminished to the size 1.8 cm.

As the object moves away from the lens, the image moves towards the focal point

(𝑢 → ∞ ⇒ 𝑣 → 𝑓) and the magnification approaches zero (𝑚 → 0)

4. A point object is placed at 20 cm from a thin plano-convex lens of focal length 15 cm


whose plane surface is silvered. Locate the position and nature of the final image.
Given: u = -20 cm ; fl =15 cm ; v = ?

Solution:

The power P of the silvered lens is P = 2Pi + Pm

35
The mirror equation is

1/v + 1/u = 1/f

(1/v) + (1/-20) = −2/15

1/v = (1/20) – (2/15)

∴ v = −12 cm

The negative sign indicates that the image is formed to the left of the Plano
convex mirror.

5. Light travels from air in to glass slab of thickness 50 cm and refractive index 1.5.
(i) What is the speed of light in glass?

(ii) What is the time taken by the light to travel through the glass slab?

(iii) What is the optical path of the glass slab? (3M)

Solution

Given, thickness of glass slab, d = 50 cm = 0.5 m, refractive index, n = 1.5

refractive index, n = c/v

i)speed of light in glass is,

ii)Time taken by light to travel through glass slab is,

iii)Optical path,

d′ = nd = 1.5×0.5 = 0.75 m = 75 cm

Light would have travelled 25 cm more (75 cm – 50 cm) in vacuum by the same
time had there not been a glass slab

36
6. What is total internal reflection? Under what conditions does it occur? Find a relation
between critical angle and refractive index and name one phenomenon which is based on
total internal reflection. (3m)

Answer:
When the ray of light travels from denser to rarer medium and if the angle of incidence is greater
than the critical angle,,the ray of light is totally reflected back to the denser medium itself.This
phenomenon is called total internal reflection.
Conditions (i) Light should travel from denser to rarer medium (ii)Angle of incidence should be
greater than critical angle.

Sin C = 1/n
Optical fiber works on total internal reflection
7. A ray of light incident on face AB of an equilateral glass prism shows a minimum
deviation of 30°. Calculate the speed of light through the prism also find the angle
of incidence at face AB so that the emergent ray grazes along with the face AC
(3m)

Answer:

37
8. Find the radius of curvature of the convex surface of a plane convex lens, whose
focal length is 0.3m and the refractive index of the material of the lens is 1.5? (3m)
Ans.

For plane convex lens

R = 0.15 m

9. Show that the limiting value of the angle of prism is twice its critical angle? Hence
define critical angle? (3 m )

Ans. Angle of the prism means

For limiting But when =C


(Maximum)
Amax = 2C
Value of angle of prism for
The angle of incidence for which angle of refraction is 90o is called critical angle.

10. Drive the expression for the angle of deviation for a ray of light passing through an
equilateral prism of refracting angle A? (3m)
Ans. At the surface AB

At the surface AC

—-(1)
In quadrilateral AQOR

From (2) and (3)

—-(2) —-(4)

Now in Substituting equation (4) in equation (1)


=i+e–A Or A+ =i+e
Or —-(3)

38
11. What is total infernal reflection of light? Hence write two advantages of total
reflecting prisms over a plane mirror? (3m)
Ans. The phenomenon of reflection of light when a ray of light traveling from a
denser medium is sent back to the same denser medium provided the angle of
incidence is greater than the angle is called total internal reflection.

Advantages
1. It does not require silvering
2. Multiple reflections do not take place in a reflecting prism hence only one image is
formed, which is very bright.

Ray Optics and Optical Instruments 5 MARKS

1. A convex lens of focal length 20 cm and a concave lens of focal length 15 cm are
kept 30 cm apart with their principal axes coincident. When an object is placed 30 cm
in front of the convex lens, calculate the position of the final image formed by the
combination. Would this result change if the object were placed 30 cm In front of the
concave tens? Give reason.
Answer:
The image of an object can be obtained at two different positions of a convex lens
for a fixed value of the distance between the object and the screen if values of u
and v are interchanged in these two positions. This situation is as shown below.

39
f1 = + 20 cm, u = – 30 cm

Here x + 20 + x = 90 cm or x = 35cm
Therefore, u = -35 cm, v = + 55 cm, f = ? Therefore, u for concave Lens = 60
– 30 = + 30 cm and f2 = – 15 cm

Now for concave lens

Or

For image formed by a convex Lens

No, the result will not change as per the principle of reversibility.

2. Draw a labelled diagram of telescope when the image is formed at the least
distance of distinct vision? Hence derive the expression for its magnifying power?

Ans.

magnifying power =

(since angles are very small)

40
—-(i)
For eye piece

Multiply by D

Substituting in e.g. (i)

3. Derive the relation


Where are focal lengths of two thin lenses and F is the focal length of
the combination in contact. (5m)

Ans. Consider two thin lenses in contact which forms the final image I
having focal length For the first
lens
Adding equation (1) & (2)

For the second lens acts as an object

41
Using lens formula

For n no. of thin lenses is contact

4. A beam of light converges at a point P. Now a lens is placed in the path of the
convergent beam 12 cm from P. At what point does the beam converge if the lens is (a)
a convex lens of focal length 20 cm, and (b) a concave lens of focal length 16 cm?

Ans.In the given situation, the


object is virtual and the image
Hence, the image is formed 7.5 cm
formed is real.
away from the lens, toward its right.
Object distance, u = +12 cm
(b) Focal length of the concave
(a) Focal length of the convex
lens, f= –16 cm
lens, f= 20 cm
Image distance = v
Image distance = v
According to the lens formula, we
According to the lens formula, we
have the relation:
have the relation:

Hence, the image is formed 48 cm away from the lens, toward its right.

5. A ray of light Is an Incident on one face of a glass prism and emerges out from the
other face. Trace the path of the ray and derive an expression for the refractive
index of the glass prism. Also, plot a graph between the Angle of incidence and
angle of deviation

42
Answer: The graph is as shown

Consider a cross-section XYZ of a prism as shown in the figure. Let A be the angle of the
prism. A ray PQ of monochromatic light is incident on face XY of the prism at an angle i.
The ray is called the incident ray and the angle is called the angle of incidence. This ray
is refracted towards the normal NQE and travels in the prism along QR. This ray is called
the refracted ray at the face XY.

Let r1 be the angle of refraction at this surface. The refracted ray QR is incident at an
angle r2 on the surface XZ. The ray QR again suffers refraction and emerges out of face
XZ at an angle e along with RS. The ray is called the emergent ray and the angle e is
called the angle of emergence. When the ray SR is extended backwards it meets the
extended ray PQ at point D such that δ is the angle of deviation of the ray.

As seen from figure we have in triangle QDR that ∠DQR = i – r1 and


∠DRQ = e – r2. Therefore from triangle QDR we have
δ = ∠DQR + ∠DRQ = (i – r1) + (e – r2)
or
δ = (i + e) – (r1 + r2) …(1)

43
Now from the quadrilateral XQER, we have
A + E = 180° …(2)
In triangle QER we have r1 + r2 + e – 180° …(3)
From equations (2) and (3) we have
A = r1 + r2 …(4)
Substituting in equation (1) we have
δ=i+e–A or
i + e = A + δ … (5)
The deviation produced by a prism depends upon (i) the angle of incidence (ii) the
angle of prism and (Hi) the refractive index of the material of the prism. It is found
that as the angle of incidence changes, the angle of deviation also changes.

A graph between the angle of incidence and the angle of deviation is shown in the
figure above. As the angle of incidence increases, the angle of deviation first
decreases becomes a minimum for a particular angle of incidence and then
increases. The minimum value of the angle of deviation is called the angle of
minimum deviation. It is denoted by δm. In this position the ray of light passes
symmetrically through the prism, i.e. the refracted ray QR is parallel to the base of
the prism. In this position, the angle of incidence is equal to the angle of
emergence, i.e. i = e. Also in this position, the angle of refraction at the faces of the
prism are equal, i.e. r1 = r2. Substituting these values in equations (4) and (5) we
have
A = r1 + r2 = r + r = 2r
or
r = A/2 …(6)
and i + i = A + δm
or
2i = A + δm

n =sin((A+δm )/ 2) / sin(A/2). Thus, above is the prism formula.

44
6. A) Below given figure shows a cross-section of a ‘light pipe’ made of a glass
fibre of refractive index 1.68.

The outer covering of the pipe is made of a material of refractive index 1.44. What
is the range of the angles of the incident rays with the axis of the pipe for which
total reflections inside the pipe take place, as shown in the figure.
B) What is the answer if there is no outer covering of the pipe?

Ans.(a) Refractive index of the glass fibre,


Refractive index of the outer covering of the pip

e, = 1.44
Angle of incidence = i
Angle of refraction = r
Angle of incidence at the interface = i‘

The refractive index of the inner core – outer core interface is given as:

For the critical angle, total internal reflection (TIR) takes place only when ,
i.e.,

Maximum angle of reflection,

Let, be the maximum angle of incidence.

45
The refractive index at the air – glass interface,
We have the relation for the maximum angles of incidence and reflection as:

Thus, all the rays incident at angles lying in the range will suffer total
internal reflection.
(b) If the outer covering of the pipe is not present, then:

Refractive index of the outer pipe, = Refractive index of air =1


For the angle of incidence , we can write Snell’s law at the air – pipe
interface as:

Since > r, All incident rays will suffer total internal reflection

7. Derive the lens maker formula. (5M)


Answer :

46
The following assumptions are taken for the derivation of lens maker
formula. Let us consider the thin lens shown in the image above with 2 refracting
surfaces having the radii of curvatures R1 and R2, respectively.

Let the refractive indices of the surrounding medium and the lens material be n1
and n2, respectively.

The complete derivation of lens maker formula is described below. Using the
formula for refraction at a single spherical surface we can say that,

For the first surface, For the second surface,

Now adding equation (1) and (2),

When u = ∞ and v = f

Therefore, we can say that,

But also,

Where μ is the refractive index of the material. This is the lens maker formula
derivation.

*********************************************

47
WAVE OPTICS
GIST OF THE CHAPTER SHORT NOTES AND FORMULAE
Light has wave as well as particle nature. A wavefront is the locus of points
which are in the same state or phase of vibration. Huygens principle states the
method of propagation of wavefront. The wavefronts are imaginary surfaces. The
light ray cuts the wavefront at right angle at every point
Types of wavefronts

Huygens Principle-“Every point on a wavefront is in itself the source of spherical


wavelets which spread out in the forward direction at the speed of light. The sum of
these spherical wavelets forms the wavefront”

Light propagating as a wavefront. Point source produces spherical


wavefront and source at infinity produces plane wavefront. Linear sources
produces cylindrical wavefront.

48
LAWS OF REFLECTION USING HUYGENS PRINCIPLE

These incident wavefront is carrying two points, point A and point B, so we can say that
from point B to point C light is travelling a distance. If ‘ v ‘ represents the speed of the
wave in the medium and if ‘ t ‘ represents the time taken by the wavefront from the point
B to C then the distance
BC = vt
In order the construct the reflected wavefront we draw a sphere of radius v t from the
point A. Let CE represent the tangent plane drawn from the point C to this sphere. So,
AE = BC = vt
If we now consider the triangles EAC and BAC we will find that they are congruent and
therefore, the angles ‘ i ‘ and ‘r ‘ would be equal. This is the law of reflection
.
We know that when a light travels from one transparent medium to another transparent
medium its path changes. So the laws of refraction state that the angle of incidence is the
angle between the incident ray and the normal and the angle of refraction is the angle
between the refracted ray and the normal.
The incident ray, reflected ray and the normal, to the interface of any two given mediums
all lie in the same plane. We also know that the ratio of the sine of the angle of incidence
and sine of the angle of refraction is constant.

We can see a ray of light is incident on this surface and another ray which is parallel to this
ray is also incident on this surface. As these rays are incident from the surface, so we call it
incident ray.

49
Let PP’ represent the medium 1 and medium 2. The speed of the light in this medium is
represented by v1 and v2. If we draw a perpendicular from point ‘A’ to this ray of light, Point
A, and point B will have a line joining them and this is called as wavefront and this
wavefront is incident on the surface.
If ‘ τ‘ represents the time taken by the wavefront from the point B to C, then the distance,
BC=v1 τ
So, to determine the shape of the refracted wavefront, we draw a sphere of radius v2 τ from
the point A in the second medium. Let CE represent a tangent plane drawn from the point C
on to the sphere.
Then, AE = v2 τ, and CE would represent the refracted wavefront. If we now consider the
triangles ABC and AEC, we obtain
sin i = BC/AC = v1 τ /AC sin r = AE/AC= v2 τ /AC
where’ i ‘ and ‘ r ‘ are the angles of incidence and refraction, respectively. Substituting the
values of v1 and v2 in terms of we get the Snell’s Law,
n1 sin i = n2 sin r (Snell’s law)

REFRACTION FROM DENSER TO RARER MEDIUM (HERE V2>V1)

BEHAVIOUR OF A PRISM,CONVEX LENS AND A CONCAVE MIRROR WHEN


A PLANE WAVEFRONT IS INCIDENT ON IT

50
PRINCIPLE OF SUPERPOSITION OF WAVES
What is Superposition of Waves?
According to the principle of superposition. The resultant displacement of a number of
waves in a medium at a particular point is the vector sum of the individual displacements
produced by each of the waves at that point.
y=y1+y2+y3+y4+…..

INTERFERENCE
The phenomenon in which two or more waves(travelling in the same direction) superpose
to form a resultant wave of greater, lower or the same amplitude. “This results in re-
distribution of energy.

Constructive interference-Results in max. intensity of resultant wave. Crest falls over crest
or trough falls over trough
Destructive interference-Results in min. intensity of resultant wave. Crest falls over
trough of the other wave.
▪ THEORY OF INTERFERENCE
Let us consider two Simple Harmonic waves travelling in the same direction having
amplitudes a and b respectively and of same frequency interfere.
At any instant at a certain position in space let the resultant displacement at the point be
represented by equations
y1=a sinωt and y2=b sin(ωt+φ) ,where φ be the phase difference between the waves.
Then, from the Principle of superposition of waves the resultant displacement y at the
point is represented by the equation y=y1+y2
i.e y= a sinωt +b sin(ωt+φ)
Simplifying and applying certain boundary conditions ,
Rcos ϴ =(a+bcos φ ) Rsin ϴ=(bsin φ) ,here a,b, φ,R and ϴ are constants.
we get, y=R sin(ωt+ϴ) ,where “R “is the resultant amplitude and “ϴ” is the resultant
wave’s phase difference.
R = √a2 + b2 + 2ab cos φ
From wave theory ,Intensity of a wave is I α (amplitude)2 or I=k(amplitude)2
Hence, Intensity of resultant wave is
∴ I = a2 + b2 + 2ab cos φ
or I== I1 + I2 + 2 √I1 I2 cos φ where I1 and I2 are the intensities of the individual waves.

51
If the waves have the same amplitude ‘a’ then the resultant intensity at a point is
I=4I0cos2(φ/2) ,where I0 is the intensity of the individual waves,4I0 is the intensity of
Central maximum
For Constructive Interference, cos φ=+1
∴Phase difference, φ =2nπ
Path difference, Δx = nλ where, n = 0, 1, 2, 3,…
Resultant amplitude is max Rmax = (a + b), I(max)=k(a+b)2

For Destructive Interference cos φ = -1


Phase difference, φ = (2n – 1)
Path difference, Δx = (2n – 1)λ / 2 where, n = 1, 2, 3, …
Resultant amplitude is min Rmin = (a – b) , I(min)=k(a-b)2
If the waves have equal amplitude i.e a= b , then I(MAX)=K4a2 I(min) =0
NOTE:
Path difference for unit phase difference is (λ/2π)
Hence, for a phase difference of ‘φ’, the corresponding path difference is
p= (λ/2π)×φ
RATIO OF INTENSITIES , I1/I2 = a2/b2 =w1/w2
Where w1 and w2 are width of slits.
Imax =(a+b)2
Imin = (a-b)2

YOUNG’S DOUBLE SLIT EXPERIMENT

When monochromatic light passing through two narrow slits illuminates a distant screen,
a characteristic pattern of bright and dark fringes is observed. This interference pattern is
caused by the superposition of overlapping light waves originating from the two slits.
Regions of constructive interference, corresponding to bright fringes, are produced when
the path difference from the two slits to the fringe is an integral number of wavelengths
of the light. Destructive interference and dark fringes are produced when the path
difference is a half-integral number of wavelengths.

The light passing through the two slits is observed on a distant screen. When the widths
of the slits are significantly greater than the wavelength of the light, the rules of
geometrical optics hold—the light casts two shadows, and there are two illuminated
regions on the screen.

The superposition principle determines the resulting intensity pattern on the illuminated
screen. Constructive interference occurs whenever the difference in paths from the two
52
slits to a point on the screen equals an integral number of wavelengths (0, λ, 2λ,…). This
path difference guarantees that crests from the two waves arrive simultaneously.
Destructive interference arises from path differences that equal a half-integral number of
wavelengths (λ/2, 3λ/2,…). Young used geometrical arguments to show that the
superposition of the two waves results in a series of equally spaced bands, or fringes, of
high intensity, corresponding to regions of constructive interference, separated by dark
regions of complete destructive interference.

Young's double-slit experimental set up

EXPRESSION FOR LINEAR FRINGE WIDTH ‘β’

The interference between two coherent waves is studied in Young's double-slit


experiment. When two coherent waves interfere, they will produce a pattern of alternate
bright and dark fringes. In Young's double-slit experiment we have two slits separated by
a distance that acts as a coherent source. The pattern is obtained using a screen placed at a
distance.
In the figure, S1 and S2 represent the two slits through which the light passes. The slit
separation is ‘d’. A screen is placed at a distance D. When monochromatic light passes
through the slits, the spherical wavefronts will interfere. As a result, we will get a pattern
on the screen.
We know that there is a difference in the paths travelled by the two rays of light S1P and
S2P. The Path difference is S2P-S1P = S2A = Δx
Δx =dsinϴ
53
For small values of θ we can write sinθ≈tanθ , From the figure we can write tanϴ=y/D
Therefore Δx =d sinϴ= d tanϴ = d y /D
We assume that ‘y’ is the distance between the centre of the screen and the point where
the fringe is formed. Then,
y = ΔxD /d (1)

The separation between two consecutive dark fringes or bright fringes is called the fringe
width.
For constructive Interference, path difference Δx =nλ Therefore, from equation (1),
y= nλ D/d (n=0,1,2….) ............ Position of ‘nth’ bright fringe from Centre of screen.
For n=0, we get the central bright fringe called central maximum.
Separation between two consecutive bright fringes is
y(n+1)-y(n)= β=λD/d =linear width of a dark fringe between them Fringe width of dark
fringe= β=λD/d
Similarly, for destructive interference, path difference, Δx =(2n-1) λ/2 Therefore from
equation (1)
y = (2n-1) λD/d (n=1,2,3….) … ........... Position of ‘nth’ dark fringe from Centre of
screen.

Separation between two consecutive dark fringes is


y(n+1)-y(n)= β=λD/d=linear width of a bright fringe between them. Fringe width of bright
fringe= β=λD/d
Fringe width is a constant and. β α λ (‘λ’ wavelength of light used) β α D (‘D’ distance
between the slits and the screen)
β α 1/d (‘d’ is distance between the slits)

Linear Width of central maximum β0 =2y1= λD/d, here, y1= λD/2d


Fringe width in YDSE when the entire apparatus is immersed in water
β(water)= βair/n = λD/n d where ‘n’ is Refractive index of water
Angular width (ϴ)
Angular width ϴ =β/D= λ/d (independent of ’D’)

54
INTENSITY PATTERN

Coherent sources of light


Coherent source of light are those sources which emit a light wave having the same
frequency, wavelength and in the same phase or they have a constant phase difference.
Coherent sources are sources of light that emit waves which have zero or constant phase
difference and same frequency A coherent source forms sustained interference patterns
when superimposition of waves occur and the positions of maxima and minima are fixed.
Example –
Laser light is an example of coherent source of light. The light emitted by the laser light has
the same frequency and phase.
Obtaining the two sources of light from the same monochromatic source of light by using
Double slits.
Non-coherent sources of light
Incoherent sources are sources of light that emit waves which have random frequencies and
phase differences.
Why are two independent sources not coherent?
Coherent sources are defined as the sources in the which initial phase difference remains
constant. In the case of two independent sources, the initial phase difference cannot remain
constant because light is emitted due to millions of atoms and their number goes on
changing in a quite random manner. The phase difference between the two waves will not
be constant.
Essential conditions to obtain sustained interference
1.The two sources must be coherent emitting light waves of same frequency
/Wavelength and monochromatic.
2.They should preferably have the same amplitude.
3.They should be very close to each other.
4.They should be point sources of light.
5.They should emit light waves continuously.

55
What would be the effect on the interference fringes if source of white light is used?
Coloured secondary fringes of varying width are obtained of varying widths and positions.
The central bright fringe is white (no phase difference/path difference). The colour closest
to the central fringe is red and the farthest is blue. After a few fringes the colour faded
away.
Colours in thin films
A thin oil film on a water surface appears coloured because of thin-film interference which
involves the interference of light waves reflecting off the top surface of a film with the waves
reflecting from the bottom surface.

If the point source S is shifted upwards then the path difference between the rays p=(S2P-
S1P) +(SS2-SS1)
Here (S2P-S1P)=yd/D and (SS2-SS1)=∆(small value)
For constructive interference=nλ,n=0,1,2…… Hence,yd/D+∆=nλ
Or y=D/d(nλ-Δ) For the central maximum n=0,therefore .we get,
y=-D Δ/d,the negative sign shows the C.M shifts downwards from the ‘zero’ level
DIFFRACTION OF LIGHT
Diffraction is the slight bending of light as it passes around the edge of an object. The
amount of bending depends on the relative size of the wavelength of light to the size of the
opening. If the opening is much larger than the light's wavelength, the bending will be
almost unnoticeable. However, if the two are closer in size or equal, the amount of bending
is considerable, and easily seen with the naked eye.

56
SINGLE SLIT DIFFRACTION

If ‘a’ is the size of the single slit AB , then when a plane wavefront falls
on the slit, the wavelets from different parts of the incident wavefront superimpose
on each other such that the central fringe is a brightest and broadest (path difference
between all the interfering waves is zero)
For a point P on the screen, the path difference between the extreme rays, i.e
BE= p=asinϴ

Condition for dark fringe at point “P”(minimum intensity) or for destructive


interference of the waves:
Path difference ,p=asinϴ=nλ (n=1,2,3,….)
Condition for bright fringe at point “P”(maximum intensity) or for constructive
interference of the waves :
Secondary bright fringes
Path difference. p=asinϴ=(2n+1)λ /2 (n=1,2,3,….)
For small angles, sinϴ~𝜃 (angular position of the fringe)

For a bright fringe at position ‘P’, 𝜽=(2n+1) λ/2a , n=1,2,3… For a dark fringe
at position ‘P’, 𝜽′=nλ/a ,n=1,2,3…..
Linear position of bright fringe at point ‘P’ on the screen yn =ϴ

57
×D=(2n+1)Dλ/2a ,n=1,2,3….

Linear position of dark fringe at point ‘P’ on the screen yn’=ϴ’×D=nλD/a


,n=1,2,3….
Width of the secondary bright and dark fringes β= λD/a i.e y(n+1)-y(n)

Width of the central bright fringe or maximum βo=2y1’=2 λD/a


Angular width(secondary maximum an minimum)

Angular width ϴ=linear width(β)/D=λ/a (independent of ‘D’)


Angular width of central maximum

Angular width ϴ0=linear width (βo)/D= 2λ/a (independent of ‘D’)


Single slit diffraction pattern

Differences between interference and diffraction

S.No Interference Diffraction


Interference is due to
superposition of two distinct Diffraction is due to superposition of
waves coming from two coherent the secondary wavelets coming from
1. sources. different parts of the same
wavefront.
Interference fringes may or may Diffraction fringes are not to be of the
2. not be of the same width. same width.
The intensity of minima is
3. generally zero. The intensity of minima is never zero.

All bright fringes are of uniform All bright fringes are not of uniform
4. intensity. intensity.

58
Write the distinguishing features between a diffraction pattern due to a single
slit and the interference fringes produced in Young’s double slit experiment?
Single slit pattern overlapping double slit pattern

To get the double slit pattern


The no. of fringes within the envelope n=size of slit /distance between the slits
n=a/d, In the event of d≈ 0,n → ∞
Double slit pattern(intensity distribution)

FRESNEL DISTANCE ZF(limiting case of Ray Optics)


Fresnel distance is defined as the minimum distance that is travelled by a ray of
light along the linear path before diffraction

Consider a slit with a width of a and a distance of screen from the slit of ZF

59
The Distance of the screen from the slit, so that the spreading of light due to
diffraction from the centre of the screen is just equal to the size of the slit is
called Fresnel distance ZF
The angular spread θ of the central maxima (due to diffraction effect) obtained
on the screen is given by,

θ=λ/a (1)
Therefore,the linear spread of the light on the screen is y= θD = λD/a (2)
Where ‘λ’ is the wavelength and ‘a’is the slit width.

At D=ZF,y=a (from definition)


So,from equation(2)we write, λZF/a =a or ZF=a2/λ-------(3) (Fresnel Distance)
For a given size of the aperture, if distance of screen , D≤ZF ,then ,Ray Optics
is valid
i.e light travels in a straight line without significant bending.. For this condition
from eqn (3) we get,
D≤a2/λ (4) Or a≥ √𝜆𝐷

Where √𝜆𝐷 is the size of Fresnel zone ‘aF’


aF =√𝝀𝑫-------(5) (size of Fresnel zone)
For a given distance of the screen from the plane of the slit ,if size of the
aperture , a≥ aF ,them, Ray optics is valid i.e light travels in a straight line
without significant bending.

60
LIST OF IMPORTANT TERMS and FORMULAE
S.N TERM DESCRIPTION
O
1. Wavefront Wave front is defined as locus of all points having
same phase at a given instant of time. The shape of
wavefront depends on the shape of the source of
disturbance. A wavefront is always normal to the light
rays. A wavefront does not propagate in the backward
direction. (Amplitude of the secondary wavelets in the
backward direction cancels out)
2. Huygen’s Principle Every point on a wavefront is in itself the source of
spherical wavelets which spread out in the forward
direction at the speed of light. The sum of these
spherical wavelets forms the wavefront.
• Secondary sources start making their own
wavelets, these waves are similar to of primary
source.
• At any instant of time, a common tangent on the
wavelets in the forward direction gives the new
wavefront.
• The sum of spherical wavelets forms the
wavefront.
3. Interference of waves • Interference of Light Waves is defined as the
modification in the distribution of light energy
when two or more waves superimpose each
other.
• For Interference the waves emitted by sources
should be with zero phase difference or no
phase difference (coherent sources)
• These sources should emit continuous waves of
same wave length and same time period. These
sources should be very close to each other.
• The sources which emit light waves should be
coherent sources. If the waves are coherent, the
interference pattern is observable and is stable.
If the waves are incoherent, the pattern is not
visible.
3. CONSTRUCTIVE • Phase difference, φ = 2nπ
INTERFERENCE • Path difference, Δx = nλ
(Condition for maximum where, n = 0, 1, 2, 3,…
intensity in interference
pattern)

61
4. DISTRUCTIVE • Phase difference, φ = (2n – 1)π
INTERFERENCE • Path difference, Δx = (2n – 1)λ / 2
(Condition for minimum • where, n = 1, 2, 3, …
intensity in interference
pattern)
5. Ratio of maximum and 𝑰𝒎𝒂𝒙 (𝒂+𝒃)𝟐
• =(𝒂−𝒃)𝟐
minimum intensity 𝑰𝒎𝒊𝒏
𝑰𝒎𝒂𝒙 𝒓+𝟏 𝟐
• =( ) here, r=a/b
𝑰𝒎𝒊𝒏 𝒓−𝟏
• I1 / I2 = a2 / b2 = ω1 / ω2
• Where ω1 and ω2 are width of slits.
6. Resultant wave • R = √a2 + b2 + 2ab cos φ
amplitude
7. Resultant wave intensity • I = a2 + b2 + 2ab cos φ
• or I== I1 + I2 + 2 √I1 I2 cos φ where I1 and I2 are
the intensities of the individual waves.
• If a=b then,
Resultant intensity
I=4a2cos2(φ/2) = I0cos2(φ/2),where I0=4a2 is
Intensity of Central maximum
𝒏𝝀𝑫
8. Distance of nth • yn= (n=0,1,2……….)
maximum (bright 𝒅

fringe)from central
maximum(YDSE)
𝑦 𝑛𝜆
9. Angular position of nth • 𝜽𝒏 = 𝑛= (n=0,1,2……….)
𝐷 𝐷
bright fringe (YDSE)
(𝟐𝐧−𝟏)𝛌𝐃
10. Distance of nth minimum • yn’= (n=1,2,3…….)
𝟐𝐝
(dark fringe)from central
maximum(YDSE)
11. Angular position of nth 𝒚′ (𝟐𝐧−𝟏)𝛌
• 𝜽𝒏 = 𝒏= (n=1,2,3…….)
𝑫 𝟐𝐝
dark fringe (YDSE)
12. Fringe width secondary • β=λD/d
maxima, minima and
Central
maximum(YDSE)
𝜷 𝝀
13. Angular fringe width • 𝜽= =
𝑫 𝒅
secondary maxima,
minima and Central
maximum(YDSE)

62
14. Diffraction Diffraction is the slight bending of light as it passes
around the edge of an object. The amount of bending
depends on the relative size of the wavelength of light
to the size of the opening. If the opening is much
larger than the light's wavelength, the bending will be
almost unnoticeable. However, if the two are closer in
size or equal, the amount of bending is considerable,
and easily seen with the naked eye.

15. Single slit diffraction Condition for nth maximum


Path difference. p=asinϴ=(2n+1)λ /2 (n=1,2,3,….)
𝐴𝑛𝑔𝑢𝑙𝑎𝑟 𝑝𝑜𝑠𝑖𝑡𝑖𝑜𝑛 𝑜𝑓 𝑛𝑡ℎ 𝑏𝑟𝑖𝑔ℎ𝑡 𝑓𝑟𝑖𝑛𝑔𝑒
2𝑛+1
𝜃=( )𝜆( n=1,2,3,….)
2𝑎
Condition for nth minimum
p=asinϴ=nλ (n=1,2,3,….)
Angular position of nth dark fringe
θ'=nλ/a ,n=1,2,3…..
16. • β=λD/a
Fringe width of
secondary maxima and
minima)(single slit
diffraction)
𝜷 𝝀
17. • 𝜽= =
Angular fringe width of 𝑫 𝒂
secondary maxima and
minima (diffraction)
18. Fringe width of central • β0=2λD/a
maximum(diffraction)
𝜷𝟎 𝟐𝝀
19. Angular width of Central • 𝜽= =
maximum(diffraction) 𝑫 𝒂

20. • ZF=a2/λ-------(Fresnel Distance)


Limiting case of ray
• For a given size of the aperture, if distance of
optics
screen , D≤ZF ,Ray Optics is valid i.e light
Fresnel distance travels in a straight line without significant
bending..
21. • aF =√𝝀𝑫-------(size of Fresnel Zone)
Size of Fresnel zone
• For a given distance of the screen from the
plane of the slit ,if size of the aperture , a≥
aF,ray optics is valid i.e light travels in a straight
line without significant bending.

63
IMPORTANT DIAGRAMS AND GRAPHS
S.No. Topic Diagram/graph
1. Huygen’s
Principle

2. Spherical
Wavefront

3. Converging,
diverging
spherical
wavefront

4. Plane wavefront

5. Cylindrical
Wavefront

64
6. Law of
reflection based
on Huygen’s
Principle

7. Law of
refraction based
on Huygen’s
Principle(rarer
to denser)

8. Law of
refraction based
on Huygen’s
Principle(denser
to rarer)

9. Behaviour of a
prism, convex
lens, concave
mirror when a
plane wavefront
is incident on
them

65
10. Yong’s Double
Slit Experiment

11. Diffraction at a
single slit

12. Interference
Intensity
distribution
Pattern

13. Diffraction at a
single slit
intensity
distribution

14. Single slit


pattern
superimposed
on double slit
pattern

66
WAVE OPTICS MIND MAP

WAVE
OPTICS

HUYGEN’S
PRINCIPLE INTERFERE
NCE OF DIFFRACTION OF
LIGHT LIGHT

wavefront and
construction & superposition single slit
shapes principle experiment
Fresnel’s
y=y1+y2+y3+…
distance
Conditions for
maxima &
Verificatio Point source- minima(Path
n of laws of spherical Conditions for difference)
reflection Distant constructive and For sec.min ϴ=nλ/a
and source-plane destructive For sec.max
refraction Linear source - interference (C.I D.I) ϴ=(2n+1)λ/2a
cylindrical
For C.I (n=1,2,3,4..)
Young’s double φ=2nπ,p=nλ..(n=0,1,
slit experiment 2,3..)
𝜆𝐷
For D.I Fringe width β=
𝑎
φ=(2n-1)π,p=(2n- For C.M β0=2β
1)λ/2
Fringe width
β=λD/d
For C.M β0= β=λD/d Differences between
interference and
diffraction pattern
67
MULTIPLE CHOICE QUESTIONS

1.The phase difference between any two points of a wavefront is:


a)00 b π/4 c) π/2 d) π

2. In the adjoining figure, a wavefront AB moving in air is incident on a plane glass


surface XY. Its position CD, after refraction through a glass slab, is as shown also
along with the normal drawn at A and D. The refractive index of glass with respect to
air will be equal to

a) BD/AC b) AB/CD c) BD/AD d) AC/AD


3. The intensity of light issuing out of two slits in young’s experiment is in ratio 1:4.
The intensity of the minimum to the maximum will be in the ratio
a)1:2 b) 1:4
c)1:9 d) none of the above
4.In Young’s double-slit experiment, if there is no initial phase difference between
the light from two slits, a point on the screen corresponding to the fifth minimum has
path difference
a) 5 λ/2 b) 10λ/2 c)9λ/2 d)11λ/2
5. A linear aperture whose width is 0.02 cm is placed immediately in front of a lens
of focal length 60 cm. The aperture is illuminated normally by a parallel beam of
wavelength 5 x 10-5cm. The distance of the first dark band of the diffraction pattern
from the centre of the screen is:
(a) 0.20 cm (c) 0.10 cm
(b) 0.15 cm (d) 0.25 cm

68
6. Two slits in young’s double slit experiment have widths in the ratio 81 :1. The
ratio of the amplitudes of light waves is

(a) 3 :1 (b) 3 : 2
(c) 9 :1 (d) 6:1

7. The colour of bright fringe nearest to central achromatic fringe in the interference
pattern with white light will be

(a) violet
(b) red
(c) green
(d) yellow
8.At the first minimum adjacent to the central maximum of a single -slit diffraction ,
the phase difference between the Huygen’s wavelet from the edge of the slit and the
wavelet from the midpoint of the slit is

(a) π radian
(b) π/8 radian
(c) π/4 radian
(d) π/2 radian

9.A parallel beam of monochromatic light of wavelength 5000A0 is incident normally


on a single narrow slit of width 0.001mm. The light is focussed by a convex lens on a
screen placed in focal plane. The first minimum will be formed for the angle of
diffraction is equal to

(a) 00 (b) 150 (c) 300 (d) 50

10. In a Young’s double slit experiment, two slits of equal width are arranged
symmetrically w.r.to the sources. The intensity of the central bright fringe is I. If one
of the slit is closed, the intensity at the Centre of the screen will be
a) I
b) I/4
c)I/2
d)2I

69
ANSWER KEY
MULTIPLE CHOICE QUESTIONS
1) a.
wavefront is the locus of all the points on a surface which are in the same
phase i.e ,phase difference is zero
2) a
refractive index , n = 𝑣1/𝑣2 = (𝐵𝐷 𝑋 𝑡)/(𝐴𝐶 𝑋 𝑡) = 𝐵𝐷/𝐴𝐶
3) c
( a1 /a2)2 = I1 /I2 = 1/4 , a1/a2 = 1/2
now, Imax / I min = (a1+a2)2 /(a1 ---a2) 2 = 9/1
I min/Imax = 1:9
4) c Path difference for nth minima = (2n−1)λ/2

For fifth minima (n = 5) = 9λ/2


5 )b
f=D = 60 cm
For first minima,
y=λ D /a = 5×10−5× 60x / 0.02 = 0.15cm
6. c W1/W2 = a12/a22
7.a

8. a

9. c
Dsinθ = nλ θ = sin−1(nλ/d/)
=s in−1((1)(5000×10−10/0.001 x10-3)) θ=30o.
10. b
When one slit is closed, then no interference pattern will be formed and the intensity at the
center is I/4

70
ASSERTION -REASON TYPE

Given below are two statements labelled as Assertion (A) and Reason (R)
Select the most appropriate answer from the options given below:
(A)Both A and R are true and R is the correct explanation of A
(B)Both A and R are true but R is not the correct explanation of A
(C)A is true but R is false.
(D) A is false and R is also false.
1.Assertion: According to Huygens’s principle, no backward wave-front is possible.
Reason: Amplitude of secondary wavelet is proportional to (1 + cos θ) where θ is the
angle between the ray at the point of consideration and the direction of secondary
wavelet.
2. Assertion: No interference pattern is detected when two coherent sources are
infinitely close to each other.
Reason: The fringe width is inversely proportional to the distance between the two
sources.
3.Assertion: On increasing the phase difference from π/3 to π/2 in young’s double
slit experiment, the intensity decreases by 1/3 rd
Reason: I=4I0COS2 (θ/2) with usual symbol meanings

4. Assertion: Two-point coherent sources of light S1 and S2 are placed on a line as


shown. P and Q are two points on that line. If at point P maximum intensity is
observed then maximum intensity should also be observed at Q

Reason: In the figure of assertion the distance |S1P−S2P| is equal to


distance |S2Q−S1Q|

5. Assertion: When tiny circular obstacle is placed in the path of light from some
distance, a bright spot is seen at the centre of the shadow of the obstacle.
Reason: Destructive interference occurs at the centre of the shadow.

71
CASE BASED QUESTIONS

1. INTERFERENCE

Interference is based on the superposition principle. According to this principle, at a


particular point in the medium, the resultant displacement produced by a number of
waves is the vector sum of the displacements produced by each of the waves. If two
sodium lamps illuminate two pinholes S1 and S2. the intensities will add up and no
interference fringes will be observed on the screen. Here the source undergoes abrupt
phase change in times of the order of 10-10 seconds

i)In Young’s double slit experiment, the central bright fringe can be identified
(a)By using white light instead of monochromatic light
(b) As it narrower than other bright fringes
(c)As it wider than other bright fringes
(d)As it has greater intensity than the other bright fringes
(ii) In a Young's double-slit experiment, the slit separation is doubled. To maintain
the same fringe spacing on the screen, the screen-to-slit distance D must be changed
to
(a) 2D (b) 4D (c) D/2 (d) D/4

(iii) In an interference experiment, third bright fringe is obtained at a point on the


screen with a light of 700 nm. What should be the wavelength of the light source in
order to obtain 5th bright fringe at the same point
(a)500nm (b)420nm (c) 750nm (d)630nm
(iv) The resultant amplitude of a vibrating particle by the superposition of the two
waves y1=a sin[ωt+π3] and y2=a sinωt is
(a) a (b)2√2 a (c) 2a (d) √3 a

72
2. SINGLE SLIT DIFFRACTION
In the single slit diffraction experiment, we can observe the bending of light or
diffraction that causes light from a source interfere with itself and produce a
distinctive pattern called the diffraction pattern. Diffraction is evident when the
sources are small enough that they are relatively the size of the wavelength of light.

i)To observe diffraction, the size of the obstacle


a) Should be λ/2 , where λ is the wavelength of light
b) Should be of the order of wave length of light
c)Has no relation to wavelength of light
d) Should be much larger than the wavelength
ii)When violet light of wavelength 415nm falls on a single slit ,it creates a central
diffraction peak that is 9.2cm wide on a screen that is 2.55m away. How wide is the slit
a)3x108m b)3.8x10-7m c)2.3x10-5m d)4.2x10-7m
iii)A single slit 1mm wide is illuminated by 450nm light. What is the width of the
central maximum in the diffraction pattern on a screen 5cm away ?
a)300x10-7m b)800x10-7cm c)450x10-7m d)500 x10-7m
iv)What happen with the diffraction pattern if the whole apparatus is immersed in
water
a)Width of central maximum increases
b)Width of central maximum decreases
c)Wavelength of light increases
d)Frequency of light decreases

73
ANSWER KEY
ASSERTION -REASON TYPE
1.b
2. a
3. a
4. b

If maximum intensity is observed at P then for maximum intensity to be also

observed at Q, S1 and S2 must have phase difference of 2 nπ (where n is an integer).


5.c
The waves diffracted from the edges of circular obstacle, placed in the path of light,
interfere constructively at the centre of the shadow resulting in the formation of a
bright spot.

CASE BASED QUESTIONS (1)

(i) a
(ii) a ( β=λ D/d)
(iii) b , 3x700D/d = 5 λ D/d λ = 420nm
(iv) d Resultant amplitude =√ a2+a2+2axacos π/3 = √3a

CASE BASED QUESTIONS (2)


1. b
2. c
a = 2λD/β =2.3x10-5 m
3. c β=2λD/a =2λD/a = 450x10-7m
4. b Wave length decrease

74
FREQUENTLY ASKED QUESTIONS

VERY SHORT ANSWER TYPE QUESTIONS (1 MARK )

1. How does the angular separation of interference fringes change in young’s


experiment, if the distance between the slits is increased?
Answer:

2. State the reason, why two independent sources of light cannot be considered as
coherent sources.
Answer:
Two independent sources of light cannot be coherent. This is because light is
emitted by individual atoms, when they return to ground state. Even the
smallest source of light contains billions of atoms which obviously cannot
emit light waves in the same phase.
3. What type of wavefront will emerge from a
(i) point source, and
(ii) distant light source?

Answer:
(i) Point source – Spherical wavefront
(ii) Distant light source – Plane wavefront.

4. Draw a diagram to show refraction of a plane wave front incident in a convex


lens and hence draw the refracted wave front.

Answer:

5. How does the fringe width, in young’s double-slit experiment, change when the
distance of separation between the slits and screen is doubled?
Answer:
Doubled, Fringe width α D

75
6. In a single slit diffraction experiment, the width of the slit is reduced to half
its original width. How would this affect the size and intensity of the central
maximum?
Answer:
Β=λD/d=2β ,when d=d/2
Iα(width)2
Hence,I=I/4
7. Define the term ‘coherent sources’ which are required to produce interference
pattern in Young’s double slit experiment.
Answer:
Two monochromatic sources, which produce light waves, having a constant
phase difference are defined as coherent sources.
8. Define the term
‘wavefront’ Answer:
The wavefront is defined as the locus of all particles of a medium, which are
vibrating in the same phase.
9. Draw the shape of the wavefront coming out of a concave mirror when
a plane wave is incident on it.
Answer:

10.How does the fringe width of interference fringes change, when the whole
apparatus of Young’s experiment is kept in a liquid of refractive index 1.3?
Answer:

β(water)=β(air)/n=β(air)/1.3

11.A interference pattern is obtained using a red light, what will be effect on
interference fringes, if the red light is replaced by blue light?
Answer:

Fringe width βis directly proportional to wavelength. The wave length of blue light is
less than that of red light, hence if red light is replaced by the blue light, the fringe
width decreases.

76
SHORT ANSWER TYPE QUESTIONS (2 MARKS)

1. .Two slits are made 1mm apart and the screen is placed away. What should be
the width of each slit to obtain 10 maxima of the double slit pattern within the
central maximum of the single slit pattern.
Answer:
As per question, width of central maxima of single slit pattern = width of10
maxima of double slit pattern
2λD/a=10(λD/d)
a=0.2d , 0.2×10−3=0.2×10−3m=0.2mm

2. Sketch a graph showing the variation of fringe width versus the distance of the
screen from the plane of the slits (keeping other parameters same) in young’s
double slit experiment What information can one obtain from the slope of this
graph?
Answer:

Information from graph: Wavelength λ = Slope x d


3. (i)State one feature by which the phenomenon of interference can be
distinguished from that of diffraction.
(ii)A parallel beam of light of wavelength 600 nm is incident normally on a slit of
width ‘a’. If the distance between the slits and the screen is 0.8 m and the distance
of 2nd order maximum from the centre of the screen is. 15 mm, calculate the
width of the slit.

Answer:
(i) In interference all the maxima are of equal intensity.
In diffraction pattern central fringe is of maximum intensity while intensity of
secondary maxima falls rapidly.
(ii) Position of nth maximum, y=(2n+1)λD/2d ,here n=2
y=5λD/2d
Substituting and solving, we get,d=80µm

77
4. State two conditions required for obtaining coherent sources In Young’s
arrangement to produce interference pattern, show that dark and bright fringes
appearing on the screen are equally spaced.
Answer:
Two conditions for obtaining coherent sources: (0 Two sources should give
monochromatic light.
(ii)Coherent sources of light should be obtained from a single source by some
device. The fringe width (dark and bright) is given by

Hence, it is same for both dark and bright fringes, so they are equally spaced on the
screen.
5. Laser light of wavelength 640 nm incident on a pair of slits produces an
interference pattern in which the bright fringes are separated by 7.2 mm.
Calculate the wavelength of another source of light which produces interference
fringes separated by 8.1 mm using same arrangement. Also find the minimum
value of the order ‘n’ of bright fringe of shorter wavelength which coincides with
that of the longer wavelength.

Answer:
Distance between two bright fringes = Fringe width
β=λD/d ,for same values of D and d ,we get

β1/β2=λ1/λ2

Substituting and solving we get ,λ2=720 nm


Calculation of minimum value of order for n to be minimum
(n + 1)th maxima of shorter wavelength should coincide with nth maximum of
longer wavelength (n+1)640= n(720),n=8.Hence minimum order of shorter
wavelength is (n+1)=8+1=9

78
6. Yellow light (λ = 6000Å) illuminates a single slit of width 1 x 10-4 m. Calculate
(i) the distance between the two dark lines on either side of the central maximum,
when the diffraction pattern is viewed on a screen kept 1.5 m away from the slit;
(ii) the angular spread of the first diffraction minimum.
Answer:
(i) Distance between two dark lines, on either side of central maximum is
=2λD/a,Substituitng and solving we get,18mm
(ii)Angular spread of the first diffraction minimum (on either side)

ϴ=λ/a
Substituting and solving we get ϴ=6*10-3 rad
7. A parallel beam of light of 500 nm falls on a narrow slit and the resulting
diffraction pattern is observed on a screen 1 m away. It is observed that the first
minimum is at a distance of 2.5 mm from the centre of the screen. Calculate the
width of the slit.

Answer:
Path difference for nth minimum is asinϴ=nλ or aϴ=nλ Also,ϴ=x/D
From this we get,
ax/D=nλ Substituting and solving we get. a=2x10-4 m
8. (i) In what way is diffraction from each slit related to the interference pattern in a
double slit experiment?

(ii) When a tiny circular obstacle is placed in the path of light from a distance source,
a bright spot is seen at the centre of the shadow of the obstacle. Explain, why.
Answer:
(i) Diffraction from each slit is related to interference pattern in a double slit
experiment in the following ways :
The intensity of minima for diffraction is never zero, while for interference it is
generally zero.
All bright fringes for diffraction are not of uniform intensity, while for interference,
these are of uniform intensity

79
(ii) Waves from the distant source are diffracted by the edge of the circular
obstacle and these diffracted waves interfere constructively at the centre of the
obstacle’s shadow producing a bright spot
9. (a)Write the conditions under which light sources can be said to be coherent.

(b) Why is it necessary to have coherent sources in order to produce an


interference pattern?
Answer:
a)Coherent sources of light. The sources of light, which emit continuously light
waves of the same wavelength, same frequency and in same phase are called
Coherent sources of light.

Two independent sources cannot act as coherent sources and interference pattern is
not obtained. This is because phase difference between the light waves emitted from
two different sodium lamps will change continuously.
(a)Conditions for interference. The important conditions for obtaining interference
of light are :
The two sources of light must be coherent. i.e. they should exist continuous waves
of same wavelength or frequency.
The two sources should be monochromatic. The phase difference of waves from
two sources should be constant. The amplitude of waves from two sources should
be equal.
The coherent sources must be very close to each other
10. Estimate the distance for which ray optics is good approximation for an aperture
of 4 mm and wavelength 400 nm.
Answer
Fresnel’s distance (Zf ) is the distance for which the ray optics is a good
approximation. It is given by the relation, Zf = a2/λ
Where, Aperture width, a = 4 mm = 4 ×10 -3 m
Wavelength of light, λ = 400 nm = 400 × 10 -9 m Zf = (4x10-3)2/400x10-9 = 40 m
Therefore, the distance for which the ray optics is a good approximation is 40 m.

80
SHORT ANSWER TYPE QUESTIONS (3 MARKS)

1. Draw the intensity pattern for single slit diffraction and double slit
interference. Hence, state two differences between interference and
diffraction patterns.

Answer:
Intensity distribution in the diffraction due to single slit

(iii) Intensity pattern for double slit interference.

(iv) Difference between Interference and Difference Patterns: Interference pattern


and Diffraction pattern :
(v) The diagram, given here, shows several fringes, due to double slit interference,
‘contained’ in a broad diffraction peak. When the separation between the slits is
large compared to their width, the diffraction pattern becomes very flat and we
observe the two slit interference pattern.

81
Basic features of distinction between interference and diffraction patterns :
(i)The interference pattern has a number of equally spaced bright and dark bands
while diffraction pattern has a central bright maximum which is twice as wide as the
other maxima.

(ii)Interference pattern is the superimposition of two waves slits originating from two
narrow slits. The diffraction pattern is a superposition of a continuous family of waves
originating from each point on a single slit.
(iii)For a single slit of width ‘a’ the first null of diffraction pattern occurs at an angle
of λ/a. At the same angle of λ/a, we get a maxima for two narrow slits separated by a
distance ‘a’.
2. Two monochromatic waves emanating from two coherent sources have the
displacements represented by y1 = a cos ωt and y2 = a cos (ωt + ϕ ), where ϕ is
the phase difference between the two displacements. (a)Show that the resultant
intensity at a point due to their superposition is given by I = 4I0 cos2 ϕ/2, where
I0 = a2.
(b) Hence obtain the conditions for constructive and destructive interference.
Answer:
(a) Let the two waves be represented by equations y1=acosωt,y2=a
cos(ωt+φ)
From the Principle of superposition of waves we get,
y=y1+y2=2acos(φ/2) cos (ωt+φ/2)
Resultant Amplitude is A=2acos(φ/2)
Hence, Resultant Intensity α (amplitude)2 =4a2cos2(φ/2)
(b) For constructive interference:

(ii)For destructive interference:

Cos (Ф/2) = 0 or Ф/2 = (2n-1)π/2 Ф = (2n-1)π

82
3. A slit of width “a” is illuminated by light of wavelength, 650nm.What will be
the value of the width ‘a’ when
a) first minimum falls at an angle of diffraction 300?
b) first maximum falls at an angle of diffraction 300?
(c) Why does the intensity of the secondary maximum become less as compared
to the central maximum?
Answer:
(i)first minimum at 300 satisfies the condition,
d sinθ=λ, d= λ/sin 30 =1300nm
(ii) first maxima at 300 satisfies the condition
d sinθ =3λ/2 d= 3λ/2sin 30 =1950nm
(b) As the order increases only 1/nth (where n is an odd number) of the slit, will
contribute in producing brightness at a point in diffraction. So the higher order
maxima are not so bright as the central maximum

4. In Young’s double slit experiment to produce interference pattern, obtain the


conditions for constructive and destructive interference. Hence deduce the
expression for the fringe width.
Answer:
Let the Resultant wave Amplitude be ‘A’
A2=a12+a22+2a1a2cosφ
For constructive interference:
Cosφ=+1,so,Phase difference is φ=2nπ and path difference is p=nλ
So,yd/D=nλ From this we get,y=nλD/d n=0,1,2…..(position of nth minimum)
Since the separation between the centres of two consecutive bright fringes is called
fringe width. It is denoted by β
β=y(n+1) - y(n)=λD/d
For destructive interference,Cosφ=-1,So φ=(2n-1)π,n=1,2,3….
Hence path difference is p= (2n-1)λ/2
Position of nth minimum is y=(2n-1)λD/2d
Fringe width β=y(n+1) -yn=λD/d

83
5. A plane wavefront is incident at an angle of incidence i on a reflecting
surface. Draw a diagram showing incident wavefront, reflected wavefront and
verify the laws of reflection.
Answer:

These incident wavefront is carrying two points, point A and point B, so we can say
that from point B to point C light is travelling a distance. If ‘ v ‘ represents the speed
of the wave in the medium and if ‘ t ‘ represents the time taken by the wavefront from
the point B to C then the distance

BC = vt

In order the construct the reflected wavefront we draw a sphere of radius v t from the
point A. Let CE represent the tangent plane drawn from the point C to this sphere. So,

AE = BC = vt

If we now consider the triangles EAC and BAC we will find that

AC is common , ˂ABC =˂AEC , AE=BC,


they are congruent and therefore, the angles ‘ i ‘ and ‘r ‘ would be equal. This is the
law of reflection
6. The intensity pattern at the central maxima in Young’s double slit experiment is I0.
Find out the intensity at a a point where the path difference is λ/4 &λ/3.
Answer:
If the path difference =λ/4
Here for the given, first case, the path difference is given as Δx=λ/4 , Δx=2π/λ× ϕ
⇒ϕ=π/2
Now substituting the value of ϕ=π/2 in the equation I=4I0cos2ϕ/2 which provides
us the intensity given as

84
⇒I=4I(π/4) ⇒I=2I0
Therefore the intensity at a point on a screen in Young’s double-slit experiment
where interfering waves of equal intensity have a path difference of λ/4 is given
as 2I0.
II. For λ/3
⇒ϕ=2π/3
⇒I=4I0cos2(2π/6 )⇒
∴I=I0
Therefore the intensity at a point on a screen in Young’s double-slit experiment
where interfering waves of equal intensity have a path difference of λ/3 is given as I0.

7. Consider two slit interference arrangements such that the distance of the screen from
the slits is half the distance between the slits. Obtain the value of D in terms of λ such
that the first minima on the screen fall at a distance D from the centre O.

Answer:
S1P = √D2 + (D – x)2 , S2P = √D2 + (D + x)2

T2P = D + x , T1P = D – x

[D2 + (D+x)2]-1/2 – [D2+(D-x)2]1/2 = λ/2 if X=D

D= λ/2(√5-1)

8. A beam of light consisting of two wavelength 800 nm and 600 nm is used to obtain
the interference fringes in young’s double slit experiment on a screen held 1.4 m
away. If the two slits are separated by 0.28 mm, calculate the least distance from the
central bright maximum, where the bright fringes of the two wavelengths coincide.

85
Answer: The two bright fringes will coincide when
n1λ1=n2λ2
⇒n1×800×10−9= n2×600×10−9
⇒4n1=3n2
n1≠=0,n2≠ 0
For y to be minimum and since n are integers, n1=3,n2=4
y=n1λ1D/d= 3x800x10-9 x1.4/0.28x10-3
⇒y=1.2×10−2m
⇒y=1.2 cm
9. A parallel beam of light of wavelength 500 nm falls on a narrow slit and the resulting

diffraction pattern is observed on a screen 1 m away. It is observed that the first

minimum is at a distance of 2.5 mm from the centre of the screen. Find the width of

the slit.

Answer:
Wavelength of light beam, λ = 500 nm = 500 × 10 -9 m

Distance of the screen from the slit, D = 1 m

For first minima, n = 1

Distance between the slits = d

Distance of the first minimum from the centre of the screen can be obtained as:

x = 2.5 mm = 2.5 × 10 -3 m

It is related to the order of minima as:

d = nλD/x = 1x500x10-9x1 / 2.5x10-3 = 2x10-4 = 0.2 mm

Therefore, the width of the slit is 0.2 mm.

86
10. In a double-slit experiment the angular width of a fringe is found to be 0.2° on a
screen placed 1 m away. The wavelength of light used is 600 nm. What will be
the angular width of the fringe if the entire experimental apparatus is immersed
in water? Take refractive index of water to be 4/3.

Answer
Given that,
Distance of the screen from the slits, D = 1 m

Wavelength of light used, λ1 =600 nm

Angular width of the fringe in air, θ1 = 0.2°

Angular width of the fringe in water = θ,

Refractive index of water, µ = 4/3

Refractive index is related to angular width as: µ = θ1/ θ2

θ2 = 3/4 θ1 = 3/4 x 0.2 = 0.15

Therefore, the angular width of the fringe in water will reduce to 0.15°.

32. In a Young’s double-slit experiment, the slits are separated by 0.28 mm and
the screen is placed 1.4 m away. The distance between the central bright fringe
and the fourth bright fringe is measured to be 1.2 cm. Determine the wavelength
of light used in the experiment.

Answer:

Here it is given that,

Distance between the slits, d = 0.28 mm = 0.28 × 10 -3 m

Distance between the slits and the screen, D = 1.4 m

87
Distance between the central fringe and the fourth (n = 4) fringe, u = 1.2 cm = 1.2
× 10 -2 m

For constructive interference, the distance between the two fringes is given by
relation: u = nλ D/d

where, n = Order of fringes

wavelength of the light can be given as: λ = ud/nD = 1.2x10-2x0.28x10-3/4x1.4 =


6x10-7 = 600 nm

Hence, the wavelength of the light is 6 x 10 -7 m.

11. Two towers on top of two hills are 40 km apart. The line joining them passes 50 m
above a hill halfway between the towers. What is the longest wavelength of radio
waves, which can be sent between the towers without appreciable diffraction effects?

Answer
Distance between the towers, d = 40 km

Height of the line joining the hills, d = 50 m.

Thus, the radial spread of the radio waves should not exceed 50 km.

Since the hill is located halfway between the towers, Fresnel’s distance can be
obtained as: Z P = 20 km = 2 × 10 4 m

Aperture can be taken as, a = d = 50 m

Fresnel’s distance is given by the relation, Zp = a2/λ

Where, λ = Wavelength of radio waves

Therefore, λ = a2/Zp = (50)2 / 2x104 = 1250x10-4 = 0.1250 m = 12.5 cm

Therefore, the wavelength of the radio waves is 12.5 cm.

88
LONG ANSWER QUESTIONS (5 MARKS)

1. State Huygen’s principle. Using this principle draw a diagram to show how a
plane wave front incident at the interface of the two media gets refracted when
it propagates from a rarer to a denser medium. Hence verify Snell’s law of
refraction.
(b) When monochromatic light travels from a rarer to a denser medium,
explain the following, giving reasons :
(i) Is the frequency of reflected and refracted light same as the frequency of
incident light?
(ii) Does the decrease in speed imply a reduction in the energy carried by light
wave?
Answer:

(a) Huygen’s Principle ; Huygen’s Principle is based on two assumptions :


(i) Each point on the primary wavefront is a source of a new disturbance called
secondary wavelets which travel in all directions with same velocity as that of
original waves.
(ii) A surface tangential to the secondary . wavelets gives the position and shape of
new wavefront at any instant. This is called secondary wavefront.
Verification of Snell’s Law, From the figure

If ‘ τ‘ represents the time taken by the wavefront from the point B to C, then the
distance, BC=v1 τ

89
So, to determine the shape of the refracted wavefront, we draw a sphere of radius v2 τ
from the point A in the second medium. Let CE represent a tangent plane drawn from
the point C on to the sphere.

Then, AE = v2 τ, and CE would represent the refracted wavefront. If we now consider


the triangles ABC and AEC, we obtain
sin i = BC/AC = v1 τ /AC sin r =AE/AC= v2 τ /AC

where’ i ‘ and ‘ r ‘ are the angles of incidence and refraction, respectively.


Substituting the values of v1 and v2 in terms of we get the Snell’s Law,

n1 sin i = n2 sin r (Snell’s law)


(b) (i) Yes, frequency is the property of source. Hence, frequency does not change
when light is reflected or refracted.
(ii) No, decrease in speed does not imply reduction in energy carried by light
wave.

This is because the frequency does not change and according to the formula E =
hv, energy will be independent of speed. Energy carried by a wave depends on the
amplitude of the wave, not on the speed of wave propagation

2. (a) In a single slit diffraction experiment, the width of the slit is made double
the original width. How does this affect the size and intensity of the central
diffraction band?

(b) In what way is diffraction from each slit related to the interference pattern
in a double-slit experiment?
(c) When a tiny circular obstacle is placed in the path of light from a distant
source, a bright spot is seen at the centre of the shadow of the obstacle. Explain
why?
(d) Two students are separated by a 7 m partition wall in a room 10 m high. If
both light and sound waves can bend around obstacles, how is it that the
students are unable to see each other even though they can converse easily.

90
(e) Ray optics is based on the assumption that light travels in a straight line.
Diffraction effects (observed when light propagates through small
apertures/slits or around small obstacles) disprove this assumption. Yet the ray
optics assumption is so commonly used in understanding location and several
other properties of images in optical instruments. What is the justification?
Answer:
(a) When width (a) of single slit is made double, the half angular width of central
maximum which is λ/a, reduces to half. The intensity of central maximum will
become 4 times. This is because area of central diffraction band would become
1/4th.
(b) If width of each slit is of the order of λ, then interference pattern in the
double slit experiment is modified by the diffraction pattern from each of the two
slits.
(c) When a tiny circular obstacle is placed in the path of light from a distant
source, a bright spot is seen at the centre of the shadow of the obstacle. This is
because light waves are diffracted from the edge of the circular obstacle, which
interferes constructively at the centre of the shadow. This constructive interference
produces a bright spot.
(d) Bending of waves by obstacles by a large angle is possible when the size of the
obstacle is comparable to the wavelength of the waves. On the one hand, the
wavelength of the light waves is too small in comparison to the size of the
obstacle. Thus, the diffraction angle will be very small. Hence, the students are
unable to see each other. On the other hand, the size of the wall is comparable to
the wavelength of the sound waves. Thus, the bending of the waves takes place at a
large angle. Hence, the students are able to hear each other.
(e) The ray optics assumption is used in understanding location and several other
properties of images in optical instruments. This is because typical sizes of
aperture involved in ordinary optical instruments are much larger than the
wavelength of light. Therefore, diffraction or bending of waves is of no
significance.

91
3. a) Describe briefly how a diffraction pattern is obtained on a screen due to a

single narrow slit illuminated by a monochromatic source of light. Hence obtain

the conditions for the angular width of secondary maxima and secondary minima.

(b) Two wavelengths of sodium light of 590 nm and 596 nm are used in turn to

study the diffraction taking place at a single slit of aperture 2 × 10-6 m. The

distance between the slit and the screen is 1.5 m. Calculate the separation

between the positions of first maxima of the diffraction pattern obtained in the

two cases.

Answer:
(a)Diffraction at a single slit. Suppose a parallel beam of monochromatic light of
wavelength λ falls normally on a slit AB of width d (of the order of the wavelength
of light). The diffraction occurs on passing through the slit. The diffraction pattern
is focused on to the screen by a convex lens. The diffraction pattern consists of a
central bright fringe (or band), having alternate dark and bright fringes of
decreasing intensity on both sides.
1. Position of central maximum. Let C be the centre of the slit AB. According to
Huygen’s principle, “when light falls on the slit, it becomes a source of secondary
wavelets.” All the wavelets originating from slit AB are in same phase. These
secondary waves reinforce each other resulting the central maximum intensity at
O.

92
Position of secondary maxima and minima. Consider a point P on the screen.
All the secondary waves travelling in a direction making angle θ with CO, reach
at a point P. The intensity at P depends on the path difference between
secondary waves.
∴ Path difference between the secondary waves reaching P from points A and B
is, BN = d sin θ
(i)If path difference ,BN =(2n+1)λ/2
d sin θ = (2n+1)λ/2
When θ is very small , then sinθ~ θ
θ = (2n+1)λ/2d
for n=1, θ=3λ/2d ,forst secondary maximum

The point P will be the position of nth secondary maxima


If path difference BN=nλ
d sin θ = nλ ,When θ is very small , then sinθ~ θ
θ=nλ/d
for n=1 , θ=λ/d ,forst secondary minima
Hence, the diffraction pattern due to single slit consists of a centre bright
maximum at O along with secondary maxima and minima on either side..

(b)First maxima due to λ1(β1) = λ1D x3/2d


First maxima doe to λ2(β2) = λ2D x3/2d

β1- β2 = 3D (λ2-λ1)/2d = 6.75 mm

****************************************

93
Chapter11: Dual Nature of Radiation and Matter
SHORT NOTES :

Three types of electron emission


(i) Thermionic emission : By suitably heating the metal, heat energy is supplied,
thereby electrons can be ejected out. The emitted electrons are called thermions.
(ii) Field emission: By applying strong electric field (108 V/m), electrons can be
emitted from the metal surface.
(iii) Photoelectric emission : when light of suitable frequency is incident on a metal
surface, electrons are emitted. Such electrons are called photoelectrons.

Work function (ф0) : The minimum energy required by an electron to just escape
from the metal surface.

Unit of work function : electron volt (eV)

Work function is less for Caesium (Cs) = 2.14 eV and high for Platinum (Pt) = 5.65 eV

Photoelectric effect :( Discovered by Heinrich Hertz)


The phenomenon of ejection of electros from a metal surface when light of
sufficiently high frequency falls on it.
Threshold frequency (ν0 ) : It is the minimum frequency of the incident photons below
which , no photoelectric emission takes place.
ф0 = h𝜈0

It depends on the nature of material of emitter plate.

Eg : Alkali metals emit photo electrons when visible light incident on it. (Li, K, Na,
Rb)
Zn, Cd , Mg etc emit electrons when UV light incident on it.

Einstein’s photo electric equation:


According to Einstein, the emission of electrons is due to the interaction of a
photon of Energy ‘hν’ with the electron. The photon is completely absorbed by the
electron. When the electron absorbs energy ‘hν’ , it uses a part of it to overcome work
function φ0 , and the emitted electron possess K.E ,
Kmax= hν – φ0.
½ mVmax2 = hν – φ0, Einstein’s photoelectric equation.
If ν0 is the threshold frequency, φ0 = hν0
hν = hν0 + ½ mVmax2

94
Photoelectric effect and wave theory of light:
According to wave theory, when light is incident on a metal surface, it spreads
evenly on metal surface. The energy is equally shared by all free electrons. But each
electron gets small energy and as time increases, electron receives more and more
energy and finally ejected out. Wave theory couldn’t explain the instantaneous
emission of electrons.
Also as per wave theory, the K.E of electron depends on the intensity of light.
But K.E is found to increase with frequency. Therefore, wave theory fails to explain
the K.E of electron.

Particle nature of light : Photon


Photon is a packet of energy (quanta of energy). Energy of each photon is E =
hν = hc/λ
Momentum of photon is P = hν/c E = mc2, m = E/C2 = h𝜈/ C2
P = mV = h𝜈 /C2 x C = h𝜈/C
Properties of photon
1. All photons of same frequency have same energy and momentum, irrespective of
intensity.
2. As intensity increases, the number of photons increases.
3. Photons are electrically neutral, not deflected by electric & magnetic Fields.
4. In a photon –electron collision, the total energy & tot. momentum remains
conserved.

Wave nature of matter :


According to de Broglie , nature has two basic entities - Matter & energy. If radiation
shows dual nature (wave & particle), matter also shows dual nature.
Matter waves : (de Broglie waves)
Waves associated with moving particle.
Properties of matter waves :
1. Independent of charge on matter.
2. They are not EM waves because EM waves are produced by accelerated charged
particle.

95
de Broglie wavelength:
According to quantum theory , E = hν, and P = hν/c = h/λ
λ = h/p = h/mv ---------------------(1) de Broglie’s wave equation for material
particles.
Where’ m’ is the mass of particle and V is velocity. This wavelength is equal to the
wavelength of EM radiation of which photon is a quantum of energy. When v = 0, λ =
∞. ie waves are associated with moving particles only.
Heisenberg’s uncertainty principle:
It is not possible to measure both the position and momentum of an electron or any
particle at the same time exactly.
There is some uncertainty ∆x in the specification of position and ∆p in momentum,
∆x .∆P ≅ h/2π.

96
IMPORTANT DIAGRAMS & GRAPHS :

(i) Effect of intensity of light on photocurrent :

Keeping the frequency of light and potential b/w the plates constant, as the intensity of
light increases, more and more photoelectrons are emitted. ie photocurrent is directly
proportional to intensity of light.

(ii) Effect of Potential on photoelectric current:

Saturation current is proportional to the intensity of light, but stopping potential is


independent of intensity, for same frequency.

97
(iii) Effect of frequency of incident radiation on stopping potential

Keeping the intensity constant, as frequency of the light increases, the stopping
potential becomes more negative but the saturation current remains same.

(iv) Variation of stopping potential V0 with frequency for given materials A and
B

(v) Verification of Einstein’s photoelectric equation : (determination of


Planck’s constant by Millikan)

V0 = hν/e - hν0/e
The graph between V0 and ν is a straight line with intercept -hν0/e . this helped to
calculate the value of h.(6.62 × 10-34 Js)

98
KEY POINTS :
• Work function (∅0 ) : The minimum energy required for an electron to escape
from the metal surface.
• Photo electric Emission : When Light of suitable frequency illuminates the
metal surface, electrons are emitted.
• Threshold Frequency : The minimum frequency of incident photons below
which no photo electric emission occurs.
• photocurrent is directly proportional to intensity of light.

• Stopping Potential : The minimum negative (retarding) potential of the


collector plate at which the photo electric current becomes zero.
• Photoelectric current is zero, when the retarding potential is able to stop the
most energetic electrons. Therefore Kmax = eV0
• ½ mVmax2 = eV0. For a given frequency of incident photon, stopping potential
is independent of intensity of light.
• As frequency of incident photon increases , K.E also increases. Kmax α 𝜈
• K.E of electrons is independent of intensity of light.
• ½ mVmax2 = hν – φ0, This is Einstein’s photoelectric equation.
• Photon is a packet of energy (quanta of energy). Energy of each photon is E =
hν = hc/λ
• Momentum of photon is P = hν/c
• de Broglie wavelength, λ = h/p = h/mv
• de- Broglie wavelength of an electron = λ = 12.27/ √𝑉 A0

IMPORTANT FORMULAE :

➢ Work function, ф0 = h𝜈0

➢ Maximum Kinetic Energy, ½ mVmax2 = eV0

➢ Photo electric equation, hν = h𝜈0 + ½ mVmax2

➢ Momentum of photon is P = hν/c

➢ de Broglie wavelength, λ = h/p = h/mv


➢ de Broglie wavelength for a charged particle accelerated through a potential ‘V’,

λ = h/√2𝑚𝑞𝑉

de Broglie wavelength of an electron, λ = 12.27/ √𝑉 A

99
CONCEPT MAP ON DUAL NATURE OF MATTER AND RADIATION

Electron emission

Thermionic Emission
Photo Electric Emission

Field
Emission

Experimental study
ofPhoto electric
effect

Effect of Potential on
Effect of Intensity of Photo current and
light on Photo current Graph
and Graph

Effect of Frequency of
Incident radiation on
stopping Potential
and Graph

Einstein’s Photo Electric


Photo electric effect and Equation
Wave Theory of Light
Kmax = hν – φ0
Wave Nature of Matter
and de Broglie relation
Dual Nature of Particle Nature of
λ=h/p Radiation & Matter light:the Photon

E= hν

100
MULTIPLE CHOICE QUESTIONS
1.Consider a beam of electrons (each electron with energy E0) incident on a metal
surface kept in an evacuated chamber. Then
(a) no electrons will be emitted as only photons can emit electrons
(b) electrons can be emitted but all with an energy E0
(c) electrons can be emitted with any energy, with a maximum of E0-φ (φ is the
work function)
(d) electrons can be emitted with any energy, with a maximum of E0
2.Threshold wavelength for a metal having work function W0 is λ. What is the threshold
wavelength for a metal having work function 2W0?
(a)4λ (b)2 λ (c) λ/2 (d) λ/4
3.A proton, a neutron, an electron and an α particle have same energy. Then their de-
Broglie wavelengths compare as
(a) λp = λn > λe > λα (b) λα < λp = λn < λe
(c) λe < λp = λn > λα (d) λe = λp = λα = λn
4. The energy of photon of wavelength 450nm is
(a) 2.5x10-17J (b) 1.25x 10-17 J (c) 4.4x 10-19J (d) 2.5 x 10-19J
5.A proton and an α-particle have the same de Broglie wavelength. What is same for
both of them?
(a) Mass (b) Energy (c) Frequency (d) Momentum
6. Wave theory of light could not explain
(a) interference (b) diffraction (c) reflection (d) photo electric effect
7.The photoelectric effect is based on conservation of
(a)momentum (b) energy (c) mass (d) angular momentum
8 In photoelectric effect, the number of photoelectrons emitted is proportional to
(a) intensity of incident beam (b) frequency of incident beam
(c)velocity of incident beam (d)work function of photocathode.
9. A photon will have less energy if its
(a) amplitude is higher (b) frequency is higher
(c) wave length is longer (d) wave length is shorter
10. Why do we not observe the phenomenon of photoelectric effect with non-metals?
i) For non metals the work function is high ii) Work function is low
iii) Work function can’t be calculated iv) For non metals, threshold frequency is low

101
ASSERTION- REASON

Directions: In each of the following questions, a statement of Assertion (A) is


given followed by a corresponding statement of Reason (R) just below it. Of the
statements, mark the correct answer as:

(A)If both assertion and reason are true and reason is the correct explanation
of assertion (B)If both assertion and reason are true but reason is not the
correct explanation of assertion

(C)If assertion is true and reason is false

(D)If both assertion and reason are false


1. Assertion: A photon has no rest mass , yet it carries definite momentum.
Reason: Momentum of photon is due to its energy and hence its equivalent mass.
(a)A (b)B (c)C (d)D
2. Assertion: Photoelectric effect demonstrates the wave nature of light.
Reason. The number of photoelectrons is proportional to the frequency of light.
(a)A b)B (c)C (d)D
3.Assertion: When light of certain wavelength falls on a metal surface it ejects
electron.
Reason: Light has wave nature.
(a)A (b)B (c)C (d)D
4. Assertion: As work function of a material increases by some mechanism, it
requires greater energy to excite the electrons from its surface.
Reason. A plot of stopping potential (V) versus frequency (v) for different
materials, has greater slope for metals with greater work functions
(a)A (b)B (c)C (d)D
5. Assertion: Light of frequency 1.5 times the threshold frequency is incident on
photosensitive material. If the frequency is halved and intensity is doubled the
photo current remains unchanged.
Reason:The photo electric current varies directly with the intensity of light and
frequency of light.
(a)A (b)B (c)C (d) D
6. Assertion: The de-Broglie wavelength of a neutron when its kinetic energy is k
is λ. Its wavelength is 2 λ when its kinetic energy is 4k.
Reason. The de - Broglie wavelength λ is proportional to square root of the kinetic
energy.
(a)A (b)B (c)C (d)D

102
CASE BASED QUESTIONS

1. Photoelectric emission is possible only if the incident light is in the form of

packets of energy, each having a definite value, more than the work function of the

metal. This shows that light is not of wave nature but of particle nature. It is due to

this reason that photoelectric emission was accounted by quantum theory of light.

Q1. Packet of energy are called

(a)electron (b)quanta (c)frequency (d)neutron

Q2. One quantum of radiation is called

(a)meter (b)meson (c) photon (d)quark

Q3. Energy associated with each photon

(a)hc (b)mc (c)hv (d)hk

Q4. Which of the following waves can produce photo electric effect?

(a). UV radiation (b). Infrared radiation (c). Radio waves (d) Microwaves

Q5. Work function of alkali metals is

(a)less than zero (b)just equal to other metals

(c) greater than other metals (d) quite less than other metals

II. According to de-Broglie a moving material particle sometimes acts as a wave and

sometimes as a particle or a wave is associated with moving material particle which

controls the particle in every respect. The wave associated with moving material

particle is called matter wave or de-Broglie wave whose wavelength called de-Broglie

wavelength, is given by λ = h/mv

103
1.The dual nature of light is exhibited by

(a) diffraction and photo electric effect (b) photoelectric effect


(c) refraction and interference (d)diffraction and reflection.

2. If the momentum of a particle is doubled, then its de-Broglie wavelength will

(a)remain unchanged (b)become four times


(c) become two times (d)become half

3.If an electron and proton are propagating in the form of waves having the same λ
, it implies that they have the same

(a)energy (b)momentum (c)velocity (d)angular momentum

4. Velocity of a body of mass m, having de-Broglie wavelength λ , is given by


relation

(a) v = λ h/m (b) v = λm/h (c) v = λ/hm (d) v = h/ λm

5. Moving with the same velocity, which of the following has the longest de Broglie
wavelength?

(a) ᵦ -particle (b) α -particle (c) proton (d) neutron.

III.The discovery of the phenomenon of photoelectric effect has been one of the
most important discoveries in modern science. The experimental observations
associated with this phenomenon made us realize that our, ‘till then’, widely
accepted picture of the nature of light – The electromagnetic (wave) theory of light
– was quite inadequate to understand this phenomenon. A ‘new picture’ of light was
needed and it was provided by Einstein through his ‘photon theory’ of light. This
theory, regarded light as a stream of particles. Attempts to understand photoelectric
effect thus led us to realize that light, which was being regarded as ‘waves’, could
also behave like ‘particles’. This led to the idea of ‘wave-particle duality’ vis-à-vis
the nature of light. Attempts to understand this ‘duality’, and related phenomenon,
led to far reaching, and very important developments, in the basic theories of
Physics.

104
1) Which of the following phenomena explain the wave nature of light?

i) Interference ii) Diffraction

iii) polarization iv) all of them

2) Wave –particle duality is shown by

i) Light only ii) matter only

iii) both light and matter iv) None of them

3) The experiment to explain the wave nature of light i.e electromagnetic wave
theory is given by

i) Hertz ii) Einstein

iii) Lenard iv) Huygen

4) The concept of photoelectric effect given by Einstein explains that the light is a

i) Photon ii)Wave

iii) Particle iv) Both

5) The practical application of the phenomenon of photoelectric effect and the


concept of ‘matter waves’ is

i) Photocells ii) Automatic doors at shops and malls

iii) automatic light switches iv) All of them

IV. Lenard observed that when ultraviolet radiations were allowed to fall on the

emitter plate of an evacuated glass tube, enclosing two electrodes (metal plates),

current started flowing in the circuit connecting the plates. As soon as the ultraviolet

radiations were stopped, the current flow also stopped. These observations proved

that it was ultraviolet radiations, falling on the emitter plate, that ejected some

charged particles from the emitter and the positive plate attracted them.

105
1) Alkali metals like Li, Na, K and Cs show photo electric effect with visible light but
metals like Zn, Cd and Mg respond to ultraviolet light. Why?

i) Frequency of visible light is more than that for ultraviolet light

ii) Frequency of visible light is less than that for ultraviolet light

iii) Frequency of visible light is same for ultraviolet light

iv) Stopping potential for visible light is more than that for ultraviolet light

2) Why do we not observe the phenomenon of photoelectric effect with non-metals?

i) For non metals the work function is high

ii) Work function is low

iii) Work function can’t be calculated

iv) For non metals, threshold frequency is low

3) What is the effect of increase in intensity on photoelectric current?

i)Photoelectric current increases ii) Decreases

iii) No change iv) Varies with the square of intensity

4) Name one factor on which the stopping potential depends

i) Work function ii) Frequency iii) Current iv) Energy of photon

5) How does the maximum K.E of the electrons emitted vary with the work function
of metal?

i) It doesn’t depend on work function

ii)It decreases as the work function increases

iii) It increases as the work function increases

iv) It’s value is doubled with the work function

106
ANSWERS
MCQ
1. (d)

2.(c) λ/2

3. (b) λα 1/√m mα > mp = mn > me

4. (c ) E = hc/ λ

5. (c)

ASSERTION- REASON

1. (A)

2. (D)

3.(B)

4.(C)

5. (D)

6.(A)

CASE STUDY

I. Q1.(b) Q2.(c) Q3.(c) Q4.(a) Q5.(d)


II. Q1.(a) Q2.(d) Q3.(b) Q4.(d) Q5.(a)

III.1) iv 2) iii 3) i 4) iii 5) iv

IV. 1) ii 2) i 3) i 4) ii 5) ii

107
DESCRIPTIVE QUESTIONS
TWO MARKS

1. Two lines, A and B, in the plot given below show the variation of de-Broglie
wavelength, λ versus 1√V, Where V is the accelerating potential difference, for two
particles carrying the same charge. Which one of two represents a particle of smaller
mass ?

Ans. B line represents particle of larger mass because slope ∝1/√m

2. An electron is accelerated through a potential difference of 100 volt. What is the


de-Broglie wavelength associated with it? To which part of the electromagnetic
spectrum does this value of wavelength correspond?
Ans. Λ = 1.227/√V nm = 1.227/√100 =1.227A0 X rays

3. Monochromatic light of frequency 6x1014 Hz is produced by a laser. The power


emitted is 2x10-3W. i) What is the energy of photon in the light? ii) How many
photons per second on the average are emitted by the source
Ans. ν = 6x1014 Hz P = 2x10-3W
i) energy of photon = E = hν = 6.6x10-34 x6x1014 = 3.98x10-19J
If N is the number of photons emitted per second by the source then Power = N x
energy of photon
P = NE N = P/E = 2x10-3/3.98x10-19 = 5 x 10 15 photons per second
4. (i) Two monochromatic beams A and B of equal intensity I, hit a screen. The
number of photons hitting the screen by beam A is twice that by beam B. Then
what inference can you make about their frequencies?.
(ii) Draw a plot showing the variation of photoelectric current versus the intensity
of incident radiation on a given photosensitive surface

108
Ans. Let n1, n2 be the number of photons hitting the screen per second by beam A
and B respectively Intensity of beam of photon, I = nhν n1 ν 1 = n2 ν 2 n1/ n2
= ν 2/ ν 1 1 As n1/ n2 = 2 ν 2/ ν 1 = 2 , ν 2 = 2 ν 1
5. Two monochromatic radiations of frequencies v1 and v2 (ν1 > v2) and having the
same intensity are, in turn, incident on a photosensitive surface to cause
photoelectric emission. Explain, giving reason, in which case
(i) more number of electrons will be emitted and
(ii) maximum kinetic energy of the emitted photoelectrons will be more.

Ans. . (i) Number of photoelectrons emitted will be same because intensity is same.

(ii) Kinetic energy is more for ν1

Kmax = hv – ϕ0
6. (i). The stopping potential in an experiment on photoelectric effect is 1.5 V. What
is the maximum kinetic energy of the photoelectrons emitted?

(ii) State one factor which determines the intensity of light in the photon picture
of light.

Ans. (i)1.5eV

(ii) Number of photons emitted per second

7. A metallic surface is irradiated with monochromatic light of variable wavelength.


Above a wavelength of 5000 Å, no photoelectrons are emitted from the surface.
With an unknown wavelength, stopping potential is 3 V. Find the unknown
wavelength.

Ans. λ = 5000 Å and VO = 3V

KMAX=3eV Using equation of photoelectric equation, w = hc/λ = 6.6x10-34x


3x108/5x10-7 = 3.96x10-19J = 2 48 eV

Kmax = E – W

E = W+ Kmax = 2.48+3 = 5.48Ev

5.48X1.6X10-19 = 6.6x10-34x 3x108/λ

λ = 2.26X10-7m

109
8. (a) Define the terms, (i) threshold frequency and (ii) stopping potential in
photoelectric effect. (b) Plot a graph of photocurrent versus anode potential for a
radiation of frequency u and intensities I1 and I2 (I1 < I2).

Ans.(a) (i) Threshold Frequency : The minimum frequency of incident light


which is just capable of ejecting electrons from a metal is called the threshold
frequency. It is denoted by ν0.

(ii) Stopping Potential : The minimum retarding potential applied to anode of a


photoelectric tube which is just capable of stopping photoelectric current is called
the stopping potential. It is denoted by V0

Graph

9. In an experiment on photoelectric emission, following observations were made 1)


Wavelength of the incident light = 2 × 10–7 m 2) Stopping potential = 3 V Find (i)
kinetic energy of photoelectrons with maximum speed (ii) work function

Ans. Vs = 3 V and Kmax = eVo,

so Kmax = 3 eV (ii) l = 2 × 10–7 m = 2000 Å.

Energy of incident photon = hc/ λ = 6.6x10-34x 3x108/ 2 × 10–7 = 6.20 eV

W = E – Kmax = 3.2 eV

10. An electron and a photon each have a wavelength 1.00 nm. Find (i) theirta, (ii) the
energy of the photon.

Ans. . (i) Momentum of photon

P = h/λ = 6.6x10-34/1x10-9 = 6.6x10-25 kgm/s

Momentum of electron

P = h/λ = 6.6x10-34/1x10-9 = 6.6x10-25 kgm/s

ii) Energy of photon = hc/λ


= 6.6x10-34x 3x108/1x10-9

=1.98x10-16J

110
THREE MARKS

1. Explain de-Broglie argument to propose his hypothesis.. b) If, deuterons and alpha
particle are accelerated through same potential, find the ratio of the associated
de-Broglie wavelengths of two.

Ans. λ = h/√2mqV

for duetron m = 2mp q=e for α particle m = 4mp q = 2e

λα/λd = √2 x 2mp eV/√2x4mpx2e = 1/2

2. State the main implications of observations obtained from various photoelectric


experiments. Can these implications be explained by wave nature of light?
Justify your answer.

Ans. Explain the features of photoelectric effect. No they cannot be explained using
wave theory. Explanation.

3. An electromagnetic wave of wavelength λ is incident on a photosensitive surface


of negligible work function. If the photoelectrons emitted from this surface have
the de-Broglie wavelength λ1, prove that λ = (2mc/h)λ12

Ans. h c/λ = ½ mv2

2hmc/λ = p2. λ1 = h/p. p = h/λ1 . 2hmc/λ = h2/λ12 . λ = (2mc/h) λ12

4. The following graph shows the variation of stopping potential V0 with the
frequency v of the incident radiation for two photosensitive metals X and Y :

(i) Which of the metals has larger threshold wavelength? Give reason.
(ii) Explain, giving reason, which metal gives out electrons, having larger kinetic
energy, for the same wavelength of the incident radiation.

(iv) If the distance between the light source and metal X is halved, how will the
kinetic energy of electrons emitted from it change? Give reason.

111
Ans. (i) X as X has smaller threshold frequency

(ii) Since фY > фX so Key > KEx Therefore x gives out electrons with large KE

(iii) no change

5. Draw a plot showing the variation of photoelectric current with collector plate
potential for two different frequencies, v1 > v2, of incident radiation having the
same intensity. In which case will the stopping potential be higher? Justify your
answer.

Ans. Graph and explanation

6. Define the terms (i) cut-off voltage and (ii) threshold frequency in relation to the
Phenomenon of photoelectric effect. Using Einstein’s photoelectric equation
show how the cut -off voltage and threshold frequency for a given photosensitive
material can be determined with the help of a suitable plot.

Ans. Definitions

From Einstein’s photoelectric equation,

eV0 = hν – φ0 for ν > ν0.

Cut off voltage V0 = (h/e) ν – (φ0/e )

h/e is the slope of the graph.

ν0 = φ0/h

112
7. Draw suitable graphs to show the variation of photoelectric current with collector
plate potential for

(i) a fixed frequency but different intensities I1> I2>I3 of radiation.

(ii) a fixed intensity but different frequencies ν1 > ν2 > ν3 of radiation.

Ans. Draw graphs.

8. The work function of Caesium metal is 2.14eV. When light of frequency 6 x 10 14


Hz is incident on the metal surface photoemission of electrons occurs. What is the

(i) maximum kinetic energy of the emitted photoelectrons

(ii) stopping potential

(iii) maximum speed of the emitted photoelectrons

Ans. (i)Kmax = hv – ϕ0
6.63x10-34x6x1014/1.6x10-19 – 2.14eV = 0.314eV

(ii) eV0 = Kmax = 0.314eV V0 = 0.314V

(iii) 345.8X103 m/s

9.The work function for a certain metal is 4.2 eV. Will this metal give photoelectric
emission for incident radiation of wavelength 330nm?

Ans. E = hc/λ = 6.6×10−34×3×108 / 330 x10-9 = 3.767eV ϕo = 4.2eV since E <


W0 no photoelectric emission.

10. The threshold frequency for a certain metal is 3.3×1014 Hz. If light of frequency
8.2×1014Hz is incident on the metal, predict the cut-off voltage for the photoelectric
emission.

Ans. Frequency of light incident on the metal, v = 8.2×1014Hz

Charge on an electron, e =1.6×10−19C Planck's constant, h = 6.626×10−34Js

Cut-off voltage for the photoelectric emission from the metal = V∘

The equation for the cut-off energy is given as: eV∘ = h(v−v∘)

V∘ = h/e(v−vo) = 6.626×10−3/(8.2×1014−3.3×1014)/1.6×10−19 =2.029eV

Therefore, the cut-off voltage for the photoelectric emission is 2.0292 V

113
11. The following table gives the values of work functions for a few sensitive metals.
S. Work
Metal
No. function(eV)

1 Na 1.92

2 K 2.15

3 Mo 4.17

If each of these metals is exposed to radiations of wavelength 330nm

which of these will not exit photoelectrons and why?

Ans. It is known that material will not emit photoelectrons whose work function is
greater than the energy of the incident radiation.

E = hc/λ = 6.6×10−34×3×108 / 33×10−8

E = 6.20×10−19J E = 6.20×10−19/1.6×10−19eV E = 3.76 eV

Hence work function of Mo is (4.17eV) which is greater than the energy of the
incident radiation (= 3.76 eV) so Mo will not emit photoelectrons.

12. Light of wavelength 2000A0falls on an aluminum surface. In aluminium4.2 eV


are required to remove an electron. What is the kinetic energy of (a) fastest (b) the
slowest photoelectron?

Ans. . Given wavelength is λ = 2000Ao = 2×10−7 m ϕo= 4.2eV

The kinetic energy is K.Emax = 1/2mV2max = hv−ϕo

(a) 1/2mV2max= hc/λ−ϕo = (6.6×10−34×3×108/2×10−7)−4.2

1/2mV2max = 2eV

This is the K.E of the fastest electron is 2eV

(b) The velocity of the slowest electron would be zero, hence the kinetic energy it
possesses is also zero.

**********************************

114
Chapter–12: ATOMS

IMPORTANT POINTS
1. All elements consists of very small invisible particles, called atom. Every atom is a
sphere of radius of the order of 10-10 m, in which entire mass is uniformly distributed
and negative charged electrons revolve around the nucleus.
2. Experimental arrangement for α-scattering experiment and trajectory followed by α
–particles

3. Impact parameter perpendicular distance of the velocity vector of a-particle from the
central line of the nucleus of the atom is called impact parameter (b).

Where, K is KE of α-particle,

θ is scattering angle,
Z is atomic number of the nucleus and 𝑒 is charge of nucleus.
4. Basic assumption of Rutherford’s atomic model
(i) Atom consists of small central core, called atomic nucleus in which whole
mass and positive charge is assumed to be concentrated.
(ii) The size of nucleus is much smaller than the size of the atom.
(iii) The nucleus is surrounded by electrons and atom is electrically neutral.
(iv) Electrons revolves around the nucleus and centripetal force is of electrostatic
nature.

115
5. Distance of Closest Approach: At a certain distance r0 from the nucleus, whole of
the KE of α-particle converts into electrostatic potential energy and α-particle cannot
go farther close to nucleus, this distance (r0) is called distance of closest approach.

6. Angle of Scattering: Angle by which a-particle gets deviated from its original path
around the nucleus is called angle of scattering.

7. Drawbacks of Rutherford’s Model:


(i) could not explained stability of atom clearly.
(ii) Unable to explain line spectrum.
8. Bohr’s Theory of Hydrogen Atom
Bohr combined classical and early quantum concepts and gave his theory in the form
of three postulates. These are
i. Bohr’s first postulate was that an electron in an atom could revolve in certain
stable orbits without the emission of radiant energy, contrary to the predictions
of electromagnetic theory.
ii. Bohr’s second postulate defines these stable orbits. This postulate states that the
electron revolves around the nucleus only in those orbits for which the angular
momentum is some integral multiple of h/2π, where h is the Planck’s constant
(= 6.6 x 10-34 J s). Thus, the angular momentum (L) of the orbiting electron is
𝑛ℎ
quantised, e. L =

As, angular momentum of electron (L) = 𝑚𝑣𝑟


∴ For any permitted (stationary) orbit, mvr = nh/2π
where, n = any positive integer i.e. 1, 2, 3,…n is principal quantum number.
iii. Bohr’s third postulate states that an electron might make a transition from one
of its specified non-radiating orbits to another of lower energy. When it does so,
a photon is emitted having energy equal to the energy difference between the
initial and final states.
The frequency of the emitted photon is then given by
ℎ𝜈= Ei– Ef
where, Ei and Ef are the energies of the initial and final states and Ei > Ef .

116
9. Limitations of Bohr’s Model
(i) Applicable only for hydrogen like atom.
(ii) Does not explain the fine structure of spectral lines in H-atom.
(iii) Does not explain about shape of orbit.
10. Important formulae related to Bohr’s model of hydrogen atoms are given
below:

117
11. Energy Level The energy of an atom is the least when its electron is revolving in
an orbit closest to the nucleus i.e. for which n = 1.

12. The lowest state of the atom is called the ground state; this state has lowest
energy. The energy of this state is -13.6 eV. Therefore, the minimum energy required
to free the electron from the ground state of the hydrogen atom is -13.6 eV.
13. (i) Emission Spectrum: Hydrogen spectrum consists of discrete bright lines a
dark background and it is specifically known as hydrogen emission spectrum.
(ii) Absorption Spectrum: There is one more type of hydrogen spectrum exists
14. The atomic hydrogen emits a line spectrum consisting of various series.

118
119
MULTIPLE CHOICE QUESTIONS

1 The ratio between Bohr radii is


(a) 1: 2: 3 (b) 2 : 4 : 6 (c) 1 : 4 : 9 (d) 1 : 3 : 5
2. The K.E. of the electron in an orbit of radius r in hydrogen atom is proportional to
𝑒2 (b) 𝑒
2
(c) 2𝑒
2
(d) 𝑒
2
(a)
𝑟 2𝑟 𝑟 3𝑟

3. In terms of Rydberg constant R, the wave number of the first Balmer line is
(a) R (b) 3R (c) 5R/36 (d) 8R/9
4. The ionisation energy of hydrogen atom is 13.6 eV. Following Bohr’s theory the
energy corresponding to a transition between 3rd and 4th orbits is
(a) 3.40 eV (b) 1.51 eV (c) 0.85 eV (d) 0.66 eV
5. The transition of electron from n = 4, 5, 6… to n = 3 corresponds to
(a) Lyman series (b) Balmer series (c) Paschen series (d) Brackett series
6. Which of the following spectral series in hydrogen atom gives spectral line of 4860 A?
(a) Lyman (b) Balmer (c) Paschen (d) Brackett
7. Which of the following postulates of the Bohr model led to the quantization of energy
of the hydrogen atom?
(a) The electron goes around the nucleus in circular orbits.
(b) The angular momentum of the electron can only be an integral multiple of h/2π.
(c) The magnitude of the linear momentum of the electron is quantized.
(d) Quantization of energy is itself a postulate of the Bohr model.
8. The transition from the state n = 5 to n = 1 in a hydrogen atom results in UV radiation.
Infrared radiation will be obtained in the transition
(a) 2 → 1 (b) 3 → 2 (c) 4 → 3 (d) 6 → 2
9. For ionising an excited hydrogen atom, the energy required (in eV) will be
(a) a little less than 13.6 eV (b) 13.6 eV
(c) more than 13.6 eV (d) 3.4 or less
10. In terms of Rydberg constant R, the shortest wavelength in Balmer series of
hydrogen atom spectrum will have wavelength
1 4 3 9
(a) (b) (c) (d)
𝑅 𝑅 2𝑅 𝑅

120
ANSWER KEY
1. Answer: c
Explanation: (c) 1: 4 : 9, In Bohr’s atomic model, rn n²
2. Answer: b
Explanation: (b) e2/2r, Since K.E = ke2/2r

3. Answer: c
Explanation:

4. Answer: d
Explanation:

5. Answer: c
Explanation: (c) In transition from n1 = 3 and n2 = 4, 5, 6…
Infrared radiation of Paschen spectral is emitted.
6. Answer: b
Explanation: (b) Since spectral line of wavelength 4860 A
lies in the visible region of the spectrum which is Balmer series of the spectrum.
7. Answer: b

8. Answer: c

9. Answer: d
Explanation: (d) As the energy of the electron is – -3.4 eV in first excited state and
magnitude is less for higher excited state.

10. Answer: b
Explanation: (b) for shortest wavelength n1 =∞, n2 = 2

121
ASSERTION –REASON TYPE QUESTIONS

Answer: A Both are correct and reason is correct explanation of assertion.


Answer: B Both are correct but reason is not the correct explanation of assertion.
Answer: C Reason is wrong.
Answer: D Both are wrong.

1. Assertion: According to Bohr’s atomic model the ratio of angular momenta of an

electron in first excited state and in ground state is 2:1.

Reason: In a Bohr’s atom the angular momentum of the electron is directly proportional

to the principal quantum number.

2. Assertion: The force of repulsion between atomic nucleus and a-particle varies with

distance according to inverse square law.

Reason: Rutherford did a-particle scattering experiment.

3. Assertion: The positively charged nucleus of an atom has a radius of almost 10−15m.

Reason: In a-particle scattering experiment, the distance of closest approach for particles

is ≃ 10−15m.

4. Assertion: Electrons in the atom are held due to coulomb forces

Reason: The atom is stable only because the centripetal force due to Coulomb’s law is

balanced by the centrifugal force.

5. Assertion: For the scattering of a-particles at a large angles, only the nucleus of the atom

is responsible.

Reason: Nucleus is very heavy in comparison to α particle.

122
CASE STUDY QUESTIONS

1. HYDROGEN SPECTRUM

Hydrogen spectrum Electrons in an atom or a molecule absorb energy and get excited, they

jump from a lower energy level to a higher energy level and they emit radiation when they

come back to their original states. This phenomenon accounts for the emission spectrum

through hydrogen also, better known as hydrogen emission spectra.

(i)If 13.6eV of energy is needed to ionize the hydrogen atom, then energy needed to remove
electron from n=2 is
a. 10.2eV b. 0eV c. 3.4eV d. 6.8eV
(ii)The longest wavelength in Balmer series is
a. Hα b. Hβ c. Hγ d. Hδ
(iii)The hydrogen spectrum which lies in the ultraviolet region is
a. Brackett b. Paschen c. Lyman d. Balmer
(iv)On moving up the energy states of H-like atoms, the energy difference between
consecutive energy states
a. Increase b. Decrease c. Remain the same d. May increase or
decrease
(v)The Bohr atom model
a. Assumes that the angular momentum of electrons is quantized
b. Uses Einstein’s photoelectric equation
c. Predicts continuous emission spectrum for atoms
d. None of these

123
2. BOHR ATOM MODEL

To study about atom various scientists perform various experiments and suggest various
models of an atom with some explanation. For example, Thomson gives the "plum
pudding" model in which he said the atom consists of a positive material known as
"pudding" with some negative materials ("plums") distributed throughout. Later, famous
scientist, Rutherford gives Rutherford's model of the atom after performing an Alpha
Particle scattering experiment.

This model is a modification of the earlier Rutherford Model. According to this model, an
atom consists of a small, positively-charged nucleus and negatively-charged electrons
orbiting around it in an orbital. These orbital can have different sizes, energy, etc. And the
energy of the orbit is also related to its size, .e The lowest energy is found in the smallest
orbit. So if the electron is orbiting in nth orbit then we will study about its Velocity in nth
orbital, Radius of nth orbital, Energy of electron in nth orbit, etc. Energy is also emitted
due to the transition of electrons from one orbit to another orbit. This energy is emitted in
the form of photons with different wavelengths. This wavelength is given by the Rydberg
formula. When electrons make transitions between two energy levels in an atom various
spectral lines are obtained. The emission spectrum of the hydrogen atom has been divided
into various spectral series like Lyman series, Balmer series, Paschen series Etc.

124
Q1. The formula which gives the wavelength of emitted photon when electron jumps from
higher energy state to lower was given by

a) Balmer

b) Paschen

c) Lyman

d) Rydberg

Q2. What is true about Bohr’s atomic Model?

a) His model was unique totally different from other

b) His model is a modification of Rutherford atomic model.

c) His model is a modification of Thomson atomic model.

d) None of the above


Q3. Bohr’s atomic model is applicable for

a) All types of atoms

b) Only for hydrogen atom

c) For hydrogen like atoms

d) For H2 gas.

Q4. The cause of rejection of Rutherford atomic model was

a) It was totally wrong

b) It could not justify its stability

c) Rutherford was unable to explain it

d) none of the above.

125
ASSERTION –REASON TYPE QUESTIONS ANSWER

1. Correct answer: A
2. Correct Answer: B
Rutherford confirmed the repulsive force on a-particle due to nucleus varies with
distance according to inverse square law and that the positive charges are
concentrated at the centre and not distributed throughout the atom.
Correct Answer: A
3. In a-particle scattering experiment, Rutherford found a small number of particles
which were scattered back through an angle approaching to 180∘. This is possible
only if the positive charges are concentrated at the centre or nucleus of the atom.
4. Correct Answer: C
According to postulates of Bohr’s atom model, the electron revolve round the
nucleus in fixed orbit of definite radii. As long as the electron is in a certain orbits it
does not radiate any energy.
5. Correct Answer: A

We know that an electron is very light particle as compared to an alpha particle. Hence
electron cannot scatter the a-particle at large angles, according to law of conservation
of momentum. On the other hand, mass of nucleus is comparable with the mass of a-
particle, hence only the nucleus of atom is responsible for scattering of a-particles

CASE STUDY QUESTIONS I


ANSWER KEY
CASE STUDY QUESTIONS II
i C ANSWER KEY
Q1 d
Q2 b
Q3 c
ii A
iii C Q4 b
2
iv B En = -13.6 / n
v A

126
SHORT ANSWER TYPE QUESTIONS (2 MARKS)

1. The ground state energy of hydrogen atom is -13.6eV . What is the Kinetic
Energies and Potential Energies of the electron in the ground state and second
excited state
Ans:
KE= 13.6 eV, PE= -27.2eV in the ground state
KE= 1.51 eV PE = -3.02eV in the second excited state
2. Find the ratio of energies of photons produced due to transition of an electron of
hydrogen atom from its (i) second permitted energy level to the first level (ii) the
highest permitted energy level to the first permitted leve
Ans: ∆E= Ei – Ef Ratio=3:4
3. In Bohr’s theory of model of Hydrogen atom name of the physical quantity which
equals to an integral multiple of h/2𝜋.How is electron velocity related to radius
Ans:
(I) Angular momentum, (II) v=nh/2πmr ( inversely proportional)
4. Calculate the de-Broglie wavelength of the electron orbiting in the n=2 state of
hydrogen atom ( Ans:Wavelength = h/√2mE= 0.67nm)
5. Give any two observations of Geiger Marsden Experiment .Write the significance
of the experiment
Ans:
Observations: Many of the α-particles pass through the foil. It means that they do
not suffer any collisions. Only about 0.14% of the incident α-particles scatter by
more than 1°; and about 1 in 8000 deflect by more than 90°
Conclusions: ( Significance- To understand the structure of atom )
(i) Atom consists of small central core, called atomic nucleus in which whole mass
and positive charge is assumed to be concentrated.
(ii) The size of nucleus is much smaller than the size of the atom.
(iii) The nucleus is surrounded by electrons and atom is electrically neutral.
(iv) Electrons revolve around the nucleus and centripetal force is of electrostatic
nature.

127
6. Define impact parameter Represent diagrammatically the shape of trajectory of
alpha particles
Ans: Impact parameter perpendicular distance of the velocity vector of a-particle
from the central line of the nucleus of the atom is called impact parameter (b).

for figure refer refer page no 115


7. Show that the speed of electron in the innermost orbit of H atom is 1/137 times the
speed light in vacuum
Ans. Speed of revolving electron

2𝜋𝑘𝑒 2 2𝜋𝑘𝑒 2 𝑐 1 𝑐
𝑣= = . = .
𝑛ℎ 𝑐ℎ 𝑛 137 𝑛

Speed of revolving electron is inversely proportional to the principal quantum


number.
8. The wavelength of Hα line of Balmer series is 6533Angstrom Calculate the value
of Rydberg constant
Ans: For Balmer series: n1=2
First line: n2=3
Putting in formula shown in the figure:
λ1=R(221−321)=365R
5R36=6563×10−12m
R=1.097×107m−1
9. The wavelength of the second line of Balmer series in Hydrogen atom is
4861Angstrom. Calculate the wave length of the first line.
Ans: For the first line in balmer series
λ1=R(1/22−1/32)=365R
For second balmer line:
4861=R(1/22−1/42)=163R Divide both equations:
λ=4861×27/20= 6562.35A0

128
10. Energy of electron in the nth orbit is -3.4eV Calculate the angular momentum of
the electron according to Bohr’s theory
Ans:
-3.4eV = -13.6eV/n2 n=2,
L=nh/2π = 2.1X 10-34 Js
11. In hydrogen atom, an electron undergoes transition from 3rd excited state to the
first excited state and then to the ground state. (i)Identify the spectral series to
which these transitions belong. (ii) Find out the ratio of the wavelengths of the
emitted radiations in the two cases.
Ans:

12. What is the ratio of radii of the orbits corresponding to first excited state and
ground state in a hydrogen atom?
( Ans; R is proportional to n2ratio of radii is 4:1)
13. The radius of innermost electron orbit of a hydrogen atom is 5.3 × 10-11 m. What is
the radius of orbit in the second excited state?
Ans:
For n=1 , r1 = 5.3x 10-11 for ground state therefore r2 = 4 r1 , and r3= 9r1
r2 = 2.12x10-10m r3= 4.77 x 10 -10 m
14. What is the maximum number of spectral lines emitted by a hydrogen atom when
it is in the third excited state? Which one will have lowest wavelength
Ans: third excited state, n2 = 4, and n1 = 3, 2, 1 Hence there are 3 spectral lines.
Greater the energy lower the wavelength
15. Using Bohr’s postulates of the atomic model, derive the expression for radius of
nth electron orbit. Hence obtain the expression for Bohr’s radius. (Refer notes)

129
16. When an electron in hydrogen atom jumps from the third excited state to the
ground state, how would the de Broglie wavelength associated with the electron
change? Justify your answer.
Ans:
Wave length decreases. P = √2𝑚𝐾𝐸 = h/λ
E is proportional to 1/ λ
17. Calculate the shortest wavelength in the Balmer series of hydrogen atom. In which
region (infra-red, visible, ultraviolet) of hydrogen spectrum does this wavelength
lie?
Ans:
For shortest wavelength in the Balmer series: n1=2 n2=∞
λmin =3.646×10−7m=364.6 nm. This wavelength lies in the ultraviolet region
18. An α-particle moving with initial kinetic energy K towards a nucleus of atomic
number z approaches a distance ‘d’ at which it reverses its direction. Obtain the
expression for the distance of closest approach ‘d’ in terms of the kinetic energy of
α-particle K

Ans.
At distance of closest 𝑟0 approach:
1 1 2𝑒. 𝑍𝑒
𝑚𝑣 2 =
2 4𝜋𝜖0 𝑑
19. Using Rutherford model of atom,derive the expression for the total energy of the
electron in hydrogen atom.What is the significance of total negative energy
possessed by the electron

Ans.
For a dynamically stable orbit in a hydrogen atom, the electrostatic force of
attraction between the revolving electron and the nucleus provides the requisite
centripetal force to keep them in their orbit of radius r.
𝑚𝑣 2 1 𝑒2
∴ =
𝑟 4𝜋𝜀0 𝑟 2

130
1 𝑒2
Kinetic energy of the revolving electron = 𝑚𝑣 2 =
2 8𝜋𝜀𝑜 𝑟

𝑒2
Potential energy of revolving electron = −
4𝜋𝜀𝑜 𝑟

∴ 𝑇𝑜𝑡𝑎𝑙 𝐸𝑛𝑒𝑟𝑔𝑦 𝑜𝑓 𝑟𝑒𝑣𝑜𝑙𝑣𝑖𝑛𝑔 𝐸𝑙𝑒𝑐𝑡𝑟𝑜𝑛


𝑒2
= 𝑘𝑖𝑛𝑒𝑡𝑖𝑐 𝑒𝑛𝑒𝑟𝑔𝑦 + 𝑝𝑜𝑡𝑒𝑛𝑡𝑖𝑎𝑙 𝑒𝑛𝑒𝑟𝑔𝑦 = −
8𝜋𝜀𝑜 𝑟
The total energy of the electron is negative. This implies that the electron is bound
to the nucleus. If it is positive, an electron will not follow a closed orbit around the
nucleus.
20. Show graphically the variation of radius of orbit with principal quantum number n

Ans.
Orbital radius is directly proportional to the square of principal quantum
number

131
SHORT ANSWER TYPE QUESTIONS (3 MARKS)

1. Show that the shortest wavelength of Lyman, Balmer and Paschen series are in the
ratio1:4:9

1 1 1
Ans. Use the equation = 𝑅[ − ]
𝜆 𝑛𝑓2 𝑛𝑖2

Shortest wavelength in Lyman series is transition from, ∞ to n=1


Shortest wavelength in Balmer series is transition from ∞ to n=2
Shortest wavelength in Paschen series is transition from ∞ to n=3
Substitute and solve

2. Using Rydberg formula calculate the wavelengths of the first four spectral lines in
the Balmer series of Hydrogen spectrum
Ans:
6575 A,
4870A,
4348A,
4109A
3. The energy levels of an atom of element X shows the following transitions Which
will emit photons of wavelength 620nm (Ans: transition D)

A B C D E

0eV
-1eV

-3eV

-10eV

132
4. The figure shows energy level diagram of hydogen atom

(a) Find out the transition which results in the emission of a photon of wavelength
496 nm.(b) Which transition corresponds to the emission of radiation of
maximum wavelength? Justify your answer

Ans:
n1= 2 and n2= 4

5. A 12.5 eV electron beam is used to excite a gaseous hydrogen atom at room


temperature. Determine the wavelengths and the corresponding series of the lines
emitted
Ans:-
13.6eV- 12.5eV= -1.1eV therefore n=3
From n=3 to n=1 wavelength= 1.025X10-7 m

n=3 to n=1 or n=4 to n=1, then wavelengths of lines so emitted lie in Lyman series.
Whereas transitions either from n=3 to n=2 or n=4 to n=2 correspond to Balmer series.

133
6. The electron, in a hydrogen atom, is in its second excited state.
Calculate the wavelength of the lines in the Lyman series that can be emitted
through the permissible transitions of this electron.
(Given the value of Rydberg constant, R = 1.1 × 107 m-1)
Ans: From n=3 to n=1 wavelength= 1.023X10-7 m
From n=2 to n=1 wavelength = 212nm
7. Find the ratio between the wavelengths of the ‘most energetic’ spectral lines in the
Balmer and Paschen series of the hydrogen spectrum

Ans: 9:4
8. The short wavelength limit for the Lyman series of the hydrogen spectrum is 913.4
A Calculate the short wavelength limit for Balmer series of the hydrogen spectrum
Ans: 365.36nm
9. The ground state energy of hydrogen atom is -13.6 eV. If an electron makes a
transition from an energy level -1.51 eV to -3.4 eV, calculate the wavelength of the
spectral line emitted and name the series of hydrogen spectrum to which it belongs.
Ans:∆E= Ei – Ef = hc/λ
∆E= -1.89 eV
Wave length= 6563Angstrom
10. The energy level diagram of an element is given. Identify, by doing necessary
calculations, which transition corresponds to the emission of a spectral line of
wavelength 102.7 nm.

Ans:
D corresponds to the given transition

134
11. Using Bohr’s postulates, obtain the expression for the total energy of the electron
in the stationary states of the hydrogen atom. Hence draw the energy level diagram
showing how the line spectra corresponding to Balmer series occur due to
transition between energy levels.
Ans:
Derivation of Total Energy of Electron in the nth orbit En= -13.6eV/n2
Transition of electron from any higher level to n=2
n=4
n=3

n=2

n=1

12. Using Bohr’s postulates, obtain the expressions for


(i) kinetic energy and
(ii) potential energy of the electron in stationary state of hydrogen atom. Draw the
energy level diagram showing how the transitions between energy levels result in
the appearance of Lyman Series.
TE of electron in the nth orbit En= -13.6eV/n2 KE= -TE PE= 2TE
Transition of electron from any higher level to n=1
.

135
13. i) State Bohr’s quantization condition for defining stationary orbits. How does de-
Broglie hypothesis explain the stationary orbits?
(ii) Find the relation between the three wave-lengths λ1, λ2 and λ3 from the energy
level diagram shown below:

Ans: i) Bohr's Quantization Rule:

Of all possible circular orbits allowed by the classical theory, the electrons are
permitted to circulate only in those orbits in which the angular momentum of an
electron is an integral multiple of 2πh, where h is Plank's constant.
Therefore, for any permitted orbit,
L=mvr=nh/2 π ; n=1,2,3,........
Where L, m, and v are the angular momentum, mass and the speed of the electron
respectively. r is the radius of the permitted orbit and n is positive integer called
principal quantum number.
The above equation is Bohr's famous quantum condition. When an electron of
mass m is confined to move on a line of length l with velocity v, the de-Broglie
wavelength λ associated with electron is:
λ=h/mv=h/P
Where P is Linear momentum
P=h/ λ =nh/2πr or Pr= nh/2π L= nh/2π
(ii) Using Rydberg's formula for spectra of hydrogen atom, we have
1/λ1=R(1/n22−1/n32) .....(1)
1/λ2=R(1/n12−1/n22) .....(2)
1/λ3=R(1/n12−1/n32) .....(3)
Now adding (1) and (2), we get
That is, 1/ λ1+1/λ2=1/λ3

136
LONG ANSWER QUESTIONS (5 MARKS)
1. Using Bohr’s postulates, derive the expression for the frequency of radiation
emitted when electron in hydrogen atom undergoes transition from higher energy
state (quantum number ni) to the lower state (nf).
When electron in hydrogen atom jumps from energy state ni = 4 to nf = 3, 2, 1,
identify the spectral series to which the emission lines belong.
2. In a Geiger- Marsden experiment calculate the distance of closest approach to the
nucleus of Z= 80 when an alpha particle of 8 MeV energy impinges on it before it
comes momentarily to rest and reverses its direction.
How will the distance of closest approach be affected when the KE of the alpha
particle is doubled. Also state how impact parameter depends on angle of
scattering
Ans( i)

Impact parameter is inversely proportional to angle of scattering


3. Using Bohr’s postulates of the atomic model derive expression for radius of nth

electron orbit. Hence obtain the expression for Bohr’s radius.

Using Bohr’s postulates derive expression for total energy of electron in the nth

orbit of Hydrogen atom. Also Prove that Energy is quantized.

137
4. (i)State Bohr’s postulates. How does de Broglie’s hypothesis explain the stability
of these orbits.
(ii) A hydrogen atom initially in the ground state absorbs a photon which excites it
to n=4 level Estimate the frequency of the photon
(iii) Based on Bohr’s postulates derive expression for Total energy of electron in
the nth orbit of Hydrogen atom.

Ans: Bohr's Postulates


(i) Bohr’s first postulate was that an electron in an atom could revolve in certain
stable orbits without the emission of radiant energy, contrary to the predictions of
electromagnetic theory.
(ii) Bohr’s second postulate defines these stable orbits. This postulate states that
the electron revolves around the nucleus only in those orbits for which the angular
momentum is some integral multiple of h/2π, where h is the Planck’s constant (=
6.6 x 10-34 J – s). Thus, the angular momentum (L) of the orbiting electron is
quantised, i. e. L = nh/2π
As, angular momentum of electron (L) = mvr
∴ For any permitted (stationary) orbit, mvr = nh/2π
where, n = any positive integer i.e. 1, 2, 3, ….
It is also called principal quantum number.
(iii) Bohr’s third postulate states that an electron might make a transition from one
of its specified non-radiating orbits to another of lower energy. When it does so, a
photon is emitted having energy equal to the energy difference between the initial
and final states.
The frequency of the emitted photon is then given by
hv – Ei– Ef
where, Ei and Ef are the energies of the initial and final states and Ei > Ef .
(ii) E=E1−E2=hc/λ
λ=E1−E2/hc (wavelength= 97nm) (frequency=3.1 X 10 15 Hz)
(iii) Refer the notes

138
5. Explain the origin of spectral lines using Bohr’s theory. On the basis of the theory
derive an expression for the wave number of radiations emitted when an electron
jumps from outer orbit to lower orbit
(For derivation use Bohr’s Third postulate and obtain frequency expression by
substituting the energy of electron for ni and nf)
Ans: The emission spectrum of hydrogen is due to the electron making transitions
between the energy levels in an atom. Using Bohr’s third postulate, Rydberg
formula is arrived at and by choosing transitions from higher energy levels to
lower ones origin of various spectral series can be explained with diagram

6. With the help of a labelled diagram describe Rutherford’s experiment on scattering


of alpha particles .Represent graphically the relation between number scattered
particles and the angle of scattering. What is the significance of the graph Also
write any one drawback of this model. (Refer the notes)

139
7. Prove that radius, velocity and Energy of electron in an atom are quantised. What

is the significance of negative value of total energy?

The electron in the given Bohr orbit has a total energy of -1.5eV Calculate its (i)

KE (ii) PE (iii) The wavelength of light emitted when electron makes a transition

to the ground state Ground state energy = -13.6eV

Ans:

(Use the concept- Centripetal force is equal to Coulomb’s force and substitute the

expression for velocity from Bohr’s quantization condition v= nh/2πmr. On solving

we get the expression for radius of the nth orbit.

TE of an electron is written by substituting the value of r.)

(: KE= 1.5eV, PE= -3eV, wavelength= 1022.7Angstrom

140
CHAPTER 13: NUCLEI
Key points and Short Notes:
Composition of the Nucleus:
● Nucleus consists of protons and neutrons.
● Nucleus is practically responsible for the whole mass of the atom
● Every atomic nucleus except that of Hydrogen has two types of particles –
protons and neutrons which are known as Nucleons.
1. Proton is a fundamental particle with positive charge 1.6 x 10-19 C and mass
1.67 x 10-27 kg (1836 times heavier than an electron).
2. Neutron is also a fundamental particle with no charge and mass 1.675 x 10-27 kg
(1840 times heavier than an electron).

Atomic Mass Unit (u):


1 th
Atomic Mass Unit (amu) is of mass of 1 atom of carbon.
12
1 12
1u= x g
12 6.023 𝑥 1023
1u = 1.66 x 10-27 kg

Neutron:
⮚ Chadwick assumed that the neutral radiation consists of a new type of
neutral particles called neutrons.
⮚ From conservation of energy and momentum, he was able to determine the
mass of new particle ‘as very nearly the same as mass of proton’.
⮚ The mass of a neutron is now known to a high degree of accuracy.
⮚ It is mn = 1.00866 u = 1.6749×10–27 kg
⮚ A free neutron, unlike a free proton, is unstable.
⮚ It decays into a proton, an electron and an antineutrino (another elementary
particle), and has a mean life of about 1000s.
1 1 0
0𝑛 → 1𝑝 + − −1𝑒 + ̅ν
It is, however, stable inside the nucleus.

Charge and mass:


Charge:
⮚ The electron has a single negative charge, -e (e = -1.60217733 x 10-19 C)
⮚ The proton has a single positive charge, +e (e = +1.60217733 x 10-19 C)
⮚ Thus, charge of a nucleus is equal to Ze
⮚ The neutron has no charge. Makes it difficult to detect

141
Mass:
It is convenient to use atomic mass units, u, to express masses
1 u = 1.660559 x 10-27 kg
Based on definition that the mass of one atom of C-12 is exactly 12 u
Mass can also be expressed in MeV/c2
From E= m c2
𝑀𝑒𝑉
1 u = 931.494
𝑐2

Proton is a fundamental particle with mass 1.67 x 10-27 kg


(1836 times heavier than an electron).
Neutron is also a fundamental particle with mass 1.675 x 10-27 kg
(1840 times heavier than an electron)

Atomic Number (Z):


The number of protons in a nucleus of an atom is called atomic number.
Atomic Mass Number (A):
The sum of number of protons and number of neutrons in a nucleus of an atom is
called atomic mass number.
A=Z+N
Isotopes:
The atoms of an element having same atomic number but different mass numbers.
are called isotopes.
e.g., 1H1, 1H2, 1H3 are isotopes of hydrogen
Isobars:
The atoms of different elements having same mass numbers but different atomic
numbers, are called isobars.
e.g. 1H3, 2He3 and 10Na22, 10Ne22
Isotones:
The atoms of different elements having different atomic numbers and different
mass numbers but having same number of neutrons, are called isotones.
e.g. 1H3, 2He4 and 6C14, 8O16

142
Size of Nucleus:
Nucleus does not have a sharp or well-defined boundary but its radius is about 10-
15
m.

R = R0 A⅓

where R0 = 1.2 x 10-5 m is a constant which is the same for all nuclei and
A is the mass number of the nucleus.

Nucleus Density:

Mass of nucleus, M = A amu = A x 1.66 x 10-27 kg

Nuclear Volume,
4 4
V = π R3 = π R03 A
3 3

4
= π (1.2 x 10-15)3 A m3
3

= 7.24 x 10-45 A m3
𝑀
Nucleus Density, ρ = = 2.29 x 1017 kg m-3
𝑉

Calculate the energy equivalent of 1 g of substance.

Here, m = 1 g = 10−3 kg,

E = m c2 = 10−3 (3 × 108) 2 = 9 × 1013 J

Calculate Energy Equivalent to 1 amu or 1 u

1 amu = 1 u = 1.66 × 1027 kg

C = speed of light = 3 × 108 m s-1

E = m c2 = 1.66×1027(3×108)2 = 1.49×10−10 Joule

1MeV = 1.6×10−13 J

E = 1.6 × 10−13 x 1.49 × 10−10 = 931.6 MeV

143
Mass Defect:

It is the difference between the rest mass of the nucleus and the sum of the masses
of the nucleons composing a nucleus is known as mass defect.

Δm = [ Zmp + (A – Z) mn ] - M

Binding Energy:
It is the energy required to break up a nucleus into its constituent parts and place
them at an infinite distance from one another.
Eb = ∆ M c2
NUCLEAR FORCE:

● The protons and neutrons are held together by the strong attractive forces
inside the nucleus. These forces are called as nuclear forces.
● These forces, which act between the nucleons, are mainly responsible for the
stability of the nucleus.

Characteristics of nuclear forces:

(i) Nuclear forces are attractive in nature: - The magnitude which depends upon
inter nucleon distance is of very high order.

(ii) Nuclear forces are charge independent: - Nature of force remains the same
whether we consider force between two protons, between two neutrons or between
a proton and a neutron.

(iii) These are short range forces: - Nuclear forces operate between two nucleons
situated in close neighbourhood only.

(iv) Nuclear forces decrease very quickly with distance between two nucleons: -
Their rate of decrease is much rapid than that of inverse square law forces. The
forces become negligible when the nucleons are more than 10-12 cm apart.

(v) Nuclear forces are spin dependent: - Nucleons having parallel spin are more
strongly bound to each other than those having anti-parallel spin.

144
Potential energy of a pair of nucleons as a function of their separation:

⮚ The potential energy is a minimum (Negative PE) at a distance r0 of about


0.8 fm.
⮚ At large distances (> r0) the force is attractive for distances larger than 0.8
fm and negative PE goes on decreasing.
⮚ For distances < r0, the forces become repulsive i.e if they are separated by
distances less than 0.8 fm. The negative PE decreases to zero and becomes
positive.
Nuclear Fission:
The phenomenon of splitting of heavy nuclei (uranium) into two small nuclei when
a high energetic neutron incident on it, is known as the Nuclear Fission.
141
1
0𝑛 + 235 92 1
92𝑈 → 36𝐾𝑟 + 56𝐵𝑎 + 0𝑛 + 200 MeV
Fission does not always produce barium and krypton. A different pair can be
produced, for example
133
1
0𝑛 + 235
92𝑈 → 236
92𝑈 → 51𝑆𝑏
99
+ 41𝑁𝑏 + 4 10𝑛
140
1
0𝑛+ 235
92𝑈 →
94 1
54𝑋𝑒 + 38𝑆𝑟 + 2 0𝑛
Types of Nuclear Fission:
There are two type of nuclear fission reaction-
1. Controlled Nuclear Fission –
Example – Nuclear Reactor
2. Uncontrolled Nuclear Fission reaction
Example – Atom Bomb

145
Nuclear Fusion:
In nuclear fusion, two nuclei with low mass numbers combine to produce a
single nucleus with a higher mass number.
Nuclear fusion is defined as a type of nuclear reaction in which two lighter
nuclei merge into one another to form a heavier nucleus accompanied by a
release of a large amount of energy.
Thermonuclear fusion:
● To generate useful amount of energy, nuclear fusion must occur in bulk matter.
● To raise the temperature of the material until the particles have enough energy –
due to their thermal motions alone – to penetrate the coulomb barrier and
undergo fusion. This process is called thermonuclear fusion.
Proton – Proton Cycle:
1H
1
+ 1H1 → 1H2 + 1e0 + 0.4 MeV
1H
1
+ 1H2 → 2He3 + n + 5.5 MeV
2He
3
+ 2He → 2He + 2 1H + 12.9 MeV
3 4 1

Important Formula:
Atomic Mass Unit (u):
1 th
Atomic Mass Unit (amu) is of mass of 1 atom of carbon.
12
1 12
1u= x g
12 6.023 𝑥 1023
1u = 1.66 x 10-27 kg
Size of Nucleus:
Nucleus does not have a sharp or well-defined boundary but its radius is about 10-
15
m.
R = R0 A⅓
where R0 = 1.2 x 10-5 m is a constant which is the same for all nuclei and
A is the mass number of the nucleus.
Nucleus Density:
Mass of nucleus, M = A amu = A x 1.66 x 10-27 kg
4 4 4
Nuclear Volume, V = π R3 = π R03 A = π (1.2 x 10-15)3 A m3
3 3 3

= 7.24 x 10-45 A m3
𝑀
Nucleus Density, ρ = = 2.29 x 1017 kg m-3
𝑉

146
Calculate the energy equivalent of 1 g of substance.
Here, m = 1 g = 10−3 kg,
E = m c2 = 10−3 (3 × 108) 2 = 9 × 1013 J
Calculate Energy Equivalent to 1 amu or 1 u
1 amu = 1 u = 1.66 × 1027 kg
C = speed of light = 3 × 108 m s-1
E = m c2 = 1.66×1027(3×108)2 = 1.49×10−10 Joule
1MeV = 1.6×10−13 J
E = 1.6 × 10−13 x 1.49 × 10−10 = 931.6 MeV
Mass Defect:
Δm = [ Z mp + (A – Z) mn ] - M
Binding Energy:
It is the energy required to break up a nucleus into its constituent parts and place
them at an infinite distance from one another.
Eb = ∆ M c2
Important Graphs
Potential energy of a pair of nucleons as a function of their separation

147
Flow chart/mind map:
Discovered by Rutherford
by Alpha Scattering NUCLEUS
Experiment

Nuclear density (𝝆) Size of Nucleus Composition


It is a constant and Radius of Nucleus Nucleus composed Mass-Energy Nuclear
independent of mass R ∝ A1/3 of protons and Relation and Reaction
number (A) R = RoA1/3 neutrons collectively Mass Defect
ρ = 2.3 x 1017 kg/m3 It is of the order of known as nucleons.
1 fermi

Nuclear Force
Atomic Number(Z) Mass Number(A)
The protons and neutrons are held
No. of protons in the Total no. of protons and together by the strong attractive
nucleus of the atom neutrons in the nucleus of forces inside the nucleus. These forces
the atom A = Np + Nn= Z+N are called as nuclear forces.
These forces, which act between the
nucleons, are mainly responsible for
the stability of the nucleus.

Isotopes: The atoms Isobars: The atoms Isotones: The atoms of


of an element of different elements different elements having
having same atomic having same mass different atomic numbers
number but numbers but and different mass
different mass different atomic numbers but having same
numbers are called numbers, are called number of neutrons, are
isotopes. isobars. e.g 31H, 32He, called isotones. e.g., 31H,
22 22
e.g., 11H, 21H, 31H 11Na, 10Ne 4 14 16
2He, and 6C, 8O

Einstein’s Mass- Atomic Mass Unit: Mass Defect:


Energy Equivalence Atomic Mass Unit It is the difference between
Mass is a form of the rest mass of the
(amu) is 1/12th of
energy equivalent to nucleus and the sum of the
mass of 1 atom of
E=mc2 masses of the nucleons.
carbon.
1 u = 931.25 MeV Δm = [ Z mp + (A – Z) mn ]-M
1eV = 1.6 x 10-19 J 1u = 1.66 x 10-27 kg

Nuclear Fission: The process Nuclear Fusion: The process


of the splitting of a heavy of combining of two lighter
nucleus into two or more nuclei to form one heavy
lighter nuclei is called nuclear nucleus, is called nuclear
fission. fusion.

148
QUESTIONS
MCQ TYPE QUESTIONS
1. The radius of a nucleus is
(a) directly proportional to its mass number
(b) inversely proportional to its atomic weight
(c) directly proportional to the cube root of its mass number
(d) None of these
2. A neutron can cause fission in
(a) Hydrogen (c) Thorium
(b) Uranium – 235 (d) Uranium – 238
3. One milligram of matter converted into energy, will give
(a) 9 Joule (d) 9 x 1010 Joule
(b) 9 x 103 Joule
(c) 9 x 105 Joule
4. The mass density of a nucleus varies with mass number A as
(a) A2 (c) Constant
(b) A (d) 1/ A
5. Two spherical nuclei have mass numbers 216 and 64 with their radii R 1 and
R2 respectively. The ratio, R1/ R2 is equal to
(a) 3 : 2 (c) 1 : 3
(b) 1 : 2 (d) 2 : 3
6. Which of the following statements is true for nuclear forces?
(a) They obey the inverse square law of distance
(b) They obey the inverse third power law of distance
(c) They are short range forces
(d) They are equal in strength to electromagnetic forces
7. Fusion reaction takes place at high temperature because
(a) Nuclei break up at high temperature
(b) Atoms gets ionised at high temperature
(c) Kinetic energy is high enough to overcome the coulomb repulsion between
nuclei
(d) Molecules break up at high temperature.

149
ANSWERS
MCQ TYPE QUESTIONS
1. c b
2. d
3. c
4. a
5. c
6. c
ASSERTION REASON TYPE QUESTIONS

Given below two statements labelled as Assertion (A) and Reason (R) .
Select the most appropriate answer from the following
(a) Both A and R are correct and R is the correct explanation of A
(b) Both A and R are correct and R is the correct explanation of A
(c) A is true but R is false
(d) Both A and R are false
1. Assertion: The nuclear force becomes weak if the nucleus contains too many
protons compared to neutrons
Reason: The electrostatic forces weaken the nuclear force.
2. Assertion: Mass is not conserved, but mass and energy as a single entity called
mass-energy.
Reason: Mass and energy are inter-convertible in accordance with Einstein’s
relation, E = mc2
3. Assertion: Two protons can attract each other.
Reason: The distance between the protons within the nucleus is about 10-15 m.

ASSERTION REASON TYPE QUESTIONS


1. c
2. a
3. a

150
CASE STUDY TYPE QUESTIONS
1. Neutrons and protons are identical in the sense that their masses are nearly the same
and the force called nuclear force, does into distinguish them. Nuclear force is the
strongest force. Stability of nucleus determined by the neutron proton ratio or mass
defect or packing fraction. Shape of nucleus is calculated by quadrupole moment and
spin of nucleus depends on odd or even mass number. Volume of nucleus depends on
the mass number. Whole mass of the atom(nearly 99%) is centred at nucleus.
(i) The correct statements about the nuclear force is/are
(a) Charge independent
(b) Short range force
(c) Non conservative force
(d) All of these.
(ii) The range of nuclear force is the order of
(a) 2 x 10-10 m (c) 1.2 x 10-4 m
(b) 1.5 x 10-20 m (d) 1.4 x 10-15 m
(iii) Two protons are kept at a separation of 40 Å. Fn is the nuclear force and
Fe is the electrostatic force between them. Then
(a) Fn << Fe (c) Fn >> Fe
(b) Fn = F e (d) F n ≈ Fe
(iv) A force between two protons is same as the force between proton and
neutron. The nature of the force is
(a) Electrical force
(b) Weak nuclear force
(c) Gravitational force
(d) Strong nuclear force

Q. No. i ii iii iv
Ans. d d b d

151
2. In a single uranium fission about 0.9 x 235 MeV (≈ 200MeV) of energy is
liberated. If each nucleus of about 50 kg of U235 undergoes fission the amount
of energy involved is about 4 x 1015J. This energy is equivalent to about
20000tons of TNT, enough for a super explosion. Uncontrolled release of
large nuclear energy is called an atomic explosion. On August 6, 1945 an
atomic device was used in warfare for the first time. The US dropped an atom
bomb on Hiroshima, Japan. The explosion was equivalent to 20000 tons of
TNT. Instantly the radioactive products devastated 10 sq km of the city which
had 343000 inhabitants. Of this number 66000 were killed and 69000 were
injured: more than 67% of the city structures were destroyed.
235
(i) When 92U undergoes fission, about 0.1% of the original mass is converted
235
into energy. The energy released when 1 kg of 92U undergoes fission is
(a) 9 x 1011 J (c) 9 x 1015 J
(b) 9 x 1013 J (d) 9 x 10 18 J
(ii) In any fission process, ratio of mass of daughter nucleus to mass of parent
nucleus is
(a) Less than 1 (d) Depends on the mass of
(b) Greater than 1 parent nucleus
(c) Equal to 1
(iii) A nuclear fission is said to be critical when multiplication factor or K
(a) K = 1 (c) K < 1
(b) K > 1 (d) K = 0
(iv) On an average, the number of neutrons and the energy of a neutron
released per fission of a uranium atom are respectively
(a) 2.5 and 2 KeV (c) 2.5 and 2 MeV
(b) 3 and 1 KeV (d) 2 and 2 KeV

Q. No. i ii iii iv
Ans. b a a c

140
VERY SHORT & SHORT ANSWER TYPE QUESTIONS

1 Group the following six nuclides into three pairs of (i) isotopes (ii) isobars and
(iii) isotones
12 3 198 3 197 14
6𝐶 , 2𝐻𝑒 , 80𝐻𝑔, 1𝐻 , 79𝐴𝑢 , 6𝐶
12
Ans.Isotopes : 6𝐶 and 146𝐶 Same Z but different A
Isobars : 32𝐻𝑒 and 31𝐻 Same A but different Z
Isotones : 198 197
80𝐻𝑔 and 79𝐴𝑢 Same A-Z
2 Natural chlorine is found to be a mixture of two isotopes of masses 34.98 amu
and 36.98 amu respectively. Their relative abundance are 75.4 and 24.6 %
respectively. Find the composite atomic mass of natural chlorine.
Ans. Average atomic mass of natural chlorine = Weighted average of
its isotopes
75.4 𝑥 34.98+24.6 𝑥 36.98
= amu
100

= 35.47 amu
3 Calculate the radius of oxygen nucleus. Take R0 = 1.1 x 10-15 m and A = 16.
Ans.Here R = R0A1/3
= 1.1 x 10-15 x (16)3
= 1.1 x 10-15 x 2.52 = 2.77 x 10-15 m
4 State Einstein’s mass energy relation.
Ans.When mass is converted into energy in any process, the energy
obtained is given by Einstein’s mass energy relation, E= mc2.
5 State the energy equivalent of 1 amu in MeV.
Ans. 1amu = 931 MeV.
6 Express 16 mg mass into equivalent energy in eV.
Ans. Here m = 16 mg = 16 x 10-3 kg c = 3 x 108 m/s
Then Equivalent energy, E = mc2
= (16 x 10-3 x (3x 108)2 ) / 1.6 x 10-19
= 9 x 1030 eV

153
7 What is meant by mass defect of a nucleus?
Ans. The difference between the sum of the rest masses of the nucleons
constituting a nucleus and the rest mass of the nucleus is called mass
defect.
Δm = Zmp + (A-Z)mn – mN
8 Two nuclei have mass numbers in the ratio 1:2. What is the ratio of their
nuclear densities?
Ans. The ratio of their nuclear densities 1. Because nuclear density does not
depend upon mass number
9 State two characteristic properties of nuclear forces.
Ans. Nuclear force is the strongest force in nature.
Nuclear force is charge independent.
10 Out of 235U and 238U isotopes, which is fissile and which one is fertile?
Ans. A material which can undergo nuclear fission easily is called a
fissile material. A material which does not undergo nuclear fission
easily but can be made fissionable is called fertile material.
Here 235U is fissile and 238U is fertile.
11 Plot a graph showing the variation of potential energy of a pair of nucleus as
a function of their separation.
Ans. Potential energy of a pair of nucleons as a function of their
separation:

154
12 Two nuclei have mass numbers in the ratio 1:8. What is the ratio of their
nuclear radii?
Ans.We know, R = R0A1/3
Therefore, R1/R2 = (A1/A2)1/3
= (1/8)1/3
= 1/2
13 Why are nuclear fusion reactions also known as thermonuclear reactions?
Ans. Nuclear fusion requires very high temperature of 106-107 K.
14 What is meant by the term isotope?
Ans. The atoms of an element having same atomic number but different
mass numbers. are called isotopes.
e.g., 11H, 21H, 31H are isotopes of hydrogen
15 What is meant by the term isobars?
Ans.The atoms of different elements having same mass numbers but
different atomic numbers, are called isobars.
e.g., 31H, 32He, 22 22
11Na, 10Ne
16 What are isotones?
Ans.The atoms of different elements having different atomic numbers
and different mass numbers but having same number of neutrons, are
called isotones.
e.g., 31H, 42He, and 146C, 168O
17 Calculate the energy released in fusion reaction: 1H2 + 1H2 3
2He + n,

where Binding Energy of 1H2 = 2.23 MeV and of 2He3 = 7.73 MeV.
2 2 3
Ans.. 1H + 1H 2He +n
Energy released
= BE of products – BE of reactants
= BE (2He3) + BE (n) – 2 x BE (1H2)
= 7.73 + 0 – 2 x 2.23
= (7.73 – 4.46)
= 3.27 MeV

155
235
18 If 200 MeV energy is released in the fission of a single nucleus of 92U ,

how many fissions must occur to produce a power of 1 kW.


Ans. . Let the number of fissions per second be ‘n’. Then,
Energy released per second = n x 200MeV
= n x 200 x 1.6 x 10-13 J
Energy required per second = Power x Time
= 1 kW x 1 s
= 1000 J
Therefor n x 200 x 1.6 x 10-13 = 1000
Or n = 1000 / (200 x 1.6 x 10-13)
n = 3.125 x 1013.
19 Show that the density of nucleus is independent of its mass number and
remains constant.
Ans. Radius of an atomic nucleus is given by
R = R0 A⅓
where R0 = 1.2 x 10-15 m
Mass of nucleus, M = A amu = A x 1.66 x 10-27 kg

Nuclear Volume,
4 4
V = π R3 = π R03 A
3 3
4
= π (1.2 x 10-15)3 A m3 = 7.24 x 10-45 A m3
3
𝑀
Nucleus Density, ρ = = 2.29 x 1017 kg m-3 .
𝑉
20 What is meant by nuclear fusion?
Ans. Nuclear fusion is defined as a type of nuclear reaction in which
two or more lighter nuclei merge into one another to form a heavier
nucleus accompanied by a release of a large amount of energy
21 What is meant by nuclear fission?
Ans. The phenomenon of splitting of heavy nuclei (uranium) into two
small nuclei when a high energetic neutron incident on it, is known as
the Nuclear Fission.
1
0n + 235
92U → 92
36Kr + 141 1
56Ba + 3 0n + 200 MeV

156
22 The fission properties of 239Pu are very similar to those of 235U. The average
energy released per fission is 180 MeV. How much energy, in MeV, is
released if all the atoms in 1 kg of pure 239Pu undergo fission?
Ans. Number of atoms present in 239g of 239 Pu = 6.023 x 1023
Therefor number of atoms present in 1kg or 1000 g of 239Pu
= (6.023 x 1023 x 1000) / 239
= 2.52 x 1024
Energy released per fission = 180 MeV
Total energy released = 2.52 x 1024 x 180MeV = 4.54 x 1026 MeV.
23 State the necessary conditions for nuclear fusion to occur.
Ans. The nuclear fusion takes place under the conditions of extreme
temperature and density due to the following reasons:
1. The high temperature is necessary for the light nuclei to have
sufficient kinetic energy so that they can overcome their mutual
columbic repulsions and come closer than the range of nuclear
force.
2. High density or pressure increase the frequency of collisions of light
nuclei and hence increase the rate of fusion
24 Assuming that four hydrogen atoms combine to form a helium atom and
two positrons, each of mass 0.000549u, calculate the energy released.
Given m(1H1) = 1.007825 u and m (2He4) = 4.002604 u.
Ans. The nuclear reaction is
4 1H1 4
2 He + 2 +1e0 +Q
The initial mass = 4 x 1.007825 = 4.031300 u
Total final mass = 4.002304 + 2 x 0.000549
= 4.003702 u
Mass defect Δm = 4.031300 – 4.031300
= 0.027598 u
Total energy released = 0.027598 x 931 MeV = 25.7 MeV

157
25 Calculate the energy released by the fission of 1g of 235
92𝑈 in Kwh. Energy

per fission is 200 MeV.


Ans. Number of atoms in 1 gram of U235 = Avogadro ’s number / Mass
number = (6.023 x1023) / 235
Energy released per fission = 200 MeV
Therefor total energy released in fission of 1 gram of U235
= [ (6.023 x 1023) / 235] x 200
= 5.126 x 1023 MeV
We know 1 Kwh = 3.6 x 106 J
Hence total energy released = (5.126 x 1023 x 1.6 x 10-13) / 3.6 x 106
= 2.278 x 104 Kwh
26 Give some important points of differences between nuclear fission and nuclear
fission
Ans.
Nuclear Fission Nuclear Fusion
The phenomenon in which a heavy The process in which two light
nucleus when excited splits into two nuclei combine to form a single
smaller nuclei of comparable heavier nucleus is called nuclear
masses is called nuclear fission. fusion.
The conditions of high temperature The conditions of high
and pressure are not necessary for temperature and pressure are
its occurrence. necessary for its occurrence.
Neutrons are the link particles for Protons are the link particle for this
this process process
It is a quick process It occurs in several steps.
Energy available per nucleon is Energy available per nucleon is
small, about 0.85 MeV large, about 6.75 MeV
It produces very harmful The products of fusion are
radioactive wastes harmless

158
27 Explain the characteristics of nuclear force.
Ans. Characteristics of nuclear forces:
a. Nuclear forces are attractive in nature: - The magnitude which
depends upon inter nucleon distance is of very high order.
(ii) Nuclear forces are charge independent: - Nature of force
remains the same whether we consider force between two
protons, between two neutrons or between a proton and a
neutron.
(iii) These are short range forces: - Nuclear forces operate between
two nucleons situated in close neighbourhood only.
(iv) Nuclear forces decrease very quickly with distance between
two nucleons: - Their rate of decrease is much rapid than that of
inverse square law forces. The forces become negligible when
the nucleons are more than 10-12 cm apart.
(v) Nuclear forces are spin dependent: - Nucleons having
parallel spin are more strongly bound to each other than those
having anti-parallel spin.
28 The potential energy(V), of a pair of nucleons varies with separation (r)
between them, in the manner shown below. Draw the important
conclusions from the graph

Ans.The potential energy of the two nucleons is minimum at a


separation of r0 of about 0.8 fm.
(i) For separation values less than r0, the P.E increases rapidly
with the decrease in separation r. This indicates the strong
repulsion between the nucleons.
(ii) For separation values greater than r0, the P.E is negative which
falls to zero for a separation more than a few femtometers.
This indicates an attractive force between the nucleon

159
ELECTRONIC DEVICES
Short notes

• In metals, the conduction band and valence band partly overlap each other and there

is no forbidden energy gap.

• In insulators, the conduction band is empty and valence band is completely filled

and forbidden gap is quite large = 6 eV. No electron from valence band can cross

over to conduction band at room temperature, even if electric field is applied. Hence

there is no conductivity of the insulators.

• In semiconductors, the conduction band is empty and valence band is totally filled.

But the forbidden gap between conduction band and valence band is quite small,

which is about 1 eV. No electron from valence band can cross over to conduction

band. Therefore, the semiconductor behaves as insulator. At room temperature,

some electrons in the valence band acquire thermal energy, greater than energy gap

of 1 eV and jump over to the conduction band where they are free to move under the

influence of even a small electric field. Due to which, the semiconductor acquires

small conductivity at room temperature.

Metals Insulators Semiconductors

160
Distinguish between Intrinsic and Extrinsic Semiconductor
Intrinsic Extrinsic
1 It is pure semiconducting 1 It is prepared by doping a small quantity of
material and no impurity atoms impurity atoms to the pure semiconducting
are added to it material.
2 Examples are crystalline forms 2 Examples are silicon and germanium
of pure silicon and germanium. crystals with impurity atoms of arsenic,
antimony, phosphorous etc. or indium,
boron, aluminium etc.
3 The number of free electron in 3 The number of free electrons and holes is
conduction band and the never equal. There is excess of electrons in
number of holes in valence n-type semiconductors and excess of holes
band is exactly equal and very in p-type semiconductors.
small indeed.
4 Its electrical conductivity is low 4 Its electrical conductivity is high.
5 Its electrical conductivity is a 5 Its electrical conductivity depends upon
function of temperature alone. the temperature as well as on the quantity
of impurity atoms doped in the structure.

Distinction between n-type and p-type semiconductors


n-type semiconductors p-type semiconductors
1 It is an extrinsic 1 It is an intrinsic semiconductors which is
semiconductors which is obtained by doping the impurity atoms of
obtained by doping the IIIrd group of periodic table to the pure
impurity atoms of Vth group germanium or silicon semiconductor.
of periodic table to the pure
germanium or silicon
semiconductor.
2 The impurity atoms added, 2 The impurity atoms added, create vacancies
provide extra electrons in the of electrons (i.e. holes) in the structure and
structure, and are called donor are called acceptor atoms.
atoms.
3 The electrons are majority 3 The holes are majority carriers and electrons
carriers and holes are minority are minority carriers.
carriers.
4 The electron density (ne) is 4 The hole density (ne) is much greater than the
much greater than the hole electron density (nh) i.e. nh>> ne
density (nh)i.e. ne>>(nh)
5 The donor energy level is close 5 The acceptor energy level is close to valence
to the conduction band and far band and is far away from the conduction
away from valence band. band.
6 The Fermi energy level lies in 6 The Fermi energy level lies in between the
between the donor energy acceptor energy level and valence band.
level and conduction band.

161
p-n junction diode

Two important processes occur during the formation of p-n junction - diffusion and drift.
The motion of majority charge carriers give rise to diffusion current.
Due to the space charge on n-side junction and negative space charge region on p-side the
electric field is set up and potential barrier develops at the junction Due to electric field e-
on p-side moves to n and holes from n-side to p-side which is called drift current.
In equilibrium state, there is no current across p-n junction and potential barrier across p-
n junction has maximum value .
The width of the depletion region and magnitude of barrier potential depends on the nature
of semiconductor and doping concentration on two sides of p-n junction.
Forward Bias
P-n junction is FB when p-type connected to the +ve of battery and n-type connected to –
ve battery
Potential barrier is reduced and width of depletion layer decreases.
Reverse Bias
P-n junction in RB p-type connected to the –ve battery and n-type connected to +ve
Resistance of p-n junction is high to the flow of current.
FORWARD BIAS REVERSE BIAS

DIODE CHARACTERISTICS HALF WAVE RECTIFIER

162
FULL WAVE RECTIFIER

SPECIAL PURPOSE DIODES

LED PHOTODIODE SOLARCELL

Forward biased Reverse biased No external biasing, It generates


emf when solar radiation falls on
it.
Recombination of Energy is supplied by Generation of emf by solar cells is
electrons and holes take light to take an electron due to three basic process
place at the junction and from valence band to generation of e-h pair, separation
emits e m radiations conduction band. and collection
It is used in Burglar It is used in photo It is used in satellites, space
alarm, remote control detectors in vehicles, calculators etc.
communication

163
PRACTICE QUESTIONS

MULTIPLE CHOICE QUESTIONS


1. With fall of temperature, the forbidden energy gap of a semiconductor
(a) increases
(b) decreases
(c) sometimes increases and sometimes decreases
(d) remains unchanged

2. To obtain p-type silicon semiconductor, we need to dope pure silicon with:

(a) aluminium (c) oxygen


(b) phosphorus (d)germanium
3. Energy bands in solids are a consequence of:
(a) Ohm’s Law
(b) Pauli’s exclusion principle
(c) Bohr’s theory
(d) Heisenberg’s uncertainty principle

4. In a semi conductor what are responsible for conduction:


(a) only electron
(b) electron and hole both
(c) only hole
(d) None of these
5. On heating, resistance of semiconductors:
(a) decreases
(b) increases
(c) remains same
(d) first increases then decreases
6. In intrinsic semiconductor at room temperature, the number of electrons and
holes are:
(a) equal
(b) unequal
(c) infinite
(d) zero

164
7. In full wave rectifier, input a.c. current has a frequency v. The output frequency
of current is :
(a) ν/2
(b) ν
(c) 2ν
(d) None

8. In semi conductor, at room temperature :


(a) the valence bond is partially empty and the conduction band is partially filled
(b) the valence band is completely filled and the conduction band is partially filled
(c) the valence band is completely filled
(d) the conduction band is completely empty

9. The material most commonly used to manufacture electronic solid state devices
is :
(a) copper
(b) silicon
(c) germanium
(d) aluminium

10. On applying reverse bias to a junction diode, it:


(a) lowers the potential barrier
(b) raises the potential barrier
(c) increases the majority carrier current
(d) decreases the minority carrier current

Answers
1. d 8. a
2. a 9. b
3. b 10. b
4. b
5. a
6. a
7. c
165
ASSERTION & REASONING TYPE QUESTIONS
These questions consist of two statements, each printed as Assertion and Reason.
While answering these questions, you are required to choose any one of the following four
responses.
(a) Both Assertion and Reason are correct and the Reason is a correct explanation of
the Assertion.
(b) Both Assertion and Reason are correct but Reason is not a correct explanation of
the Assertion.
(c) Assertion is correct but Reason is incorrect.
(d) Both the Assertion and Reason are incorrect.
1. Assertion : A pure semiconductor has negative temperature coefficient of resistance.
Reason : In a semiconductor on raising the temperature, more charge carriers are released,
conductance increases and resistance decreases.
2. Assertion : Silicon is preferred over germanium for making semiconductor devices.
Reason : The energy gap in germanium is more than the energy gap in silicon.
3. Assertion : When two semi conductors of p and n type are brought in contact, they form
p-n junction which act like a rectifier.
Reason : A rectifier is used to convert alternating current into direct current.
4. Assertion : The diffusion current in a p-n junction is from the p-side to the n-side.
Reason : The diffusion current in a p-n junction is greater than the drift current when the
junction is in forward biased.
5. Assertion : Doping a germanium crystal with indium results in the formation of n-type
germanium crystal.
Reason : Indium is a pentavalent impurity atom.
Answers
1
.a
2.c
3. b
4. b
5. d

166
CASE BASED QUESTIONS
1. Read the Case Study given below and answer the questions that follow: A
Photodiode is again a special purpose p-n junction diode fabricated with a
transparent window to allow light to fall on the diode. It is operated under reverse
bias. When the photodiode is illuminated with light (photons) with energy (hν) greater
than the energy gap (E) of the semiconductor, then electron-hole pairs are generated
due to the absorption of photons. The diode is fabricated such that the generation of
e-h pairs takes place in or near the depletion region of the diode. Due to electric field
of the junction, electrons and holes are separated before they recombine. The
direction of the electric field is such that electrons reach n-side and holes reach p-side.
Electrons are collected on n-side and holes are collected on p-side giving rise to an
emf. When an external load is connected, current flows. The magnitude of the
photocurrent depends on the intensity of incident light.
I. Photo Diode is
a) forward biased b) reverse biased c) Not biased
II. Which of the following is true about photodiode?
a) E > hv b) E = hv c) E < hv d) None of these
III. Magnitude of photocurrent depends on
a) Intensity of light b) Biasing
c) Potential d) None of these
IV. Electrons and holes are separated before they recombine by:
a) Diffusion current
b) Drift current
c) Electric field
d) Electric potential
V. Direction of electric field is such that
a) electrons reach n- side
b) holes reach p- side
c) Both (a) and (b)
d) holes reach n side

167
2. This diode is a heavily doped p-n junction which under forward bias emits

spontaneous radiation. The diode is encapsulated with a transparent cover so that

emitted light can come out. When the diode is forward biased, electrons are sent from

n → p (where they are minority carriers) and holes are sent from p → n (where they

are minority carriers). At the junction boundary the concentration of minority

carriers increases compared to the equilibrium concentration (i.e., when there is no

bias). Thus at the junction boundary on either side of the junction, excess minority

carriers are there which recombine with majority carriers near the junction. On

recombination, the energy is released in the form of photons. Photons with energy

equal to or slightly less than the band gap are emitted. When the forward current of

the diode is small, the intensity of light emitted is small. As the forward current

increases, intensity of light increases and reaches a maximum. Further increase in the

forward current results in decrease of light intensity.

I. Which special purpose diode is mentioned above?


a) Solar cell b)LED
c) Photodiode d) Zener Diode
II. Which of the following device is forward biased?
a) Solar cell b)LED
c) Photodiode d) Zener Diode
III) Energy is released in the form of
a) Electron
b) Proton
c) Photon
d) None of these

168
IV. Which of the following is false?
a) As forward current increases, intensity of light increases.
b) Increase in further forward current decreases the intensity of light.
c) Photons with energy greater than band gap is emitted.
d) Photons with energy less than band gap is emitted.
V. What happens at the junction?
a) Excess minority carriers recombine with majority majority carriers.
b)Excess majority carriers recombine with majority majority carriers.
c)Excess minority carriers recombine with majority minority carriers.
d) None of these
3. A solar cell is basically a p-n junction which generates emf when solar radiation
falls on the p-n junction. It works on the same principle (photovoltaic effect) as the
photodiode, except that no external bias is applied and the junction area is kept much
larger for solar radiation to be incident because we are interested in more power. A
p-Si wafer of about 300 µm is taken over which a thin layer (~0.3 µm) of n-Si is grown
on one-side by diffusion process. The other side of p-Si is coated with a metal (back
contact). On the top of n-Si layer, metal finger electrode (or metallic grid) is deposited.
This acts as a front contact. The metallic grid occupies only a very small fraction of
the cell area so that light can be incident on the cell from the top. The generation of
emf by a solar cell, when light falls on, it is due to the following three basic processes:
generation, separation and collection— (i) generation of e-h pairs due to light (with
hν > E).
I. Working principle of solar cell is same as:
a) Photodiode c) LED
b) Zener diode d) Half wave rectifier
II.Which type of external biasing is applied in Solar cell?
a) Forward c) both (a) and (b)
b) Reverse d) No external biasing is applied
III. Which of the following is true for a solar cell?
a) hv < E
b) hv>E
c) E = hv
d) None of these

169
IV. What is the principle of Solar cell?
V. In a solar cell,
a) Junction area is large c) Junction area is negligible
b) Junction area is small d) None of these
4. Consider a thin p-type silicon (p-Si) semiconductor wafer. By adding precisely a small
quantity of pentavelent impurity, part of the p-Si wafer can be converted into n-Si.
There are several processes by which a semiconductor can be formed. The wafer now
contains p-region and n-region and a metallurgical junction between p-, and n- region.
Two important processes occur during the formation of a p-n junction: diffusion and
drift. We know that in an n-type semiconductor, the concentration of electrons (number
of electrons per unit volume) is more compared to the concentration of holes. Similarly,
in a p-type semiconductor, the concentration of holes is more than the concentration of
electrons. During the formation of p-n junction, and due to the concentration gradient
across p-, and nsides, holes diffuse from p-side to n-side (p → n) and electrons diffuse
from n-side to p-side (n → p). This motion of charge carries gives rise to diffusion
current across the junction.
I.How can a p-type semiconductor be converted into n- type semiconductor?
a) adding pentavalent impurity
b)adding trivalent impurity
c)not possible
d) heavy doping
II.Which of the following is true about n type semiconductor?
a) concentration of electrons is less than that of holes.
b)concentration of electrons is more than that of holes.
c)concentration of electrons equal to that of holes.
d)None of these
III.Which of the following is true about p type semiconductor?
a) concentration of electrons is less than that of holes.
b)concentration of electrons is more than that of holes.
c)concentration of electrons equal to that of holes.
d)None of these

170
lV. Which of the following is the reason about diffusion current?
a) diffusion of holes from p to n
b) diffusion of electrons from n to p
c) both (a) and (b)
d) None of these
V. What are the processes that occur during formation of a p-n junction?
a) drift b) diffusion c) both (a) and (b) d)None of these
Answers
Q.1
I. B
II. C
III. A
IV. C
V. C
VI.

Q.2
I. B
II. B
III. C
IV. C
V. A

Q.3
I. A
II. D
III. B
IV. Photovoltaic
Effect
V. A

Q.4
I. A
II. B
III. A
IV. C
V. C

171
VERY SHORT ANSWER QUESTIONS (1 mark)
1. What is meant by doping?
Ana: The technique of adding impurities to a pure semiconductor is known as doping
2. What are n-type semiconductors?
Ans: n-type semiconductors are extrinsic semiconductors in which dopant atoms
provide additional conduction electrons.
3. What are p-type semiconductors?
Ans: p-type semiconductors are extrinsic semiconductors in which dopant atoms
provide additional holes for conduction.
4. What is the difference between an n-type and a p-type intrinsic semiconductor?

Ans:

5. The graph shown in the figure represents a plot of current versus voltage for a
given semi-conductor. Identify the region, if any, over which the semi-conductor
has a negative resistance.

Answer:

In the region between B and C, the semiconductor has a negative resistance.


6. Show the variation of resistivity of Si with temperature in a graph.

Answer:

7. How does the width of depletion layer change, in reverse bias of a p-n junction
diode?

Answer: Increases

172
8. In a semiconductor concentration of electron is 8 x 1013cm-3 and holes 5 x 1012
cm-3: is it P or N type semiconductor?

Ans: N type

9. In a given diagram, is the diode reverse (or) forward biased?

Ans: Reverse biased.

SHORT ANSWER QUESTIONS (2 marks)

1. The intrinsic carrier concentration in germanium crystal at 300 K is 2.5x1013 per


cm3.If the electron density in an n-type germanium crystal at 300 K is 0.5x1017 per
cm3, what will be the hole density in this crystal at 300 K?
Answer:
n e n h = n i2 nh =1.25 x 1010 per cm3
2. Mention any TWO differences between an intrinsic semiconductor and a p-type
semiconductor.
Ans:

3. Give reason, why a p-type semiconductor crystal is electrically neutral


although nh >> ne?
Ans: In a p-type semiconductor, the trivalent impurity atom shares its three
valence electrons with the three tetravalent host atoms while the fourth bond
remains unbounded. The impurity atom as a whole is electrical neutral. Hence the
p-type semiconductor is also neutral.

173
4. In the given following diagram ‘S’ is a semiconductor. Would you increase
or decrease the value of R to keep the reading of the ammeter A constant when S is
heated? Give reason for your answer.

Ans: The value of ‘R’ would be increased since the resistance of ‘S’, a semi
conductor decreases on heating.

5. State the factor which controls wave length and intensity of light emitted by
LED.
Ans: (i) Nature of semi-conductor (ii) Forward Current
6. Pure Si at 300K have equal electron and holes concentration 1.5 x 1016 per
m3. Doping by Indium increases hole concentration to 4.5 x 1022 per m3. Calculate
new electron concentration.
Ans: Hint : nenh = ni2
7. A germanium diode is preferred to a silicon one for rectifying small
voltages. Explain why?
Ans: Because the energy gap for Ge (Eg = 0.7 ev) is smaller than the energy gap
for Si (Eg = 1.1eV) or barrier potential for Ge<Si.
8. Why semiconductors are opaque to visible light but transparent to infrared
radiations?
Ans: The photons of infrared radiation have smaller energies, so they fall to
excite the electrons in the valence band. Hence infrared radiations pass through the
semiconductors as such; i.e. a semiconductor is transparent to infrared radiation
9. The ratio of number of free electrons to holes ne/nh for two different materials
A and B are 1 and <1 respectively. Name the type of semiconductor to which A and
B belongs.
Ans: If ne/nh =1 . Hence A is intrinsic semiconductor. If ne/nh<1 , ne<nh hence B is
P-type.

10. With the help of a diagram show the biasing of light emitting diode. Give
two advantages over conventional incandescent Lamp.
Ans: Mono chromatic, Consume less power.

174
11. Which of the diodes are forward biased and which are reverse biased?

Ans: (i) is forward biased, remaining all reverse biased

SHORT ANSWER QUESTIONS (3 marks)


1. In the case of n-type Si semiconductor, the donor level is slightly below the bottom
of conduction band. whereas in p-type semiconductor, the acceptor energy level is
slightly above the top of the valence band. Explain, what role do these energy levels
play in conduction and valence bands?

Ans: Role of energy levels in conduction and valence bands : In the energy band
diagram of n-type Si semiconductor, the donor energy level ED is slightly below the
conduction band and electrons from this level moves into conduction band with very
small supply of energy. At room temperature, most of the donor atoms get ionised, but
very few (~ 10-12) atoms of Si atom get ionised. So the conduction band will have most of
the electrons coming from donor impurities.
For p-type semiconductor, the acceptance energy level EA is slightly above the
valence band. With very small supply of energy, an electron from the valence band can
jump to the level EA and ionise the acceptor negatively. At room temperature, most of the
acceptor atoms get ionised leaving holes in the valence band.

175
2. Distinguish between n-type and p-type semi-conductors on the basis of energy
band diagrams. Compare their conductivities at absolute zero temperature and at
room temperature.
Answer:

Distinction between n-type and p-type semiconductors on the basis of energy level
diagram
(i) In n-type semi conductors an extra energy level (called donor energy level) is
produced just below the bottom of the conduction band, while in the p-type
semiconductor, this extra energy band (called acceptor energy level) is just above the top
of the balanced band.
(ii) In n-type semiconductors, most of the electrons come from the donor impurity while
in p-type semiconductor, the density of holes in the valence band is predominantly due to
the impurity in the extrinsic semiconductors.
(iii) At absolute zero temperature conductivities of both types of semi-conductors will be
zero.
(iv) For equal doping, an n-type semiconductor will have more conductivity than a p-type
semiconductor, at room temperature.

176
3. Determine the current through resistance “R” in each circuit. Diodes D1 and D2
are identical and ideal.

Answer: In circuit (i) Both D1 and D2 are forward biased hence both will conduct
current and resistance of each diode is “0”.Therefore I = 3/15 = 0.2 A
Diode D1 is forward bias and D2 is reverse bias, therefore resistance of diode D1 is
“0” and resistance of D2 is infinite. Hence D1 will conduct and D2 do not conduct.
No current flows in the circuit.

4. A photodiode is fabricated from a semiconductor with a band gap of 2.8eV.


Can it detect a wavelength of 600nm? Justify?

Ans: Energy corresponding to wavelength 600 nm is


E=hc/  = 6.6x10-34 x 3x108 joule = 0.2eV.
600x10-9
It cannot detect because E < Eg

5. Draw a circuit diagram to show how a photodiode is biased. Draw its


characteristic curves for two different illumination intensities.
Ans:

177
LONG ANSWER QUESTIONS (5 marks)

1. Draw the Energy band diagrams for (i) insulator (ii) conductor and (iii) pure

semiconductor. How does the energy band, for a pure semiconductor, get

affected when this semiconductor is doped with (a) an acceptor impurity and

(b) donor impurity? Hence discuss why the ‘holes’, and the ‘electrons’

respectively, become the ‘majority charge carriers’ in these two cases?

Answer:

Distinguishing features between conductors, semiconductors and insulators :

(i) Insulator. In insulator, the valence band is completely filled. The conduction

band is empty and forbidden energy gap is quite large. So no electron is able to go

from valence band to conduction band even if electric field is applied. Hence

electrical conduction is impossible. The solid/ substance is an insulator.

(ii) Conductors (Metals). In metals, either the conduction band is partially filled or the

conduction and valence band partly overlap each other. If small electric field is applied

across the metal, the free electrons start moving in a direction opposite to the direction of

electric field. Hence, metal behaves as a conductor.

(i) Semiconductors. At absolute zero kelvin, the conduction band is empty and the

valence band is filled. The material is insulator at low temperature. However the

energy gap between valence band and conduction band is small. At room

temperature, some valence electrons acquire thermal energy and jump to

conduction band where they can conduct electricity. The holes left behind in

valence band act as a positive charge carrier.

178
(ii) ‘Energy Band’ diagrams :

(a) When the semiconductor is doped with an acceptor impurity, it results in an additional

energy level a little above the top of the valence band.

(b) The donor impurity results in an additional energy level a little below the bottom of

the conduction band.

In the first case, electrons from the valence band, easily jump over to the acceptor level,

leaving ‘holes’ behind. Hence, ‘holes’ becomes the majority charge carriers.

In the second case, electrons from the donor level, easily ‘jump over’ to the conduction

band. Hence, electrons become the majority charge carriers.

2. Explain the working of p-n junction diode as a full wave rectifier? Draw the

input output waveforms.

Ans: A Rectifier is a device which converts AC into DC.

Using two diodes and a centre tapped transformer, a full wave rectifier can be

constructed, as shown in figure.

179
During the positive half cycle of input AC, the diode D1 conducts and D2 do not conduct

and the current flows from B to A.

During the positive half cycle of input AC, the diode D2 conducts and D1 do not conduct

and the current flows from B to A.

180
181
182
183
184
185
186
187
188
189
190
191
KENDRIYA VIDYALAYA SANGATHAN
ERNAKULAM REGION
TERM II EXAMINATION- (2021-22)
SAMPLE QUESTION PAPER I
CLASS-XII (PHYSICS)
BLUE PRINT

UNIT NAME OF UNIT 2 MARK 3 CASE TOTAL


NO: QSTN MARK BASED
QSTN QUESTION
V Electromagnetic
Waves ----- 3(4) 5(1) 17
VI Optics
VII Dual Nature of
Radiation and
Matter
VIII Atoms and Nuclei 2(3) 3(4) ---- 18
IX Electronic Devices

TOTAL 6(3) 24 (8) 5(1)

Note:
1) The number outside the bracket represents the marks & the number inside the
bracket represents the total number of questions.
2) No overall choice to be given. However, an internal choice must be provided in one
question of two marks and two questions of three marks. Student needs to attempt
only one of the choices in such questions.

192
KENDRIYA VIDYALAYA SANGATHAN
ERNAKULAM REGION
SAMPLE QUESTION PAPER 1
CLASS XII PHYSICS THEORY
TERM II SESSION 2021 – 22
Time: 2 hours Max. Marks: 35

General Instructions:
(i) There are 12 questions in all. All questions are compulsory.
(ii) This question paper has three sections: Section A, Section B and Section C.
(iii) Section A contains three questions of two marks each, Section B contains
eight questions of three marks each, Section C contains one case study-based
question of five marks.
(iv) There is no overall choice. However, an internal choice has been provided in
one question of two marks and two questions of three marks. You have to
attempt only one of the choices in such questions.
(v) You may use log tables if necessary but use of calculator is not allowed.

SECTION: A

1) Draw energy band diagram of n-type and p-type semiconductor at


temperature T > 0 K. Mark the donor and acceptor energy level.

2) Green light ejects photoelectrons from a given photosensitive surface, whereas


yellow light does not. What will happen in the case of violet and red light? Give
reason for your answer.
OR
If the frequency of light falling on a metal is doubled, what will be the effect
on
photocurrent and the maximum kinetic energy of emitted photoelectrons?

3) Assuming that the two diodes D1 and D2 used in the electric circuit are ideal,
find out the value of current flowing through the 1Ω resistor.

193
SECTION: B
4) A beam of light consisting of red green and blue colours is incident on a right angled
prism. The refractive index of the material of the prism for the above red, green and
blue wavelengths are 1.39, 1.44 and 1.47, respectively. Complete the ray diagram &
give reasons.

OR

A glass prism of refractive index 1.5 is immersed in water (refractive index 4/3). A
light beam normally on the face AB is totally reflected to reach on the face BC if θ is the
angle of incidence at BC. Prove that Sin θ ≥ 8/9.

5) (i) Name the EM waves which are suitable for radar systems & so used in
aircraft
navigation. Write the range of frequency of these waves.

(ii) If the earth did not have atmosphere, would its average surface
temperature be
higher or lower than what it is now? Explain

(iii) Optical and radio telescopes are built on the ground while X-ray astronomy is
possible only from satellites orbiting the earth. Why?

194
6) Draw the electric circuit diagram of a full wave rectifier and explain the working.
Draw the input and output wave form and give the relation between the frequency
of the input and output wave.

OR
i) What are the important processes that occur during the formation of a p-n
Junction ?

ii) How is a p-n junction diode connected to get a forward bias and
reverse bias.

iii) Draw the V-I characteristics corresponding to forward and reverse bias.

7) A screen is placed 2m away from the lens to obtain the diffraction pattern in the
focal plane of the lens in a single slit diffraction experiment.

i) What will be the slit width if the first minimum lies 5 mm on either side of
the central maximum when plane light waves of wavelength 4000 Å are
incident on the slit?

ii) Also give two points of comparison between interference pattern &
diffraction pattern.

8) Plot a graph showing the variation of photoelectric current with collector


potential for two different frequencies, ʋ1 > ʋ2, of incident radiation having the
same intensity. In which case will the stopping potential be higher? Justify your
answer.

9) Using postulates of Bohr atom model of hydrogen atom obtain the expression
for the wavelength of radiation emitted when atom makes a transition from the
higher energy state with quantum number ni to the lower energy level with
quantum number nf (nf < ni).

10) Derive Lens maker’s formula.

11) i) Define the term binding energy.

iii) Calculate the binding energy per nucleon of 20 Ca40 nucleus. Given that
mass of neutron is 1.008665u ,mass of 20 Ca40 = 39.962591u, mass of
proton is 1.007825u and 1u = 931Mev.

195
Section: C

12) Interference occurs when several waves are added together provided that
the phase differences between them remain constant over the observation time.
Interference is essentially an energy redistribution process. The energy which is
lost at the destructive interference is regained at the constructive interference.

i) What does happen to the interference pattern the two slits S1 and S2 in
Young’s double experiment are illuminated by two independent but identical
sources?
(a) The intensity of the bright fringes doubled.
(b) The intensity of the bright fringes becomes four times.
(c) Two sets of interference fringes overlap.
(d) No interference pattern is observed .

ii) In Young’s double slit experiment, a maximum intensity is obtained when


the path difference between the interfering waves is
(a) n λ
(b) n λ / 2
(c) (2n+1) λ / 2
(d) (2n – 1) λ / 4

iii) The intensity of light emerging from the two slits, in Young’s experiment is
in the ratio 1 : 4. The ratio of, the intensity of the minimum to that of the
consecutive maximum will be :
(a)1:4
(b)1:9
(c)1:16
(d) 2 : 3

196
iv) In a Young’s Double Slit Experiment, band width is found to be 1mm & the
setup is shifted from air to water. The new band width is:
(a) 1mm
(b) 1.33mm
(c) 0.5mm
(d) 0.75mm

v) In Young’s double slit experiment, the distance between the slit and the
screen is doubled and the separation between the slits is reduced to half. The
fringe width:
(a) is doubled
(b) becomes four times
(c) is halved
(d) remains unchanged

197
KENDRIYA VIDYALAYA SANGATHAN, ERNAKULAM REGION
SAMPLE PAPER – II
CLASS : XII MAX MARKS : 35
SUBJECT : PHYSICS TIME : 2 hours
SECTION A

1 Two identical Silicon wafers A and B are doped with Boron and Antimony 2
respectively. (i) Identify the extrinsic property of A and B after doping (ii)
Draw the energy band diagram of extrinsic semiconductor formed from A

2 Every metal has a definite work function. Then why do photoelectrons not 2
come out with same energy, even if the incident radiation is monochromatic?
Why is there an energy distribution of photoelectrons?

OR

Light from Balmer series of Hydrogen is able to eject photoelectrons from a


metal. What can be the maximum work function of the metal? Justify your
answer with necessary mathematical relations

3 Study the I-V characteristics given below and answer the following: 2
(i) Identify the device and state one use of it
(ii) Draw the circuit diagram used to obtain the typical I – V
characteristics of the device

198
SECTION B
4 (i) How does the angular width of central band of diffraction pattern affect 3
when (a) the width of the slit is doubled? (b) blue light is replaced by
yellow light?
(ii) Two students are separated by a 7m partition wall in a room 10m high. If
both light and sound waves can bend around the obstacles, explain why
the students are unable to see each other even though they can converse
easily?

(iii) A light wave is incident normally over a slit of width 24 x10-5cm. The
angular position of second minima from the central maxima is 30o. What
is the wavelength of light?
5 A monochromatic light source of power 5mW emits 8X1015 photons per 3
second. This light ejects photoelectrons from a metal surface. The stopping
potential for this set up is 2V. Calculate the work function of the metal.
6 3
A ray of light while travelling from a denser medium to a rarer medium
undergoes total reflection. Derive the expression for the critical angle in terms
of the speed of light in the respective media.
Determine the value of the angle of incidence for a ray of light, travelling from
a medium of refractive index n1 = √2 into a medium of refractive index n2 = 1,
so that it just grazes along the surface of separation
OR
(i) Radio waves and Gamma rays both are transverse in nature and
electromagnetic in character with same speed in vacuum. In what
respects are they different, write any two points.
(ii) Electromagnetic waves with wavelengths
(a) λ1 used to treat muscular strain (b) λ2 used in FM broadcasting (c)
λ3 are produced in nuclear reactions (d) λ4 are produced by
bombarding metal target by high speed electrons. Identify these waves
and arrange them in the decreasing order of their wavelengths.

199
7 Two narrow slits are illuminated by a monochromatic source. Name the pattern 3
obtained on a screen kept at a distance. Draw the schematic diagram showing
the arrangements to obtain the pattern. Derive the expression for positions of
bright and dark fringes.
235
8 Name the reaction which takes place when slow neutron beam strikes 92𝑈. 3
Using the data given below, calculate the energy released when Uranium splits
into Krypton and Barium with the release of 2 neutrons.
235
Mass of 92𝑈 = 235.043933 amu
Mass of 141
56𝐵𝑎 = 140.917700amu
92
Mass of 36𝐾𝑟 = 91.895400 amu
Mass of 10𝑛 = 1.008665 amu
9 Draw a ray diagram showing the image formation of a distant object by a 3
refracting type telescope in normal adjustment.
Define magnifying power and write two important factors considered to
increase the magnifying power.
OR
Draw a ray diagram to show the working of a compound microscope while
forming the image at near point.
In a compound microscope, an object is placed at a distance of 1.5cm from the
objective of focal length 1.25cm. If the eyepiece has focal length 5cm and final
image is formed at near point, estimate the magnifying power of microscope.
10 An external potential difference is applied across a p-n junction diode in such a 3
way that the applied potential difference opposes the internal potential barrier.
(i) Draw the circuit arrangement to study the V-I characteristics
(ii) Explain the working of junction diode in the above arrangement
and define knee voltage.
11 (i) In hydrogen atom, an electron undergoes transition from 2nd excited state to 3
the first excited state and then to the ground state. Identify the spectral series to
which these transitions belong.
(ii) Find out the ratio of the wavelengths of the emitted radiations in the two
cases.

200
12 CASE STUDY:-
A prism is a portion of a transparent medium bounded by two plane faces
inclined to each other at a suitable angle. A ray of light suffers two refractions
on passing through a prism and hence deviates through a certain angle from its
original path. The angle of deviation of a prism is, δ = (n- 1) A, through which a
ray deviates on passing through a thin prism of small refracting angle A.
If ‘n’ is refractive index of the material of the prism, then prism formula is,
𝐴 + 𝛿𝑚 )
𝑆𝑖𝑛(
𝑛= 2
𝐴
𝑆𝑖𝑛 ( ⁄2)

(i) For which colour, angle of deviation is minimum?


(a) Red (b) Yellow (c) Violet (d) Blue
(ii) When white light moves through vacuum
(a) all colours have same speed (b) different colours have different speeds
(c) violet has more speed than red (d) red has more speed than violet.
(iii) For which colour, the refractive index of the material of prism is
maximum?

(a) Red (b) Violet (c)Blue (d) Equal for all colours

(iv) What is the deviation produced by a prism of angle 6°? (Refractive index of
the material of the prism is 1.644).
(a) 3.864° (b) 4.595° (c) 7.259° (d) 1.252°
(v) A ray of light falling at an angle of 50° is refracted through a prism and
suffers minimum deviation. If the angle of prism is 60°, then the angle of
minimum deviation is
(a) 45° (b) 75° (c) 50° (d) 40°

201
KENDRIYA VIDYALAYA SANGATHAN
ERNAKULAM REGION
SAMPLE QUEDSTION PAPER III

CLASS XII TERM 2 (2021-22)

Time: 2 hours Max. Marks: 35

-----------------------------------------------------------------------------------------------------
General instructions

(i) There are 12 questions in all. All questions are compulsory.


(ii) This question paper has three sections: Section A, Section B and Section C.
(iii) Section A contains three questions of two marks each, Section B
contains eight questions of three marks each, and Section C contains
one case study-based question of five marks.

(iv) There is no overall choice. However, an internal choice has been


provided in one question of two marks and two questions of three marks.
You have to attempt only one of the choices in such questions.
(v) You may use log tables, if necessary, but use of calculator is not allowed.

SECTION A

1. Two thin lenses of power –4 D and +2 D are placed in contact coaxially. Find the
focal length of the combination
OR
How does the angle of minimum deviation of glass prism of refractive index 1.5
change, if it is immersed in a liquid of refractive index 1.3?

2. Draw the circuit diagram of a full-wave rectifier and explain its working

3. An electron and a proton are accelerated through the same potential difference.

Which one of the two has greater value of de-Broglie wavelength associated with

it? Why?

202
SECTION B

4. (a)Show giving an example, how electromagnetic wave carry energy and


momentum.
(b) How are microwaves produced? Why is it necessary in microwave ovens to
select the frequency of microwaves to match the resonant frequency of water
molecule?
5. Draw a labeled ray diagram of astronomical telescope of the near point
adjustment.
You are given three lenses of power 0.5 D, 4D&10D.State with reason which two
lenses will you select for constructing a good astronomical telescope
OR
The image of a needle placed 45 cm from a lens is formed on a screen placed 90

cm on the other side of the lens. Find the displacement of the image if the object is

moved by 5 cm away from the lens.

6. What is Total internal Reflection? Mention the conditions required for it to

happen. Hence mention one of its practical applications.

7. Describe Young’s double slit experiment to produce interference pattern due to a

monochromatic source of light. Deduce the expression for the fringe width

8. Draw the diagram showing intensity distribution of light on the screen for diffraction

of light at a single slit. How is the width of central maximum affected if,

i) the width of the slit is doubled

ii) the wavelength of light used is increased? Justify your answer in each case.

9. Write Einstein’s photo electric equation. Explain how Planck’s constant and work

function can be determined from the above.

10. Draw a graph showing the variation of potential energy of a pair of nucleons as

a function of their separation. Indicate the region in which nuclear force is

i) attractive ii) repulsive.

Write any three characteristic features of nuclear force.

203
11. The ground state energy of hydrogen atom is -13.6eV.

i) What is the kinetic energy of an electron in the second excited state?

ii) What is the potential energy of an electron in the third excited state?

iii) If the electron jumps to the ground state from the third excited state, calculate the

wavelength of the photon emitted.

SECTION C (CASE STUDY)

12. A heavily doped p-n junction diode under forward bias can emit spontaneous
radiation. The diode is encapsulated with a transparent cover so that emitted light can
come out. When the diode is forward biased, electrons are sent from n → p (where
they are minority carriers) and holes are sent from p → n (where they are minority
carriers). At the junction boundary, the concentration of minority carriers increases as
compared to the equilibrium concentration (i.e., when there is no bias).
Thus at the junction boundary on either side of the junction, excess minority carriers
are there which recombine with majority carriers near the junction. On recombination,
the energy is released in the form of photons. Photons with energy equal to or slightly
less than the band gap are emitted. When the forward current of the diode is small, the
intensity of light emitted is small. As the forward current increases, intensity of light
increases and reaches a maximum. Further increase in the forward current results in
decrease of light intensity

The V-I characteristics is similar to that of a Si junction diode. But the threshold
voltages are much higher and slightly different for each colour. The reverse
breakdown voltages are very low, typically around 5 V. So, care should be taken that
high reverse voltages do not appear across them. It can emit red, yellow, orange, green
and blue light are commercially available.
(i) Which special purpose diode is mentioned above?

a) Solar cell d) Zener Diode

b) LED

c) Photodiode

204
(ii) During recombination at the junction, emitted photons have:

(a) energy equal to or slightly less than the band gap.

(b) Energy greater than the band gap.

(c) Energy which has no relation with the band gap.

(d) Very low energy compared to band gap.

(iii) Which of the following is true about n type semiconductor?

a) Concentration of electrons is less than that of holes

b) Concentration of electrons is more than that of holes.

c) Concentration of electrons equal to that of holes.

d) None of these

(iv) Which of the following device is forward biased?

a) Solar cell b)LED

c) Photodiode d) Zener Diode

(v) Threshold voltage of LED is:


(a) lower compared to other p-n junction diodes and slightly different for each
colour.
(b) lower compared to other p-n junction diodes and same for all colours.
(c) higher compared to other p-n junction diodes and same for all colours.
( d) higher compared to other p-n junction diodes and slightly different for each
colour.

205
KENDRIYA VIDYALAYA SANGATHAN
ERNAKULAM REGION
SAMPLE QUESTION PAPER IV
CLASS XII PHYSICS (THEORY)
TERM II SESSION 2021 - 22
MM : 35 TIME : 2 Hours
………………………………………………………………………………………

General Instructions:

(i) There are 12 questions in all. All questions are compulsory.


(ii) This question paper has three sections: Section A, Section B and Section C.
(iii) Section A contains three questions of two marks each, Section B contains
eight questions of three marks each, Section C contains one case study-based
question of five marks.
(iv) There is no overall choice. However, an internal choice has been provided in
one question of two marks and two questions of three marks. You have to
attempt only one of the choices in such questions.

SECTION- A

1. The work function for a given photosensitive surface is 2.5 eV. When light of

frequency υ falls on this surface, the retarding potential needed to completely

stop the photoelectrons is found to be 4.1 V. What is the value of υ ?

OR

Calculate the frequency of the photon which can excite an electron to -3.4

eV from -13.6 eV

2. Name the electronic device used for detecting optical signals and mention the

biasing in which it is operated. Draw its I-V characteristics.

206
3. Determine the current through resistance “R” in each circuit. Diodes D1 and D2 are
identical and ideal.

SECTION-B
4. Name the following constituent radiations of electromagnetic spectrum which-
(i) are used in satellite communication/in radar and geostationary satellite
(ii) are used for studying crystal structure of solids
(iii) are similar to the radiations emitted during decay of radioactive nuclei
(iv) are used for water purification/ are absorbed from sunlight by ozone layer
Arrange the radiations in the increasing order of wavelength.
5. Draw a ray diagram of a compound microscope. Obtain the expression for its
magnifying power.
6. Write the conditions for total internal reflection to take place.
Three rays of light, red (R), green (G) and blue (B) are incident on the face AB of a
right angled prism as shown in the figure. The refractive indices of the material
prism for red, green and blue colours are 1.39, 1.44, and 1.47 respectively. Which
one of the three rays will emerge out of the prism? Give reason to support your
answer.

OR

207
In a two slit experiment with monochromatic light, fringes are obtained on a screen

placed at some distance D from the slits. If the screen is moved 5 x 10-2 m towards

the slits, the change in fringe width is 3 x 10-5 m. If the distance between the slit is

10-3 m. Calculate the wavelength of the light used.

7. Define the term ‘wave front’. Using Huygens’s construction of a wave front,

explain the refraction of a plane wave front at a plane surface and hence deduce

Snell’s law.

8. Why macroscopic objects in our daily life do not show wave like properties?

An electron, an alpha particle and a proton have the same kinetic energy, which

one of these particles has (i) the shortest and (ii) the largest, de-Broglie wavelength?

9. What is the energy possessed by an ē for n= ∞ ?

The ground state energy of hydrogen atom is -13.6eV. What is the K.E & P.E of

the electron in this state?

10.At a given instant there are 25% un-decayed radioactive nuclei in a sample. After

10 seconds the number of un-decayed nuclei reduces to 12.5 %.calculate the i)

mean life of the nuclei ii) the time in which the number of the un-decayed nuclei

will further reduce to 6.25 % of the reduced number.

OR

(a) State the postulates of Bohr's model of hydrogen atom.

(b) Show that the radius of the orbit in hydrogen atom varies as n2 , where n is the

principal quantum number of the atom.

11. With the help of a suitable diagram, explain the formation of depletion region in a

p-n junction. How does its width change when the junction is: (i) forward biased? &

(ii) reverse biased?

208
SECTION-C

12 Lens maker's formula.


The lens maker’s formula relates the focal length of a lens to the refractive index of
its material and the radii of curvature of its two surfaces . This formula is used to
manufacture a lens of particular focal length from the glass of a given refractive
index. For this reason , it is called the lens maker’s formula .

1. For a plano-convex lens of radius of curvature 10 cm the focal length is 30 cm . If


the refractive index of the material of the lens is
(i).2.0 (ii).1.33 (iii).1.66 (iv).1.5
2 . A convex lens of focal length 20 cm is placed in contact with a diverging lens of
unknown focal length . The lens combination acts as a converging lens and has a
focal length of 30 cm . What is the focal length of diverging lens
(i).-90 cm (ii).-60 cm (iii).-30 cm (iv).-10 cm
3. An image is formed on the screen by a convex lens when upper half part of lens is
covered with black paper then
(i).half image is formed (ii).full image is formed
(iii).intensity of image is enhanced (iv).all of the above
4. The focal length of a biconvex lens of radii of each surface 50 cm and refractive
index 1.5, is
(a) 40.4 cm (b) 75 cm (c) 50 cm (d) 80 cm
5. How does the focal length of a convex lens change if mono chromatic red light is
used instead of violet light?
(a) Focal length is increased when red light is used
(b) Focal length is decreased when red light is used
(c) Focal length remains same when red light is used
(d) Does not depend on colour of light.

209
KENDRIYA VIDYALAYA SANGATHAN
ERNAKULAM REGION
SAMPLE QUESTION PAPER V
CLASS XII PHYSICS THEORY
TERM II SESSION 2021 - 22
MM: 35 TIME: 2 Hours
…………………………………………………………………………………………..
General Instructions:
(i) There are 12 questions in all. All questions are compulsory.
(ii) This question paper has three sections: Section A, Section B and Section C.
(iii) Section A contains three questions of two marks each, Section B contains eight
questions
of three marks each, Section C contains one case study-based question of five
marks.
(iv) There is no overall choice. However, an internal choice has been provided in
one question
of two marks and two questions of three marks. You have to attempt only one of
the
choices in such questions.
(v) You may use log tables if necessary but use of calculator is not allowed.

SECTION A
1. In half wave rectification, what is the output frequency, if the input frequency is
50 Hz. What is the output frequency of a full wave rectifier for the same input
frequency?
2. Three photo diodes D1, D2, and D3 are made of semiconductors having band
gaps of 2.5eV, 2eV and 3eV respectively. Which one will be able to detect light
of wavelength 6000Å?
3. There is material which absorb photon of shorter wavelengths and emit photon
of longer wavelengths. Can there be stable substance which absorb photon of
larger wavelengths and emit light of shorter wavelength?
OR
The total energy of an electron in the first excited state of hydrogen atom is
about -3.36eV. What is the kinetic energy of the electron in first excited state?
What is the Potential energy of the electron in the first excited state.

210
SECTION B
4. In hydrogen atom an electron undergoes transition from second excited state to
the first excited state and then to the ground state. Identify the spectral series to
which these transitions belong. Find out the ratio of the wavelengths of the
emitted radiation in the two cases.
5. Describe the working of LED. Write the name of semiconductors, which are
preferred to make LED and why? Give two advantages of using LED over
conventional incandescent low power lamp
6. Suppose we think of fission of iron nucleus into two equal fragment of
aluminium. Is the fission energetically possible? Explain. Given mass of
Fe= 55.93494 u and mass of Al = 27.98191 u
7. Two sources of intensity I and 4I are used in an interference experiment. Find the
intensity at a point, where the waves from two sources superimpose with a phase
difference of a) zero b) 900 c) 1800 d) also find the ratio of maximum and
minimum intensities in these cases.
8. A tank is filled with water to a height of 12.5 cm. The apparent depth of a needle
lying at the bottom of the tank is measured by a microscope to be 9.4cm. What is
the refractive index of water? If water is replaced by a liquid of refractive index
1.63 up to the same height, by what distance should the microscope have to be
moved to focus on the needle again?
9. Calculate the radius of curvature of an equi-concave lens of refractive index 1.5,
when it is kept in a medium of refractive index 1.4 to have a power of -5D
OR

Define power of a lens. Write its unit. Deduce the relation 1/f= 1/f1+ 1/f2 for 2
thin lenses kept in contact co-axially.
10. Electric field amplitude of an EM wave Is E0= 120N/C and its frequency Is
50MHz. Determine B0, ω, k, λ. Find expression for E and B

211
OR
Which segment of e.m. wave has highest frequency? How are these produced.
Give one use of these waves. Which e.m. wave lie near the high frequency end
of visible part of e.m. spectrum? Give any one use of this. In what way this
component has harmful effect on human beings.
11. Describe briefly three experimentally observed features in the phenomenon of
photoelectric effect. Discuss briefly how wave theory of light cannot explain
these features

SECTION C
12.

An optical fibre is a thin tube of transparent material that allows light to pass through
it without being refracted into the air or another external medium. It makes use of total
internal reflection. These fibres are fabricated in such a way that light reflected at one
side of the inner surface strike the other at an angle larger than critical angle. Even if
fibre Is bent, light can easily travel along the length.
1) Which of the following Is based on the phenomenon of total internal
reflection of light?
a) Sparkling of diamond
b) Optical fibre communication
c) Instrument used in endoscopy
d) All the above
212
2) A ray of light will undergo total internal reflection if
a) It goes from rarer to denser medium
b) It goes from denser to rarer medium
c) Strike normally at the interface
d) incident at an angle less than critical angle
3) If angle of incidence in the core of optical fibre is equal to critical angle, then
angle of refraction will be
a) 00
b) 450
c) 900
d) 1800
4) In optical fibre relation of refractive index of core and cladding is (core –
inner, cladding-Outer coating ---N for core, n for cladding
a) N>n
b) n>N
c) n=N
d) N+n=1
5) If the value of critical angle is 30 for total internal reflection from a given
optical fibre, then speed of light in that fibre is
a) 3x 10 8 m/s
b) 1.5x 10 8 m/s
c) 6 x 10 8 m/s
d) 4.5x 10 8 m/ s

213
MARKING SCHEME

MARKING SCHEME 1 (SAMPLE QUESTION PAPER I )


Q NO: Answer key Mark

1. Correct diagram 1
Marking of energy level of donor and acceptor atom 1

2. Violet light – photo electric emission and red light no emission 1


Energy of incident radiation is hc/λ . .λv<λg . Hence violet has 1

more energy.
OR
There is no change in the photo electric 1
1
current
Kinetic energy increases
3. D1 forward biased hence resistance 0 ½
D2 reverse biased hence resistance infinity ½

Therefore, resistance = 3Ω
1
I =V/R =2A
4. ITR when i> ic , 45>ic 1
ITR when Sin45 > Sin ic , 1/Sin45 < 1/Sin ic ½
½
ITR when 1.44 < μ
1
Ray diagram (Green & violet do ITR & Red gets refracted.)
OR
½
A light beam incident normally on the face AB is totally reflected to
reach on the face BC if θ≥θc ½
The beam is grazed on face BC, e=900 1
Refractive index of water, n1=4/3 and that of prism, n2=1.5
Snell's law of refraction, n2 sinθc=n1 sine Sinθc = 8/9 1
For total internal reflection sinθ ≥ sinθc
Hence sin θ ≥ 8/9

214
5 Micro waves ½
Range of frequency (3X 1011 to 108 Hz) ½
T low, No Green House Effect ½, ½
Low wavelength for x-ray, so penetrates through matter easily ½, ½
6 Circuit diagram 1
Working 1
Wave form ½
Output frequency = double of input frequency ½

OR
Drift and diffusion of charge and formation of barrier potential 1
Correct diagram 1
Graph 1
7 Slit width, d= 2ʎD/distance 1
d= 0.32mm 1
1
2 points
8 Graph 1
Stopping potential is higher for higher frequency 1
1
Frequency increases then kinetic energy also increases and
thereby increases the stopping potential
9 Ei – Ef = hʋ ½
Derivation 2½

10 Ray diagram 1
Equations among 2 spherical surfaces 1
Assuming object to be at infinity ½
Final equation ½
11 Definition 1
BE = mass defect x c2 ½
Mass defect calculation 1
Calculation ½
12 i)d , (no coherent waves) 1
ii)a, 1
iii)b (Imax / Imin = ( ΓIA + ΓIB )2 ( ΓIA - ΓIB )2 1
iv)d (β’ = β/n) 1
v)b (β = ʎD /d ) 1

215
MARKING SCHEME II (SAMPLE QUESTION PAPER II )
KENDRIYA VIDYALAYA SANGATHAN, ERNAKULAM REGION
SAMPLE PAPER – II: MARKING SCHEME
CLASS : XII MAX MARKS : 35
SUBJECT: PHYSICS TIME: 2 hours
1 Extrinsic property of A : p – type ; B : n - type 1
1

2 The work function is the minimum energy required to eject photo 2


electrons. The electrons in the metal surface belong to various energy
levels. So the they gain different kinetic energies from photons of same
energy.
OR
Energy equation for Balmer series ½
Maximum energy from Balmer series using Rydberg equation = 3.4 eV 1
Maximum work function = 3.4eV ½

3 Photo diode ½
Any one use ½
1
Circuit diagram of photo diode
SECTION B
4 𝜆
𝐴𝑛𝑔𝑢𝑙𝑎𝑟 𝑤𝑖𝑑𝑡ℎ =
𝑎
(i) (a) Become half (b) increases 1
(ii) Diffraction is prominent when the size of the aperture/obstacle 1
is of the order of wavelength. Sound waves have larger
wavelength than that of light. So diffraction takes place with
large obstacle 1
(iii) For second minima d Sinθ=2λ
=24×10−5×10−2×sin30∘
λ=6×10−7m=6000Å

216
5 𝑃 5 𝑚𝑊 2
𝐸= = = 6.25 × 10−19 𝐽 = 3.9eV
𝑛 8 𝑥 1015
1
Work function = E = eVo = 3.9 – 2 = 1.9 eV
6 Derivation with diagram 2
When grazes, angle of refraction = 90o
Using Snell’s law, angle of incidence = 45o 1
OR
Any two differences ( production/uses/detection) 1
λ1 – IR ; λ2 – Radio ; λ3 – Gamma ; λ4 – X-rays 1
λ2 > λ1 > λ4 > λ3 1
7 Interference pattern ½
Experimental set up ½
Derivation 2
8 Nuclear fission ½
Δm=[235.043933−140.9177−91.8954−2(1.008665)] 2½

=0.213503 amu
E=0.21503×931.5MeV
=198.87 MeV
9 Ray diagram, Expression for magnifying power, two correct suggestions 1,1,1
OR
Ray diagram
1
Calculate vo using formula 1/fo=1/vo−1/ uo 2
Magnification, m=vo / uo (1+D/fe) = 30
10 Circuit diagram of p-n junction with forward bias 1
Explanation and definition of knee voltage 2

11 (i) Balmer series ; Lyman series 1


(ii) Wavelength in Balmer series λB = (36/5 ) R 2

Wavelength in Lyman series λL = (4/3 ) R


Ratio = 27 : 5
12 (i) (a) 1
(ii) (a) 1
(iii) (b) 1
(iv) (a) 1
(v) (d) 1

217
MARKING SCHEME III (SAMPLE QUESTION PAPER III )

1 1
1. P=P1+P2 = -2D : F= = = −0.5𝑚
𝑃 −2

Or
𝐴+𝐷
sin( )
2 = 1.5…………….. (1)
𝒏𝒈𝒂 = 𝐴
𝑠𝑖𝑛
2

𝐴+𝐷′
sin( ) 1.5
2
𝒏𝒈𝒘 = 𝐴
=
…..(2)
𝑠𝑖𝑛 1.3
2

Since eqn1 is greater than eqn2: D’ is less than D.


2. Diagram and explanation.
ℎ 1
3. λ= ,λ.∝
√2𝑚𝑒𝑉 𝑚

Electron has less mass and hence its de Broglie wavelength will be more.
4. (a) An electric charge in a plane perpendicular to the direction of propagation of
wave, will be set in motion by the electric and magnetic fields of the wave This
shows that it carries energy and momentum. (1)
(b)Microwaves are produced by special vacuum tubes like klystron/magnetron. When
the frequency of the microwaves matches with the frequency of the water molecules,
energy is efficiently transferred to the kinetic energy of the molecules. (2)
5. RAY DIAGRAM -labelled (2)
For constructing astronomical telescope, the lens of 0.5D should be used as
objective because of its large focal length. Lens of 10 D should be used as eyepiece
because of its smaller focal length.
𝑓0
𝑚=
𝑓𝑒

OR

1 1 1
u= -45 cm v=90 cm − = F=30 cm
𝑣 𝑢 𝐹

U’=-(45+5) =-50cm 1/v’ =1/f +1/u’ =1/30 -1/50 V’=75 cm.


Displacement of image =90-75=15 cm towards the lens

218
6. Definition & 2 conditions
7 .Discussion of YDS and derivation of fringe width.
8. Graph
β=.λ D/a
1. If a is doubled width is halved
2. If wavelength is increased width will increase

9.
Graph between stopping potential and frequency of incident radiation.
Slope of the graph= h/e
Work function=e* intercept on y axis.
10. Graph and explanation.
11. E1=-13.6 eV ,E3=-1.51 eV ,E4=-0.85eV,
(i) KE=-E3 =1.51 eV
(ii)PE=2E4 =-1.7 eV

12. (i) b (ii). a (iii). b (iv). b (v). d

************************************************

219
MARKING SCHEME IV (SAMPLE QUESTION PAPER IV )

Q.NO ANSWER VALUE


POINT
1 hυ = eV0 +W0 1m
4.1 eV + 2.5 eV= 6.6 eV
1m
υ= 6.6.eV/h= 6.6 x 1.6 x10 /6.6x 10
-19 -34 15
= 1.6 x10 Hz
1m
OR
1m
E2 – E1 = hυ -3.4- -13,6 = 10.2 eV
υ= 10.2.eV/h= 10.2 x 1.6 x10-19/6.6x 10-34 = 2.5 x 1015 Hz
2 Photo diode in reverse bias ½ +½
1m
NCERT text book vol-II fig. 14.23
3 In circuit (i) Both D1 and D2 are forward biased hence both will 1m
conduct current and resistance of each diode is “0”.Therefore I = 3/15
= 0.2 A 1m
In circuit(ii) Diode D1 is forward bias and D2 is reverse bias,
therefore resistance of diode D1 is “0” and resistance of D2 is
infinite. Hence D1 will conduct and D2 do not conduct. No current
flows in the circuit.
4 Microwaves, X-rays,γ -rays, UV rays ½ +½+½

λgama < λX < λ uv < λmicro
1m
5 1½ +1½
Diagram + derivation of m= -L/fo(1 +D/fe)

6 Light travel from denser medium to rare medium. 1


a 1
µb = 1/sinC For TIR, I > C, sin i> sin C, 1/sin I < 1/sin C , aµb>
1
1.414
1
green and blue will undergo TIR, red will emerge out
1
OR 1
β = Dλ/d; β ‘= D’λ/d.
β - β’ = (D-D’)λ/d.
λ = (β - β’ )d / (D-D’) = 3 x 10-5 x 10-3 / 5 x 10-2 = 6 x 10-7m = 6000
A0

220
7 1
A locus of points, which oscillate in phase is called a wavefront.
1
NCERT Vol-II ,P.No 357,fig 10.5 1
Proof n1 sin i = n 2 sin r (Derivation)

8 Macroscopic objects in our daily life do not show wave like 1


properties because the wave length associated with them is very small
and beyond the scope of any measurement. 1
ℎ 1 1
𝜆= ,λα
√2𝑚𝐸 √𝑚

mα>mp>me so λα < λp < λe


9 Zero 1
1
T.E = -13.6 eV, K.E = 13.6 eV
1
P.E = -27.2 eV
10 t1/2=10s, λ=.0693/s, τ=1/λ=14.43s, 1
𝑁 6.25 1
=(1/2)t/T = (1/2)2 1
𝑁0 25

t/T =2 or t= 2 x 10 = 20s
11 NCERT vol-II ,P.no-478 Fig.14.10 1
1
Formation of p-n junction
½ +½
1)decreases ii) increases
12 1 1 1 1 1 1 1
1. = (μ -1) ( - ), = (μ -1) ( - ) or μ = 1.33
𝑓 𝑅1 𝑅2 19 30 𝛼
1 1 1 1 1 1 1
2. = + , = + or f2 =60 cm
𝐹 𝑓1 𝑓2 30 10 𝑓2

3.(ii) 1
1 1 1 1 1
4. = (μ -1) ( - ) , = (1.5-1)(2/50)= 1/50, f= 50 cm
𝑓 𝑅1 𝑅2 𝑓
1 1 1
5. f ∝ ,μα so f ∝ λ Focal length increases.
𝜇 𝜆

221
MARKING SCHEME V (SAMPLE QUESTION PAPER V )
CLASS XII PHYSICS THEORY
TERM II SESSION 2021 – 22
………………………………………………………………………………….
1. Output frequency of half wave rectifier -50 Hz ,Frequency of full wave
rectifier= 2 x 50=100Hz
2. Energy of a photon = hv = hc/λ=2.06eV. hence D2 as it has band gap less than
the radiation
3. Not possible for a stable substance. Here energy of the incident photon is low
and the energy of the emitted photon is high. Hence the material has to supply
excess energy needed for the emission which is impossible for a stable
substance.
OR
Kinetic energy = -E =3.36eV, Potential energy =2E = -6.72eV

4. First transition – Balmer , second transition – Lyman


λ1 = hc /E = hc/ 1.9
λ2 = hc /E =hc /10.2
λ1/λ2=10.2/1.9 =5.3
5. Working of LED. Semiconductor with appropriate band gap close to 1.5eV is
preferred. Material should have high optical absorption, availability, low cost.
6. E = mc 2
E =∆ m x 931MeV
∆m = - 0.2888 Hence fission is not possible
7. I = I1+I2+2√ I1I2 Cos ᵠ
ᵠ =0 Resultant intensity =9I
ᵠ =90 Resultant intensity = 5I
ᵠ =180 Resultant intensity = I
Ratio of maximum to minimum = 9:1

222
8. Refractive index of water = real depth/ apparent depth = 12.5/9.4 =1.33
1.63 =12.5/ apparent depth
Apparent depth =12.5/1.63 =7.67cm
Therefore microscope Is to be shifted by 9.4-7.67 =1.73cm
9. Use lens makers formula and solve R = 2.86 cm
Or
Definition, unit, Derivation with fig
10. B0 =E0/c =40x 10-8 =400 nanotesla
W =2∏√ =2 x 3.14 x 50x 10 6
K =w /c =1.05 rad/m
λ= c/√ =6m
E = E0 sin(kx-wt)
B = B0 sin(kx-wt) substitute for k and w
Or gamma ray , production and use, uv- any one use and harmful effect
11. Explain ---Variation of photocurrent with intensity
---------------------Kinetic energy of emitted electron increases with frequency
No time lag between incidence of radiation and emission of photoelectron
According to wave theory absorption of energy by electrons take place
continuously over the entire wave front of the radiation. Hence it will take hour
for a single electron to come out of the metal which contradict the experimental
fact that photoelectron emission is instantaneous.
12. 1)d 2) b 3) c 4) a 5) sin 30 =v /c ,ans. b

*******************************************************

223
KENDRIYA VIDYALAYA (RO)
CHANDIGARH REGION
CLASS XII
PHYSICS

1
INDEX
CHAPT GIST PAGES
ER
8 Electromagnetic Waves 1-36
i) Gist, MLM 1-8
ii) One mark Questions 8-9
iii) Two mark Questions 9-12
iv) Three mark Questions 12-14
v) MCQ 14-19
vi) Assertion Reason 19-21
vii) Case Study Based 21-26
viii) Answer key 26-36
9 Ray Optics and Optical Instruments 37-84
i) Gist, MLM 37-41
ii) One mark Questions 41-43
iii) Two mark Questions 43-46
iv) Three mark Questions 46-52
v) MCQ 52-59
vi) Assertion Reason 59-62
vii) Case Study Based 62-70
viii) Answer key 70-84
10 Wave Optics 85-136
i) Gist, MLM 85-94
ii) One mark Questions 95-96
iii) Two mark Questions 96-99
iv) Three mark Questions 99-104
v) MCQ 104-111
vi) Assertion Reason 112-115
vii) Case Study Based 115-119
viii) Answer key 119-136
11 Dual nature of Radiation and matter 137-181
i) Gist, MLM 137-144
ii) One mark Questions 144-146

2
iii) Two mark Questions 146-150
iv) Three mark Questions 150-156
v) MCQ 156-162
vi) Assertion Reason 162-164
vii) Case Study Based 164-169
viii) Answer key 169-181
12 & 13 Atom & Nuclei 182-212
i) Gist, MLM 182-191
ii) One mark Questions 191-192
iii) Two mark Questions 192-192
iv) Three mark Questions 193-193
v) MCQ 193-199
vi) Assertion Reason 199-202
vii) Case Study Based 202-204
viii) Answer key 204-212
14 Semiconductor- Electronics: 213-274
Material devices and simple
circuits
i) Gist, MLM 213-221
ii) One mark Questions 221-223
iii) Two mark Questions 223-229
iv) Three mark Questions 230-232
v) MCQ 232-238
vi) Assertion Reason 238-241
vii) Case Study Based 241-253
viii) Answer key 254-274

3
Chapter 8: Electromagnetic Waves
Gist:
1. Electromagnetic Waves An electromagnetic wave is a wave
radiated by an accelerated or oscillatory charge in which
varying magnetic field is the source of electric field and
varying electric field is the source of magnetic field. Thus
two fields becomes source of each other and the wave
propagates in a direction perpendicular to both the fields.
2. Electromagnetic waves are transverse in nature, i.e. electric
and magnetic fields are perpendicular to each other and to
the direction of wave propagation. Electromagnetic waves
are not deflected by electric and magnetic fields.

3. E (electric field) and B (magnetic field) in


electromagnetic waves are in same phase.
8. Speed of electromagnetic wave

4. The energy in electromagnetic wave is divided on


average equally between electric and magnetic fields.
5. Energy associated with an electromagnetic wave is

6. Linear momentum delivered to the surface, p= U/c,


where, U = total energy transmitted by electromagnetic
waves and c = speed of electromagnetic wave.

4
7. Electromagnetic Spectrum The systematic sequential
distribution of electromagnetic waves in ascending or
descending order of frequency or wavelength is known as
electromagnetic spectrum. The range varies from 10-12 m,
to 104 m, i.e. from γ-rays to radio waves.

8. Elementary facts about the uses of electromagnetic waves


Radio waves
(i) In radio and TV communication.
(ii) In astronomical field.
Microwaves
(i) In RADAR communication.
(ii) In analysis of molecular and atomic structure.
5
(iii) For cooking purpose.
Infrared waves
(i) In knowing molecular structure. (ii) In remote control
of TV VCR, etc.
Ultraviolet rays
(i) Used in burglar alarm. (ii ) To kill germs in minerals.
X-rays
(i) In medical diagnosis as they pass through the muscles
not through the bones.
(ii) In detecting faults, cracks, etc., in metal products,
γ-rays
(i) As food preservation. (ii) In radiotherapy.
9. Properties of Electromagnetic Waves:
1. Variations in both electric and magnetic fields occur
simultaneously. Therefore, they attain their maxima and
minima at the same place and at the same time.
2. The direction of electric and magnetic fields are mutually
perpendicular to each other and as well as to the direction
of propagation of wave.
3. The electric field vector E and magnetic field vector B are
related by
c = E0 / B0
where E0 and B0 are the amplitudes of the respective fields
and c is speed of light.
4. The velocity of electromagnetic waves in free space, c =
1 / √μ0ε0
5. The velocity of electromagnetic waves in a material
medium = 1 / √με
where μ and ε are absolute permeability and absolute
permittivity of the material medium.
6. Electromagnetic waves obey the principle of
superposition.
6
7. Electromagnetic waves carry energy as they propagate
through space. This energy is divided equally between
electric and magnetic fields.
8. Electromagnetic waves can transfer energy as well as
momentum to objects placed on their paths.
9. For discussion of optical effects of EM wave, more
significance is given to Electric Field, E. Therefore,
electric field is called ‗light vector‘.
10. Electromagnetic waves do not require material
medium to travel.
11. An oscillating charge which has non-zero
acceleration can produce electromagnetic waves.

EM
S. No Range of λ Range of ν Source Uses Detection
Wave
A few km A few Hz
Radio Oscillating Radio and TV Receiver‘s
1 to to
Wave electronic circuits broadcasting aerials
0.3 m 109 Hz
Radar, analysis of
fine details of
0.3 m 109 Hz
Micro Oscillating atomic and Point contact
2 to to
wave electronic circuits molecular diodes
10-3 m 3 x 1011 Hz
structures &
Microwave oven
Industry,
medicine,
Thermopile,
astronomy, night
10-3 m 3 x 1011 Hz Bolometer,
Infra Red Molecules and vision device,
3 to to infrared
waves hot bodies green house,
7.8 x 10-7 m 4 x 1014 Hz photographic
revealing secret
film
writings on
ancient walls, etc.
Optics and
Atoms and The eye,
7.8 x 10-7 m 4 x 1014 Hz Optical
Visible molecules when photocell,
4 to to Instruments,
rays electrons are photographic
3.8 x 10-7 m 8 x 1014 Hz Vision,
excited films
photography, etc.
Medical
Atoms and
application,
3.8 x 10-7 m 8 x 1014 Hz molecules in Photodiodes,
Ultra sterilization,
5 to to electrical photographic
violet killing bacteria
6 x 10-10 m 3 x 1017 Hz discharges and films
and germs in food
Sun
stuff, detection of

7
invisible writing,
forged
documents, finger
print, etc.
X-ray
photography,
treatment of
cancer, skin
disease & tumor,
locating cracks
10-9 m 3 x 1017 Hz Inner or more Photographic
and flaws in
6 X- Rays to to tightly bound films, Geiger
finished metallic
6 x 10-12 m 5 x 1019 Hz electrons in atoms tube
objects, detection
of smuggled
goods in bags of a
person, study of
crystal structure,
etc.
They overlap Information about
Photographic
the upper limit 3 x 1018 Hz structure of
Gamma Radioactive films,
7 of the X-Ray. to nuclei,
rays substances ionisation
10-10 m to 3 x 1022 Hz astronomical
-14 chamber
10 m research, etc.

Short Answer Question (1 marks)


1. Name the part of the electromagnetic spectrum of
wavelength 10-2 m and mention its one application.
2. Write the following radiations in ascending order in
respect of their frequencies ;
X-rays, Microwaves, UV rays and radiowaves.
3. Name the part of electromagnetic spectrum which is
suitable for
(a) radar systems used in aircraft navigation
(b)treatment of cancer tumours.
4. Name the EM waves used for studying crystal structure
of solids. What is its frequency range?
5. Which part of electromagnetic spectrum has largest
penetrating power?
6. Which part of electromagnetic spectrum is absorbed from
sunlight by ozone layer?

8
7. Which part of electromagnetic spectrum is used in radar
systems?
8. Name the part of electromagnetic spectrum whose
wavelength lies in the range of 10-10 m. Give its one use.
9. Which of the following has the shortest wavelength :
Microwaves, Ultraviolet rays, X-rays.
10. Arrange the following in descending order of
wavelength :
X-rays, Radio waves, Blue light, Infrared light.
11. A plane electromagnetic wave travels in vacuum
along z-direction. What can you say about the direction
of electric and magnetic field vectors?
12. A plane electromagnetic wave travels in vacuum
along x-direction. What can you say about the direction
of electric and magnetic field vectors?
13. A plane electromagnetic wave travels in vacuum
along y-direction. What can you say about the direction
of electric and magnetic field vectors?
14. How are radio waves produced?
15. How are X-rays produced?
16. How are microwaves produced?
17. Name the physical quantity which remains same for
microwaves of wavelength 1 mm and UV radiations of
1600 Å in vacuum.
18. What are the directions of electric and magnetic field
vectors relative to each other and relative to the direction
of propagation of electromagnetic waves?
19. The speed of an electromagnetic wave in a material
medium is given by v=1√με,μ the permeability of the
medium and ε its permittivity. How does its frequency
change?
Short Answer Type SA-(2 Marks)
20. Welders wear special goggles or face masks with
glass windows to protect their eyes from electromagnetic

9
radiations. Name the radiations and write the range of
their frequency.
21. To which part of the electromagnetic spectrum does
a wave of frequency 5 × 1019 Hz belong?
22. To which part of the electromagnetic spectrum does
a wave of frequency 3 × 1013 Hz belong?
23. Why are microwaves considered suitable for radar
systems used in aircraft navigation?
24. The oscillating magnetic field in a plane
electromagnetic wave is given by
By = (8 × 10-6) sin [2 × 10-11 t + 300 π x] T
(i) Calculate the wavelength of the electo-magnetic wave.
(ii) Write down the expression for the oscillating electric
field.
25. The oscillating electric field of an electromagnetic
wave is given by :
E = 30 sin [2 × 1011 t + 300 π x] Vm-1
(a) Obtain the value of the wavelength of the
electromagnetic wave.
(b) Write down the expression for the oscillating
magnetic field.
26. How does a charge q oscillating at certain frequency
produce electromagnetic waves? Sketch a schematic
diagram depicting electric and magnetic fields for an
electromagnetic wave propagating along the Z-direction.
27. Arrange the following electromagnetic radiations in
ascending order of their frequencies:
(i) Microwave
(ii) Radiowave
(iii) X-rays
(iv) Gamma rays
Write two uses of any one of these.
28. Draw a sketch of a plane electromagnetic wave
propagating along the z-direction. Depict clearly the

10
directions of electric and magnetic fields varying
sinusoidally with z.
29. How are infrared waves produced? Why are these
referred to as ‗heat waves‘? Write their one important
use.
30. (a) An em wave is travelling in a medium with a
velocity v→=vi^. Draw a sketch showing the propagation
of the em wave, indicating the direction of the oscillating
electric and magnetic fields.
(b) How are the magnitudes of the electric and magnetic
fields related to the velocity of the em wave?
31. (a) How are electromagnetic waves produced?
(b) How do you convince yourself that electromagnetic
waves carry energy and momentum?
32. (a) Arrange the following electromagnetic waves in
the descending order of their wavelengths :
1. Microwaves
2. Infra-red rays
3. Ultra-violet radiation
4. Gamma rays
(b) Write one use each of any two of them.
33. Name the types of e.m. radiations which
1. are used in destroying cancer cells,
2. cause tanning of the skin and
3. maintain the earth‘s warmth.
Write briefly a method of producing any one of these
waves.
34. For a plane electromagnetic wave, propagating along
the Z-axis, write the two (possible) pairs of expression for
its oscillating electric and magnetic fields. How are the
peak values of these (oscillating) fields related to each
other?
35. An e.m. wave, Y1, has a wavelength of 1 cm while
another e.m. wave, Y2, has a frequency of 1015 Hz. Name

11
these two types of waves and write one useful application
for each.
36. Identify the electromagnetic waves whose
wavelengths vary as
(a) 10-12 < λ < 10-8 m
(b) 10-3 m < λ < 10-1 m
Write one use for each.
37. Identify the electromagnetic waves whose
wavelengths lie in the range
(a) 10-11 m < λ < 10-8 m
(b) 10-4 m < λ < 10-6 m Write one use of each.
Long Answer Type LA-(3 Marks)
38. Identify the following electromagnetic radiations as
per the wavelengths given below. Write one application
of each.
(a) 10-3 nm
(b) 10-3 m
(c) 1 nm
39. Identify the following electromagnetic radiations as
per the wavelengths given below. Write one application
of each.
(a) 1 mm
(b) 10-12 m
(c) 10-8 m
40. (a) When the oscillating electric and magnetic fields
are along the x- and indirection respectively
1. point out the direction of propagation . of electromagnetic
wave.
2. express the velocity of propagation in terms of the
amplitudes of the oscillating electric and magnetic fields.
(b) How do you show that the em wave carries energy and
momentum
41. Answer the following :
(a) Name the em waves which are suitable for radar
systems used in aircraft navigation. Write the range of
12
frequency of these waves.
(b) If the. earth did not have atmosphere, would its
average surface temperature be higher or lower than what
it is now? Explain.
(c) An em wave exerts pressure on the surface on which it
is incident. Justify.
42. Answer the following :
(a) Name the em waves which are used for the treatment
of certain forms of cancer. Write their frequency range.
(b) Thin ozone layer on top of stratosphere is crucial for
human survival. Why?
(c) Why is the amount of the momentum transferred by
the em waves incident on the surfrace so small?
43. Answer the following questions :
(a) Name the em waves which are produced during
radioactive decay of a nucleus. Write their frequency
range.
(b) Welders wear special glass goggles while working.
Why? Explain.
(c) Why are infrared waves often called as heat waves?
Give their one application.
44. Answer the following questions:
(i) Show, by giving a simple example, how em waves
carry energy and momentum.
(i) How are microwaves produced? Why is it necessary in
microwave ovens to select the frequency of microwaves
to match the resonant frequency of water molecules?
(iii) Write two important uses of infrared waves.
45. Name the parts of the electromagnetic spectrum
which is
(a) suitable for radar systems used in aircraft navigation.
(b) used to treat muscular strain.
(c) used as a diagnostic tool in medicine.
Write in brief, how these waves can be produced.

13
46. Name the e.m. waves in the wavelength range 10 nm
to 10-3 nm. How are these waves generated? Write their
two uses.
47. Name the type of e.m. waves having a wavelength
range of 0.1 m to 1 mm. How are these waves generated?
Write their two uses.
48. Name the type of e.m. waves having a wavelength
range 10-7 m to 10-9 m. How are these waves generated?
Write their two uses.
49. To which part of the electromagnetic spectrum does
a wave of frequency 5 x 1019 Hz belong?
50. To which part of the electromagnetic spectrum does
a wave of frequency 3 x 1013 Hz belong?
51. Write three uses of microwaves.
52. Write three uses of infrared rays.
53. Write three uses of X-rays.
What is the frequency of electromagnetic waves produced
by oscillating charge of frequency V=105Hz?

Multiple Choice Questions


1. One requires 11 eV of energy to dissociate a carbon
monoxide molecule into carbon and oxygen atoms. The
minimum frequency of the appropriate electromagnetic
radiation achieve the dissociation lies in
(a ) visible region (b ) infrared region
(c ) ultraviolet region (d ) microwave region
2. A plane electromagnetic wave travelling along X – axis
has a wave length 10.0 mm. The electric field points
along Y- direction and has a peak value of 30 V/M. Then
the magnetic field in terms of x in metre and t in second
may be expressed as
(a) 30 sin 200 ( )
(b) 10-7 sin 200 ( )
(c ) 30 sin 2 ( )
14
(d) 10-7 sin 2 ( )
3. A linearly polarised electromagnetic wave given as E =
E0 cos ( kz – ωt)i is incident normally on a perfectly
reflecting infinite wall at z = a . Assuming that the
material of the wall is optically inactive, the reflected
wave will be given as
(a) Er = E0 cos ( kz – ωt)i
(b) Er = E0 cos ( kz + ωt) i
(c ) Er = - E0 cos ( kz + ωt) i
(d ) Er = - E0 sin (kz – ωt) i
4. Light with an energy flux of 20 W/cm2 falls on a non–
reflecting surface at normal incidence. If the surface has
an area of 30 cm2, the total momentum delivered for
complete absorption during 30 minutes is
(a ) 36 x 10 -5 kg m/s (b ) 36 x 10 -4 kg m/s
4
(c ) 108 x 10 kg m/s (d ) 1.08 x 10 7 kg m/s
5. The part of electromagnetic spectrum belonging to 2.7 K
is
(a ) lnfrared (b ) Ultraviolet
(c ) X – rays (d ) Microwaves
6. The correct option, if speeds of gamma rays, X-rays and
microwave are Vg, Vx an Vm respectively will be.
(a) Vg > Vx > Vm
(b) Vg < Vx < Vm
(c) Vg > Vx > Vm
(d) Vg = Vx = Vm
7. The electric intensity produced by the radiations coming
from 100w bulb at a 3 m distance is E. The electric field
intensity produced by the radiations coming from 50 W
bulb at the same distance is
(a ) E/2 (b) 2E (c ) E √ (d ) √ E
8. If E and B represent electric and magnetic field vectors of
the electromagnetic wave, the direction of propagation of
electromagnetic wave is along.
(a ) E (b ) B (c ) B X E (d ) E X B
15
9. An electromagnetic wave radiates outwards from a dipole
antenna, with E0 as the amplitude of its electric field
vector . The electric field E0 which transports significant
energy from the source fall of as
(a ) 1/r3 (b )1/r2 (c )1/r (d ) 1/r2
10. A plane electromagnetic wave propagating along x
direction can have the following pairs of E and B
(a) Ez , BY (b ) EY , BZ(c ) Ex , BY (d ) both a and b
11. The ratio of contributions made by the electric field
and magnetic field components to the intensity of an em
wave is
(a) c : 1 (b) c2: 1 (c) 1:1 (d) √
12. Which of the following has maximum penetrating
power?
(a) Ultraviolet radiation
(b) Microwaves
(c) γ-rays
(d) Radio waves
13. Which of the following is called heat radiation?
(a) X-rays
(b) γ-rays
(c) Infrared radiation
(d) Microwave
14. Electromagnetic waves are transverse in nature is
evident by
(a) polarisation.
(b) interference,
(c) reflection.
(d) diffraction
15. Which of the following are not electromagnetic
waves?
(a) Cosmic rays
(b) γ-rays
(c) β-rays
(d) X-rays
16
16. 10 cm is a wavelength corresponding to the spectrum
of
(a) infrared rays
(b) ultraviolet rays
(c) microwaves
(d) X-rays
17. The condition under which a microwave over heats
up a food item containing water molecules most
efficiently is
(a) The frequency of the microwaves must match the
resonant frequency of the water molecules.
(b) The frequency of the microwaves has no relation with
natural frequency of the water molecules.
(c) Microwaves are heat waves, so always produce
heating.
(d) Infrared waves produce heating in a microwave oven.
18. An electromagnetic wave can be produced, when
charge is
(a) moving with a constant velocity
(b) moving in a circular orbit
(c) falling in an electric field
(d) both (b) and (c)
19. Which of the following has/have zero average value
in a plane electromagnetic wave?
(a) Both magnetic and electric fields
(b) Electric field only
(c) Magnetic field only
(d) None of these
20. A charged particle oscillates about its mean
equilibrium position with a frequency of 109 Hz. The
frequency of electromagnetic waves produced by the
oscillator is
(a) 106 Hz (b) 107 Hz
8
(c) 10 Hz (d) 109 Hz
21. The wavelength of signal weather frequency of 300
17
MHz is
(a) 2m (b) 20 m
(c ) 10m (d) 1 m
22. Human body radiates
(a) microwavws (b) X-rays
(c) infrared rays (d) gamma rays
23. The phase difference between electric field and
magnetic field in an em wave is
(a) 0 (b)
© (d)
24. The structure of solids is investigated by using
(a) cosmic rays (b) X-rays
(c) γ-rays (d) infrared rays
25. The condition under which a microwave over heats
up a food item containing water molecules most
efficiently is
(a) The frequency of the microwaves must match the
resonant frequency of the water molecules.
(b) The frequency of the microwaves has no relation with
natural frequency of the water molecules.
(c) Microwaves are heat waves, so always produce
heating.
(d) Infrared waves produce heating in a microwave oven.
26. Which radiations are used in treatment of muscle
ache?
(a) Infrared (b) Ultraviolet
(c) Microwave (d) X-rays
27. Waves in decreasing order of their wavelength are
(a) X-rays, infrared rays, visible rays, radio waves
(b) radio waves, visible rays, infrared rays, X-rays.
(c) radio waves, infrared rays, visible rays, X-rays.
(d) radio waves, ultraviolet rays, visible rays, X-rays.
28. The electric field associated with an e.m wave in
vacuum is given by =40 cos (kz – 6 × 108t) , where
E, Z and t are in volt/m, metre and seconds respectively.
18
The value of wave vector K is
(a) 2 m-1 (b) 0.5 m-1 (c) 6 m-1 (d) 3 m-1
29. Electromagnetic waves with wavelength λ are used
by a FM radio station for broadcasting. Here λ belongs to
(a) radio waves (b) VHF radio waves
(c) UHF radio waves (d) microwaves
30. The oscillating magnetic field in a plane
electromagnetic wave is given as By = (8 × 10-6) sin [2 ×
1011t+ 300πx] T, wavelength of the em wave is
(a) 0.80 cm (b) 1 × 103m
(c) 2 × 10-2cm (d) 0.67 cm
Assertion Reason Type questions
Two statements are given- one labelled
Assertion & other labelled Reason.
Select the correct answer to these
questions from the codes (a), (b), (c),
(d) are given below
(a) Both A & R are true & R is the correct explanation of A
(b) Both A & R are true & R is not the
correct explanation of A
(c) A is true but R is false
(d) A is false but R is also false
1. Assertion : Electromagnetic wave are transverse in
nature.
Reason : The electric and magnetic fields in
electromagnetic waves are perpendicular to each other
and the direction of propagation.
2. Assertion : Electromagnetic waves interact with matter
and set up oscillations.
Reason : Interaction is independent of the wavelength of
the electromagnetic wave.
3. Assertion : Electromagnetic waves carry energy and
momentum.
Reason : Electromagnetic waves can be polarised.
19
4. Assertion : Electromagnetic waves exert radiation
pressure.
Reason : Electromagnetic waves carry energy.
5. Assertion : The electromagnetic wave is transverse in
nature.
Reason : Electromagnetic wave propagates parallel to the
direction of electric and magnetic fields.
6. Assertion : The velocity of electromagnetic waves
depends on electric and magnetic properties of the
medium.
Reason : Velocity of electromagnetic waves in free space
is constant.
7. Assertion : The basic difference between various types of
electromagnetic waves lies in their wavelength or
frequencies.
Reason : Electromagnetic waves travel through vacuum
with the same speed.
8. Assertion : Microwaves are better carrier of signals than
optical waves.
Reason : Microwaves move faster than optical waves.
9. Assertion : Infrared radiation plays an important role in
maintaining the average temperature of earth.
Reason : Infrared radiations are sometimes referred to as
heat waves
10. Assertion : Dipole oscillations produce
electromagnetic waves.
Reason: Accelerated charge produces electromagnetic
waves.
11. Assertion: The frequencies of incident, reflected and
refracted beam of monochromatic light incident from one
medium to another are same.
Reason: The incident, reflected and refracted rays are
coplanar.
12. Assertion: Sound waves can not travel in vacuum but
light waves can travel in vacuum.
20
Reason: Sound waves are longitudinal waves and can not be
polarized but electromagnetic waves are polarized and
can be polarized.
13. Assertion: Radio waves can be polarized.
Reason: Sound waves in air are longitudinal in nature.
14. Assertion: The earth without its atmosphere would
be inhospitably cold.
Reason: All heat would escape in the absence of atmosphere.
15. Assertion: EM Waves are transverse in nature
Reason: The electric and magnetic fields of an em wave
perpendicular to each other and also perpendicular to the
direction of wave propagation.
16. Assertion : X-Rays astronomy is possible only from
satellites orbiting the earth.
Reason: Efficiency of X-rays telescope is large as
compared to any other telescope.
17. Assertion: In em waves electric field and magnetic
field lines are perpendicular to each other.
Reason: Electric field and magnetic field are self
sustaining.
18. Assertion: Electromagnetic radiations exert pressure.
Reason: Electromagnetic radiations carry both
momentum and energy.
19. Assertion: UV radiation cause damage to the ozone
layer.
Reason: Ozone hole is resulting in global warming and
climate change.
20. Assertion: When a charged particle moves in a
circular path, it produces em wave.
Reason: Charged particle has acceleration.

CASE STUDY BASED QUESTIONS


I. Gamma rays are used in radiotherapy to treat cancer.
They spot the tumors. They kill the living cells and
damage the malignant tumor. The gamma knife machine
21
is used for this purpose. The patient‘s head is enclosed in
a helmet which focuses the narrow beam of gamma rays
to target a tumor in the brain.
1. What is the source of gamma rays?
a. Radioactive decay of nucleus
b. Hot bodies and molecules
c. Klystron valve
d. Accelerated motion of charged particles in a wire
2. What is the other use of gamma rays?
a. In aircraft navigation
b. In telecommunications
c. In sterilization of surgical items
d. In haze photography
3. Which one of the following option has correct radiation
order?
a. Gamma> alpha> beta
b. Alpha> beta > gamma
c. Beta> alpha >gamma
d. Gamma>beta > alpha
4. Chose the correct order of increasing wavelength
a. X- rays<gamma rays< radiowaves
b. X- rays < radiowaves< gamma rays
c. Radiowaves< X- rays<gamma rays
d. X- rays<gamma rays< microwaves
II. All the known radiations from big family of em waves
which stretch over a large range of wavelengths. The
orderly distribution of em waves in accordance with their
wavelength or frequency into distinct groups having
widely differing properties is called em spectrum. The
main aprts of the em spectrum are gamma rays, x-rays,
UV rays, visible light, infra red waves, microwaves and
radiowaves in order of increasing wavelength.
1. The correct option, if speeds of gamma rays, X-rays and
microwave are Vg, Vx an Vm respectively will be.
(a) Vg > Vx > Vm
22
(b) Vg < Vx < Vm
(c) Vg > Vx > Vm
(d) Vg = Vx = Vm
2. The structure of solids is investigated using
(a) Cosmic rays
(b)X-rays
(c) Gamma rays
(d)Infra red radiation
3. Microwave oven acts on the principle of
(a) Giving rotational energy to water molecules
(b)Giving vibrational energy to water molecules
(c) Giving translational energy to water molecules
(d)Transferring electrons from lower to higher energy
levels in water molecule
4. The condition under which a microwave oven heats up a
food item containing water molecules most efficiently, is
(a) The frequency of microwaves must match the
resonant frequency of water molecules.
(b) Infrared radiations produce heating in a microwave
oven.
(c) Microwaves are heat waves, so always produce
heating.
(d) The frequency of the microwaves has no relation
with the frequency of water molecules.

III. X-rays are a form of em radiation, similar to visible


light. Unlike, visible light, however X-rays have higher
energy and pass through the most of the objects,
including the body. Medical X-rays are used to generate
images of tissues and structures inside the body.
1. What is the most common way of production of X-rays?
(a) Radioactive decay of nucleus
(b)Vibration of atoms and molecules
(c) Bombardment of metal by high energy electrons
(d)Magnetron valve
23
2. Which of the following equipments/instrument can detect
X-rays?
a. Photocells, photographic film
b. Thermopiles, bolometer
c. Photographic film, Geiger tube
d. Geiger tube, human eye
3. Where do X-rays fall in em spectrum?
a. Between UV region and infrared region
b. Between infrared and microwaves
c. Between microwaves and radiowaves
d. Between gamma rays and UV region

4. What is the use of rays lying beyond X-rays region in the


em spectrum?
a. Used to kill microbes
b. Used to detect heat loss in the insulated systems
c. Used in standard broadcast radio and television
d. Used in oncology, to kill cancerous cells

IV. Green house effect is a natural process that warms


the Earth‘s surface. When the sun‘s energy reaches the
earth‘s atmosphere, some of it is reflected back to space
and rest is absorbed and re-radiated by the green house
gases.The absorbed energy warms the atmosphere and
the surface of earth.
1. Which rays are known as heat waves?
a. gamma rays
b. radio waves
c. infra red
d. X-rays
2. Which of the following is not considered as naturally
occurring green house gas?
a. Metane
b. CFCs
24
c. Carbon monoxide
d. Nitrous oxide
3. Which of the following is the best method for the
production of infrared rays?
(a) Radioactive decay of nucleus
(b)Vibration of atoms and molecules
(c) Bombardment of metal by high energy electrons
(d)Magnetron valve
4. Infrared radiations are the waves of _____ wavelength.
a. Shorter
b. Longer
c. Infinite
d. Zero
V. Microwaves are used in aircraft navigation. A radar guns
out a short bursts of microwaves and it reflects back from
the oncoming aircraft and are detected by the receiver
gun. The frequency of the reflected wave is used to
compute the speed of incoming aircraft.
1. How are microwaves produced?
a. Radioactive decay of nucleus
b. Vibration of atoms and molecules
c. Bombardment of metal by high energy electrons
d. Magnetron and klystron valve
2. Which of the following is the use of microwaves?
a. In treatment of cancer
b. To observe change in blood flow
c. To kill microbes
d. Studying details of atoms and molecules
3. Why are microwaves used for aircraft navigation?
a. Due to low wavelength
b. Due to high wavelength
c. Due to lower frequency
d. Due to their frequency modulation power
4. Where do microwaves fall in em spectrum?
a. Between UV region and infrared region
25
b. Between infrared and visible
c. Between infrared and radiowaves
d. Between gamma rays and UV region
Answer Key (1 Mark)
1. Name of the part: Microwave
Applications :It is used in radar communication, It is used
in microwave ovens, It is also used in analysis of fine
details of molecular and atomic structure.
2. Radiowaves, microwaves, UV-rays and X-rays.
3. (1) Micro-waves (2) Gamma-rays
4. X-rays frequency range : 1017 Hz to 1020 Hz
5. γ-rays are the electromagnetic waves of frequency range
3 × 1018 Hz to 5 × 1022 Hz and have the highest
penetrating power.
6. Ultraviolet rays are absorbed from sunlight by ozone
layers.
7. Microwave region of electromagnetic spectrum is used in
radar systems.
8. Name : X-rays
Use : In medical diagnosis to look for broken bones;
treatment study of crystal structure.
9. X-rays have the shortest wavelength.
10. Decreasing order ➝ Radio waves, Infrared light,
Blue light, X-rays.
11. The direction of electric field vector is along X-axis.
Magnetic field vector is along Y-axis.
12. The electric field and magnetic field vectors are in
YZ-plane in the Y-direction and Z-direction respectively.
13. The electric field and magnetic field vector are in
ZX-plane in the X-direction and Z-direction respectively.
14. Radio waves are produced by the accelerated motion
of charges in conducting wires.
15. X-rays are produced by sudden deceleration or
acceleration of electrons in an X-ray tube.

26
16. Microwaves are produced by Klystron valve or
magnetron valve.
17. Speed/Velocity of light remains the same.
18. The oscillations of E→ and B→ fields are
perpendicular to each other as well as to the direction of
propagation of the wave.
19. Frequency remains unchanged
Answer Key (2 Marks)
20. The name of radiations is ultraviolet radiation. Its
frequency range is 1015 to 1017 Hz.
21. A wave of frequency 5 × 1019 Hz belongs to γ-rays
region of electromagnetic spectrum.
22. Infra-red region of electromagnetic spectrum.
23. Due to their short wavelengths, microwaves are
considered suitable for radar systems in aircraft
navigation.
24. Given: By = 8 × 10-6 sin [2 × 1011 t + 300 π x] T
(i) Standard equation is,

The oscillations of E→ and B→ fields are perpendicular to


each other as well as to the direction of propagation of the
wave. So we take electric field in z-direction because
oscillating magnetic field is in y-di recti on and
propagation of the wave is in x-direction.
27
25. (a) We compare the given expression with

26. As the charge q moves accelerating, the electric field


and magnetic field produced will change the space and
time E and B varying with time produced the other field
B and E respectively and sustain the E.M. pattern.This is
from the interpretation of Maxwell supported by

27. In ascending order of their frequencies :


Radiowave < Microwave < X-rays < Gamma rays.
Two uses of microwaves are :
1. In microwave ovens.
2. In aircraft navigation.
28. Sketch of a plane electromagnetic wave propagating
along the z-direction with oscillating electric field E
along the x-direction and the oscillating magnetic field B
along the y-direction.

28
29. Infrared rays are produced by hot bodies and
molecules. This may involve vibration and bending of
molecules. Infrared band lies adjacent to low-frequency
or long-wavelength end of the visible spectrum. Infrared
waves are sometimes referred to as heat waves.
Use: Infrared rays are used to take photographs in darkness.
These are also used to study secret writing. They are also
used in physical therapy.
30.

31. (a) Electromagnetic Waves : Accelerating electric


charge produces electromagnetic waves.
(b) Einstein‘s explanation of photoelectric effect led de
Broglie to the wave-particle duality, i.e., matter exhibits
wave as well as particle properties.
Electromagnetic waves are characterised by wave
properties, such as periodicity in space-time, wavelength,
amplitude, frequency, wave velocity etc. It transports
energy but no matter.
The term wave-particle duality refers to the behaviour where
both wave-like and particle-like properties are exhibited
under different conditions by the same entity. Hence
electromagnetic waves show particle properties such as
definite position, size, mass, velocity, momentum, energy
etc.For a photon of momentum (p), an associated
wavelength is given by λ=hp.
32. (a) Arrangement:

29
1. Microwaves
2. Infra-red rays
3. Ultra-violet radiation
4. Gamma rays
(b) Uses :
1. Microwaves are used in radar system.
2. Infra-red rays are used for protecting dehydrated fruits.
3. Ultra-violet rays are used in the study of molecular
structure.
4. Gamma rays are used to kill micro-organisms in food
industry
33.
1. γ-rays
2. Ultraviolet rays
3. Infrared rays
Mode of production
1. γ-rays are produced by radioactive decay of nucleus.
2. Ultraviolet rays are produced when inner shell electrons
in atoms move from one energy level to an other energy
level.
3. Infrared rays are produced due to vibration of atoms and
molecules.
34. For the e.m. wave, propagating along the z-axis, we
have

The two possible forms for electric and magnetic fields


are :

The peak values of these two fields are related by

30
35. (i) Y1 ➝ Microwaves
Applications : Microwaves are used in Microwave ovens,
Aircraft Navigators etc.
(ii) Y2 ➝ Ultraviolet waves
Applications : Ultraviolet rays are used in sterilizing
surgical instruments, food preservation etc.
36. (a) X-rays—Used in medical science for the purpose
of detection of fractures, stones in gall bladder, stones in
kidney etc.
(b) Microwaves—Used in radar systems for aircraft
navigation.
37. (a) Uses of X-Rays and Gamma rays :
X-rays are used as a diagnostic tool in medicine and as a
treatment for certain forms of cancer. Gamma rays are
used in medicine to destroy cancer cells.
(b) Uses of Infrared, visible and microwaves :
 Infrared waves are widely used in remote switches of
household electronic systems such as remotes for TVs,
video recorders etc.
 Visible rays provide us information about the world.
 Microwaves are used in the radar systems in aircraft
navigation.
Answer key (3 Marks)
38. (a) 10-3 nm : γ-rays
Application :
1. γ-rays are used in the treatment of cancer and tumour.
2. γ-rays are used in radiation therapy. (any one)
(b) 10-3m : Microwave
Application : Microwaves are used in Radar systems for
aircraft navigation.
(c) 1 nm : X-rays Application :
1. Infra-red waves are used for taking photographs during
the conditions of fog, smoke etc.

31
2. These are also used as a diagonostic tool for the detection
of fractures, (any one)
39. (a) 1 mm : Microwaves
Application : In aircraft navigation for the radar system.
Also used in microwave ovens.
(b) 10-12 m : Gamma rays
Application : Gamma rays are used as medicine to
destroy cancer cells
(c) 10-8 m : Ultraviolet rays
Application : Ultraviolet rays are used in LASIK eye
surgery.
40. (a)
1. Along z-direction.
2. Velocity of propogation will be, C=E0B0
(b) Photoelectric effect shows the particle nature of
electromagnetic waves. As such the photons carry energy
and momentum. The energy is given by

41. (a) Microwaves are used in radar systems. Its


frequency range : 1010 to 1012 Hz
(b) In the absence of earth‘s atmosphere, there would have
no ozone layer to prevent ultraviolet radiations reaching
the earth, the temperature on earth‘s surface would have
been lower due to green house effect, making it difficult
for human survival.
(c) Since em wave carries both energy and momentum,
hence exerts pressure on the surface on which it is
incident.
An em wave exerts negligibly very small pressure on the
surface on which it is incident.
It is due to the fact that momentum of the photon is
extremely small, which can be

32
calculated by de-Broglie relation (λ=hp)

42. (a) Gamma (γ) rays are used for the treatment of
certain forms of cancer. Their frequency range is 1018 Hz
to 1022 Hz.
(b) The thin ozone layer on top of stratosphere absorbs most
of the harmful ultraviolet rays coming from the Sun
towards the Earth. They include UVA, UVB and UVC
radiations, which can destroy the life system on the Earth.
Hence, this layer is crucial for human survival.
(c) Thus, the amount of the momentum transferred by the
em waves incident on the surface is very small, because
of small value of planks constant. For example, an
electromagnetic wave of wavelength 1.00 nm will
provide momentum (p) according to de-Broglie‘s
relation,

It is extremely small value of the momentum.


43. (a) γ-rays; Frequency range : 1018 Hz to 1022 Hz
(b) Because to protect eyes from intense ultra-violet
radiations produced during welding; and also to protect
from glare and flying sparks.
(c) Because infrared waves are em waves of higher
wavelength (less frequency) and are produced by highly
vibrating molecules of hot bodies.
Applications :
1. used in the remote switches of household electronic
systems.
2. used for protecting dehydrated fruits.
3. used in solar water heaters and cookers. (Any one)

33
44. (i) Consider a plane perpendicular to the direction of
propagation of the wave. An electric charge, on the plane,
will be set in motion by the electric and magnetic fields
of em wave, incident on this plane. This illustrates that
em waves carry energy and momentum.
(ii) Microwaves are produced by special vacuum tubes like
the Klystron/Magnetron/Gunn diode.
In microwave ovens, the frequency of microwaves is
selected to match the resonant frequency of water
molecules, so that energy is transferred efficiently to the
kinetic energy of the molecules.
(iii) Important uses of infra-red waves :
1. These are associated with the green house effect.
2. These are used in remote switches of household
electrical appliances.
45. (a) Microwaves
Production : Klystron/magnetron
(b) Infrared Radiations
Production ; Hot bodies/vibrations of atoms and
molecules.
(c) X-Rays
Production : Bombarding high energy electrons on a
metal target.
46.
-3
 e.m. waves in the wavelength range 10 nm to 10 nm are
X-rays.
 X-rays are generated by bombarding a metal target with
high energy electrons.
Uses :
1. Diagnosis of bone fractures.
2. Treatment of some forms of cancer.
47. (i)e.m. waves having a wavelength range 0.1 m to 1
mm are MICROWAVES. (ii)Microwaves are generated
by special vacuum tubes such as klystron, magnetron and
gunn diodes.
34
(iii)Microwaves are used in :
 Radar system in aircraft navigation
 Ovens for heating and cooking.
48. (i) E.M. waves having a wavelength range 10-7 m to
-9
10 m are ultra violet rays.
(ii) Sun is an important source of UV rays. Some special
lamps and very hot bodies also produce UV rays.
Uses :
 UV rays are used in lasik eye surgery.
 UV lamps are being used to kill germs in water purifiers.
49. Gamma rays
50. Infrared waves
51. Uses (i) RADAR communication, (ii) analyzing
molecular and atomic structure
52. Uses (i) knowing molecular structure, (ii) in remote
control of TV, VCR etc.
53. Any three
Answer Key (MCQs)
S. NO. Correct S. NO. Correct
option option
1 C 16 C
2 B 17 A
3 B 18 D
4 B 19 A
5 B 20 D
6 D 21 D
7 C 22 C
8 D 23 A
9 C 24 B
10 D 25 A
11 C 26 A
12 C 27 C
13 C 28 A
14 A 29 B
35
15 C 30 D
Answer Key (ASSERTION REASON)
S. NO. Correct S. NO. Correct
option option
1 A 11 B
2 C 12 B
3 B 13 B
4 A 14 A
5 C 15 A
6 B 16 C
7 A 17 B
8 D 18 A
9 B 19 B
10 A 20 A

I.CASE STUDY II.CASE STUDY


S. NO. Correct S. NO. Correct option
option
1 A 1 D
2 C 2 B
3 D 3 B
4 C 4 A
III. CASE STUDY IV.CASE STUDY
S. NO. Correct S. NO. Correct option
option
1 C 1 C
2 C 2 B
3 D 3 B
4 D 4 B
V. CASE STUDY
S. NO. Correct
option
1 D
2 D
3 A
4 C

36
Chapter 9: Ray Optics and Optical Instruments
Gist
1. Refraction
2. Total Internal Reflection
3. Refraction at Spherical Surfaces and by Lenses
4. Refraction through a Prism
5. Dispersion by a Prism
6. Optical Instruments.
QUICK RECAP
Optics: It is the branch of physics which deals with the study
of light and the phenomena associated with it. It is
divided into two branches: X Geometrical optics or ray
optics physical optics or wave optics
Geometrical optics or ray optics: It treats propagation of
light in terms of rays and is valid only if wavelength of
light is much lesser than the size of obstacles. It deals
with the formation of images by ordinary geometrical
methods and the laws of reflection and refraction.
Refraction of light: When a ray of light passes from one
medium to another, in which it has
a different velocity, there occurs a change in the direction of
propagation of light except when it strikes the surface of
separation of two media normally. This bending of a ray
of light is known as refraction.
The angles made by the incident ray and the refracted ray
with the normal to the separating surface at the point of
incidence are known as the angles of incidence and of
refraction
respectively.
Laws of refraction: the two laws of refraction are as follows
:the incident ray, the normal and the refracted ray all lie
in the same plane

37
.the ratio of the sine of angle of incidence to the sine of
angle of refraction for any two media is constant for a
light of definite colour. µis constant .µ= sini/sinr.
Total internal reflection:The phenomenon of reflection of
light when a ray of light traveling from a denser medium
is sent back to the same denser medium provided the
angle of incidence is greater than the angle called critical
angle is called total internal reflection.

Conditions:-1. Light should travel from a denser to a rarer


medium.
2. Angle of incidence in denser medium should be greater
than the critical angle for the pair of media in contact.
Applications of total internal reflection
1. The brilliance of diamond is due to the phenomenon of
total internal reflection.
2. Mirages in deserts are also due to total internal reflection.
3. Totally reflecting prism
4. The working of optical fibre is based on the phenomenon
of total internal reflection.
Refraction from a spherical surfaces:

38
Lens Makers formula-

Lens Formula: 1/f = 1/ v – 1/u


Lens In Combination:-

P = P1 + P2
REFRACRION THROUGH PRISM

COMPOUND MICROSCOPE-

39
ASTRONOMICAL TELESCOPE-

REFLECTING TELESCOPE-

40
Short Answer Question (1 marks)
1. A convex lens is placed in contact with a plane mirror. A
point object at a distance of 20 cm on the axis of this
combination has its image coinciding with itself. What is
the focal length of the lens?
2. For the same value of angle of incidence, the angles of
refraction in three media A, B and C are 15°, 25° and 35°
respectively. In which media would the velocity of light
be minimum?
3. When monochromatic light travels from one medium to
another its wavelength changes but frequency remains the
same. Explain.
4. State the criteria for the phenomenon of total internal
reflection of light to take place?
5. A concave lens of refractive index 1.5 is immersed in a
medium of refractive index 1.65.What is the nature of the
lens?
6. A biconvex lens made of a transparent material of
refractive index 1.25 is immersed in water of refractive
index 1.33. Will the lens behave as a converging or a
diverging lens? Give reason.

41
7. A biconcave lens made of a transparent material of
refractive index 1.25 is immersed in water of refractive
index 1.33. Will the lens behave as a converging or a
diverging lens? Give reason.
8. A ray of light falls on a transparent sphere with centre C
as shown in the figure. ray emerges from the sphere
parallel to the line AB. Find the angle of refraction at A if
refractive index of the material of the sphere is 3 .
9. When red light passing through a convex lens is replaced
by light of blue colour, how will the focal length of the
lens change?
10. Under what condition does a biconvex lens of glass
having a certain refractive index act as a plane glass sheet
when immersed in a liquid?
11. How does focal length of a lens change when red
light incident on it is replaced by violet light? Give
reason for your answer.
12. Two thin lenses of power –4 D and 2 D are placed in
contact coaxially. Find the focal length of the
combination.
13. A glass lens of refractive index 1.45 disappears when
immersed in a liquid. What is the value of refractive
index of the liquid?
14. A converging lens is kept coaxially in contact with a
diverging lens, both the lenses being of equal focal
lengths. What is the focal length of the combination?
15. Two thin lenses of power + 6 D and –2 D are in
contact. What is the focal length of the combination?
16. A glass lens of refractive index 1.5 is placed in a
trough of liquid. What must be the refractive index of the
liquid in order to make the lens disappear?
17. Write the relationship between angle of incidence ‗i‘,
angle of prism ‗A‘ and angle of minimum deviation dm
for a triangular prism.

42
18. How does the angle of minimum deviation of a glass
prism vary, if the incident violet light is replaced with red
light?
19. You are given following three lenses. Which two
lenses will you use as an eyepiece and as an objective to
construct an astronomical telescope?
Lenses Power (P) Aperture (A)
L1 3D 8 cm
L2 6D 1 cm
L3 10 D 1 cm
20. When viewing through a compound microscope, our
eyes should be positioned not on the eyepiece but a short
distance away from it for best viewing. Why? How much
should be that short distance between the eye and
eyepiece?
Short Answer Type SA-(2 Marks)
1. How does the refractive index of a transparent medium
depend on the wavelength of incident light used?
Velocity of light in glass is 2 X 108 m/s and in air is 3
X108 m/s. If the ray of light passes from glass to air,
calculate the value of critical angle.
2. (a) Write the necessary conditions for the phenomenon of
total internal reflection to occur.
(b) Write the relation between the refractive index and
critical angle for a given pair of optical Media.
3. A fish in a water tank sees the outside world as if it (the
fish) is at the vertex of a cone such that the circular base
of the cone coincides with the surface of water. Given the
depth of water, where fish is located, being ‗h‘ and the
critical angle for water-air interface being ‗ic‘, find out by
drawing a suitable ray diagram the relationship between
the radius of the cone and the height ‗h‘.
4. An equi-convex lens of focal length ‗f‘ is cut into two
identical plane convex lenses. How will the power of
each part be related to the focal length of the original
43
lens? A double convex lens of +5 D is made of glass of
refractive index 1.55 with both faces of equal radii of
curvature. Find the value of its radius of curvature.
5. A convex lens of focal length 25 cm is placed coaxially in
contact with a concave lens of focal length 20 cm.
Determine the power of the combination. Will the system
be converging or diverging in nature?
6. A convex lens of focal length f1 is kept in contact with a
concave lens of focal length f2. Find the focal length of
the combination.
7. A beam of light converges at a point P. A concave lens of
focal length 16 cm is placed in the path of this beam 12
cm from P. Draw a ray diagram and find the location of
the point at which the beam would now converge.
8. Find radii of curvature of the faces of a double convex
lens are 10 cm and 15 cm. If focal length of the lens is 12
cm, find the refractive index of the material of the lens.
9. A convex lens of refractive index 1.5 has a focal length of
18 cm in air. Calculate the change in its focal length
when it is immersed in water of refractive index 4/3.
10. What are optical fibres? Give their one use?
11. What is mirage? On what principal it is based on?
12. the following table gives the values of the angle of
deviation, for different values of the angle of incidence,
for a triangular prism :
Angle of Incidence 33° 38° 42° 52° 60° 71°
Angle of Deviation 60° 50° 46° 40° 43° 50°

(a) For what value of the angle of incidence, is the angle of


emergence likely to be equal to the angle of incidence
itself?
(b) Draw a ray diagram, showing the passage of a ray of
light through this prism when the angle of incidence has
the above value.

44
13. Define refractive index of a transparent medium. A ray of
light passes through a triangular prism. Plot a graph
showing the variation of the angle of deviation with the
angle of incidence
14. (a) Why does white light disperse when passed through a
glass prism?
(b)Using lens maker‘s formula, show how the focal length of
a given lens depends upon the colour of light incident on
it.
15. Violet colour is seen at the bottom of the spectrum when
white light is dispersed by a prism State reasons to
explain these observations
16. What is dispersion of light ? What is its cause ?
17. Write the conditions for observing a rainbow. Show by
drawing suitable diagrams, how one understands the
formation of a rainbow.
18. The bluish colour Predominantes in clear sky why?
19. You are given two converging lenses of focal lengths
1.25 cm and 5 cm to design a compound microscope. If it
is desired to have a magnification of 30, find out the
separation between the objective and the eyepiece
20. A small telescope has an objective lens of focal length
150 cm and eyepiece of focal length 5 cm. What is the
magnifying power of the telescope for viewing distant
objects in normal adjustment. If this telescope is used to
view a 100 m tall tower 3 km away, what is the height of
the image of the tower formed by the objective lens
21. Draw a schematic arrangement of a reflecting telescope
(Cassegrain) showing how rays coming from a distant
object are received at the eye-piece. Write its two
important advantage over a refracting telescope.
22. A biconvex lens made of a transparent material of
refractive index 1.25 is immersed in water of refractive
index 1.33. Will the lens behave as a converging lens?
Give reason.
45
23. The angle subtended at the eye by an object is equal to
the angle subtended at the eye by the virtual image
produced by a magnifying glass. In what sense then does
a magnifying glass provide angular magnification?
24. In viewing through a magnifying glass, one usually
positions one‘s eyes very close to the lens. Does angular
magnification change if the eye is moved back?
25. Magnifying power of a simple microscope is inversely
proportional to the focal length of the lens. What then
stops us from using a convex lens of smaller and smaller
focal length and achieving greater and greater magnifying
power?
26. On what factor refractive index depends.
27. A virtual image, we always say, cannot be caught on a
screen. Yet when we‘see‘a virtual image, we are
obviously bringing it on to the ‗screen‘ (i.e., the retina) of
our eye. Is there a contradiction?
28. A diver under water, looks obliquely at a fisherman
standing on the bank of a lake.
Does the apparent depth of a tank of water change if
viewed obliquely? if so, does the apparent depth increase
of decrease?
29. The refractive index of diamond is much greater than that
of ordinary glass. is this fact of some use to a diamond
cutter?
30. The image of a small electric bulb fixed on the wall of a
room is to be obtained on the opposite wall 3 m away by
means of a large convex lens. What is the maximum
possible focal length of the lens required for the purpose?
Long Answer Type LA-(3 Marks)
1. Define the term ‗critical angle‘ for a pair of media. A
point source of monochromatic light ‗S‘ is kept at the
centre of the bottom of a cylinder of radius 15.0 cm. the
cylinder contains water (refractive index 4/3) to a height

46
of 7.0 cm. Draw the ray diagram and calculate the area of
water surface through which the light emerges in air.
2. Designed prisms make use of total internal reflection to
obtain inverted image of the object by deviating rays (i)
through 90° and (ii) through 180°.
3. A convex lens of focal length 20 cm is placed coaxially
with a convex mirror of radius of curvature 20 cm. the
two are kept at 15 cm from each other. A point object lies
60 cm in front of the convex lens. Draw a ray diagram to
show the formation of the image by the combination.
Determine the nature and position of the image formed.
4. Define power of a lens. Write its units. Deduce the
relation 1/f = 1/f1+ 1/f 2 for two thin lenses kept in
contact coaxially.
5. Draw a ray diagram to show the formation of the image
of an object placed on the axis of a convex refracting
surface of radius of curvature ‗R‘, separating the two
media of refractive indices ‗n1‘ and ‗n2‘ (n2 > n1). Use
this diagram to deduce the relation
n2/v – n1/u = n2- n1/R
where u and v represent respectively the distance of the
object and the image .
6. A convex lens made up of glass of refractive index 1.5 is
dipped, in turn, in (i) a medium of refractive index 1.65,
(ii) a medium of refractive index 1.33.
(a) Will it behave as a converging or a diverging lens in the
two cases?
(b) How will its focal length change in the two media?
7. A convex lens of refractive index 1.5 has a focal length of
20 cm in air. Calculate the change in its focal length
when it is immersed in water of refractive index 4/3.
8. A double convex lens of glass of refractive index1.6 has
its both surface of equal radii of curvature of 30 cm each.
An object of height 5 cm is placed at a distance of 12.5
cm from the lens. Calculate the size of the image formed.
47
9. An illuminated object and a screen are placed 90 cm
apart. Determine the focal length and nature of the lens
required to produce a clear image on the screen, twice the
size of the object.
10. A point object is placed in front of a double convex
lens (of refractive index n = n2/n1 with respect to air)
with its spherical faces of radii of curvature R1 and R2.
Show the path of rays due to refraction at first and
subsequently at the second surface to obtain the
formation of the real image of the object. Hence obtain
the lens-maker‘s formula for a thin lens.
11. Three light rays red (R), green (G) and blue (B) are
incident on the right angled prism a b c at face ab. The
refractive indices of the material of the prism for red,
green and blue wavelengths are respectively 1.39, 1.44
and 1.47. Trace the paths of these rays reasoning out the
difference in their behaviour.

12. A ray of light, incident on an equilateral glass prism


(µ=√ ) moves parallel to the base line of the prism inside
it. Find the angle of incidence for this ray.
13. Draw a ray diagram to show refraction of a ray of
monochromatic light passing through a glass prism.
Deduce the expression for the refractive index of glass in
terms of angle of prism and angle of minimum deviation.
14. Draw a plot showing the variation of power of a lens
with the wavelength of the incident light. A diverging
lens ofrefractive index 1.5 and of focal length 20 cm in
air has the same radii of curvature for both sides. If it is
immersed in a liquid of refractive index 1.7, calculate the
focal length of the lens in the liquid.
48
15. (i) A giant refracting telescope has an objective lens
of focal length 15 m. If an eye piece of focal length 1.0
cm is used, what is the angular magnification of the
telescope?
(ii) If this telescope is used to view the moon, what is the
diameter of the image of the moon formed by the
objective lens? The diameter of the moon is 3.48 × 106 m
and the radius of lunar orbit is 3.8 × 108 m.
16. Which two of the following lenses L1, L2, and L3
will you select as objective and eyepiece for constructing
best possible (i) telescope (ii) microscope? Give reason to
support your answer.
Lens Power (P) Aperture (A)
L1 6D 1 cm
L2 3D 8 cm
L3 10 D 1 cm
17. (a) Draw a labelled ray diagram showing formation
of a final image by a compound microscope at least
distance of distinct vision.
(b) The total magnification produced by a compound
microscope is 20. The magnification produced by the eye
piece is 5. The microscope is focussed on a certain object.
The distance between the objective and eyepiece is
observed to be 14 cm. If least distance of distinct vision is
20 cm, calculate the focal length of the objective and the
eye piece.
18. (i) Draw a schematic labelled ray diagram of a
reflecting type telescope.
(ii) Write two important advantage justifying why reflecting
type telescopes are prefer over refracting telescopes.
(iii) The objective of a telescope is of large focal length and
of larger aperture (compared to the eyepiece). Why?
Give reasons.
19. (a) A small telescope has an objective lens of focal
length 140 cm and an eyepiece of focal length 5.0 cm.
49
Find the magnifying power of the telescope for viewing
distant objects when
(i) The telescope is in normal adjustment,
(ii) The final image is formed at the least distance of distinct
vision.
(b) Also find the separation between the objective lens and
the eye piece in normal adjustment.
20. Draw a ray diagram showing the image formation
by a compound microscope when the final image is
formed at the normal adjustment and derive expression
for magnifying power.
21. Two convex lenses of focal length 10 cm and 1 cm
constitute a telescope. The telescope is focussed on a
point which is 1m away from the objective. Calculate the
magnification produced and the length of the tube, if the
final image is formed at a distance of 25 cm from the
eyepiece.
22. (a) Draw a neat labelled ray diagram of an
astronomical telescope in normal adjustment.
(b) Derive its magnifying power.
23. a) How is the working of a telescope different from
that of a microscope?
b) The focal lengths of the objective and eyepiece of a
microscope are 1.25 cm and 5 cm respectively. Find the
position of the object relative to the objective in order to
obtain an angular magnification of 30 in normal
adjustment..
24. Monochromatic light of wavelength 589 nm is
incident from air on a water surface. If for water is 1.33,
find the wavelength, frequency and speed of the refracted
light.
25. Explain, with the help of a ray diagram, the working
of an astronomical telescope. the magnifying power of a
telescope in its normal adjustment is 20. If the length of

50
the telescope is 105 cm in this adjustment, find the focal
lengths of the two lenses.
26. A ray PQ incident normally on the refracting face
BA is refracted in the prism BAC made of material of
refractive index 1.5. Complete the path ray through the
prism. From which face will the ray emerge? justify your
answer.

27. Explain the following, giving reasons :


I. When monochromatic light is incident on a surface
separating two media, the reflected and refracted light
both have the same frequency as the incident frequency.
II. When light travels from a rarer to a denser medium, the
speed decreases. Does this decrease in speed imply a
reduction in the energy carried by the wave?
28. (I). The line AB in the ray diagram of the given
figure represents a lens. State whether the lens
represented by AB is convex or concave?

(II) You are given three lenses each of focal length20 cm


.An object is kept at 40 cm in front of L1. The final real

51
image is formed at the focus of L3. Find the separation
between the lenses.
29. A small pin fixed on a table top is viewed from
above from a distance of 50 cm. By what distance would
the pin appear to be raised if it is viewed from the same
point through a 15 cm thick glass slab held parallel to the
table? Refractive index of glass = 1.5. Does the answer
depend on the location of the slab?
30. Answer the following question:
(a) Magnifying power of a simple microscope is inversely
proportional to the focal length of the lens. What then
stops us from using a convex lens of smaller and smaller
focal length and achieving greater and greater magnifying
power?
(c) A tank is filled with water to a height of 12.5 cm. The
apparent depth of a needle lying at the bottom of the tank
is measured by a microscope to be 9.4 cm. What is the
refractive index of water? If water is replaced by a liquid
of refractive index 1.63 upto the same height, by what
distance would the microscope have to be moved to focus
on the needle again?

Multiple Choice Questions


1. Suppose while sitting in a parked car, you notice a jogger
approaching towards you in the rear view mirror of R = 2
m. If the jogger is running at a speed of 5 ms-1, how fast
is the image of the jogger moving, when the jogger is 39
m away ?
(a)1/150 m/s (b) 1/150 m/s (c)
1/150 m/s (d)none of these
2. In optical fibers, propagation of light is due to
(a) diffraction (b) total internal reflection (c)
reflection (d) refraction

52
3. In figure the points C1 and C2 denote the centres of curvatures
then the focal length of the thin lens (μ=1.5) is

(a) 10cm (b) 20 cm (c)


30 cm (d)40 cm
4. The optical length of an astronomical telescope with
magnifying power of 10, for normal vision is 44cm. What
is focal length of the objective?
(a) 40 cm (b) 440cm (c) 42
cm (d)none of these
5. The angle of prism is 60° and angle of deviation is 30°. In
the position of minimum deviation, the values of angle of
incidence and angle of emergence are:
(a) i = 45°; e = 50° (b) i = 30°; e = 45° (c) i = 45°; e =
45° (d) i = 30°; e = 30°
6. If the refractive index of a material of equilateral prism is
√ 3 then angle of minimum deviation of the prism is
(a) 60° (b) 45° (c) 30° (d) 75°
7. Air bubble in water behaves as
(a) sometimes concave, sometimes convex lens
(b) concave lens
(c) convex lens
(d) always refracting surface

53
8. The length of an astronomical telescope for normal vision
(relaxed eye) will be

9. A metal coin is at bottom of a beaker filled with a liquid


of refractive index = 4/3 to height of 6 cm. To an
observer looking from above the surface of liquid, coin
will appear at a depth
(a) 1.5 cm (b) 6.75 cm (c) 4.5 cm
(d) 7.5 cm
10. If a convex lens of focal length 80 cm and a concave
lens of focal length 50 cm are combined together, what
will be their resulting power?
(a) + 6.5 D (b) – 6.5 D (c) + 7.5 D
(d) – 0.75 D
11. In an experiment to find focal length of a concave
mirror, a graph is drawn between the magnitude of u and
v. The graph looks like

12. A double convex lens of refractive index µ1 is


immersed in a liquid of refractive index µ2. The lens will
act as transparent plane sheet when
(a) µ1 = µ2 (b) µ1 > µ2 (c) µ1 < µ2 (d) µ1 = 1/μ2
13. For a total internal reflection, which of the following
is correct?
54
(a) Light travels from rarer to denser medium.
(b) Light travels from denser to rarer medium.
(c) Light travels in air only.
(d) Light travels in water only.
14. Two beams of red and violet color are made to pass
separately through a prism (angle of the prism is 60°). In
the position of minimum deviation, the angle of refraction
will be
(a) 30° for both the colors
(b) greater for the violet color
(c) greater for the red color
(d) equal but not 30° for both the colors
15. An astronomical refractive telescope has an objective
of focal length 20 m and an eyepiece of focal length 2
cm. Then
(a) the magnification is 1000
(b) the length of the telescope tube is 20.02 m
(c) the image formed of inverted
(d) all of these
16. You are given four sources of light each one
providing a light of a single colour – red, blue, green and
yellow. Suppose the angle of refraction for a beam of
yellow light corresponding to a particular angle of
incidence at the interface of two media is 90°. Which of
the following statements is correct if the source of yellow
light is replaced with that of other lights without changing
the angle of incidence?
(a) The beam of red light would undergo total internal
reflection.
(b) The beam of red light would bend towards normal

55
while it gets refracted through the second medium.
(c) The beam of blue light would undergo total internal
reflection.
(d) The beam of green light would bend away from the
normal as it gets refracted through the second medium.
17. The direction of ray of light incident on a concave
mirror is shown by PQ while directions in which the ray
would travel after reflection is shown by four rays
marked 1, 2, 3 and 4. Which of the four rays correctly
shows the direction of reflected ray?

(a) 1 (b) 2 (c) 3


(d) 4
18. How does the angle of minimum deviation of a glass
prism of refractive index 1.5 change, if it is immersed in a
liquid of refractive index 1.3?
(a) It will decreases (b) It will increases
(c) It will remain same (d) None of these
19.A fish is a little way below the surface of a lake. If the critical
angle is 49o, then the fish could see things above the water
surface within an angular range at θo where

56
.
(a) θ = 49° (b) θ = 98° (c) θ = 90° (d) 24.5°
20. The power of a thin convex lens of glass is 5 D.
When it is immersed in a liquid, then it behaves like a
divergent lens of focal length 100 cm. The refractive
index of the liquid is:
(Given, refractive index of glass = μ = 1.5)
(a) 3/5 (b) ½ (c) 2/3 (d) 5/3
21. An equiconvex lens of focal length f and power P is
cut into two halves in thickness. The focal length and
power of each half is
(a) f/2 (b) 2f (c) f
(d) zero
22. The magnifying power of an astronomical telescope
in normal adjustment is 100. The distance between the
objective and the eyepiece is 101 cm. The focal length of
the objectives and eyepiece is
(a) 10 cm and 1 cm respectively
(b) 100 cm and 1 cm respectively
(c) 1 cm and 100 cm respectively
(d) 1 cm and 10 cm respectively
23. Angle of minimum deviation for a prism refractive
index 1.5 is equal to the angle of the prism. Then the
angle of prism _______ (given, sin 48° 36' = 0.75)
(a) 62° (b) 82° (c) 60°
(d) 41°
24. The dispersive power will be maximum for
(a) Flint glass (b) Crown glass
(c) Mixture of glass (d) None of these

57
25. Which two of the following lenses L1, L2 and
L3 will you select as objective and eyepiece for
constructing best possible (i) telescope,

Power Aperture
Lens
(P) (A)

L1 3D 8 cm

L2 6D 1 cm

L3 10D 1 cm

(a)L1 as objective L3 as eye piece (b) L2 as


objective L3 as eye piece (c) L3 as objective L2 as eye
piece (d) none of these
26. An optical instrument uses a lens of power 100 D for
objective lens and 50 D for its eyepiece. When the tube
length is kept at 25 cm. the final image is formed at
infinity.Calculate the magnification produced by the
instrument.
(a) 312.5 cm (b)300cm (c) 450
cm (d)100 cm
27. A ray PQ is incident normally on the face AB of a
triangular prism of refracting angle of 60°, made of a
transparent material of refractive index 2/√3, as shown in
the figure. Trace the path of the ray as it passes through
the prism. Calculate the angle of emergence
58
(a) 900 (b) 600 (c) 300
(d) 450
28. An astronomical telescope uses two lenses of power
10 D and 1 D. What is its magnifying power in the
normal adjustment?
(a) -10 cm (b) -21 cm (c)-20 cm
(d)1 cm
29. You are given two converging lens of focal lengths
1.25 cm and 5 cm to design a compound microscope. If it
is desired to have a magnification of 30, find out the
separation between the objective and the eyepiece.
(a) 6.25 cm (b) 4.02 cm (c) 10 cm
(d) 5 cm
30. Two lenses of power +15D and -5D are in contact
with each other forming a combination lens what is the
focal length of this combination?
(a) 20 cm (b) 10 cm (c) 15 cm
(d) none of these
Assertion Reason Type questions
In each of the following questions, a statement of Assertion
(A) is given followed by a corresponding statement of
Reason(R) just below it. Of the statements, mark the
correct answer as
(a) If both assertion and reason are true, and reason is the
true explanation of the assertion.

59
(b) If both assertion and reason are true but reason is the true
explanation of the assertion.
(c) If assertion is true, but reason is false.
(d) If both assertion and reason are false.
1. Assertion : A ray of light is incident from outside on a
glass sphere surrounded by air. This ray may suffer total
internal reflection at second interface.
Reason : If a ray of light goes from denser to rarer
medium, it bends away from the normal.
2. Assertion : The focal length of the convex mirror will
increase, if the mirror is placed in water.
Reason : The focal length of a convex mirror of radius R
is equal to , f = R/2.
3. Assertion : The optical instruments are used to increase
the size of the image of the object.
Reason : The optical instruments are used to increase the
visual angle.
4. Assertion: A thick lens shows more chromatic aberration.
Reason: Thick lens behave as many thin lenses.
5. Assertion: The edges of the images of white object formed by
a concave mirror on the screen appear white.
Reason: Concave mirror does not suffer chromatic aberration.
6. Assertion: Optical fibers make use of total internal
reflection.
Reason: Light goes through successive total internal
reflections as it moves through an optical fiber.
7. Assertion :In a movie, ordinarily 24 frames are projected
per second from one end to the other of the complete
film.
Reason :The image formed on retina of eye is sustained upto
1/10 second after the removal of stimulus.
8. Assertion: A double convex lens (m = 1.5) has focal
length 10 cm. When the lens is immersed in water (m =
4/3) its focal length becomes 40 cm.
60
Reason :1/f=μl−μm/μm(1/R1−1/R2)
9. Assertion: The focal length of the mirror is f and distance
of the object from the focus is u, the magnification of the
mirror is f / u.
Reason: Magnification =Size of image/Size of object
10. Assertion : In optical fibre, the diameter of the core
is kept small.
Reason : This smaller diameter of the core ensures that
the fibre should have incident angle more than the critical
angle required for total internal reflection.
11. Assertion: When an object is placed between two plane
parallel mirrors, then all the images found are of equal
intensity.
Reason :In case of plane parallel mirrors, only two images are
possible.
12. Assertion : Sky is maximum red in morning
Reason : Smallest wavelength scatter maximum
13. Assertion :A ray of light is incident from outside on a
glass sphere surrounded by air. This ray may suffer total
internal reflection at second interface.
Reason :If a ray of light goes from denser to rarer medium, it
bends away from the normal.
14. Assertion: The resolving power of both microscope
and telescope depends on the wavelength of light used.
Reason: the resolving power of a lens is the ability to resolve
the two images so they are distinctly identified.
15. Assertion:- In modem microspore multicomponent
lenses are used for both objective and eye piece.
Reason:- Multicomponent lenses increase the magnification.
16. Assertion: Microscope magnifies the image.
Reason: Angular magnification for image is more than
object in microscope.
17. Assertion : A prism deviates a ray of light towards its
base.
Reason : Both refracting sides scatter the light.
61
18.Assertion: A beam of white light gives a spectrum on passing
through a hollow prism.
Reason: Speed of light outside the prism is different from the
speed of light inside the glass prism.
19. Assertion: The apparent depth of a tank of water
decreases if viewed obliquely.
Reason: Real depth decreases if viewed obliquely.
20. Assertion: A convex lens and a concave lens are kept
in contact. They will behave as a diverging lens if focal
length of convex lens is more.
Reason: Power of a concave lens is always less than the
power of a convex lens, as power of concave lens is
negative whereas power of convex lens is positive.

CASE STUDY BASED QUESTIONS


Case study -1
A prism has three rectangular surfaces and two triangular
faces inclined at an angle. The angle between the two
refracting surfaces is called the angle of the prism. The
angle between the incident ray and the emergent ray is
called the angle of deviation of the prism. Dispersion is
defined as the separation of white light into different
colours when the light is passed through the prism. The
amount of scattering depends on the wavelength of light.

62
Q1: The critical angle between an equilateral prism and air is
45. If the incident ray is perpendicular to the refracting
surface, then
a. It is reflected totally from the second surface and emerges
perpendicular from the third surface.
b. It gets reflected from second and third surfaces and
emerges from the first surface
c. It keeps reflecting from all the three sides of the prism
and never emerges out
d. After deviation, it gets refracted from the second surface
Q2: Which ray is least deviated by a prism?
a. Violet ray
b. Green ray
c. Red ray
d. Yellow ray
Q3: A prism (μ=1.5) has a refracting angle of 300. The
deviation of a monochromatic ray incident normally on
its one surface will be
a. 180 36‘
b. 200 30‘
c. 180
d. 190 30‘
Q4: The dispersive power of prism depends upon
a. The shape of the prism
b. The material of the prism
c. The angle of the prism
d. Height of the prism

Q5: The refractive angle of a prism for a monochromatic


light is 600 and refractive index is √2. For minimum
deviation, the angle of incidence will be
a. 600
b. 450
c. 300
d. 750
63
Case study -2
Optical instruments are the devices which process light
waves to enhance an image for a more clear view. The
optical microscope is of two types: a simple microscope
and a compound microscope. An optical microscope uses
a single lens or a group of lenses for magnification.

Q1: What is the ratio of resolving power of an optical


microscope for wavelengths λ1= 6000 Å and λ2= 9000 Å?
a. 2:3
b. 3:2
c. 6:25
d. 16:9
Q2: Two identical glass (μg=3/2) equi – convex lenses of
focal length f each are kept in contact. The space between
the two lenses is filled with water ((μw=4/3). The focal
length of the combination is
a. f/3
b. f
c. 4f/3
d. 3f/4
Q3: Resolving power of a microscope depends upon
a. The focal length and aperture of the eye lens
b. The focal length and objective of the eye lens
64
c. The apertures of the objective and the eye lens
d. The wavelength of light illuminating the object
Q4: The magnification power of a compound microscope
does not depend upon
a. The focal length and aperture of the eye lens
b. The apertures of the objective lens
c. Tube length of the microscope
d. None of the above
Q5: Resolving power of light microscope is
a. 2 mm
b. 0.2 mm
c. 0.1 mm
d. 1 mm
Case study -3
An optical fibre is a thin tube of transparent material that
allows light to pass through, without being refracted into
the air or another external medium. It make use of total
internal reflection. These fibres are fabricated in such a
way that light reflected at one side of the inner surface
strikes the other at an angle larger than critical angle.
Even, if fibre is bent, light can easily travel along the
length.

Q1. Which of the following is based on the phenomenon


of total internal reflection of light?
(a) Sparkling of diamond (c) Instrument used by doctors for endosco
(b) Optical fibre (d) All of these
(ii) A ray of light will undergo rotal internal reflection inside
the optical fibre, if it
Q2. (a) goes from rarer medium to denser medium
65
(b) is incident at an angle less than the critical angle
(c) strikes the interface normally
(d) is incident at an angle greater than the critical angle
Q3. If in core, angle of incidence is equal to critical angle,
then angle of refraction will be
(a) 0° (b) 45° (c) 90 (d) 180°
Q4. In an optical fibre (shown), correct relation for
refractive indices of core and cladding is

(a) n1 = n2 (b) n1 > n2 (c) n1 < n2 (d) n1 + n2 = 2


Q5. If the value of critical angle is 30° for total internal
reflection from given optical fibre, then speed of light in
that fibre is
(a) 3 x 108 m S-1 (b) 1.5 x 108 m S-1 (c) 6 x 108 m s-1 (d) 4.5 x 108 m s-1
Case Study -4
A compound microscope is an optical instrument used for
observing highly magnified images of tiny
objects.Magnifying power of a compound microscope is
defined as the ratio of the angle subtended at the eye by
the final image to the angle subtended at the eye by the
object, when both the final image and the object are
situated at the least distance of distinct vision from the
eye. It can be given that:m=me×mom=me×mo where
me is magnification
produced by eye lens and mo is magnification produced
by objective lens. Consider a compound microscope that
consists of an objective lens of focal length 2.0 cm and an
eyepiece of focal length 6.25 cm separated by a distance

66
of 15 cm.
Q1. The intermediate image formed by the objective of a
compound microscope is
(a) real, inverted and magnified (b) real, erect, and magnified
(c) virtual, erect and magnified (d) virtual, inverted and magnified
Q2. The magnifying power of a compound microscope
increases with
(a) the focal length of objective lens is increased and that of eye lens is
(b) the focal length of eye lens is increased and that of objective lens is
(c) focal lengths of both objects and eye-piece are increased
(d) focal lengths of both objects and eye-piece are decreased.
Q3. What is the magnifying power of the microscope in
case of least distinct vision?
(a) 20 (b) 30 (c) 40 (d) 10
Case study -5
If v1 and v2 denote the velocity of light in medium 1 and
medium 2 respectively and λ1 and λ2 denote the
wavelength of light in medium 1 and medium 2. Thus

The above equation implies that when a wave gets


refracted into denser medium (v1 > v2) the wavelength
and the speed of propagation decreases but the frequency
v
Q1.When light travels from one medium to another medium
which are separated by a sharp boundary, the
characteristic which does not change is:
a.Velocity
b.Wavelength
c.Frequency
d.Amplitude

67
Q2. When light travels from a rarer to a denser medium, the
speed of light in the medium:
a. Increases
b. Decreases
c. Remains the same
d. First increases and then decreases

Case Study -6
Real Depth is actual distance of an object beneath the
surface, as would be measured by submerging a perfect
ruler along with it. Apparent depth in a medium is the
depth of an object in a denser medium as seen from the
rarer medium.

Q1. The speed of light in air is 3×108ms−1. Calculate the speed of


light in glass. The refractive index of glass is 1.5.
(a)3x108 m/s (b) 2x108(c) 1.5x108(d) 2.5x108
Q.2 A water pond appears to be 2.7 m raised when seen
from above. Refractive index of water is 4/3. Find the
real depth.
(a)10.8 m (b) 15 m
(c) 9m (d) 6m

68
Case Study -7
When a light travels from denser to rarer medium the angle
of refraction is more than the angle of incidence. So,
whenever light travels from denser to rarer medium light
tends to move away from the normal. And when light
travels from rarer to denser medium light tends to move
towards normal.

1. A small air bubble in glass shpere of radius 2 cm appears


to be 1 cm from the surface when looked at, along a
diameter. If the refractive index of glass is 1.5, find the
true position of the air bubble
(a)-1.2 cm (b)2 cm
(c) 8cm (d) 6cm
2. A point ‗ O‘ marked on the surface of a glass sphere of
diameter 20cm is viewed through glass from the position
directly opposite to the point ‗ O‘. If the refractive index
of the glass is 1.5, find the position of the image formed.
69
(a)-40 cm (b)-22 cm
(c) -20cm (d) 10cm

Answer Key (1 Mark)


1. 20cm
2. Refractive index, µ=sini/sinr=c/v
As sin15° < sin25° < sin35°
So, vA < vB < vC
Hence in medium A, velocity of light is minimum.
3. Frequency being a characteristic of source of light does
not change with change of medium.
Refractive index of medium is defined as, Refractive index,
µ=sini/sinr=c/v
v = c/λ µ∝1/λ2 ( λ is same in different media)
Hence, wavelength of light is different in different media.
4. Essential conditions for total internal re_ection :
(i) Light should travel from a denser medium to a rarer
medium.
(ii) Angle of incidence in denser medium should be greater
than the critical angle for the pair of media in contact.
5. Focal length of a concave lens is negative. Using lens
maker‘s formula ( )( ),
Here, µl = 1.5, µm = 1.65 Also, is negative and focal length
of the given lens becomes positive. Hence, it behaves as a
convex lens.
6. The lens will act as a diverging lens as the refractive
index of water is greater than that of lens.
7. The lens will act as a converging lens as the refractive
index of water is greater than that of biconcave lens.
8. r = 30° by snells law , µ= sini/sinr
9. Focal length of the lens decrease when red light is
replaced by blue light
10. When the refractive index of the biconvex lens is
equal to the refractive index of the liquid in which lens is
70
immersed then the biconvex lens behaves as a plane glass
sheet. In this case, 1 /f =1/0 or f →∞.
11. Using lens maker‘s formula ( )( ),
V R the increase in refractive index would
result in decrease of focal length of lens. Hence, we can
say that replacing red light with violet light, decreases the
focal length of the lens used.
12. Net power P = P1 + P2 = –4 + 2 = –2 D Focal length
F = -50 cm
13. The value of refractive index of the liquid is 1.45.
14. Infinity.
15. Power P = + 6 D – 2 D = 4 D f=1/P=1/4=0.25m =
25cm
16. The refractive index of the liquid must be equal to
1.5.
17. the relation between the angle of incidence i, angle of
prism A, and the angle of minimum deviation , for a

triangular prism is given by,


18. When incident violet light is replaced with red light,
the angle of minimum deviation of a glass decreases.
19. For a telescope, lens L1 is used as objective as its
aperture is largest. the lens L3 is used as eye piece as
its focal length is smallest.
20. On moving the eye backward away from lens the
angular magnification decreases slightly, as both the
angle subtended by the
image at eye ‗a‘ and by the object at eye ‗α‘ decreases.
Although the decrease in angle subtended by object a is
relatively smaller.
Answer Key (2 Marks)
0
1. C = 41.3
2. a) Essential conditions for total internal reflection :

71
(i) Light should travel from a denser medium to a rarer
medium.
(ii) Angle of incidence in denser medium should be greater
than the critical angle for the pair of media in contact.
b) µ = 1/sinc
3. Radius r = htanC = sinC/cosC =

4. (i) P=1/2f
(ii the radius of curvature of the lens is 22 cm
5. The focal length of the combination= 1 m = 100 cm. the
system will be diverging in nature as the focal length is
negative.
6. F =
7. The beam converges at a distance 48 cm from the lens or
48 – 12 = 36 cm after P.
8. 1.5
9. Change in focal length, 3f = 54 cm
10. Optical fibres consist of thin and long strands of fine
quality glass or quartz coated with a thin layer of material
of refractive index less than the refractive index of
strands. They work on the principle of total internal
reflection so they do not suffer any loss.
Uses: The optical fibres are used in medical investigations
i.e. one can examine the inside view of stomach and
intestine by a method called endoscopy.
11. Mirage is an optical illusion that occur in hot
summer days.it is based on total internal reflection.
12. The angle of incidence is likely to be equal to 52°.
Using the equation =i+e–A
40° = 52° + e – 60° ⇒ e = 48°
13. Refractive index of a medium is defined as the ratio
of speed of light in vacuum to the speed of light in that
medium i.e. µ=c/v

72
Graph showing the variation of the angle of deviation with
angle of incidence from ncert book
14. the refractive index of prism-material depends on the
wavelength λof light. It is inversely proportional to
square of wavelength . Accordingly refractive index is
maximum for violet(λ= 4000 Å) and minimum for red
(λ= 7500 Å). So the deviation caused by prism = (µ–
1) A is maximum for violet and minimum for red; hence
on passage through the prism; the different colours are
separated; thus causing dispersion of white light.
15. When white light is dispersed by a prism, the violet
colour deviates through maximum angle.
Hence it is seen at the bottom of the spectrum.
16. ―Dispersion‖ of light is the phenomenon of splitting
up of white light into its constituent colours. the band of
seven colours then obtained on the screen is called
―VIBGYOR‖.the colour pattern obtained on the screen is
also called spectrum
Cause of dispersion : White light consist of continuous
forward wavelength range 400 nm to 700 nm. Refractive
index of glass is different for different wavelengths
17. The conditions for observing a rainbow are :
(i) Afer the rain, the sky is almost clear and there is bright
sunshine
with clouds in the east.
(ii) The sun is at the back of the observer. When a
rainbow is seen,
the sun, observer‘s eye and the centre of the arc of the
rainbow are all in a straight line. Sometimes, we see a
rainbow in the morning. Rainbow is also sometimes seen
while using a lawn sprinkler.
18. the amount of scattering as per Rayleigh‘s law
depends upon wavelength. Scattering µ=1/λ4, As λB <
λR Hence blue colour scatters more and also blue colour
73
is most sensitive to our eyes than any colour like violet
and indigo. thus the part of atmosphere which we observe
as sky has scattering of blue colour mostly, thus the sky
appears to be bluish.
19. 11.67 cm
20. –5 cm
21.

Advantages:
(i) It is free from chromatic and spherical aberrations.
(ii) Its resolving power is greater than refracting telescope
due to larger aperture of mirror
22.

23. In magnifying glass the object is placed closer than


25 cm, which produces image at 25 cm. This closer
74
object has larger angular size than the same object at 25
cm. In this way although the angle subtended by virtual
image and object is same at eye but angular magnification
is achieved.
24. Here, the retina is working as a screen, where the
rays are converging, but this screen is not at the position
of formed virtual image, in fact the reflected divergent
rays are converged by the eye lens at retina. Thus, there is
no contradiction.
25. If we decrease focal length, the lens has to be thick
with smaller radius of curvature. In a thick lens both the
spherical aberrations and chromatic aberrations become
pronounced. Further, grinding for small focal length is
not easy. Practically we can not get magnifying power
more than 3 with a simple convex lens.
26.

27. Here, the retina is working as a screen, where the


rays are converging, but this screen is not at the position
of formed virtual image, in fact the reflected divergent
rays are converged by the eye lens at retina. Thus, there is
no contradiction.
28. An observer in denser medium will observe the
fisherman taller than actual height, due to refraction from
rare to denser medium.
Apparent depth decreases if viewed obliquely as compared
to when observed near normally.
29. As μ=1/sinChence, sin C=1/μ refractive index of
diamond is much greater than that of ordinary glass,
hence critical angle C for diamond is much smaller (24°)
as compared to that of glass (42°).
A skilled diamond cutter thus can take the advantage of
75
such large range of angle of incidence available for total
internal reflection 24° to 90°. The diamond can be cut
with so many faces, to ensure that light entering the
diamond does multiple total internal reflections before
coming out. This behavior produce brilliance i.e.,
sparkling effect in the diamond.
30. condition for image to be obtained on the screen,
i.e.m real image. 9 – 12ƒ > 0 or 9 > 12ƒ or f < 0.75 m. so,
maximum focal length is 0.75 m.

Answer key (3 Marks)


1. Diagram ,Area of water surface= 200.28 cm2
2.

3. Diagram
Final image is formed at the distance of 30 cm from the
convex mirror (or 45 cm from the convex lens) to the
right of the convex mirror._the final image formed is a
virtual image.
4. Power of lens : It is the reciprocal of focal length of a
lens.P =1/f (f is in metre)
Unit of power of lens : Dioptre
correct derivation of 1/F = 1/F1 + 1/F2
5. Derivation
76
6. (i) fA = –5.5 fair As the sign of fA is opposite to that of
fair the lens will behave as a diverging lens.
(ii) fB = 3.91 fair As the sign of fB is same as that of
fair, the lens will behave as a converging lens
7. fw = 80 cm
8. h1 10cm
9. u = - 30 cm, v = 60 cm f = + 20 cm A convex lens of
focal length 20 cm is required.
10. Correct derivation from NCERT.
11.

77
12. i = 60°
13. Correct diagram and derivation from NCERT.
14.

15. (i) Here, fo = 15 m = 1500 cm and fe = 1.0 cm


Angular magni_cation by the telescope in normal
Adjustment m = f o/fe 1500/1 .0= 1500

78
(ii) the image of the moon by the objective lens is formed on
its focus only as the moon is nearly at infinite distance as
compared to focal length. fo = 15 m
Height of object i.e., Radius of moon Rm 3 48 2 x106
m
Rm = 1.74 × 106 m
Distance of object = Radius of lunar orbit
R0 = 3.8 × 108 cm
Distance of image for objective lens is the focal
length of objective lens, f0 = 15 m
Radius of image of moon by objective lens can be
calculated.
tanRm/Ro =h//f 0
h = 6.87 × 10–2 m
Diameter of the image of moon,
DI = 2h = 13.74 × 10–2 m = 13.74 cm
16. An astronomical telescope should have an objective
of larger aperture and longer focal length while an eye
piece of small aperture and small focal length. therefore,
we will use L2 as an objective and L3 as an eyepiece. For
constructing microscope, L3 should be used as objective
and L1 as eyepiece because both the lenses of microscope
should have short focal lengths and the focal length of
objective should be smaller than the eyepiece.
17. (a) correct diagram (b) f o = 3 5. Cm
18.

79
(iii) (a) the objective of a telescope have a larger focal length
to obtain large magnifying power and greater intensity of
image.
(b) the aperture of objective lens of a telescope is taken
as large because this increases the light gathering
capacity of the objective from the distant object.
Consequently, a brighter image is formed.
19. (a) (i) Given f0 = 140 cm, fe = 5 cm When _nal
image is at in_nity, magnifying power,
M=fo / fe m = –28
Negative sign shows that the image is inverted.
(ii) When final image is at the least distance of distinct
vision, magnifying power,
m=f o/fe(1+ fe /D)
= –33.6
(b) Separation between objective and eye piece when final
image is formed at infinity,
L = f0 + fe
L = 140 cm + 5.0 cm
L = 145 cm
20. Correct diagram and derivation from NCERT.
21. Magnifying power of telescope, m = me × m0=
−26x1/9=2.89
Length of telescope tube = v0 +u0= 12.1 cm( hint)
22. Correct diagram and derivation from NCERT.
80
23.

24. λ/µ = 442.89 nm 5.09 x 1012


hz
= 2.25
25. Correct diagram from ncert

81
The focal length of objective f0 is 100 cm and focal length
of eye piece fe is 5 cm.
26.

27. I. Frequency is intrinsic property of a wave,


which does not change when it goes
from one medium to another. Only wavelengths and
speed of the wave changes.
II. No. Because energy is E = hv and v remains same.
28. (I).Convex lens.
(II) f1 = f2 = f 3 =20cm
u1 = -40 cm By lens formula v1 = 40 cm
For lens L3 ,f3 = 20cm v 3 = 20cm ,on calculation
u3 = ∞
Distance between L1 and L 2 =40 + 20 +60 cm.
As the image formed by L2 lies at infinity ,distance
between L2and L3 can have
any value.
29. The shift in the image by the thick glass slab can be
calculated. Here, shift only depend upon thickness of
glass slab and refractive index of glass.
Shift = Real thickness – Apparent of thickness

82
The answer does not depend on the location of the
slab.
30. (a) If we decrease focal length, the lens has to be
thick with smaller radius of curvature. In a thick lens both
the spherical aberrations and chromatic aberrations
become pronounced. Further, grinding for small focal
length is not easy. Practically we cannot get magnifying
power more than 3 with a simple convex lens
(b) .

Now if the water is replaced by other liquid, the apparent


depth will change and microscope will have to be further
moved to focus the image. With new liquid

Now the microscope will have to shift from its initial


position to focus on the needle again which is at 7.67 cm
depth. Shift distance = 9.4 – 7.67 = 1.73 cm.
Answer Key (MCQs)
83
1 A 11 C 21 B
2 B 12 A 22 B
3 D 13 B 23 B
4 A 14 A 24 A
5 C 15 D 25 A
6 A 16 C 26 A
7 B 17 B 27 A
8 D 18 A 28 A
9 C 19 B 29 A
10 d 20 D 30 B
Answer Key (ASSERTION REASON)
1 B 6 B 11 D 16 A
2 D 7 C 12 A 17 C
3 D 8 A 13 B 18 D
4 C 9 A 14 A 19 C
5 A 10 A 15 B 20 C
Answer Key (case study based)
Case 1 Case 3 Case 5
1 A 1 D 1 C
2 C 2 D 2 B
3 A 3 C Case 6
4 B 4 B 1 B
5 B 5 B 2 A
Case 2 Case 4 Case 7
1 B 1 A 1 A
2 D 2 D 2 A
3 D 3 C
4 B
5 B

84
Chapter 10: Wave Optics
Gist :
1. Wave Optics Describes the connection between waves
and rays of light. According to wave theory of light, light
is a form of energy which travels through a medium in
the form of transverse wave.
2. Wavefront: It is the continuous locus of all such particles
of the medium which are vibrating in the same phase of
oscillation at any instant.
Depending upon the shape of the source of light,
wavefront are of different shapes.
(i) Plane wavefront
(ii) Spherical wavefront
(iii) Cylindrical wavefront

3. Ray: A line perpendicular to a wave front is called a ray.


The direction of rays are always perpendicular to the
wave front along the direction of propagation of wave.
4. Huygens‘ Principle:

85
 According to Huygens‘ Each point on the given wave
front (called primary wave front) acts as a fresh source of
new disturbance, called secondary wavelet, which travels
in all directions with the velocity of light in the given
medium.
 A surface touching these secondary wavelets, tangentially
in the forward direction at any instant gives the new
wavefront at that instant. This is called secondary
wavelets.
 Principle of Huygens‘ Construction:

 It is based on the principle that every point on a


wavefront is a source of secondary wavefront.

 The envelope of these wavefronts i.e., the surface tangent


to all the secondary wavefront gives the new wavefront.
The laws of reflection and refraction can be verified using
Huygens‘ wave theory.

86
As, frequency v is characteristic of the source, therefore v =
1/T remains the same as light travels from one medium to
another.
Wavelength is inversely proportional to refractive index (μ)
of the medium
i.e. λ‘ = λ/μ
5. Laws of reflection on the basis of Huygens‘ wave theory
As shown in figure, consider a plane wave front AB incident
on the reflecting surface XY, both the wave front and the
reflecting surface being perpendicular to the plane of
paper.

6. Law of refraction on this basis of Huygens‘ wave theory


Consider a plane wavefront AB incident on a plane surface
XY, separating two media 1 and 2, as shown in Figure.
Let v1 and v2 be the velocities of light in two media, with
v1 <v2.

87
This proves Snell‘s law of refraction. The constant 1μ2 is
called the refractive index of the socond medium with
respect to first medium.
7. Behaviour of a Prism, Lens and Spherical Mirror towards
Plane Wave front

88
8. Principle of superposition:- It states that a number of
waves travelling, simultaneously, in a medium behave
independent of each other and the net displacement of the
particle, at any instant, is equal to the sum of the
individual displacements due to all the waves.
9. The phenomenon of redistribution of energy in the region
of superposition of waves is called interference. The
points of maximum intensity in the regions of
superposition of waves are said to be in constructive
interference whereas the points of minimum intensity are
said to be in destructive interference.
Conditions for interference:-
(a) The two sources should emit, continuously, waves of
same wavelength or frequency.
(b) The amplitudes of the two waves should be either or
nearly equal
(c) The two sources should be narrow.
(d) The sources should be close to each other.
(e) The two sources should be coherent one.
 Condition for constructive interference:-
Path difference = (2n)λ/2
Phase difference = (2n)π
 Condition for destructive interference:-
Path difference = (2n+1)λ/2
Phase difference = (2n+1)π
10. Young‘s double slit experiment:-

89
 For a point P on the screen, the path

difference
Where d is the separation between two slits, D is the
distance between the slits and the screen and x is the
distance of the point of P from the central fringe.
For constructive interference (bright band), the path
difference must be an integer multiple of , i.e.-The
separation between adjacent bright (or dark) fringes
is using which can be measured.
 Constructive Interference:
1. Phase difference : where n is an integer
2. Path difference: where n is an integer
 Destructive interference:
1. Phase difference : ( ) , where n is an
integer
2. Path difference: ( ) , where n is an
integer

 Interference fringes with white light:- When the slits are


illuminated with white light, the interference pattern
consist of a central white fringe having on both sides a
90
few coloured fringes and then a general illumination.
Conditions for sustained interference:-
i. Two sources of light must be coherent.
ii. The frequencies (or wavelength) of the two waves should
be equal.
iii. The light must be monochromatic.
iv. The amplitudes of the interfering waves must be equal or
nearly equal.
v. The two sources must be narrow.

 Interference pattern.

 Diffraction:The phenomenon of bending of light around


the corners of an obstacle is called the diffraction of light.

 Diffraction at a Single Slit

91
1. The single-slit diffraction pattern shows the central
maximum (at ), zero intensity at angular
separation ( )
2. Angular spread of the central maxima 2 /d
 Width of the central maxima: 2 d
Where D is the distance of the slit from the screen, d is
the slit width

 Condition for the Minima on the either side of the Central


Maxima:
d sin , where n = 1,2,3,….

 Relation between phase difference & path difference:

Where is the phase difference & is the path


difference.
 Important Points

92
 A soap bubble or oil film on water appears coloured in
white light due to interference of light reflected from
upper and lower surfaces of soap bubble or oil film.
 In interference fringe pattern all bright and dark fringes
are of same width,
 In diffraction fringe pattern central bright fringe is
brightest and widest. and I remaining secondary maximas
are of gradually decreasing intensities.
 The difference between interference and diffraction is
that the interference is the superposition between the
wavelets coming from two coherent sources while the
diffraction is the superposition between the wavelets
coming from the single wavefront

93
 MIND MAP

94
Short Answer Question (1 marks)
1. What is the shape of the wave front when light is
diverging from a point source?
2. What is the phase difference between any two points
on a wavefront?
3. What is the shape of wavefront obtained from a point
source at a (i) small distance (ii) large distance?
4. Under what conditions a cylindrical wavefront is
obtained?
5. What type of wavefront is obtained when a plane
wave is reflected by a concave mirror?
6. Do all waves exhibit diffraction or only light?
7. What is the maximum intensity of light in Young‘s
double slit experiment if the intensity of light
emerging from each slit is Io?
8. Name two devices produce two coherent sources?
9. Can we use white light to get sustained interference
pattern? Give reason.
10. Instead of using two slits as in Young‘s experiment,
if two separate but identical sodium lamps are used,
what is the effect on interference pattern?
11. What is the effect on interference fringes when
yellow light is replaced by blue light in Young‘s
double slit experiment?
12. How does the fringe width in interference pattern
vary with the wavelength of incident light?
13. What is the effect on the interference fringes in a
Young‘s double-slit experiment when the
monochromatic source is replaced by a source of

95
white light?

14. How does the fringe width in interference vary with


the intensity of incident light?
15. If Young‘s double slit experiment is performed in
water
keeping the rest of the set-up same, the fringes
will….
16. Does the law of conservation of energy holds good in
interference and in diffraction?
17. How wave front and ray of light are related to each
other.
18. When light suffers reflection at the interface between
water and glass, the change of phase in the reflected
wave is.
19. Can two 60 W bulbs be used as sources to produce
interference pattern?
20. Which color of light undergoes diffraction to
maximum extent?
Short Answer Type SA-(2 Marks)
1. State the conditions that must be satisfied for two
light sources to be coherent?
2. In young‘s double slit experiment. The distance
between the slits is halved, what change in the fringe
width will take place?
3. State the reason, why two independent sources of
light cannot be considered as coherent sources.
4. How would the . separation of interference fringes in
Young‘s double slit experiment change when the
distance between the slits and screen is doubled?
96
5. The light of wavelength 600 nm is incident normally
on a slit of width 3mm. Calculate the linear width of
central maximum on a screen kept 3 m away from
the slit.
6. Write the conditions for constructive and destructive
interference to take place.
7. Why is the contribution of wavelets lying on the back
of secondary wave front zero?
8. Draw the shape of wave front originating from (i) a
point source and (ii) a line source?
9. What are two assumptions on which Huygens‘
Principle is based?
10. Why no interference pattern is observed, when two
coherent sources are (i) infinitely close to each other
(ii) far apart from each other?
11. What is the ratio of slit widths when the amplitudes
of light waves from them have a ratio 3:1?
12. If the two slits in Young‘s experiment have width
ratio 4:1, calculate the ratio of intensity at maxima
and minima in the interference pattern.
13. Find the ratio of intensities at two points in a screen
in YDSE when waves from the two slits have path
difference is (1) zero (2) λ/4
14. Sketch the refracted wave front emerging from a
convex lens if a plane wave front is incident
normally on it.
15. Why narrow sources are to be used for producing
interference?
16. Differentiate between a ray and a wavefront.
17. Light from two coherent sources of same amplitude

97
reaching at a point differ in path by 3(λ)/2. What will
be the colour of fringe at this point?
18. Give the graphical representation to show the
variation of intensity of light in single slit diffraction.
19. A slit of width 'd' is illuminated by light of
wavelength 6500.Å. For what value of d will the first
secondary maximum fall at an angle of diffraction of
30 degree?
20. How does the fringe width of interference fringes
change, when the whole apparatus of Young‘s
experiment is kept in a liquid of refractive index 1.3?

21. How does the angular separation of interference


fringes change, in Young‘s experiment, if the
distance between the slits is increased?
22. When two light waves interfere at some point to
produce darkness, what happens to the light energy ?
23. What is the shape of the wavefront in each of the
following cases:
(a) Light emerging out of a convex lens when a point
source is placed at its focus.
(b) The portion of the wavefront of light from a
distant star intercepted by the Earth.
24. Radio waves can diffract easily around a window but
not light waves. Give reason.
25. Why is the diffraction of Sound waves more evident
in daily experience than that of light wave?
26. In a single-slit diffraction experiment, the width of
the slit is made double the original width. How does
this affect the size and intensity of the central

98
diffraction band
27. In a single slit diffraction experiment, the width of
the slit is reduced to half its original width. How
would this affect the size and intensity of the central
maximum?
28. State one feature by which the phenomenon of
interference can be distinguished from that of
diffraction.
A parallel beam of light of wavelength 600 nm is
incident normally on a slit of width ‗a‘. If the
distance between the slits and the screen is 0.8 m and
the distance of 2nd order maximum from the centre of
the screen is. 15 mm, calculate the width of the slit.
29. Difference between interference and diffraction of
light
30. Why is it necessary to have coherent sources in order
to produce an interference pattern?
Long Answer Type LA-(3 Marks)
1. How will the angular separation and visibility of
fringes in Young‘s double slit experiment change
when (i) screen is moved away from the plane of the
slits, and (ii) width of the source slit is increased?
2. When monochromatic light travels from a rarer to a
denser medium, explain the following, giving reasons
:
(i) Is the frequency of reflected and refracted light
same as the frequency of incident light?
(ii) Does the decrease in speed imply a reduction in
the energy carried by light wave?
3. Using Huygens principle and drawing the sketches of

99
wave fronts, show how a parallel beam of light is
reflected from polished surface and hence verify that
<i= <r.
4. What is the effect of the interference fringes in a
Young‘s double slit experiment due to each of the
following operations.
a) The screen is moved away from the plane of the
slits:
b) The monochromatic source is replaced by another
monochromatic source of shorter wavelength:
c) The separation between the two slits is increased.
5. In YDSE using light of wavelength 600nm,the
angular width of the fringe formed on a distant screen
is 0.10 . Find the spacing between the two slits
6. In a Young‘s double slit experiment the two parallel
slits are made 1 mm apart and screen is placed 1m
away .What is the fringe separation when blue green
light of wavelength 500 nm is used?
7. Describe Young‘s double slit experiment to produce
interference pattern due to a monochromatic source
of light. Deduce the expression for the fringe width.
8. In Young's double slit experiment, the slits are 2 mm apart
and are illuminated by photons of two wavelengths λ1
=12000A0 and λ2=10000A0. At what minimum distance
from the common central bright fringe on the screen 2 m
from the slit will a bright fringe from one interference
pattern coincide with a bright fringe from the other?
9. State Huygen‘s principle. Using this principle draw a
diagram to show how a plane wave front incident at
the interface of the two media gets refracted when it

100
propagates from a rarer to a denser medium. Hence
verify Snell‘s law of refraction.
10. Draw interference of light and diffraction pattern of
light.
11. Illustrate with the help of suitable diagram, action of
the following when a plane wavefront incidents.
(i) a prism (ii) a convex lens and (iii) a concave
mirror.
12. Derive the expression for fringe width in
interference. Show that it is same for bright and dark
fringes
13. Explain Fraunhoffer diffraction at a single slit and
establish the relation for width of central maxima.
14. The figure drawn here, shows a modified Young's double
slit experimental set up in which ss2−ss1=λ/4,
(i) State the condition for constructive and destructive
interference.
(ii) Obtain the expression for fringe width.

15. i) Two monochromatic waves emanating from two


coherent sources have the displacements represented
101
by y1 = a cos ωt and y2 = a cos (ωt + ϕ) where ϕ is
the phase difference between the two displacements.
Show that the resultant intensity at a point due to
their superposition is given by I = 4 I0 cos2 ϕ/2 where
I0 = a2.
ii) Hence obtain the conditions for constructive
and destructive interference.
16. Two wavelength of sodium light 590 nm and 596 nm
are used, in turn, to study the diffraction taking
placed at a single slit of aperture 2 xx 10^(-4)m. The
distance between the slit and screen is 1.5 m.
Calculate the separation between the positions of first
maxima of diffraction pattern obtained in the two
cases.
17. i) In what way is diffraction from each slit related to
the interference pattern in a double slit experiment ?
(ii) When a tiny circular obstacle is placed in the
path of light from a distant source, a bright spot is
seen at the centre of the shadow of the obstacle.
Explain, why.
18. The intensity at the central maxima (O) in a Young's
double slit experiment is I0. If the distance OP equals
one-third of the fringe width of the pattern, show that

the intensity at point P would be I0/4.

102
19. In Young‘s double-slit experiment using
monochromatic light of wavelength λ, the intensity of
light at a point on the screen where path difference is
λ, is K units. What is the intensity of light at a point
where path difference is λ /3?
20. Explain why the maxima at θ = (n + ½) λ/a become
weaker and weaker with increasing n.
21. In Young‘s double slit experiment, the two slits 0. 15
mm apart are illuminated by monochromatic light of
wavelength 450 nm. The screen is 1.0 m away from
the slits.
(a) Find the distance of the second
(i) bright fringe,
(ii) dark fringe from the central maximum.
(b) How will the fringe pattern change if the screen is
moved away from the slits?
22. Use Huygens‘ principle to show how a plane
wavefront propagates from a denser to rarer medium.
Hence verify Snell‘s law of refraction.
23. A plane wavefront is incident at an angle of
incidence i on a reflecting surface. Draw a diagram
showing incident wavefront, reflected wavefront and
verify the laws of reflection.
24. The ratio of the intensities at minima to the maxima
in the Young‘s double slit experiment is 9 : 25. Find
the ratio of the widths of the two slits
25. Is energy conserved in interference? Explain.
26. What is Fresnel distance?
27. Estimate the distance for which ray optics is a good
approximation for an aperture of 4mm and
103
wavelength 400nm.
28. The human eye has an approximate angular
resolution of ɸ = 5.8 x 10-4 rad and a typical
photoprinter prints a minimum of 300 dpi (dots per
inch, 1 inch = 2.54 cm). At what minimal distance z
should a printed page be held so that one does not see
the individual dots?
Multiple Choice Questions
1. Two identical and coherent sources of light are used in
Young's double-slit experiment and resultant intensity at
the centre of the screen is found to be I₁. When two
identical sources of intensity same as before but
incoherent are used for the experiment, then resultant
intensity at the centre of the screen is found to be I₂. What
is the value of I₁/I₂?
a. 1
b. 2
c. 4
d. 0.5
2. Contrast of the fringe pattern obtained in Young's double-
elit experiment depends on
a. wavelength
b. phase difference between sources
c. intensity ratio of the sources.
d. distance between plane of slits and screen
3. In Young's double-slit experiment, coordinate system is
selected in such a manner that Y-coordinate of central
maximum is 1 cm and the same for 9th maximum is 9 cm.
If the entire set-up is immersed in a fluid with refractive

104
index 4/3, then what will be new Y-coordinates of central
maximum and 9th maximum?
a. 1 cm, 9 cm
b. 3/4 cm, 27/4 cm
c. 4/3 cm, 7 cm
d. 1 cm, 7 cm
4. In a standard Young's double-slit experiment set up, two
points P and Q are marked on the screen. Path difference
corresponding to point P is λ/2 and for point Q it is λ/4.
Here λ is wavelength of light being used. If Ip and IQ are
the resultant intensities at points P and Q, then Ip/IQ is
a. 2
b. 1/2
c. 0
d. infinity
5. A source emitting light of wavelengths λ₁ and λ2 is used
in Young's double-slit experiment. If fourth bright of λ₁
coincides with sixth bright of λ2, then
a. 2λ₁ = 3λ₂
b. 3λ₁ = 2λ₂
c. 4λ₁ = 3λ₂
d. 3λ₁ = 4λ₂
6. Select best monochromatic source of light.
a. Bulb
b. Candle
c. Tube light
d. Laser
7. In Young's double-slit experiment, d is separation
between the slits. Separation between plane of the slits

105
and screen is D. Wavelength of light used is λ. Number of
fringes per unit distance on the screen is
a. λ D/d
b. d/ λD
c. λ d/D
d. D/ λd
8. When light ray is refracted from one medium to another,
then which of the following does not change?
a. Amplitude
b. Velocity
c. Frequency
d. Wavelength
9. Two light sources are said to be coherent if they produce
waves
a. of equal wavelength
b. of equal frequency
c. of equal velocity
d. of equal frequency and having constant phase difference
between them.
10. In standard Young's double-slit experiment, 9 fringes
are found to be formed in a portion of screen. Wavelength
of light used is 600 nm. How many fringes will be
formed in the same portion of screen if wavelength of
light is changed to 450 nm?
a. 12
b. 18
c. 14
d. 20
11. Oil drop spread on water surface during the day light
exhibits brilliant colours due to

106
a. Polarisation
b. Scattering
c. Interference
d. reflection
12. If entire Young's double-slit experiment set-up is
immersed in water, then
a. fringe width will remain unaffected
b. fringe width will increase
c. fringe width will decrease
d. experiment cannot be performed in water.
13. What is the shape of wavefront from distant source
of light?
a. Planar
b. Spherical
c. Cylindrical
d. Depends on shape of the source
14. There are two points on same wavefront by a
distance λ/2. What will be the phase difference between
these two points?
a. π
b. π /2
c. π/4
d. 0
15. In Young's double-slit experiment, two coherent
sources of different intensities are used to make
interference pattern. Ratio of the maximum to minimum
intensity of pattern is found to be 25. What will be the
ratio of intensities of sources?
a. 625:1
b. 9:4

107
c. 25:1
d. 5:1
16. Two identical coherent sources are used to perform
interference experiment. Intensity of light at the point of
maxima is found to be Io. If one of the slits is closed, then
what will be intensity of light at the same point?
a. Io
b. 4Io
c. Io/4
d. 2Io
17. Angular width (θ) of central maximum of diffraction
pattern of a single slit does not depend upon
a. distance between slit and screen
b. wavelength of light used
c. width of the slit
d. frequency of light used.
18. The linear width of the diffraction band varies
a. inversely as the wavelength
b. directly as the width of the slit
c. directly as the distance between the slit and the screen
d. inversely as the size of the source from which the slit is
illuminated.
19. A diffraction pattern is obtained by using a beam of
red light. What will happen, if the red light is replaced by
the blue light?
a. bands disappear
b. bands become broader and farther apart
c. no change will take place
d. diffraction bands become narrower and crowded together.

108
20. A parallel beam of a monochromatic light of
wavelength 900 nm passes through a long slit of width
0.4 mm. The angular divergence in which most of the
light is diffracted, is
a. 4.5×10-3 rad
b. 9.0 × 10-³ rad
c. 2.25 x 10-3 rad
d. none of these
21. A beam of light of wavelength 600 nm from a distant
source falls on a single slit 1 mm wide and the resulting
diffraction pattern is observed on a screen 2 m away. The
distance between the first dark fringes on either side of
the central bright fringe is
a. 1.2 cm
b. 1.2 mm
c. 2.4 cm
d. 2.4 mm
22. A single slit of width d is illuminated by violet light
of wavelength 400 nm and the width of the diffraction
pattern is measured as y. When half of the slit width is
covered and other half illuminated by yellow light of
wavelength 600 nm, the width of the diffraction pattern is
a. the pattern vanishes and the width is zero
b.
c. 3y
d. none of the above.
23. In a single slit diffraction pattern, the distance
between the first minimum on the left and the first
minimum on the right is 5 mm. The screen on which the
diffraction pattern is displayed is at a distance of 80 cm
109
from the slit. The wavelength is 6000 Å. The slit width in
(mm) is about
a. 0.576
b. 0.348
c. 0.192
d. 0.096
24. Angular width of central maximum in the
Fraunhoffer's diffraction pattern is measured. Slit is
illuminated by the light of wavelength 6000 Å. If slit is
illuminated by light of another wavelength, angular width
decreases by 30%. Wavelength of light used is
a. 3500 Å
b. 4200 Å
c. 4700 Å
d. 6000 Å
25. For an aperture of size 'a' illuminated by a parallel
beam of light having wavelength λ, the Fresnel distance is
a. λ
b.
c. λ
d.
26. For what distance is ray optics a good approximation
when the aperture is 4 mm wide and the wavelength is
500 nm?
a. 32 m
b. 64 m
c. 16 m
d. 8 m

110
27. What happens if one of the slits, say S1 in Young‘s
double, slit experiment-is covered with a glass plate
which absorbs half the intensity of light from it?
a. The bright fringes become less-bright and the dark
fringes have a finite light intensity
b. The bright fringes become brighter and the dark fringes
become darker
c. The fringe width decreases
d. No fringes will be observed
28. For sustained interference, we need two sources
which emit radiations :
a. of the same intensity
b. of the same amplitude
c. having a constant phase difference
d. None of these
29. The fringe width (β) of a diffraction pattern and the
slit width d are related as:
a. β ∝ d
b. β ∝
c. β ∝ √d
d. β ∝
30. What happens to the interference pattern the two slits
S1 and S2 in Young‘s double experiment are illuminated
by two independent but identical sources?
a. The intensity of the bright fringes doubled
b. The intensity of the bright fringes becomes four times
c. Two sets of interference fringes overlap
d. No interference pattern is observed

111
Assertion Reason Type questions
The questions given below consist of an assertion and the
reason. Use the following key to choose appropriate
answer:

(a) If both assertion and reason are correct and reason is a


correct explanation of the assertion.

(b) If both assertion and reason are correct but reason is not
the correct explanation of assertion.

(c) If assertion is correct but reason is incorrect.

(d) If assertion is incorrect but reason is correct.

(e) If both assertion and reason are incorrect.

1. Assertion: When two light sources are placed near to each


other, energy is distributed non-uniformly around them.
Reason: Light waves from two sources interfere each other
and redistribution of energy takes place due to
phenomenon of interference.
2. Assertion: Energy is created in constructive interference
and energy is destroyed in destructive interference.
Reason: There is maximum intensity at the point of
constructive interference and minimum intensity at the
point of destructive interference.
3. Assertion: Light shows the phenomena of interference
and diffraction.
Reason: Light can behave as particle.
4. Assertion: Corpuscular theory of light cannot explain
change in velocity of light when it changes medium.
112
Reason: According to corpuscular theory of light, speed of
light is more in denser medium than in rarer medium.
5. Assertion: Phase difference between two points separated
by a distance equal to λ/2 on same wavefront is π.
Reason: Path difference (∆x) and phase difference δ are
related as δ = ∆x
6. Assertion: The sun is visible a few minutes before it is
actually above horizon.
Reason: Light rays bend due to diffraction.
7. Assertion: When light ray is refracted in some other
medium, then its frequency remains unchanged.
Reason: Incident ray, refracted ray and normal are coplanar.
8. Assertion: Two light sources with different frequencies
cannot be coherent.
Reason: Phase difference between coherent sources remains
constant with time.
9. Assertion: According to Huygen‘s principle, no backward
wave-front is possible.
Reason: Amplitude of secondary wavelet is proportional to
(1 + cos θ) where θ is the angle between the ray at the
point of consideration and the direction of secondary
wavelet.
10. Assertion: No interference pattern is detected when
two coherent sources are infinitely close to each other.
Reason: The fringe width is inversely proportional to the
distance between the two sources.
11. Assertion: In YDSE, if a thin film is introduced in
front of the upper slit, then the fringe pattern shifts in the
downward direction.

113
Reason: In YDSE if the slit widths are unequal, the minima
will be completely dark.
12. Assertion: The pattern and position of fringes always
remain same even after the introduction of transparent
medium in a path of one of the slits.
Reason: The central fringe is bright or dark depends upon
the initial phase difference between the two coherent
sources.
13. Assertion: To observe diffraction of light, the size of
obstacle/ aperture should be of the order of 10-7 m.
Reason: 10-7 m is the order of wavelength of visible light.
14. Assertion: Colours are seen in thin layers of oil on
the surface of water.
Reason: White light is composed of several colours.
15. Assertion: No interference pattern is detected when
two coherent sources are infinitely close to each other.
Reason: The fringe width is inversely proportional to the
distance between the two sources.
16. Assertion: White light falls on a double slit with one
slit is covered by a green filter. The bright fringes
observed are of green colour.
Reason: The fringes observed are coloured.
17. Assertion: In Young‘s double slit experiment if
wavelength of incident monochromatic light is just
doubled, number of bright fringe on the screen will
increase.
Reason: Maximum number of bright fringe on the screen
is inversely proportional to the wavelength of light used.
18. Assertion: Interference pattern is made by using
yellow light instead of red light, the fringes becomes

114
narrower.
Reason: In YDSE, fringe width is given by β=λD/d
19. Assertion: Coloured spectrum is seen when we look
through a muslin cloth.
Reason: It is due the diffraction of white light on passing
through fine slits.
20. Assertion: Diffraction takes place for all types of
waves mechanical or non-mechanical, transverse or
longitudinal.
Reason: Diffraction‘s effect is perceptible only if
wavelength of wave is comparable to dimensions of
diffracting device.
CASE STUDY BASED QUESTIONS
PARAGRAPH 1: Refraction of Light

Huygens‘ Principle is the basis of wave theory of light. It


tells how a wavefront propagates through a medium.
According to the Huygen's principle, each point on a
wavefront is a source of secondary waves, which add up
to give a wavefront at any later time. This principle can
be used to prove the laws of refraction. The given figure
shows a surface XY separating two transparent media -
medium-1 and medium-2. The lines ab and cd represent
wavefronts of a light wave travelling in medium-1 and
incident on XY. The lines ef and gh represent wavefronts
of the light wave in medium-2 after refraction.

115
QUESTIONS

1. Select the right option in the following:


a. Christian Huygens, a contemporary of Newton's
established the wave theory of light by assuming that
light waves were transverse.
b. Maxwell provided the theoretical evidence that light is
transverse wave.
c. Thomas Young experimentally proved the wave
behaviour of light and Huygens' assumption
d. All the statements given above, correctly answer the
question "What is light?"
2. Speed of light is
a. same in medium-1 and medium-2
b. larger in medium-1 than in medium-2
c. larger in medium-2 than in medium-1
d. different at b and d.
3. The phases of the light wave at c, d,e and f are ϕc, ϕd, ϕe
and ϕf respectively. It is given that ϕc = ϕf.
a. ϕc cannot be equal to ϕd
b. ϕd cannot be equal to ϕe

116
c. (ϕd – ϕf) is equal to (ϕc – ϕe)
d. (ϕd - ϕc) is not equal to (ϕf - ϕe).
4. Light travels as a
a. parallel beam in each medium
b. convergent beam in each medium
c. divergent beam in each medium
d. divergent beam in one medium and convergent beam in
the other medium.
5. Light waves travelling through air strike the surface of
water at an angle. Which of the following statements
about the light's wave properties upon entering the water
is correct?
a. The light's speed, frequency and wavelength all stay the
same
b. The light's speed, frequency and wavelength all change
c. The light's speed and frequency change, but the
wavelength stays the same
d. The light's wavelength and speed change, but the
frequency stays the same.
PARAGRAPH 2: Interference of Light
Interference of visible light is not easy to observe because of
the short wavelength (400 nm - 700 nm). To maintain
stable interference pattern individual waves must
maintain a constant phase relationship with one another.
Light waves whose phase difference is either zero or
constant are known as coherent waves. Sources of light
which emit such waves are called coherent sources. Such
coherent light sources can be obtained by illuminating a
screen containing two narrow slits by a monochromatic
light source.

117
Constructive interference (p = nλ) produces a bright band.
Destructive interference [p = (n + ½) λ] produces a
completely dark band if the amplitudes of the two waves
are equal. Young's formula for bright band separation is β
= λ D/d. where d is the slit separation and D is the
relatively large distance of the bands from the slits.
The given figure shows an interference pattern obtained
in a double slit experiment using light of wavelength 600
nm. Numbers 1, 2, 3, 4 and 5 are marked on five fringes.

QUESTIONS

1. The phase difference between two superposing waves at


the place of constructive interference is
a. a multiple of π
b. an even multiple of π
c. an odd multiple of π
d. a multiple of π/2
2. The path difference between two superposing waves at
the place of destructive interference is
a. m
u
l
t
iple of λ
b. multiple of λ /2
c. even multiple of λ /2
d. odd multiple of λ /2
3. The third order bright fringe is marked
a. 2
b. 3
118
c. 4
d. 5
4. Which fringe results from a phase difference of 4π
between the light waves falling on the two slits?
a. 2
b. 3
c. 4
d. 5
5. If pA and pc represent the path differences between the
waves interfering at points marked 1 and 3 respectively,
then |pA-pc| is equal to
a. 150 nm
b. 200 nm
c. 300 nm
d. 600 nm

Answer Key (1 Mark)


1. Spherical
2. Zero
3. Spherical, plane
4. A cylindrical wave front is obtained at a small
distance from a linear source of light.
5. Spherical wavefront (converging).
6. Diffraction can occur with any kind of wave.
7. 4 Io
8. Fresnel biprism, Young‘s double slit, Laser,
Lloyd‘s mirror
9. No
10. Interference pattern disappears
11. If we replace yellow light with blue light, i.e.,

119
longer wavelength with shorter one, therefore the
fringe width decreases
12. The fringe width is directly proportional to
the wavelength of incident light.
13. The central fringe is white. The fringe closest on
either side of the central white fringe is red and the
farthest will appear blue.
14. The fringe width is not affected by the intensity of
incident light.
15. decrease. as λw=λa/μ
16. the energy is simply redistributed among the bright
and dark fringes which also implies that the energy
is conserved
17. rays are always normal to the wavefront.
18.
19. Two independent sources of same
frequencies can not sunstaine interference just
because they have not constant phase difference
20. In the visible wavelengths of the electromagnetic
spectrum, red, with the longest wavelength, is
diffracted most

Answer Key (2 Marks)


1. They must emit waves continuously of same
wavelengths.
The phase difference between the waves must
be zero or constant
2.

120
3. Coherent sources are defined as the sources in
the which initial phase difference remains
constant. In the case of two independent
sources, the initial phase difference cannot
remain constant because light is emitted due
to millions of atoms and their number goes on
changing in a quite random manner

4. Hint ( β=Dλ/d)
5. Hint ( β=2Dλ/d)
Here, λ=600nm = 6×10-7 m,
d=3mm=3×10-3m
D=3m, (2x)=? D=3m,( 2x)=?
2x=2λD/d==12×10−4m=1.2mm
6. Constructive Interference:
Phase difference : where n is an
integer
Path difference: where n is an integer
Destructive interference:
Phase difference : ( ) , where n is
an integer
Path difference: ( ) , where n is
an integer
7. Wavefront is the locus of all the particles in the
same state of vibration. Hence, the wavefront
also travels along the same direction. The
flow of energy does not happen from the

121
lower potential to higher potential. Therefore,
there is no backward wavefront, flowing from
lower disturbance to higher disturbance.
8. The wave front originating from a point source is
spherical in shape. It is because, the locus of
all such points, which are equidistant from the
point source, is a sphere

.
ii) The wave front originating from a line
source is cylindrical in shape. It is because, all
the points, which are equidistant from the
linear source, lie on the surface of a cylinder

9. 1. Each point on a wavefront may be regarded as


a source of secondary waves
2. The position of the wavefront at a later time is
determined by the envelope of these
secondary wave

10. When coherent sources are placed very close,

122
fringe width is very large and a single fringe
may occupy the whole field of view and the
pattern would not be detected. Similarly, if
the coherent sources are far apart, fringe with
will be very small and the pattern cannot be
detected
11. w1/w2 = I1/I2 =a/b
(Ans 9:1)
12.

13.

(Ans 2:1)

123
14.

15. It is because a broad source is equivalent to a


large number of narrow sources lying close to
each other. Different pairs of narrow sources
will produce their own interference patterns
which will overlap each other
16. A wavefront is defined as a surface of constant
phase. A wavefront is the locus of all points
in the medium that have the same phase.
The ray, at each point of a wavefront, is normal
to the wavefront at that point. The ray
indicates the direction of propagation of wave
while the wavefront is the surface of constant
phase.
17. Dark
18.

19. For first secondary maximum of the diffraction


124
pattern,
d sin θ= 3λ/2
∴d=3λ/2sinθ
= 3×6,500×10−10m2×sin30∘=
1.95×10−6 m
20. Fringe width, β = Dλ/d => β ∝ λ for same D
and d. When the whole apparatus is immersed
in a transparent liquid of refractive index n
=1×3, the wavelength decreases to λ = λ/n
= λ/1.3. so, fringe width decreases to 1/1.3
time.
21. The angular separation of interference
fringes (α=λ/d)) decreases as the distance .d.
between the slits is increased.
22. Energy will appear at some other place where
constructive interference is taking place.
23. (a) Plane wavefront; (b) Plane wavefront.
24. The phenomenon of bending of waves around
the corners of an obstacle or opening placed
in its path is called diffraction. ... But the
wavelengths of the light waves are much
shorter as compared to the size of the
obstacles and openings. That is why radio
waves diffract around the buildings while
light waves do not
25. The frequencies of sound waves lie between 20
Hz to 20 kHz, their wavelength ranges
between 15 m to 15 mm. The diffraction
occurs if the wavelength of waves is nearly
equal to slit width.The wavelength of light

125
waves is 7000 x 10-10 m to 4000 x 10-10 m.
For observing diffraction of light we need
very narrow slit width. In daily life
experience we observe the slit width very near
to the wavelength of sound waves as
compared to light waves. Thus, the diffraction
of sound waves is more evident in daily life
than that of light waves.

26. If the width of the diffraction slit is doubled, the


size of the central diffraction band will
become half and its intensity will become four
times of its original value.
27.

28.

29. Interference Diffraction

126
1 Interference is due Diffraction is due to
to superposition of superposition of the
two distinct waves secondary wavelets
coming from two coming from different
coherent sources. parts of the same
wavefront.
2 Interference fringes Diffraction fringes are not
may or may not be to be of the same width.
of the same width.
3 The intensity of The intensity of minima is
minima is generally never zero.
zero.
4 All bright fringes All bright fringes are not
are of uniform of uniform intensity.
intensity.
30. Conditions for interference. The important
conditions for obtaining interference of light
are :
1. The two sources of light must be coherent. i.e.
they should exist continuous waves of same
wavelength or frequency.
2. The two sources should be monochromatic.
3. The phase difference of waves from two
sources should be constant.
4. The amplitude of waves from two sources
should be equal.
5. The coherent sources must be very close to
each other.

127
Answer key (3 Marks)
1. (i) Angular separation βθ = β/D = λ/d It is
independent of D; therefore, angular separation
remains unchanged if screen is moved away from
the slits. But the actual separation between
fringes β = λD/d increases, so visibility of fringes
increases
(ii) When width of source slit is increased, then
the angular fringe width remains unchanged but
fringes becomes less and less sharp; so visibility
of fringes decreases. If the condition s/S < λ/d
is not satisfied, the interference pattern
disappears.
2. i) Yes, frequency is the property of source.
Hence, frequency does not change when light is
reflected or refracted.
(ii) No, decrease in speed does not imply
reduction in energy carried by light wave.
This is because the frequency does not change
and according to the formula E = hv, energy will
be independent of speed. Energy carried by a
wave depends on the amplitude of the wave, not
on the speed of wave propagation.
3. Ref. to point 5 .
4. Ref to NCERT solved example 10.4
5. .

128
6. Given that, 2d = 1 mm = 1 x 10–3 m, D = 1m

7. Ref to NCERT
8. λ1/λ2=n2/n1=12000/10000=6/5
x=n1λ1/Dd
( )

6mm
9. Ref to point 6 in gist of chapter

129
10.

11. Ref to point 6 in gist of chapter


12. Ref to gist of chapter
13. Ref to gist of chapter
14.

15.

130
16.

17. (a) If slit width in interference Pattern is reduced


to the size of wavelength of light used; the
diffraction will also takes place along with
interference.
(b) The diffraction pattern is itself due to the
interference of wavelength belonging to same. (ii)
Waves diffracted from the edge of the circular
obstacle interfere constructively at the centre of
the shadow producing a bright spot
18.
Fringe width β = λD/d y = β/3 = λD/3d

131
19.

20. Maxima become weaker and weaker with


increasing n. This is because the effective part of
the wavefront contributing to the maxima.
becomes smaller and smaller with increasing n.
21.

132
22. Huygens‘ geometrical construction for a plane
wave propagation. Let AB be a section of primary
wavefront at any instant t. Take points 1, 2, 3, 4,
… on the wavefront AB. Taking each point as
centre, draw spheres of radius r = ct, where c is
the velocity of light in the medium.
Draw a surface A1B1 touching tangentially at the
secondary wavelets in the forward direction. The
surface A1B1 is the secondary wavefront after
time t.

23. Ref. from NCERT text book

133
24.

25. During interference of light, all the bright fringes


formed have maximum and equal intensity and all
dark fringes formed have minimum intensity and
are completely dark. It means the energy is
equally distributed among the bright and the dark
fringes which implies energy is conserved in case
of interference of light.
26. "Fresnel distance is the minimum distance a beam
of light has to travel before its deviation from
straight line path becomes significant".
Fresnel distance (ZF)=a2/λ, Where a=width of the
aperture, λ=wave length

27. Fresnel‘s distance (ZF) is the distance for which


the ray optics is a good approximation.
It is given by the relation (ZF)=a2/λ,,
Where, Aperture width, a = 4 mm = 4 ×10−3 m
Wavelength of light, λ = 400 nm = 400 × 10−9 m
134
Therefore, the distance for which the ray optics is
a good approximation is 40 m.
28.

Answer Key (MCQs)


S. Correct S. Correct S. Correct
N option N opti N opti
O O O
on on
. . .
1 B 16 C 21 D
2 C 17 A 22 B
3 D 18 C 23 C
4 D 11 C 24 B
5 A 12 C 25 B
6 D 13 A 26 A
7 B 14 D 27 A
8 C 15 B 28 C
9 D 19 D 29 B
135
10 A 20 A 30 D
Answer Key (ASSERTION REASON)
S. NO. Correct S. NO. Correct
option option
1 A 11 D
2 D 12 D
3 B 13 A
4 A 14 B
5 D 15 A
6 B 16 C
7 B 17 A
8 A 18 A
9 B 19 B
10 A 20 A
Answer Key (case study based)
S. NO. I Correct S. NO. II Correct
option option
1 B 1 B
2 B 2 D
3 D 3 D
4 A 4 C
5 B 5 C

136
Chapter 11 Dual Nature of Radiation and
Matter
Gist and MLM:
1. Photoelectric Effect The phenomenon of emission of
photoelectron from the surface of metal, when a light
beam of suitable frequency is incident on it, is called
photoelectric effect. The emitted electrons are called
photoelectrons and the current so produced is called
photoelectric current.
Hertz’ Observation The phenomenon of photo electric
emission was discovered in 1887 by Heinrich Hertz
during his electromagnetic wave experiment. In his
experimental investigation on the production of
electromagnetic waves by means of spark across the
detector loop were enhanced when the emitter plate was
illuminated by ultraviolet light from an arc lamp.
Lenard’s Observation Lenard observed that when
ultraviolet radiation were allowed to fall on emitter plate
of an evacuated glass tube enclosing two electrodes,
current flows. As soon as, the ultraviolet radiations were
stopped, the current flows also stopped. These
observations indicate that when ultraviolet radiations fall
on the emitter plate, electrons are ejected from it which
are attracted towards the positive plate by the electric
field.
2. Terms Related to Photoelectric Effects
There are many terms related to photoelectric effects
which are of follow.
(i) Free Electrons In metals, the electrons in the outer
shells (valence electrons) are loosely bound to the atoms,

137
hence they are free to move easily within the metal
surface but cannot leave the metal surface. Such electrons
are called free electrons.
(ii) Electron Emission The phenomenon of emission of
electrons from the surface of a metal is called electron
emission.
(iii) Photoelectric Emission It is the phenomenon of
emission of electrons from the surface of metal when
light radiations of suitable frequency fall on it.
(iv) Work Function The minimum amount of energy
required to just eject an electron from the outer most
surface of metal is known as work function of the metal.

(v) Cut-off Potential For a particular frequency of


incident radiation, the minimum negative (retarding)
potential V0 given to plate for which the photoelectric
current becomes zero, is called cut-off or stopping
potential.

(vi) Cut-off Frequency The minimum frequency of light


which can emit photoelectrons from a material is called
threshold frequency or cut-off frequency of that material.
(vii) Cut-off Wavelength The maximum wavelength of
light which can emit photoelectrons from a material is
called threshold wavelength or cut-off wavelength of that
material.
3. Effect of Intensity of Light on Photo current For a
138
fixed frequency of incident radiation, the photoelectric
current increases linearly with increase in intensity of
incident light.

4. Effect of Potential on Photoelectric Current For a


fixed frequency and intensity of incident light, the
photoelectric current increases with increase in the
potential applied to the collector. When all the
photoelectrons reach the plate A, current becomes
maximum it is known as saturation current.

5. Effect of Frequency of Incident Radiation on


Stopping Potential We take radiations of different
frequencies but of same intensity. For each radiation, we
139
study the variation of photoelectric current against the
potential difference between the plates.

6. Laws of Photoelectric Emission


(i) For a given material and a given frequency of incident
radiation, the photoelectric current number of
photoelectrons ejected per second is directly proportional
to the intensity of the incident light.
(ii) For a given material and frequency of incident
radiation, saturation current is found to be proportional to
the intensity of incident radiation, whereas the stopping
potential is independent of its intensity.
(iii) For a given material, there exists a certain minimum
frequency of the incident radiation below which no
emissions of photoelectrons takes place. This frequency
is called threshold frequency.
Above the threshold frequency, the maximum kinetic
energy of the emitted photoelectron or equivalent
stopping potential is independent of intensity of incident
light but depends only upon the frequency (or
wavelength) of the incident light.
(iv) The photoelectric emission is an instantaneous
140
process. The time lag between the incidence of radiations
and emission of photoelectron is very small, less than
even 10-9 s.
7. Einstein Photoelectric Equation Energy Quantum of
Radiation,

Kmax = hv – Ф0 where, hv = energy of photon and Ф =


work-function
NOTE. According to Planck‘s quantum theory, light
radiations consist of tiny packets of energy called quanta.
One quantum of light radiation is called a photon which
travels with the speed of light.
8. Relation between Stopping Potential (V0) and
Threshold Frequency (v0)

9. Important Graphs related to Photoelectric Effect

141
(iii) Frequency (v) and photoelectric current (I) graph.
This graph shows that the photoelectric current (I) is
independent of frequency of the incident light till
intensity remains constant.

142
(iv) Intensity and stopping potential (V0) graph

(v) Photoelectric current (I) and time lag (t) graph is given by

143
ALL POSSIBLE FORMULA

Short Answer Question (1 marks)


1. Mention one physical process for the release of electron
from the surface of a metal?
2. The maximum kinetic energy of photoelectron is 2.8 eV.
What is the value of stopping potential?
3. Show graphically how the stopping potential for a given
photosensitive surface varies with the frequency of
incident radiations?

144
4. How does the stopping potential applied to a photocell
change if the distance between the light source and the
cathode of the cell is doubled?
5. On what factor does the retarding potential of a photocell
depend?
6. Electron and proton are moving with same speed, which
will have more wavelength?
7. Define intensity of radiation in photon picture of light.
8. The stopping potential in an experiment on photoelectric
effect is 1.5 V. What is the maximum kinetic energy of
the photoelectrons emitted?
9. The maximum kinetic energy of a photoelectron is 3 eV.
What is its stopping potential?
10. The wavelength of electromagnetic radiation is
doubled. What will happen to the energy of photon?
11. If the maximum kinetic energy of electrons emitted
by a photocell is 4 ev. What is the Stopping potential?
12. What is the energy associated in joules with a photon
of wavelength 4000A0?

145
13. The photoelectric cut-off voltage in a certain
experiment is 1.5V. What is the maximum Kinetic
energy?
14. Calculate the work function of a metal in eV. If its
threshold wavelength is 6800A0?
15. What is the momentum of a photon of energy 120
MeV?
16. What is the de-Broglie wavelength (in A0)
associated with an electron accelerated through a
Potential of 100V?
17. Calculate the ratio of de-Broglie wavelength
associated with a deuteron moving with velocity 2V and
an alpha particle moving with velocity V?
18. Two beams, one of red light and other of blue light,
of same intensity are incident on a metallic surface to
emit photoelectrons. Which one of the two beams emits
electrons of greater kinetic energy?
19. What is the stopping potential applied to a photocell,
in which electrons with a maximum kinetic energy of 5.6
eV are emitted.
20. Work functions of cesium m and lead are 2 .14 eV
and 4.25 eV respectively. Which of the two has a higher
threshold wavelength?
21. The stopping potential in an experiment on
photoelectric effect is 2 V. What is the maximum kinetic
energy of the photoelectrons emitted?
Short Answer Type SA-(2 Marks)
1. In the wave picture of light, intensity of light is
determined by the square of the amplitude of the wave.

146
what determines the intensity in the photon picture of
light?
2. Monochromatic light of frequency 6.0 × 1014 Hz is
produced by a laser the power emitted is 2.0 × 10-3 W.
estimate the number of photons emitted per second on an
average by the source.
3. A Proton and an α-particle have the same de-Broglie
wavelength. determine the ratio of
(I) their accelerating potentials
(II) their speeds
4. A Proton and an Alpha particle are accelerated through
the same potential, which one of the two has
(I) greater value of de Broglie wavelength associated with it,
and
(II) less kinetic energy. give reason to support your
answer.
5. Two monochromatic radiation of frequency v1 and v2
(v1 > v2) and having the same intensity are in turn,
incident on a photosensitive surface to cause
photoelectric emission.
explain giving reason in which case
(I) more number of electrons will be emitted and
(II) maximum kinetic energy of the emitted
photoelectrons will be more.
6. To monochromatic radiations, blue and violet of the same
intensity are incident on a photosensitive surface and
cause photoelectric emission. Would
(I) the number of electrons emitted per second and
(II) the maximum kinetic energy of the electrons be
equal in two cases? justify your answer
7. Write the Einstein‘s Photoelectric equation. state clearly
the three salient features observed in Photoelectric effect
which can be explained on the basis of above equation.
147
8. Describe briefly three experimentally observed features
in the phenomenon of Photoelectric effect.
9. An α-particle and a Proton are accelerated through the
same potential difference. Find the ratio of their de-
Broglie wavelengths.
10. The wavelength λ of a photon and the de-Broglie
wavelength of an electron have the same value. show that
energy of photon is (2λ mc / h) times the kinetic energy
of electron, where m, c and h have their usual meaning.
11. When an electron in hydrogen atom jumps from the
third excited state to the ground state, how would the de-
Broglie wavelength associated with the electron change?
justify your answer.
12. State de-Broglie hypothesis.
13. A proton and deutron are accelerated through the
same accelerating potential. which of the two has
(I) greater value of de Broglie wavelength associated with it
and
(II) less Momentum? give reasons to justify your answer
14. A deuteron and an α-particle are accelerated with the
same accelerating potential. which one of the two has
(I) greater value of the de-Broglie wavelength, associated
with it and
(II) less kinetic energy?
15. An electron is revolving around the nucleus with a
constant speed of 2.2 × 108 m/s. find the de- Broglie
wavelength associated with it.
16. An electron is accelerated through a potential
difference of 100 V. what is the de-Broglie wavelength
associated with it? to which part of the electromagnetic
spectrum does this value of wavelength correspond?
17. Find the ratio of de-Broglie wavelengths associated
with
148
(I) Protons, accelerated through a potential of 128 V and
(II) α-particle is accelerated through a potential of 64 V.
18. The ratio between the de-Broglie wavelengths
associated with protons, accelerated through a potential
of 512 V and α-particles, accelerated through a potential
of X volt is found to be one. find the value of X.
19. Calculate the ratio of the accelerating potential
required to accelerate a Proton and an α-particle to have
the same de-Broglie wavelength associated with them.
20. Crystal diffraction experiments can be performed
either by using electrons accelerated through appropriate
voltage or by using X-rays. if the wavelength of these
probes (electron or X-rays) is 1 Å, estimate which of the
two has greater energy.
21. Write Einstein‘s Photoelectric equation. explain the
terms
(I) threshold frequency, and
(II) stopping potential.
22. Compute the typical de-Broglie wavelength of an
electron in a metal at 27° C and compass it with the mean
separation between two electrons in a metal which is
given to be about 2 × 10-10 m.
23. An electron is accelerated through a potential
difference of 100 volts what is the de Broglie wavelength
associated with it? to which part of the electromagnetic
spectrum does this value of wavelength correspond?
24. An electron and a Proton are moving in the same
direction with same kinetic energy. find the ratio of de-
Broglie wavelengths associated with this particles.
25. For what kinetic energy of a Neutron, will the
associated de-Broglie wavelength be 1.32 × 10-10 m?
given that mass of a neutron = 1.675 10-27 kg.

149
26. A proton and deutron are accelerated through the
same accelerating potential. which one of the two has
(a) greater value of de Broglie wavelength associated with it,
and
(b) less Momentum? give reason to justify your answer.
27. Show that the wavelength of electromagnetic
radiation is equal to the de-Broglie wavelength of its
Quantum (Photon). T
28. he threshold frequency for a certain metal is 3.3 x
1014 Hz. If the light of frequency 8.2 x 1014 Hz is
incident on the metal, predict the cut-off voltage for the
photoelectric emission.
29. The work function for a certain metal is 4.2 eV. Will
this metal give photoelectric emission for incident
radiation of wavelength 330 nm?
Long Answer Type LA-(3 Marks)
1. (I) write the important properties of photons which are
used to establish Einstein‘s Photoelectric equation.
(II) use this equation to explain the concept of (a) Threshold
frequency and (b) stopping potential.
2. (I) Describe briefly three experimentally observed
features in the phenomenon of Photoelectric effect.
(II) discuss briefly how wave theory of light cannot explain
this features.
3. A beam of monochromatic radiation is incident on a
photosensitive surface answer the following question
giving reasons.
(I) do the emitted photoelectrons have the same kinetic
energy?
(II) does the kinetic energy of the emitted electrons
depend on the intensity of incident radiation?

150
(III) on what factor does the number of emitted
photoelectrons depend?
4. (I) Why Photoelectric effect cannot be explained on the
basis of wave nature of light? give reasons.
(II) write the basic feature of photon picture of
electromagnetic radiation on which Einstein‘s
Photoelectric equation is based.
5. Define the terms threshold frequency and stopping
potential in the study of photoelectric emission. explain
briefly the reasons why wave theory of light is not able to
explain the observed features in Photoelectric effect?
6. (a) In Photoelectric effect, do all the electrons which
absorb a photon come out as photoelectrons irrespective
of their location? Explain.
(b) a source of light, of frequency greater than the threshold
frequency, is placed at a distance ‗d‘ from the cathode of
a photocell. the stopping potential is found to be V. if the
distance of the light source is produced to d/n (where n >
1), explain the changes that are likely to be observed in
the (i) photoelectric current and (ii) stopping potential.
7. In a plot of photoelectric current versus anode potential,
how does
(i) the saturation current vary with anode potential for
incident radiations of different frequency but same
intensity?
(ii) the stopping potential vary for incident radiation of
different intensities but same frequency? (iii)
photoelectric current vary for different intensities but
same frequency of incident radiations?
justify your answer in each case.

151
8. (I) Write Einstein‘s Photoelectric equation state clearly
how this equation is obtained using the photon picture of
electromagnetic radiation.
(II) write the three salient features observed in Photoelectric
effect which can be explained using this equation.
9. (a) What is Photoelectric effect?
(b)using Photon picture of light show how Einstein‘s
Photoelectric equation can be established.
© write three salient feature of observed in
Photoelectric effect which can be explained using this
equation.
10. Describe briefly the experimentally observed
features in the phenomenon of Photoelectric effect.
11. Write three characteristic features in Photoelectric
effect which cannot be explained on the basis of wave
theory of light, but can be explained only using Einstein‘s
equation.
12. Write the basic features of the photon picture of
electromagnetic radiation on which Einstein‘s
Photoelectric equation is based.
13. Write two characteristic feature of observed in
Photoelectric effect which support the photon picture of
electromagnetic radiation.
14. Two monochromatic radiations of frequency v1 and
v2 (v1 > v2) and having the same intensity are, in turn,
incident on a photosensitive surface to cause
photoelectric emission. Explain, giving reason, in which
case
(i) more number of electrons will be emitted and
(ii) maximum kinetic energy of the emitted
photoelectrons will be more.

152
15. A beam of monochromatic radiation is incident on a
photosensitive surface answer the following questions
giving reasons .
(i) do the emitted photoelectrons have the same kinetic
energy?
(ii) does the kinetic energy of the emitted electrons
depend on the intensity of incident radiation ?
(iii) on what factors does the number of emitted
photoelectrons depend?
16. Light of intensity ‗I‘ and frequency ‗v' is incident on
a photosensitive surface and causes photoelectric
emission. what will be the effect on anode current when
(i) the intensity of light is gradually increased,
(ii) the frequency of incident radiation is increased, and
(iii) the anode potential is increased? in each case, all
other factors remain the same. Explain, giving
justification in each case.
17. State how in a photo-cell, the work function of the
metal influence the kinetic energy of emitted electrons.
(a) if the intensity of incident radiation is doubled what
changes occur in
(i) the stopping potential and
(ii) the photoelectric current?
(b) if the frequency of the incident radiation is doubled what
changes occur in the
(i) stopping potential and
(ii) photoelectric current?
18. In the frequency of the incident radiation of the
cathode of a photocell is doubled how will the following
change.
(i) kinetic energy of the electrons?
(ii) photoelectric current?
153
(iii) stopping potential ?
justify your answer.
19. Radiation of frequency 1015 Hz is incident on three
photosensitive surfaces A, B and C. following
observation are recorded.
surface A . no photo emission occurs.
surface B . photo-emission occurs but the photoelectrons
have zero Kinetic Energy.
Surface C . photo-emission occurs and photoelectrons have
some kinetic energy.
based on Einstein‘s Photoelectric equation explain the three
observations.
20. (I) State two important features of Einstein‘s
Photoelectric equation.
(II) Radiation of frequency 1015 Hz is incident on two
photosensitive surface P and Q. there is no photoemission
from surface P. photoemission occurs from surface Q but
photoelectrons have zero kinetic energy. explain this
observation and find the value of work function for
surface Q.
21. The work function of caesium metal is 2.14 eV. When
light of frequency 6 × 1014 Hz is incident on the metal
surface, photoemission of electrons occurs. What is the
(a) maximum kinetic energy of the emitted electrons
(b) Stopping potential
(c) maximum speed of the emitted photoelectrons?
22. Monochromatic light of wavelength 632.8 nm is
produced by a
helium-neon laser. The power emitted is 9.42 mW.
(a) Find the energy and momentum of each photon in the
light beam

154
(b) How many photons per second, on the average, arrive at
a target irradiated by this beam? (Assume the beam to
have a uniform cross-section which is less than the target
area)
(c) How fast does a hydrogen atom have to travel in order to
have the same momentum as that of the photon?
23. Light of frequency 7.21 x 1014 Hz is incident in a metal
surface. Electrons with a maximum speed of 6.0 x 105
m/s are ejected from the surface. What is the threshold
frequency for photoemission of electrons?
24. Light of wavelength 488 nm is produced by an argon
laser which is used in the photoelectric effect. When light
from this spectral line is incident on the emitter, the
stopping (cut-off) potential of photoelectrons is 0.38 V.
Find the work function of the material from which the
emitter is made.
25. Calculate the
(a) momentum, and
(b) the de Broglie wavelength of the electrons accelerated
through a potential difference of 56 V.
26. What is the.
(a) Momentum,
(b) Speed, and
(c) De Broglie wavelength of an electron with a kinetic
energy of 120 eV.
27. What is the de Broglie wavelength of.
(a) a bullet of mass 0.040 kg travelling at the speed of 1.0
km/s,
(b) a ball of mass 0.060 kg moving at a speed of 1.0 m/s, and
(c) a dust particle of mass 1.0 × 10-9 kg drifting with a speed
of 2.2 m/s?

155
28. An electron and a photon each have a wavelength of
1.00 nm. Find.
(a) Their momenta,
(b) The energy of the photon, and
(c) The kinetic energy of the electron.
29. (a) For what kinetic energy of a neutron will the
associated de Broglie wavelength be 1.40 x 10-10 m?
(b) Also find the de Broglie wavelength of a neutron, in
thermal equilibrium with matter, having an average
kinetic energy of (3/2) kT at 300 K.
30. Show that the wavelength of electromagnetic radiation is
equal to the de Broglie wavelength of its quantum
(photon).
Multiple Choice Questions
1. Protons and alpha particles have the same de-
Broglie wavelength. What is same for both of
them ?
(a) Energy
(b) Time period
(c) Frequency
(d) Momentum
2. Kinetic energy of emitted electrons depends
upon :
(a) frequency
(b) intensity
(c) nature of atmosphere surrounding the
electrons
(d) none of these
3. The work function of photoelectric material is
3.3 eV. The threshold frequency will be equal
to:
(a) 8 × 1014 Hz
(b) 8 × 1010 Hz
(c) 5 × 1010 Hz
156
(d) 4 × 1014 Hz
4. Which of the following shows particle nature of
light?
(a) Refraction
(b) Interference
(c) Polarization
(d) Photoelectric effect
5. The momentum of an electron that emits a
wavelength of 2 Å. will be:
(a) 6.4 × 10-36 kgms-1
(b) 3.3 × 10-24 kgms-1
(c) 3.3 × 10-34 kgms-1
(d) none of these
6. In photo electric emission, for alkali metals the
threshold frequency lies in the:
(a) visible region
(b) ultraviolet region
(c) infrared region
(d) far end of the infrared region
7. What is the de-Broglie wavelength of an
electron accelerated from rest through a
potential difference of 100 volts?
(a) 12.3 Å
(b) 1.23 Å
(c) 0.123 Å
(d) None of these
8. When a yellow light is incident on a surface, no
electrons are emitted while green light can emit
electrons. If the red light is incident on the
surface then:
(a) no electrons are emitted
(b) photons are emitted
(c) electrons of higher energy are emitted
(d) electrons of lower energy are emitted
9. When an electron jumps across a potential
157
difference of 1 V, it gains energy equal to:
(a) 1.602 × 10-19 J
(b) 1.602 × 1019 J
(c) 1.602 × 1024 J
(d) 1 J
10. The de-Broglie wavelength of particle of mass 1
mg moving with a velocity of 1 ms-1, in terms
of Planck‘s constant h, is given by (in metre):
(a) 105 h
(b) 106 h
(c) 10-3 h
(d) 103 h
11. Light of frequency 1.9 times the threshold
frequency is incident on a photosensitive
material. If the frequency is halved and intensity
is doubled, the photocurrent becomes

(a) Doubled
(b) Quadrupled
(c) Halved
(d) zero
12. For a metal having a work function W0, the
threshold wavelength is λ. What is the threshold
wavelength for the metal having work function
2W0?
(a) λ/4
(b) λ/2
(c) 2λ
(d) 4λ
13. The stopping potential V0 for photoelectric
emission from a metal surface is plotted along
with the y-axis and frequency v of incident light
along the x-axis. A straight line is obtained as
shown. Planck‘s constant is given by
158
(a) product of the slope of the line and charge
on the electron
(b) intercept along y-axis divided by the charge
on the electron
(c) product of the intercept along x-axis and
mass of the electron
(d) the slope of the line

14. What is the ratio of the de Broglie wavelengths


proton and an α particle if they are accelerated
by the same potential difference?

(a) 2√2: 1
(b) 3:2
(c) 3√2: 1
(d) 2: 1
15. Which of the following statements is true
regarding the photoelectric experiment?
(a) The stopping potential increases with the
increase in the intensity of incident light.
(b) The photocurrent increases with the
intensity of light.
(c) The photocurrent increases with the
increase in frequency
(d) All of the above
16. Photoelectric emission occurs only when the
incident light has more than a certain minimum
159
(a) power
(b) wavelength
(c) intensity
(d) frequency
17. The work-function of a metal is
(a) the minimum current required to take out
electron from the metal surface
(b) the maximum frequency required to take out
electron from the metal surface
(c) the minimum amount of energy required to
take out the electron from the metal surface
(d) None of these
18. The work function of a metal is independent of
(i) nature of the surface of the metal
(ii) dimensions of the metal
(iii) properties of the metal
(iv) abundance of the metal
(a) (i) only
(b) (i) and (iii)
(c) (ii) and (iii)
(d) (ii) and (iv)
19. The photoelectric current does not depend upon
the
(i) frequency of incident light
(ii) work function of the metal
(iii) stopping potential
(iv) intensity of incident light
(a) (i) and (iv) only
(b) (ii) and (iii) only
(c) (iii) only
(d) (ii) only
20. The stopping potential is directly related to
(a) the work function of the metal
(b) intensity of incident radiation
(c) the saturation current for the given frequency
160
(d) the kinetic energy gained by the
photoelectrons
21. Photoelectric effect shows
(a) wave like behaviour of light
(b) paritcle like behaviour of light
(c) both wavelike and paticle like behaviour
(d) neither wave like nor particle like behaviour
of light
22. In Einstein‘s picture of Photoelectric emission,
the photoelectric emission does not take place
by
(a) continuous emission of energy from
radiation
(b) continuous absorption of energy from
radiation
(c) discrete absorption of energy from radiation
(d) discrete emission of energy from radiation
23. Photons are deflected by
(a) electric field only
(b) magnetic field only
(c) electromagnetic field
(d) None of these
24. Electrically, photons are
(a) positively charged
(b) negatively charged
(c) neutral
(d) strongly charged, may be positive or
negative
25. Of the following properties, the photon does not
possess
(a) rest mass
(b) momentum
(c) energy
(d) frequency
26. The momentum of a photon of wavelength λ is
161
(a) hλ
(b) h/λ
(c) λ/h
(d) h/cλ
27. The photoelectric effect is based on the law of
conservation of
(a) momentum
(b) energy
(c) angular momentum
(d) mass
28. When light is incident on a metal surface the
maximum kinetic energy of emitted electrons
(a) vary with intensity of light
(b) vary with frequency of light
(c) vary with speed of light
(d) vary irregularly
29. The minimum energy required to eject an
electron, from the metal surface is called
(a) atomic energy
(b) mechanical energy
(c) electrical energy
(d) work function
30. The work function for photoelectric effect
(a) is different for different metals
(b) is same for all metals
(c) depends upon the intensity of incident light
(d) depends upon the frequency of incident light
Assertion Reason Type questions
For question numbers 1 to 10, two statements are given-one
labelled Assertion (A) and the other labelled Reason (R).
Select the correct answer to these questions from the codes
(a), (b), (c) and (d) as given below.
a) Both A and R are true and R is the correct explanation of
A

162
b) Both A and R are true but R is NOT the correct
explanation of A
c) A is true but R is false
d) A is false and R is also false
1. Assertion : In process of photoelectric emission, all
emitted electrons do not have same kinetic energy.
Reason : If radiation falling on photosensitive
surface of a metal consists of different wavelength
then energy acquired by electrons absorbing
photons of different wavelengths shall be different.
2. Assertion : Though light of a single frequency
(monochromatic) is incident on a metal, the
energies of emitted photoelectrons are different.
Reason : The energy of electrons emitted from
inside the metal surface, is lost in collision with the
other atoms in the metal.
3. Assertion : Photoelectric saturation current
increases with the increase in frequency of incident
light.
Reason : Energy of incident photons increases with
increase in frequency and as a result photoelectric
current increases.
4. Assertion : Photosensitivity of a metal is high if its
work function is small.
Reason : Work function = hf0 where f0 is the
threshold frequency.
5. Assertion : The kinetic energy of photoelectrons
emitted from metal surface does not depend on the
intensity of incident photon.
Reason : The ejection of electrons from metallic
surface is not possible with frequency of incident
photons below the threshold frequency.
6. Assertion: Photoelectric effect demonstrates the
wave nature of light.
Reason: The number of photoelectrons is
163
proportional to the frequency of light.
7. Assertion: If a proton and electron are moving
same velocity, then wavelength of de-Broglie wave
associated with electron is longer than that
associated with proton.
Reason: The wavelength of de-Broglie wave
associated with a moving particle is inversely
proportional to its mass.
8. Assertion: If a photon and electron have same de-
Broglie wavelength, they will possess equal energy.
Reason: Both the photon and electron possess rest
mass energy.
9. Assertion: Higher the work function of a metal,
greater is the threshold frequency.
Reason: The work function of alkali metals is
usually lower than that for other metals.
10. Assertion: The slope of the plot of Einstein‘s
photoelectric equation is equal to Planck‘s constant.
Reason: The intercept made by the plot of
Einstein‘s photoelectric equation on the frequency
axis is equal to threshold frequency of the metal.
CASE STUDY BASED QUESTIONS
Case 1
Photoelectric effect is the phenomenon of emission of
electrons from a metal surface, when radiations of suitable
frequency fall on them. The emitted electrons are called
photoelectrons and the current so produced is called
photoelectric current.

(i) With the increase of intensity of incident radiations on a


photo tube, the number of photoelectrons emitted per unit
time
(a) increases (b) decreases
(c) remains same (d) none of these

164
(ii) It is observed that photoelectron emission stops at a
certain time t after the light source is switched on. The
stopping potential (V) can be represented as
(a) 2(KEmax/e) (b) (KEmax/e)
(c) (KEmax/3e) (d) (KEmax/2e)
(iii) A point source of light of power 3.2 x 10-3 W emits
monoenergetic photons of energy 5.0 eV and work function
3.0 eV. The efficiency of photoelectron emission is 1 for
every 106 incident photons. Assume that photoelectrons are
instantaneously swept away after emission. The maximum
kinetic energy of photon is
(a) 4 eV (b) 5 eV
(c) 2 eV (d) Zero
(iv) Which of the following device is the application of
Photoelectric effect?
(a) Light emitting diode (b) Diode
(c) Photocell (d) Transistor
(v) If the frequency of incident light falling on a
photosensitive metal is doubled, the kinetic energy of the
emitted photoelectron is
(a) unchanged (b) halved
(c) doubled (d) more than twice its initial value

Case 2
According to Einstein, when a photon of light of frequency v
or wavelength λ is incident on a photosensitive metal surface
of work function ϕ0 here ϕ0< hv (here, h is Planck's
constant), then the emission of photoelectrons takes place.
The maximum kinetic energy of the emitted photoelectrons
is given by Kmax = hv - ϕ0. If the frequency of the incident
light is v0 called threshold frequency, the photoelectrons are
emitted from metal without any kinetic energy. So hv0 = ϕ0.
(i) A metal of work function 3·3 eV is illuminated by light
of wavelength 300 nm. The maximum kinetic energy of
photoelectrons emitted is (taking h = 6·6 x 10-34 Js
165
(a) 0.413 eV (b) 0.825 eV (c) 1.65 eV (d) 1.32 eV
(ii) The variation of maximum kinetic energy (Kmax) of the
emitted photoelectrons with frequency (v) of the incident
radiations can be represented by

(iii) The variation of photoelectric current (i) with the


intensity of the incident radiation (I) can be represented by

166
(iv) The graph between the stopping potential (V0)
and (1/λ) is shown in the figure. ϕ1, ϕ2, and ϕ3 are work
function. Which of the following options is correct?

(a) ϕ1:ϕ2:ϕ3 =1:2:3


(b) ϕ1:ϕ2:ϕ3 =4:2:1
(c) ϕ1:ϕ2:ϕ3 =1:2:4
(d) None of the above.
(v) Which of the following figures represent the variation of
particle momentum and the associated de- Broglie
wavelength?

167
Case 3
According to de-Broglie a moving material particle
sometimes acts as a wave and sometimes as a particle or a
wave is associated with moving material particle which
controls the particle in every respect. The wave associated
with moving material particle is called matter wave or de-
Broglie wave whose wavelength called de-Broglie
wavelength, is given by λ = h/mv.
(i) The dual nature of light is exhibited by
(a) diffraction and photo electric effect
(b) photoelectric effect
(c) refraction and interference
(d)diffraction and reflection.
(ii) If the momentum of a particle is doubled, then its de-
Broglie wavelength will
(a)remain unchanged
(b)become four times
(c) become two times
(d)become half

168
(iii) If an electron and proton are propagating in the form of
waves having the same λ, it implies that they have the same
(a)energy
(b)momentum
(c)velocity
(d)angular momentum
(iv) Velocity of a body of mass m, having de-Broglie
wavelength λ, is given by relation
(a) v = λ h/m
(b) v = λm/h
(c) v = λ/hm
(d) v = h/ λm
5. Moving with the same velocity, which of the following
has the longest de Broglie wavelength?
(a) Electron
(b) α -particle
(c) proton
(d) neutron.
Answer Key (1 Mark)
1. Photoelectric emission.
2. 2.8 V
3. threshold of frequency or cut off frequency

4. Stopping potential does not depend on the intensity of the


light source which changes due to the change in distance
from the light source.

169
5.It depends upon the frequency of the incident light.
6. Since so electron being lighter will have more
wavelength.
7. The photon incident on unit area of a surface in unit time.

8. KEmax = eV0 =>KEmax = 1.5 eV


9. KEmax = 3eV => 3eV = eV0 => V0 = 3V
10. Half
11. 4 Volt
12. 4.96 X 10 -19 joule
13. 2.4 × 10−19J
14. W =1.825 eV
15. 5.92 X 10 -24 Kg m/sec
16. 1.23 A0
17. 1.1
18. Blue Light
19. 5.6 V
20. Threshold wavelength inversely proportional to Work
function.
Hence· cesium has a higher threshold wavelength for
photoelectric emission.
21. KEmax = eV0 = e(2V) = 2 eV
Answer Key (2 Mark)
1. For a given frequency, intensity of light in the photon
picture is determined by
I = Energy of photons / area × time
= n × hv / A × t
Where, n is the number of photons incident normally on
crossing area A in time t.
2. Here frequency v = 6.0 × 1014 Hz and power of laser P =
2.0 × 10-3 W
170
number of photons emitted per second by laser source n = P
/ hv
n = 2.0 × 10-3 / (6.63 × 10-34) × (6.0 × 1014) = 5.03 × 1015
3. As per question λp = λα but λ = h / √2qm V = h/mv
(I) ∴ h/√2empvp = h/√2(2e) (4mp) vα
⇒ 2empvp = 16empVα ⇒ vp/vα = 8/1
(II) Again λ = h/mpvp = h/ mα vα = h/mpvα
⇒ mpvp = mpVα ⇒ vp/ vα = 4/1
4. We know de-Broglie wavelength associated with a
particle of charge q, mass m, accelerated through a
potential V is given by
λ = h/√2 qmV
(I) Here accelerating potential V is constant for Proton as
well as α-particle hence λ ∝ 1 / √qm. As value of ‗qm' is
less for Proton hence de-Broglie wavelength for Proton is
more.
(II) the kinetic energy of charged particle K = qv. as charge
of proton is less than that of α-particle, the kinetic energy
of proton is less.
5. (I) same as intensity is same
(II) hv = Ф0 + KEmax
∴ for given Ф0 (work function of metal)
Kmax increases with v
∴ Maximum kinetic energy of emitted photoelectron will be
more for monochromatic light of frequency v1 (as v1 >
v2).
6. The intensities for both the monochromatic radiation are
same but their frequency are different. This,
(I) the number of electrons ejected in two cases are same
because it in it depends on the number of incident
photons.
(II) As, KEmax = hv - Ф0 [Einstein‘s Photoelectric current]
171
the KEmax of violet radiation will be more.
7. Correct answer
8. Correct answer
9. We have de-Broglie wavelength
λ = h / √2mqv
Where v is potential difference, q is charge of the particle
and m is mass of the particle.
Given, Vα = vp = V
λα / λp = √2mpqpv / √2mαqαv = √mpqp / √mαqα = √1 / √4
× 2 = 1 / 2√2
10. Given that λ is the wavelength of the photon. the de-
Broglie wavelength of the electron is λ = h/mv. kinetic
energy of electron,
Ee = ½ mv2 = ½ m( h/mλ)2 = h2/2mλ2
We know at energy of photon is Ep = hc/ λ
On dividing Eq. (I) by Eq. (II),
Ee / Ep = h2/2m λ2 × λ/hc ⇒ Ep = 2 λmc/h Ee
11. we know that, λ = h/p = h/mv
mv = h/ λ
mvr = hr/ λ = nh/2λ
λ = 2π/nh × hr = 2πr/n
As, r ∝ n2
λ ∝ 1/(n2) = n
Thus, we can say that, λ3 / λ1 = 3/1 ⇒ λ1 = λ3 /3
Thus, wavelength decreases three times as an electron jumps
from third excited state to the ground state.
12. De-Broglie hypothesis a moving object sometimes
acts as a wave and sometimes as a particle or a wave is
associated with the moving particle which control This
particle in every respect. this wave associated with the

172
moving particle is called matter wave or de-Broglie wave.
its wavelength is given by
λ = h/mv
where, h = Planck's Constant, m = mass of object, v =
velocity of the object.
13. (I) de-Broglie wavelength is given by
λ = h/√2mV0q
λ∝ 1/√m
[ since V0 and q are same, because proton are deuteron have
been accelerated same potential and have same charge]
Since, mass of proton is less as compared to a deuteron. so
it will have higher value of de-Broglie wavelength
associated with it.
(I) De-Broglie wavelength is given by
λ = h/p
p = h/λ
As, λd < λp
So, pd > pp
14. (I) De-Broglie wavelength is given by
λ = h/√2mV0q
λ ∝ 1/√mq
λd/ λa = 1/√2me / 1/√4m2e = 2/1
Wavelength of deutrons is 2 times the wavelength of Alpha
particle.
(II) KEd/ KEa = V0e/ V0 2e = ½
KE of deutron is half of KE of Alpha particle.
15. Given, v = 2.2 × 108 m/s
De-Broglie wavelength is given by
λ = h/mv
Here, m = 9.1 × 10-31 kg
h = 6.63 × 10-34 kg-m2-s
173
Substituting all values in Eq. (I), we get
λ = 6.63 × 10-34 / 9.1 × 10-31 × 2.2 × 108
λ = 3.31 × 10-12 m
16. Given, V = 100V. wavelength of accelerated
electron beam from De-Broglie equation
λ = 12.27/√V Å
For V = 100 V
λ = 1.227 Å
this wavelength belongs to the X-ray part of the
electromagnetic radiation.
or 1.02 Å
this wavelength belongs to X-ray part of electromagnetic
spectrum.
Or λ = 1.5 Å and X-ray.
17. De-Broglie wavelength is given by
λ = h/√2mK = h/√2mqV
λ ∝ 1/√mqV
Where m = mass of charged particle, q = charge, V =
potential difference
Therefore ratio of de-Broglie wavelength of proton and
Alpha particle.
λα / λp = √mαqαVα / mpqpvp =√(mα / mp) (qα / qp ) (Vα /
Vp)
Here, mα / mp = 4, qα / qp = 2
[( since Alpha particle is 4 times heavier than proton and it
has double the charge than that of proton)]
Vα / Vp = 64/128 = ½
Therefore λα / λp = √4×2×1/2 = 2 or λα . λp = 2.1
18. De Broglie wavelength of accelerated charged
particle is given by
λ = h/√2mqV
174
λ ∝ 1/√mqV
Ratio of wavelengths of protons and Alpha particle
λα / λp = √(mα / mp) (qα / qp ) (Vα / Vp)
here, mα / mp = 4, qα / qp = 2
Vα / Vp = X/512 λα / λp = 1
1 = √ 4 × 2 × X/512 = X/64
X = 64 V
19. Since De-Broglie matter wave equation for
accelerating charged particle is given by
λ = h/√2mqV
Where, h = Planck‘s Constant
m = mass of charged particle
q = charge of charged pparticle
V = potential difference
λα / λp = √(mα / mp) (qα / qp ) (Vα / Vp)
λα / λp = 1, mα / mp = 4, qα / qp = 2, (Vα / Vp) = ?
1 = √4×2×(Vα / Vp)
1 = 8 × Vα / Vp
(Vα / Vp) = 8
Vα . Vp = 8 . 1
20. For an accelerated electron beam, the de-Broglie
matter Wave Equation states that
λ = h/√2emV = h/√2mK
K = h2 / 2m λ2
For x-ray of same wavelength, λ = 1 Å
E` = hv = hc/ λ
K/E` = h2 / 2m λ2 / hc/ λ2
K/E` = h2 / 2m λ2 × λ2 / hc = h/ 2mcλ
Where, h = 6.6 × 10-34 J-s, c = 3 × 108 m/s
m = 9.1 × 10-31 kg
λ = 1 Å = 1 × 10-10 m
175
K/E` = 6.6 × 10-34 / 2 × 9.1 × 10-31 × 3 × 108 × 1 × 10-10
K/E` = 0.012
K/E` < 1
K < E`
Energy possessed by X-ray is more than electron.
21. Correct answer
22. Electrons in metal behave like an electron gas.
for electron λde Broglie = h/p = h/√3m kB T
= 6.63 × 10-34 / √3 × 9.11 × 10-31 × 1.38 × 10-23 × 300 =
6.2 × 10-9 m
Given r = 2 × 10-10 m
λde Broglie / r = 6.2 × 10-9 / 2 × 10-10 = 31
Therefore, λde Broglie >> mean separation between
electrons in a metal.
23. The accelerating voltage V = 100 V
therefore de-Broglie wavelength of electron
λ = h/√2emV = 6.626 × 10-34 / √2 × 1.602 × 9.1 × 10-31 ×
100
= 1.227 Å
24. We know that De Broglie wavelength λ = h/√m v =
h/ √2m K, where K is the kinetic energy as mp > me ,
hence it is clear that for same kinetic energy.
λe /λp = √mp / me
λe > λp i.e., de Broglie wavelength of electron will be
greater than of proton.
25. It is given that λ = 1.32 × 10-10 m and mn = 1.675 ×
10-27 kg
λ = h/√2mn K
K = h2 / 2mn λ2
Therefore kinetic energy of neutron

176
K = (6.63 × 10-34 )2 / 2 ×(1.675 × 10-27 ) × (1.32 × 10-10
)2 = 7.53 × 10-21 J
26. For an accelerating potential V the de-Broglie
wavelength of a charged particle of charge q and mass m
is given by
λ = h/√2mqV and momentum of particle p = √2mqV
As mass of deuteron is greater than that of proton i.e., mD >
mp , hence we conclude that
(a) de Broglie wavelength of proton is greater than that of
deuteron, and
(b) momentum of proton is less than that of deuteron.

27. Photon has an energy E = h v = hc/ λ hence


Momentum p =E/c = h/ λ
Therefore de Broglie wavelength of photon λde Broglie =
h/p = h/h/ λ = λ
λde Broglie = λem radiation.
28 . Vo = 2.0291 V
29. E = 3.76 V. The energy of the incident radiation is
less than the work function of the metal. Hence, there is
no photoelectric emission taking place.
Answer Key (3 Mark)
1 Correct answer
2 Correct answer
3 Correct answer
4 Correct answer
5 Correct answer
6 Correct answer
7 Correct answer
8 Correct answer
9 Correct answer.
10 Correct answer.
177
11 Correct answer
12 Correct answer
13 Correct answer
14 Correct answer
15 Correct answer
16 Correct answer
17 Correct answer
18 Correct answer
19 Correct answer
20 Correct answer.
21 (a) The maximum energy (kinetic) by the photoelectric
effect.
K = 0.345 eV
Hence, 0.345 eV is the maximum kinetic energy of the
emitted electrons.
(b) For stopping potential Vo, we can write the equation
for kinetic energy as.
K = eVo
= 0.345 V
Hence, 0.345 V is the stopping potential of the material.
(c) Maximum speed of photoelectrons emitted = ν
Therefore, ν = 3.323 x 105 m/s = 332.3 km/s
Hence, 332.3 km/s is the maximum speed of the emitted
photoelectrons.
22 Monochromatic light having a wavelength, λ = 632.8 nm
= 632.8 × 10-9 m
Given that the laser emits the power of, P = 9.42 mW = 9.42
× 10-3 W
Planck‘s constant, h = 6.626 × 10-34 Js
Speed of light, c = 3 × 108 m/s
Mass of a hydrogen atom, m = 1.66 × 10-27 kg
(a) E = 3.141 x 10-19 J
178
P = 1.047 x 10-27 kg m/s
(b) Number of photons/second arriving at the target
illuminated by the beam = n
Assuming the uniform cross-sectional area of the beam is
less than the target area.
Hence, equation for power is written as.
P = nE = 3 x 1016 photons/s
(c) ν = speed of hydrogen atom = 0.621 m/s
23 νo = 4.738 x 1014 Hz
24 Work function = 2.16 eV
25 p = mv
= 9.1 x 10-31 x 4.44 x 106= 4.04 x 10-24 Kg m/s
(b) de Broglie wavelength = 0.1639 nm
26 (a) Momentum of the electron, p = mv = 9.1 × 10-31×
6.496 × 106
= 5.91 × 10-24 kg m/s
(b) speed of the electron, v = 6.496 × 106 m/s
(c) de Broglie wavelength of an electron having a
momentum p, is given as.
= 0.112 nm
27 (a) Mass of the bullet, m = 0.040 Kg
Speed of the bullet, v = 1.0 km/s = 1000 m/s
Planck‘s constant, h = 6.6 x 10-34 Js
de Broglie wavelength of the bullet is given by the
relation .
λ=h/mv = 1.65 x 10-35 m
(b) Mass of the ball, m = 0.060 Kg
Speed of the ball, v = 1.0 m/s
de Broglie wavelength of the ball is = 1.1 x 10-32 m
(c) Mass of the dust particle, m = 1 x 10-9 Kg
speed of the dust particle, v = 2.2 m/s

179
de Broglie wavelength of the dust particle is = 3.0 x
10-25 m
28 (a) The momentum of an elementary particle is given by
de Broglie relation.
= 6.63 x 10-25 Kg m/s
(b) The energy of the photon is 1.243 keV.
(c) The kinetic energy (K) of an electron having
momentum p, is given by the relation.
K = 1.51 eV
1.51eV is the kinetic energy of the electron.
29 (a) kinetic energy of the neutron is 6.75 x 10-21 J or
4.219 x 10-2 eV.
(b) 0.146nm is the de Broglie wavelength of the neutron.
30 The momentum of a photon having energy (hv) is given
as.
p = h/λ
λ = h/p ………. (i)
Where,
λ = wavelength of the electromagnetic radiation
c = speed of light
h = Planck‘s constant
De Broglie wavelength of the photon is given as.
λ =h/mv
But, p = mv
Therefore, λ=h/p ……………(ii)
Where, m = mass of the photon
v = velocity of the photon
From equation (i) and (ii) it can be concluded that the
wavelength of the electromagnetic radiation and the
de Broglie wavelength of the photon are equal.

180
Answer Key (MCQs)
Q.No. Ans Q.No. Ans Q.No. Ans
Q1 D Q11 d Q21 b
Q2 A Q12 b Q22 b
Q3 A Q13 a Q23 d
Q4 D Q14 a Q24 c
Q5 B Q15 b Q25 a
Q6 A Q16 d Q26 b
Q7 B Q17 c Q27 b
Q8 A Q18 d Q28 b
Q9 A Q19 c Q29 d
Q10 B Q20 d Q30 a
Answer Key (ASSERTION REASON)
Q.No. Ans Q.No. Ans
Q1 b Q6 d
Q2 a Q7 a
Q3 d Q8 d
Q4 b Q9 b
Q5 b Q10 a
Answer Key (case study based)
Q1 Ans Q2 Ans Q3 Ans
(i) A (i) b (i) a
(ii) B (ii) c (ii) d
(iii) C (iii) a (iii) b
(iv) C (iv) c (iv) d
(v) D (v) d (v) a

181
Chapter 12&13 Atom and Nuclei
BASIC CONCEPTS/GIST OF CHAPTERS
1. All elements consists of very small invisible particles,
called atom. Every atom is a sphere of radius of the
order of 10-10 m, in which entire mass is uniformly
distributed and negative charged electrons revolve
around the nucleus.
2. Experimental arrangement for α-scattering experiment
and trajectory followed by α –particles

3. Impact parameter perpendicular distance of the velocity


vector of a-particle from the central line of the nucleus
of the atom is called impact parameter (b).

where, K is KE of α-particle, θ is scattering angle, Z is


atomic number of the nucleus and e is charge of
nucleus.
182
4. Basic assumption of Rutherford’s atomic model
(i) Atom consists of small central core, called atomic
nucleus in which whole mass and positive charge is
assumed to be concentrated.
(ii) The size of nucleus is much smaller than the size of
the atom.
(iii) The nucleus is surrounded by electrons and atom is
electrically neutral.
(iv) Electrons revolves around the nucleus and
centripetal force is of eletrostatic nature.
5. Distance of Closest Approach At a certain distance
r0 from the nucleus, whole of the KE of α-particle
converts into electrostatic potential energy and α-
particle cannot go farther close to nucleus, this distance
(r0) is called distance of closest approach.

6. Angle of Scattering Angle by which a-particle gets


deviated from its original path around the nucleus is
called angle of scattering.
7. Drawbacks of Rutherford’s Model
(i) Could not explained stability of atom clearly.
(ii) Unable to explain line spectrum.
8. Bohr’s Theory of Hydrogen Atom Bohr combined
classical and early quantum concepts and gave his
theory in the form of three postulates. These are
(i) Bohr‘s first postulate was that an electron in an atom
could revolve in certain stable orbits without the
emission of radiant energy, contrary to the predictions
of electromagnetic theory.
(ii) Bohr‘s second postulate defines these stable orbits.
This postulate states that the electron revolves around
the nucleus only in those orbits for which the angular
momentum is some integral multiple of h/2π, where h is
183
the Planck‘s constant (= 6.6 x 10-34 J – s). Thus, the
angular momentum (L) of the orbiting electron is
quantised, i. e. L = nh/2π
As, angular momentum of electron (L) = mvr
∴ For any permitted (stationary) orbit, mvr = nh/2π
where, n = any positive integer i.e. 1, 2, 3, ….
It is also called principal quantum number.
(iii) Bohr‘s third postulate states that an electron might
make a transition from one of its specified non-radiating
orbits to another of lower energy. When it does so, a
photon is emitted having energy equal to the energy
difference between the initial and final states.
The frequency of the emitted photon is then given by
hv – Ei– Ef
where, Ei and Ef are the energies of the initial and final
states and Ei > Ef .
9. Limitations of Bohr’s Model
(i) Applicable only for hydrogen like atom.
(ii) Does not explain the fine structure of spectral lines
in H-atom.
(iii) Does not explain about shape of orbit.
10. Energy Level The energy of an atom is the least when
its electron is revolving in an orbit closest to the nucleus
i.e. for which n = 1.
11. The lowest state of the atom is called the ground state,
this state has lowest energy. The energy of this state is -
13.6 eV. Therefore, the minimum energy required to
free the electron from the ground state of the hydrogen
atom is -13.6 eV.
12. (i) Emission Spectrum Hydrogen spectrum consists of
discrete bright lines a dark
background and it is specifically known as hydrogen
emission spectrum.
(ii) Absorption Spectrum There is one more type of
hydrogen spectrum exists where we get dark lines on
184
the bright background, it is known as absorption
spectrum
13. The atomic hydrogen emits a line spectrum consisting
of various series.

185
NUCLEI
1. In every atom, the positive charge and mass are densely
concentrated at the centre of the atom forming its
186
nucleus. More than 99.9% mass of the atom is
concentrated in the nucleus.
2. Atomic Mass Unit (amu) The unit to express atomic
masses is called atomic mass
unit. Atomic mass unit is defined as 1/12 th of the mass
of carbon atom (C12).

3. Composition of Nucleus The composition of a nucleus


can be described by using the following terms and
symbols.
(i) Atomic Number Z Atomic number of an element is
the number of protons present inside the nucleus of an
atom of the element.
Atomic number = Number of protons = Number of
electrons
(ii) Mass Number A Mass number of an element is the
total number of protons and neutrons inside the atomic
nucleus of the element.
Mass number = Number of protons + Number of
neutrons = Number of electrons + Number of neutrons
i.e. A=Z.+ N
4. Size of Nucleus If R is the radius of the nucleus having
mass number A, then

187
5. Nuclear Density Density of nuclear matter is the ratio
of mass of nucleus and its volume.

Nuclear Fission If the nucleus of a heavy atom–such as


uranium–absorbs a neutron, the nucleus can become
unstable and split. This is called nuclear fission. Fission
releases energy in the form of heat.
Nuclear Fusion-Fusion is the opposite reaction of
fission. In fusion, atoms are fused together.
For a fusion reaction to occur, it is necessary to bring two
nuclei so close that nuclear forces become active and glue
the nuclei together.

NUCLEAR FORCES The nuclear force is the force that


binds the protons and neutrons in a nucleus together. This
force can exist between protons and protons, neutrons and
protons or neutrons and neutrons. This force is what holds
the nucleus together.
The charge of protons, which is +1e, tends to push them
away from each other with a strong electric field repulsive
force, following Coulomb‘s law. But nuclear force is strong
188
enough to keep them together and to overcome that
resistance at short range.
Properties of Nuclear Force
 It is attractive in nature but with a repulsive core.
That is the reason that the nucleus is held together
without collapsing in itself.
 The range of a nuclear force is very short. At 1
Fermi, the distance between particles in a nucleus is
tiny. At this range, the nuclear force is much stronger
than the repulsive Coulomb‘s force that pushes the
particles away. However, if the distance is anything
more than 2.5 Fermi, nuclear force is practically non-
existent.
 The nuclear force is identical for all nucleons. It does
not matter if it is a neutron or proton, once the
Coulomb resistance is taken into consideration,
nuclear force affects everything in the same way.
 At a distance of less than 0.7 Fermi, this force
becomes repulsive. It is one of the most interesting
properties of nuclear force, as this repulsive
component of the force is what decides the size of
the nucleus. The nucleons come closer to each other
till the point that the force allows, after which they
cannot come any closer because of the repulsive
property of the force.
FORMULAS CHAPTER-ATOMS AND NUCLEI
SR PHYSIC FORMULA SYMBAL
N AL MEANING
O QUANT
ITY
1 Impact b = Ze2cot(/2)/40(K) K =KE
paramete
r
2 Distance r0 =4kZe2/mv2 K =KE
of closest
189
approach
3 The h= planks
frequenc constant
y (ν) of
the
emitted
photon
4 Bohr's L= angular
quantisat L= , momentum
ion
condition
5 , the h= planks
electron constant
orbits
radii
6 total h= planks
energy of constant
electrone
7 frequenc Lyman R=1.097
y of any series: x107/m
line in a
series
 Balmer
Series:

 Paschen
Series:

 Brackett
Series:

190
 Pfund
Series:

8 AMU

9 nuclear where R0 = a
radius constant =
1.2 fm.
10 mass A= mass no
defect Z- atomic no

Short Answer Question (1 marks)

1. Define mass defect? (1 mark)


2. What is electron volt?
3. Name the series of hydrogen spectrum which has
least wavelength.
4. Why is the classical (Rutherford) model for an atom
of electron orbitting around the nucleus not able to
explain the atomic
5. Write the expression for Bohr‘s radius in hydrogen
atom.
6. Two nuclei have mass numbers in the ratio 1: 2.
What is the ratio of their nuclear densities?
7. Two nuclei have mass numbers in the ratio 1: 8.
What is the ratio of their nuclear radii?
8. Two nuclei have mass numbers in the ratio 8:125.
What is the ratio of their nuclear radii?

191
9. What is angular momentum of an electrone in the
thired orbit of an atom)
10. Why do we use very thin foil in Rutherford
experiment.
11. What is meaning of negative energy of an electron
12. What are stationary orbits?
13. In Rutherford experiment, large angle scattering is
possible due to nucleus Why?
14. Why neutrons with lower energy should be capable
of causing fission
15. What happens when a neutron is absorbed by a
nucleus of an atom of U235?
Short Answer Type SA-(2 Marks)
1. State the postulates of Bohr‘s atomic theory.
2. What are the limitations of Rutherford‘s atomic model?
3. What are the failures of Bohr‘s atomic theory?
4. Sketch the energy level diagram for hydrogen atom.
Mark the transition corresponding to Lyman and Balmer
series.
5. What is the distance of closest approach when a 5.0
MeV proton approaches a gold nucleus?
6. 2.3 eV separates two energy levels in an atom. What is
the frequency of radiation emitted when the atom transits
from the upper level to the lower level?
7. The ground state energy of hydrogen atom is -13.6 ev.
What are the kinetic and potential energy of the electron
in that state?
8. The ground state energy of hydrogen atom is −13.6 eV.
What are the kinetic and potential energies of the
electron in this state?
9. What's the difference between nuclear fission and
nuclear fusion?
10. What is a controlled nuclear reaction? What are uses of
controlled nuclear reaction?

192
Long Answer Type LA-(3 Marks)
1. The radius of the inner most electron orbit of a hydrogen
atom is 5.3x10-11 m. What are the radii of the n = 2 and
n = 3 orbits?
2. The energy level diagram of an element is given below.
Identify, by doing necessary calculations for each ,
which transition corresponds to the emission of a
spectral line of wavelength 102.7 nm.

3. Define atomic mass unit (a.m.u.) and calculate its value


in SI unit of mass. Also find energy equivalent in MeV
corresponding to it.
4. What is the shortest wavelength present in the Paschen
series of spectral lines?
5. A difference of 2.3 eV separates two energy levels in an
atom. What is the frequency of radiation emitted when
the atom makes a transition from the upper level to the
lower level?
6. Describe Rutherford scattering experiment. What are its
observations and conclusion?
7. What are nuclear forces? What are their properties.
8. What is nuclear fission and fusion. What is difference
between them.

Multiple Choice Questions


1. Balmer series lies in which spectrum?
(a) visible
(b) ultraviolet

193
(c) infrared
(d) partially visible, partially infrared
2. According to the Rutherford‘s atomic model, the
electrons inside the atom are
(a) stationary
(b) not stationary
(c) centralized
(d) None of these
3. Rutherford‘s α-particle experiment showed that the
atoms have
(a) Proton
(b) Nucleus
(c) Neutron
(d) Electrons
4. Electrons in the atom are held to the nucleus by
(a) coulomb‘s force
(b) nuclear force
(c) vander waal‘s force
(d) gravitational force
5. The Rutherford α-particle experiment shows that most of
the α-particles pass through almost unscattered while
some are scattered through large angles. What
information does it give?
(a) Atom is hollow.
(b) The whole mass of the atom is concentrated in a
small
centre called nucleus
(c) Nucleus is positively charged
(d) All of the above
6. Electrons in the atom are held to the nucleus by
(a) coulomb‘s force
(b) nuclear force
(c) vander waal‘s force
(d) gravitational force

194
7. Rutherford‘s atomic model was unstable because
(a) nuclei will break down
(b) electrons do not remain in orbit
(c) orbiting electrons radiate energy
(d) electrons are repelled by the nucleus
8. According to Bohr‘s model of hydrogen atom
(a) the linear velocity of the electron is quantised.
(b) the angular velocity of the electron is quantised.
(c) the linear momentum of the electron is quantised.
(d) the angular momentum of the electron is quantised.
9. As the quantum number increases, the difference of
energy between consecutive energy levels
(a) remain the same
(b) increases
(c) decreases
(d) sometimes increases and sometimes decreases.
10. According to the Bohr theory of H-atom, the speed of
the
electron, its energy and the radius of its orbit varies with
the principal quantum number n, respectively, as
(a)
1/n, n2, 1/n2
(b) n, 1/n2, n2
(c) n, 1/n2, 1/n2
(d) 1/n, 1/n2, 1/n2
11. When hydrogen atom is in its first excited level, it‘s
radius is
(a) four times, it ground state radius
(b) twice times, it ground state radius
(c) same times, it ground state radius
(d) half times, it ground state radius.
12. The angular momentum of the electron in hydrogen
atom in the ground state is
(a) 2h
(b) h/2

195
(c) h/2π
(d) h/4π
13. When an electron jumps from the fourth orbit to the
second orbit, one gets the
(a) second line of Paschen series
(b) second line of Balmer series
(c) first line of Pfund series
(d) second line of Lyman series
14. Which of the following series in the spectrum of
hydrogen atom lies in the visible region of the
electromagnetic spectrum?
(a) Paschen series
(b) Balmer series
(c) Lyman series
(d) Brackett series
15. In a hydrogen atom, which of the following electronic
transitions would involve the maximum energy change
(a) n = 2 to n = 1
(b) n = 3 to n = 1
(c) n = 4 to n = 2
(d) n = 3 to n = 2
16. Hydrogen atom excites energy level from fundamental
state to n = 3. Number of spectral lines according to
Bohr, is
(a) 4
(b) 3
(c) 1
(d) 2
17. Bohr‘s atom model is the modification of Rutherford‘s
atom model by the application of
(a) newton‘s theory
(b) huygen‘s theory
(c) maxwell‘s theory
(d) planck‘s quantum theory

196
18. In Bohr‘s model electrons are revolving in a circular
orbits around the nucleus called as
(a) stationary orbits
(b) non radiating orbits
(c) Bohr‘s orbits
(d) all of these
19. According to Bohr the difference between the energies
of the electron in the two orbits is equal to
(a) hν
(b) hc/λ
(c) both (a) and (b)
(d) neither (a) nor (b)
20. The angular momentum of electrons in an atom
produces
(a) magnetic moment
(b) ZEEMAN effect
(c) light
(d) nuclear fission
21. The nuclear radius is of the order of
(a) 10–10 m
(b) 10–6 m
(c) 10–15 m
(d) 10–14 m
22. The radius of a nucleus is
(a) directly proportional to its mass number
(b) inversely proportional to its atomic weight
(c) directly proportional to the cube root of its mass
number
(d) None of these
23. A nuclei having same number of neutron but different
number of protons / atomic number are called
(a) isobars
(b) isomers
(c) isotones
(d) isotopes
197
24. Which one of the following has the identical property
for isotopes?
(a) Physical property
(b) Chemical property
(c) Nuclear property
(d) Thermal property
25. When the number of nucleons in nuclei increases, the
binding energy per nucleon
(a) increases continuously with mass number
(b) decreases continuously with mass number
(c) remains constant with mass number
(d) first increases and then decreases with increase of
mass number
26. Mp denotes the mass of a proton and Mn that of a
neutron.
A given nucleus, of binding energy B, contains Z
protons
and N neutrons. The mass M(N, Z) of the nucleus is
given
by (c is the velocity of light)
(a) M(N, Z) = NMn + ZMp + B/c2
(b) M(N, Z) = NMn + ZMp – Bc2
(c) M(N, Z) = NMn + ZMp + Bc2
(d) M(N, Z) = NMn + ZMp – B/c2
27. Mass energy equation was propounded by
(a) Newton
(b) Madam Curie
(c) C. V. Raman
(d) Einstein
28. Which of the following statement is not true regarding
Einsteins mass energy relation?
(a) Mass disappears to reappear as energy.
(b) Energy disappears to reappear as mass.
(c) Mass and energy are two different forms of the same

198
entity.
(d) Mass and energy can never be related to each other.
29. Nuclear forces are
(a) spin dependent and have no non-central part
(b) spin dependent and have a non-central part
(c) spin independent and have no non-central part
(d) spin independent and have a non-central part
30. Nuclear forces exists between
(a) neutron – neutron
(b) proton – proton
(c) neutron – proton
(d) all of these
Assertion Reason Type questions
Directions: These questions consist of two statements,
each printed as Assertion and Reason. While answering
these questions, you are required to choose any one of
the following four responses.
(a) If both Assertion and Reason are correct and the
Reason is a correct explanation of the Assertion.
(b) If both Assertion and Reason are correct but Reason
is not a correct explanation of the Assertion.
(c) If the Assertion is correct but Reason is incorrect.
(d) If both the Assertion and Reason are incorrect.
1. Assertion : The force of repulsion between atomic
nucleus and α-particle varies with distance according to
inverse square law.
Reason : Rutherford did α-particle scattering
experiment.
2. Assertion : According to classical theory the proposed
path of an electron in Rutherford atom model will be
parabolic.
Reason : According to electromagnetic theory an
accelerated particle continuously emits radiation.
3. Assertion : Bohr had to postulate that the electrons in
stationary orbits around the nucleus do not radiate.
199
Reason: According to classical physics all moving
electrons radiate.
4. Assertion : Electrons in the atom are held due to
coulomb forces.
Reason : The atom is stable only because the centripetal
force due to Coulomb‘s law is balanced by the
centrifugal force.
5. Assertion : Hydrogen atom consists of only one
electron but its emission spectrum has many lines.
Reason : Only Lyman series is found in the absorption
spectrum of hydrogen atom whereas in the emission
spectrum, all the series are found.
6. Assertion : Between any two given energy levels, the
number of absorption transitions is always less than the
number of emission transitions.
Reason : Absorption transitions start from the lowest
energy level only and may end at any higher energy
level. But emission transitions may start from any
higher energy level and end at any energy level below
it.
7. Assertion : In Lyman series, the ratio of minimum and
maximum wavelength is 3/4
Reason : Lyman series constitute spectral lines
corresponding to transition from higher energy to
ground state of hydrogen atom.
8. Assertion : The whole mass of the atom is concentrated
in the nucleus
Reason : The mass of a nucleus can be either less than
or more than the sum of the masses of nucleons present
in it
9. Assertion : Density of all the nuclei is same.
Reason : Radius of nucleus is directly proportional to
the cube root of mass number.
10. Assertion : Neutrons penetrate matter more readily as
compared to protons.
200
Reason : Neutrons are slightly more massive than
protons.
11. Assertion : The mass number of a nucleus is always
less than its atomic number.
Reason : Mass number of a nucleus may be equal to its
atomic number.
12. Assertion : The binding energy per nucleon, for nuclei
with atomic mass number A > 100, decrease with A.
Reason : The forces are weak for heavier nuclei.
13. Assertion : A free neutron decays to a proton but a free
proton does not decay to a neutron. This is because
neutron is an uncharged particle and proton is a charged
particle.
Reason : Neutron has larger rest mass than the proton.
14. Assertion : Energy is released when heavy nuclei
undergo fission or light nuclei undergo fusion.
Reason : For heavy nuclei, binding energy per nucleon
increases with increasing Z while for light nuclei it
decreases with increasing Z.
15. Assertion : Nuclear fusion reactions are considered as
thermos-nuclear reactions
Reason : The source of stellar energy is nuclear fusion
16. Assertion : The mass of an atom is the mass of
nucleus.
Reason : The nucleus is at the centre.
17. Assertion : An atom is electrically neutral.
Reason : Atoms have equal number of protons and
electrons.
18. Assertion : An α-particle is a doubly ionised helium
atom.
Reason : An α-particle carries 2 units of positive
charge.
19. Assertion : Bohr had to postulate that the electrons in
stationary orbits around the nucleus do not radiate.

201
Reason : According to classical physics, all moving
electrons radiate.
20. Assertion : The different lines of emission spectra (like
Lyman, Balmer etc.) of atomic hydrogen gas are
produced by different atoms.
Reason : The sample of atomic hydrogen- gas consists
of millions of atoms.

CASE STUDY BASED QUESTIONS


Case I
Read the passage given below and answer the
questions from 1 to 5:
Neutrons and protons are identical particle in the sense
that their masses are nearly the same and the force,
called nuclear force, does into distinguish them. Nuclear
force is the strongest force. Stability of nucleus is
determined by the neutron proton ratio or mass defect or
packing fraction. Shape of nucleus is calculated by
quadrupole moment and spin of nucleus depends on
even and odd mass number. Volume of nucleus depends
on the mass number. Whole mass of the atom (nearly
99%) is centred at the nucleus.
1. The correct statements about the nuclear force is/are
(a) charge independent
(b) short range force
(c) non-conservative force
(d) all of these.
2. The range of nuclear force is the order of
(a) 2 x 10-10 m
(b) 1.5 x 10-20 m
(c) 1.2 x 10-4 m
(d) 1.4 x 10-15 m
3. A force between two protons is same as the force
between proton and neutron. The nature of the force
is
202
(a) electrical force
(b) weak nuclear force
(c) gravitational force
(d) strong nuclear force
4. Two protons are kept at a separation of 40 A0. Fn is
the nuclear force and Fe is the electrostatic force
between them. Then
(a) Fn <<Fe
(b) Fn= Fe
(c) Fn >> Fe
(d) Fn ≈ Fe
5. All the nucleons in an atom are held by
(a) nuclear forces
(b) vander waal‘s forces
(c) tensor forces
(d) coulomb forces
Case II
Read the passage given below and answer the
questions from 6 to 10:
The spectral series of hydrogen atom were accounted
for by Bohr using the relation

where, R=Rydberg constant = 1.097 x 107 m-1


Lyman series is obtained when an electron jumps to first
orbit from any subsequent orbit. Similarly, Balmer
series is obtained when an electron jumps to 2nd orbit
from any subsequent orbit. Paschen series is obtained
when an electron jumps to 3rd orbit from any subsequent
orbit. Whereas Lyman series in U.V. region, Balmer
series is in visible region and Paschen series lies in
infrared region. Series limit is obtained when n2=∞.
203
1. The wavelength of first spectral line of Lyman series is
(a) 1215.4 A0
(b) 1215.4 cm
(c) 1215.4 m
(d) 1215. 4 mm
2. The wavelength limit of Lyman series is
(a) 1215.4 A0
(b) 511.9 A0
(c) 951.6 A0
(d) 911.6 A0
3. The frequency of first spectral line of Balmer series is
(a) 1.097 x 107 Hz
(b) 4.57 x 1014 Hz
(c) 4.57 x 1015 Hz
(d) 4.57 x 1016 Hz
4. Which of the following transitions in hydrogen atom
emit photon of highest frequency?
(a) n=1 to n=2
(b) n=2 to n=6
(c) n=6 to n=2
(d) n=2 to n=1
5. The ratio of minimum to maximum wavelength in
Balmer series is
(a) 5 : 9
(b) 5 : 36
(c) 1 : 4
(d) 3 : 4
Answer Key (1 Mark)
1. The difference between the sum of the masses of the
neutrons constituting a nucleus and the rest mass of the
nucleus is known as mass defect.
2. 1eV is the unit of energy.1eV=1.6X10-19J.
3. Lyman series
4. The classical method could not explain the atomic
structure as the electron revolving around the nucleus
204
are accelerated and emits energy as the result, the radius
of the circular paths goes on decreasing. Ultimately
electrons fall into the nucleus, which is not in practical.
5.

6.

205
7.

8.

9. using L=nh/2 l=3.15 x 10 -34 Js


10. . In thick foil Kinetic Energy and hens alpha partic will
be absorbed and unable to penetrate
11. It mean electron is bound to nucleus , it cant escape
from atom.
12. A certain discrete non-radiating orbit in which angular
momentum of electron is an integral multiple of h/2
13. Large angle scattering is only due to repulsion of
positive charges.
14. Due to collisions with various nuclei, initial high kinetic
energy of fission neutron decreases. Thus for a
sustained reaction, eve neutrons with lower energy
should be capable of causing fission. Only neutrons can
result in sustained reaction as two or three neutrons are
released for each one absorbed by fission.
15. When a neutron is absorbed by a nucleus of an atom
U235, a U236 isotope is formed. This isotope is highly
unstable which lasts for one millionth of a second and
splits into two equal parts releasing energy of 200MeV.
Answer Key (2 Marks)
1. Refer NCERT
2. can not explain stability of an atom and line spectrum
3. applicable to hydrogen like single electron atoms, could
not explain Zeeman effect, Stark effect, could not tell
anything about relative intensities of various spectral
lines.
206
4. Sketch the energy level diagram for hydrogen atom.
Mark the transition corresponding to Lyman and Balmer
series.
5. Given, E = 5.0 MeV = 5 × 1.6 × 10-13 J = 8 × 10-13 J
For gold, Z1 = 79
For proton, Z2 = 1
E = 1.6 × 10-19 C
1 z z e2
Distance of closest approach, r0  . 1 2
4 0 E

r0 

9  109  79  1  1.6  10 19 
2

8  10 13
 2.27  10 14 m
6. E  2.3ev  2.3  1.6  10 19 J
 3.68  10 19 J
E  h
E 3.68  10 19
or   
h 6.626  10 34
 5.65  1014 Hz
7. The potential energy of electron is given by –
1 e2
Ep  .
4 0 r
and Kinetic energy is
1 1 e2 1
Ek  . .   Ep
2 4 0 r 2
Ep  Ek  13.6ev
 1 
Or, Ep    Ep   13.6ev
 2 
1
Or, Ep  13.6
2
Or, Ep  27.2ev
 Ek  13.6ev
207
8. Ground state energy of hydrogen atom, E = − 13.6 eV
This is the total energy of a hydrogen atom. Kinetic
energy is equal to the negative of the total energy.
Kinetic energy = − E = − (− 13.6) = 13.6 eV
Potential energy is equal to the negative of two times of
kinetic energy.
Potential energy = − 2 × (13.6) = − 27 .2 eV
9. Refer NCERT
10. It is a reaction that takes place under controlled
conditions. It is used in producing electricity and
powering submarines.
Answer key (3 Marks)
1. The radius of the nth orbit of a hydrogen atom
4 0 n 2 h 2
rn 
4 2 mc 2
or, rn  Kn 2
for n = 1, r1 = K × (1)2
or, K = 5.3 × 10-11 m
for n = 2, r2 = K(2)2
= 5.3 × 10-11 × 4
= 2.12 × 10-10 m
for n = 3, r3 = K(3)2
= 5.3 × 10-11 × 9
= 4.77 × 10-10 m
2. Using E2-E1 = hc/λ, Transition D corresponds to λ
=102.7nm

3. Atomic mass unit is defined as th of mass of one


atom.
According to Avogadro‘s hypothesis number of atoms
in 12 g of is equal to Avogadro number. i.e
6.023x1023 .

208
Therefore the mass of one carbon atom = =
1.99 x10-23g = 1.99 x10-26kg

Or, 1a.m.u. = x1.99 x10-26=1.665x10-27kg


Energy equivalent of 1 a.m.u ,

∆m = 1 a.m.u = 1.665x10-27kg

E = ∆m C2 J = 1.665x10-27 x ( 3x108 )2 / 1.6 x 10-


13
MeV = 931.5 MeV
4. Rydberg‘s formula is given as:

Where,
h = Planck‘s constant = 6.6 × 10−34 Js
c = Speed of light = 3 × 108 m/s
(n1 and n2 are integers)
The shortest wavelength present in the Paschen series of
the spectral lines is given for values n1 = 3 and n2 = ∞.

5. Separation of two energy levels in an atom,


E = 2.3 eV
= 2.3 × 1.6 × 10−19
= 3.68 × 10−19 J
Let ν be the frequency of radiation emitted when the
atom transits from the upper level to the lower level.
209
We have the relation for energy as:
E = hv
Where,
h = Planck‘s constant

Hence, the frequency of the radiation is 5.6 × 1014 Hz.


6. In Rutherford  - particle scattering expect high
energy  - particles emitted from radioactive source
polonium falls on a gold foil. In the experiment the
thickness of gold foil taken is of the order of 10-6 m.
The foil is taken thin to avoid multiple scattering of 
- particles. The entire apparatus is placed in a vacuum
chamber to prevent any energy loss of  - particles due
to their collisions with air molecules.

When  - particle beam falls on gold foil, the  -


particles are scattered due to collision with gold atoms.
The scattering takes place in all possible directions.
The number of  - particles scattered in any direction
is counted by scintillation counter.
Observations & Conclusions:
210
1. Mest of the  - particles pass through the gold
foil undeflected. This implies that most part of
the atoms is hollow.
2.  - particles are scattered through all angles.
Some  - particles suffer scattering through
angles more than 90, while a smaller number
retrace their path. This implies that when fast
moving positively charged  - particle come
near gold atom, then a few of them experience
such a repulsive force that they turn back. On
this basis Rutherford concluded that whole of the
positive charge of atom is concentrated in a
small central core called the nucleus.
3. The negative charges do not influence the
scattering process. This implies that nearly
whole mass of atom is concentrated in nucleus.
4. By this experiment the Coulomb‘s square law is
verified.
5. The amount of positive charge in nuclei of
different metals is different.
7. Refer NCERT
8. Refer NCERT

Answer Key (MCQs)


S. Correct S. Correct S. Correct
NO. option NO. option NO. option
1 B 11 A 21 D
2 B 12 C 22 C
3 B 13 B 23 C
4 A 14 B 24 B
5 D 15 B 25 D
6 A 16 B 26 D

211
7 B 17 D 27 D
8 D 18 D 28 D
9 C 19 C 29 B
10 D 20 A 30 d
Answer Key (ASSERTION REASON)
S. Corre S. Corre S. Corre S. Corre
NO ct NO ct NO ct NO ct
. option . option . option . option
1 B 6 A 11 D 16 C
2 D 7 B 12 C 17 A
3 B 8 C 13 D 18 B
4 C 9 A 14 D 19 C
5 B 10 B 15 B 20 B
Answer Key (case study based)
S. NO. I Correct S. NO. II Correct
option option
1 D 1 A
2 D 2 B
3 D 3 D
4 A 4 B
5 A 5 A

212
Chapter 14 Semiconductor Electronics : Material
,Devices and simple circuits
Gist and MLM:
Classification of solids on the basis of conductivity
1. Conductor Conductors are those substances through which
electricity can pass easily, e.g., all metals are conductors.
2. Insulator Insulators are those substances through which
electricity cannot pass, e.g., wood. rubber, mica etc.
3. Semiconductor Semiconductors are those substances whose
conductivity lies between conductors and insulators. e.g.,
germanium, silicon,
Energy Bands of Solids
4. Energy Band
In a crystal due to interatomic interaction valence electrons of
one atom are shared by more than one atom in the crystal.
Now splitting of energy levels takes place. The collection of
these closely spaced energy levels is called an energy band.
5. Valence Band
This energy band contains valence electrons. This band may
be PartIally or completely filled with electrons but never
be empty. The electrons in this band are not capable of
gaining energy from external electric field to take part in
conduction of current.
6. Conduction Band
This band contains conduction electrons. This band is either
empty or Partially filled with electrons. Electrons present
in this band take part in the conduction of current.
Semiconductor is of two type
p-type Semiconductor: majority carriers are holes and
minority carriers are electrons nh>ne. It is formed by
213
adding trivalent impurities atoms like Al, B etc

 At equilibrium nh.ne = ni2


 Minimum energy required to create a hole electron pair is
hυ= Eg where Eg is energy gap.
 Electric current I = eA(ne.ve + nh.vh )
 Mobility of charge carriers, µ = v/E, where E is applied
electric field.
 Electrical conductivity, ζ = 1/ρ= e(μe.ne + μh.nh )
 PN Junction: A p-n junction is an arrangement made by a
close contact of n-type semiconductor and p type
semiconductor.

214
Circuit diagram and VI characteristics for reverse biase

215
 Diode as Rectifier The process of converting alternating
voltage/current into direct voltage/current is called
rectification. Diode is used as a rectifier for converting
alternating current/voltage into direct
current/voltage.There are two ways of using a diode as a
rectifier i.e.
(i)Diode as a Half-Wave Rectifier Diode conducts
corresponding to positive half cycle and does not conduct
during negative half cycle. Hence, AC is converted by
diode into unidirectional pulsating DC. This action is
known as half-wave rectification.
Circuit diagram of p-n junction diode as half-wave
rectifier is shown below:

216
The input and output waveforms have been given below

(ii) Diode as a Full-Wave Rectifier In the full-wave


rectifier, two p-n junction diodes, D1 and D2 are used.
217
The circuit diagram or full-wave rectifier is shown below:

The input and output waveforms have been given below:

218
 Optoelectronic Devices Semiconductor diodes in which
carriers are generated by photons, i.e. photo-excitation,
such devices are known as optoelectronic devices.
These are as follows:
(i) Light Emitting Diode (LED) It is a heavily doped p-n
junction diode which converts electrical energy into light
energy.

 LEDs has the following advantages over conventional


incandescent low power lamps

219
(a)Fast action and no warm up time required
(b)It is nearly monochromatic
(c)Low operational voltage and less power consumed
(d)Fast ON-OFF switching capability.
(ii) Photo diode A photodiode is a special type of junction
diode used for detecting optical
signals. It is a reverse biased p-n junction made from a
photosensitive material.
Its symbol is

220
Short Answer Question (1 marks)
1. Draw the energy band diagram for a p-type
semiconductor..
2. Draw the voltage-current characteristic of a p-n junction
diode in forwarding bias and reverse bias.
3. For a extrinsic semiconductor, indicate on the energy
band diagram the donor and acceptor levels?.
4. Draw the energy band diagram for n-type semiconductor.
5. An AC input signal of frequency 60 Hz is rectified by a
(i) Half wave and a (ii) Full-wave rectifier. Write the
output frequency in each case.
6. Give the ratio of the number of holes and the number of
conduction electrons in an intrinsic semiconductor..
7. What is the depletion region in a p-n junction?.
8. Name an impurity which when added to pure silicon
makes it a
(i) p-type semiconductor (ii) n-type semiconductor

221
9. Which type of biasing gives a semiconductor diode very
high resistance?.
10. Identify the biasing in the figure given below.

11. Draw the circuit symbol of (a) photodiode, and (b)


light-emitting diode.
12. What is the function of a photodiode?
13. When a p-n junction diode is forward biased, how
will its barrier potential be affected?
14. Name the junction diode whose l-V characteristics
are drawn below:

15. How does the width of the depletion region of a p-n


junction vary if the reverse bias applied to it decreases?
16. How does the width of the depletion region of a p-n
junction vary if the reverse bias applied to it decreases?
17. Why is the conductivity of n-type semiconductors
greater than that of p-type semiconductors even when
both of these have the same level of doping?
18. How does the conductance of a semiconducting
material change with rising in temperature?

222
19. How is a sample of an n-type semiconductor
electrically neutral though it has an excess of negative
charge carriers?
20. How is the bandgap, Eg, of a photodiode related to
the maximum wavelength, λm, that can be detected by it?
21. Determine the currents through resistance R of the
circuits (i) and (ii) when similar diodes are connected as
shown in the figure.

22. Can the potential barrier across a p-n junction be


measured by simply connecting a voltmeter across the
junction? (NCERT Exemplar)
Short Answer Type SA-(2 Marks)
1. Draw a labeled circuit diagram of a full-wave rectifier
using a p-n junction.
2. What is a solar cell? How does it work? Give one of its
uses.
3. Draw the output signal in a p-n junction diode when a
square input signal of 10 V as shown in the figure is
applied across it. (CBSE AI 2019)

223
4. The following diagrams, indicate which of the diodes are
forward biased and which are reverse biased.

5. Mention the important considerations required while


fabricating a p-n junction diode to be used as Light-
Emitting Diode (LED). What should be the order of
bandgap of an LED if it is required to emit light in the
visible range?
6. In the given circuit diagram shown below, two p-n
junction diodes D1 and D2 are connected with a resistance
R and a dc battery E as shown. Redraw the diagram and
indicate the direction of flow of appreciable current in the
circuit. Justify your answer.

224
7. The diagram below shows a piece of pure semiconductor
S in series with a variable resistor R and a source of
constant voltage V. Would you increase or decrease the
value of R to keep the reading of ammeter (A) constant
when semiconductor S is heated? Give reason.

8. Two semiconductor materials X and Y showed in the


figure are made by doping germanium crystal with
indium and arsenic respectively. The two are joined end
to end and connected to a battery as shown,
(i) Will the junction be forward or reverse biased?
(ii) Sketch a V-l graph for this arrangement.

9. Draw the output waveform across the resistor (figure).


(NCERT)

225
10. (i) Name the type of a diode whose characteristics
are shown in figure
(a) and figure (b).
(ii) What does point P in figure (a) represent?
(iii) What do the points P and Q in figure (b) represent?
(NCERT Exemplar)

11. Draw a pn junction with reverse bias? Which biasing


will make the resistance of a p-n-junction high
12. Two material bars A and B of the equal area of the
cross-section are connected in series to a dc supply. A is
made of usual resistance wire and B of an n-type
semiconductor.
(i) In which bar is the drift speed of free electrons
greater?
(ii) If the same constant current continues to flow for a
long time, how will the voltage drop across A and B be
affected? Justify each answer. (CBSE Sample Paper
2018-19).
13. Explain how the width of the depletion layer in a p-n
junction diode changes when the junction is (i) forward
biased and (ii) reverse biased. (CBSE Delhi 2018C).

226
14. With the help of necessary circuit diagram, explain
the working of a photo diode used for detecting optical
signals. [All India 2014 C].
15. With the help of circuit diagrams, distinguish
between forward biasing and reverse biasing of p-n
junction diode.
16. Write two characteristics features to distinguish
between n-type and p-type semiconductors. [All India
2012].
17. What are the advantages and disadvantages of
semiconductor devices over vacuum tubes?
18.

.
19. What do you mean by hole in a circuit? Write its two
characteristics?
20. What do you mean by depletion region and potential
barrier in junction diode?
21. graph of potential barrier versus width of depletion
region for an unbiased diode is shown in A. In
comparison to A, graphs B and C are obtained after
biasing the diode in different ways. Identify the type of
biasing in B & C and justify your answer.

227
The graph of potential barrier versus width of depletion
region for an unbiased diode is shown in A. In
comparison to A, graphs B and C are obtained after
biasing the diode in different ways. Identify the type of
biasing in B & C and justify your answer.
22. Why semiconductors are opaque to visible light but
transparent to infrared radiations?
23. V-I characteristics of SI diode is
given.Calculate diode resistance for bias voltage 2V

24. Germanium diode is preferred to a silicon one for


rectifying small voltages.Explain why?
25. The ratio of number of free electron to holes ne/nh
for two different materials A and B 1 and <1respectively.
Name the type of semiconductor to which A and B
belongs.
26. Why is a photodiode operated in reverse bias mode?
Fig. shows reverse bias current, under different
illumination intensities I1,I2, I3 and I4for a given
photodiode. Arrange the intensities I1,I2, I3 and I4 in
decreasing order of magnitude.
228
27. Why are photodiodes used preferably in reverse basis
condition? A Photodiode is fabricated from a
semiconductor with band gap of 2.8 ev. Can it detect a
wavelength using 6000nm? Justify.
28. Germanium and silicon junction diodes are
connected in parallel. A resistance R,a 12 V battery, a
milliammeter (mA)and Key(K) isclosed, a current began
to flow in the circuit.What willbe the maximum reading
of voltmeter connected across the resistance R?
29. How will classify the material on the basis of band
theory of solids?
30. A semiconductor has equal electron and hole
concentrations of 6x108/m3.On doping with a certain
impurity, the electron concentration increases to
9x1012/m3.
Identify the new semiconductor obtained after doping.
Calculate the new hole concentration.

229
Long Answer Type LA-(3 Marks)
1. Define the terms ‗potential barrier‘ and ‗depletion region‘
for a p – n junction diode. State how the thickness of the
depletion region will change when the p-n junction diode
is (i) forward biased and (ii) reverse biased.
2. Explain (i) forward biasing and (ii) reverse biasing of a p-
n junction diode.
3. Draw V-l characteristics of a p-n junction diode. Answer
the following questions, giving reasons:
(i) Why is the current under reverse bias almost
independent of the applied potential up to a critical
voltage?
(ii) Why does the reverse current show a sudden increase
at the critical voltage? Name any semiconductor device
which operates under the reverse bias in the breakdown
region. (CBSEAI 2013)
4. Draw the energy band diagrams of (i) n-type and (ii) p-
type semiconductor at temperature T > 0 K.
In the case of n-type Si semiconductors, the donor energy
level is slightly below the bottom of the conduction band,
whereas in p-type semiconductors, the acceptor energy
level is slightly above the top of the valence band.
Explain what role do these energy levels play in
conduction and valence bands. (CBSE AI 2015 C)
5. Give reasons for the following:
(i) High reverse voltage does not appear across an LED.
(ii)Sunlight is not always required for the working of a
solar cell.(iii) The electric field, of the junction of a
Zener diode, is very high even for a small reverse bias
voltage of about 5 V. (CBSE Delhi 2016C)

230
6. State the reason why the photodiode is always operated
under reverse bias. Write the working principle of
operation of a photodiode. The semiconducting material
used to fabricate a photodiode has an energy gap of 1.2
eV. Using calculations, show whether it can detect light
of a wavelength of 400 nm incident on it. (CBSE Al
2017C)
7. Explain the two processes involved in the formation of a
p-n junction diode. Hence define the term ‗barrier
potential‘. (CBSE Delhi 2017C)
8. Explain briefly how a photodiode operates.
9. Name the p-n junction diode which emits spontaneous
radiation when forward biased. How do we choose the
semiconductor, to be used in these diodes, if the emitted
radiation is to be in the visible region?
10. The figure shows the V-l characteristic of a
semiconductor diode designed to operate under reverse
bias. (CBSE Al 2019)

(a) Identify the semiconductor diode used.


(b) Draw the circuit diagram to obtain the given
characteristics of this device.
(c) Briefly explain one use of this device.
231
11. With the help of a diagram, show the biasing of a
light-emitting diode (LED). Give its two advantages over
conventional incandescent lamps.
12. Explain working of photodiode with diagram. How it
is fabricated?
13. Draw the circuit diagram of a full-wave rectifier and
explain its working. Also, give the input and output
waveforms. (CBSE Delhi 2019)
14. Draw the circuit diagram to show the use of a p-n
junction diode as a half-wave rectifier. Also show the
input and the output voltages, graphically. Explain its
working.
15. Distinguish between conductors, insulators, and
semiconductors on the basis of the band theory of solids.
16. Explain briefly with the help of a circuit diagram
how V-l characteristics of a p-n junction diode are
obtained in (i) forward bias and (ii) reverse bias.
17. (i) Describe briefly with the help of a necessary
circuit diagram, the working principle of a solar cell
(ii) Why are Si and GaAs preferred materials for solar cells?
Explain. (CBSE AI 2011C)
18. Write the two processes that take place in the
formation of a p-n junction. Explain with the help of a
diagram, the formation of depletion region and barrier
potential in a p-n junction. (CBSE Delhi 2017)
Multiple Choice Questions
1. In a semiconductor:
(a ) there are no free electrons at 0 K
(b) there are no free electrons at any temperature
(c) the number of free electrons increases with pressure

232
(d) the number of free electrons is more than that in a
conductor
2. The resistivity of a semiconductor at room temperature
is in between:
(a) 10–2 to 10–5 Ω cm (b) 10–3 to 106 Ω cm
(c) 106 to 108 Ω cm (d) 1010 to 1012 Ω cm
3. Electric conduction in a semiconductor takes place due
to
(a) electrons only
(b) holes only
(c) both electrons and holes
(d) neither electrons nor holes
4. At absolute zero, Si acts as a:
(a) metal (b) semiconductor
(c) insulator (d) none of these
5. A p-type semiconductor is:
(a) positively charged
(b) negatively charged
(c) uncharged
(d) uncharged at 0 K but charged at higher temperatures
6. Let nh and ne be the number of holes and conduction
electrons in an extrinsic semiconductor. Then:
(a) nh > ne
(b) nh = ne
(c) nh < ne
(d) nh ≠ ne
7. A strip of copper and another of germanium are cooled
from room temperature to 80 K. The resistance of:
(a) each of these decreases
(b) copper strip increases and that of germanium
decreases
(c) copper strip decreases and that of germanium
increases
(d) each of these increases

233
8. If a small amount of antimony is added to germanium
crystal:
(a) it becomes a p-type semiconductor
(b) the antimony becomes an acceptor atom
(c) there will be more free electrons than holes in the
semiconductor
(d) its resistance is increased
9. If the energy of a photon of sodium light (λ = 589 nm)
equals the band gap of semiconductor, the minimum
energy required to create hole electron pair:
(a) 1.2 eV
(b) 1.3 eV
(c) 2.1 eV
(d) 3.1 eV
10. In a crystal, the permitted energy states of electrons are
present:
(a) in the conduction band and the forbidden gap
(b) only in the forbidden gap.
(c) in the valence band and conduction band.
(d) in the forbidden gap and the valence band.
11. The forbidden energy band gap in conductors,
semiconductors and insulators are Eg1, Eg2, and Eg3
respectively.
The relation among them is:
(a) Eg1 = Eg2 = Eg3
(b) Eg1 < Eg2 < Eg3
(c) Eg1 > Eg2 > Eg3
(d) Eg1 < Eg2 > Eg3
12. The forbidden energy gap is maximum for:
(a) mercury (b) silicon
(c) diamond (d) silver
13. Forbidden energy gap for diamond is about:
(a) 1 eV
(b) 1.5 eV

234
(c) 6 eV
(d) 0.6 eV
14. In n-type semiconductor when all donor states are
filled, then the net charge density in the donor states
becomes:
(a) 1 (b) < 1, but not zero
(c) > 1 (d) zero
15. The dominant mechanism for motion of charge carriers
in forward and reverse biased silicon p-n junction are:
(a) drift in forward bias, diffusion in reverse bias
(b) diffusion in forward bias, drift in reverse bias
(c) diffusion in both forward and reverse bias
(d) drift in both forward and reverse bias
16. In an unbiased p-n junction, holes diffuse from the p-
region to n region because:
(a) free electrons in the n region attract them
(b) hole concentration in p-region is more as compared
to n region
(c) they move across the junction with same potential
(d) all of these
17. A pure semiconductor:
(a) has low resistance.
(b) is an intrinsic semiconductor.
(c) allows inadequate current to pass through it.
(d) is an extrinsic semiconductor.
18. The intrinsic semiconductor becomes an insulator at:
(a) 0 C (b) −100 C
(c) 300 K (d) 0 K
19. In a pure semiconductor crystal if current flows due to
breakage of crystal bonds, then semiconductor is called
as:
(a)Acceptor
(b) Donor
(c) Intrinsic semiconductor
(d) Extrinsic semiconductor
235
20. Doping materials are called impurities because they
(a) change the number of chaise carriers.
(b) alter the crystal structure.
(c) change chemical properties.
(d) make semiconductor less pure.
21. A semiconductor has equal electron and hole
concentration of 6 10× 8 −3 m . On doping with certain
impurity,
electron concentration increases to 9 10× 12 −3 m . The
new hole concentration is :
(a) 4 × 105 m–5
(b) 4 × 104 m–5
(c) 4 × 105 m–5
(d) 4 × 106 m–5
22. When arsenic is added as an impurity to silicon, the
resulting material is:
(a) n-type conductor
(b) n-type semiconductor
(c) p-type conductor
(d) p-type semiconductor
23. On doping an intrinsic semiconductor with a group V
element, free electrons in the conduction band are the
Majority charge carriers and the resulting
semiconductor has a net:916
(a) negative charge
(b) positive charge
(c) charge zero
(d) negative, zero or positive charge
24. A p-type semiconductor is
i. a silicon crystal doped with arsenic impurity.
ii. a silicon crystal doped with aluminium impurity.
iii. a germanium crystal doped with boron impurity.
iv. a germanium crystal doped with phosphorus
impurity.

236
(a) (i) and (ii) are correct
(b) (ii) and (iii) are correct
(c) (i) and (iv) are correct
(d) only (i) is correct
25. The average value of output direct current in a full wave
rectifier is:
(a) I0 /π
(b) I0 /2
(c) πI0 /2
(d) 2 I0 /π
26. In a p-n junction having depletion layer of thickness
10−6 m the potential across it is 0.1 V. The electric field
is
(a) 107 V/m
(b) 10–6 V/m
(c) 105 V/m
(d) 10–5 V/m
27. The breakdown in a reverse biased p-n junction diode is
more likely to occur due to:
1. large velocity of the minority charge carriers if the
doping concentration is small
2. large velocity of the minority charge carriers if the
doping concentration is large
3. strong electric field in a depletion region if the
doping concentration is small
4. none of these
28. In the half wave rectifier circuit operating from 50 Hz
mains frequency, the fundamental frequency in the
ripple would be:
(a) 25 Hz
(b) 50 Hz
(c) 70.7 Hz
(d) 100 Hz
29. In the middle of the depletion layer of a reverse biased
p-n junction, the:
237
(a) potential is zero.
(b) electric field is zero.
(c) potential is maximum.
(d) electric field is maximum.
30. LED is the abbreviation of:
(a) Light Editing Diode
(b) Light Editing Display
(c) Light Emitting Display
(d) Light Emitting Diode
Assertion Reason Type questions
Directions: In the following questions, a statement of
assertion is followed by a statement of reason. Mark the
correct choice as:
If both assertion and reason are true and reason is the
correct explanation of assertion.
(b) If both assertion and reason are true but reason is not the
correct explanation of assertion.
(c) If assertion is true but reason is false.
(d) If both assertion and reason are false..

1 Assertion: If there is some gap between the


conduction band and the valence band, electrons in
the valence band all remain bound and no free
electrons are available in the conduction band. Then
the material is an insulator.
Reason: Resistance of insulators is very low.
2 Assertion: Diode lasers are used as optical sources in
optical communication.
Reason: Diode lasers consume less energy.
3 Assertion: A pure semiconductor has negative
temperature ocefficient of sriestance.
Reason: In a semiconductor on raising the
temperature, more charge carriers are
released,conductance increases and resistance
decreases.
238
4 Assertion: If the temperature of a semiconductor is
increased then it‘s resistance decreases.
Reason: The energy gap between conduction band
and valence band is very small.
5 Assertion: At a fixed temperature, silicon will have a
minimum conductivity when it has a smaller
acceptor doping.
Reason: The conductivity of an intrinsic
semiconductor is slightly higher than that of a lightly
doped p-type semiconductor.
6 Assertion: An N-type semiconductor has a large
number of electrons but still it is electrically neutral.
Reason: An N-type semiconductor is obtained by
doping an intrinsic semiconductor with a pentavalent
impurity.
7 Assertion: Semiconductors do not Obey‘s Ohm‘s
law.
Reason: Current can not be determined by the rate of
flow of charge carriers.
8 Assertion: The energy gap between the valence band
and conduction band is greater in silicon than in
germanium.
Reason: Thermal energy produces fewer minority
carriers in silicon than in germanium. Semiconductor
Diode
9 Assertion: Two p-n junction diodes placed back to
back, will work as a npn transistor.
Reason: The p-region of two p-n junction diodes
back to back will form the base of npn transistor.
10 Assertion: If the temperature of a semiconductor is
increased then its resistance decreases.
Reason: The energy gap between conduction band
and valence band is very small.
11 Assertion: A p-n junction with reverse bias can be
used as a photo-diode to measure light intensity.
239
Reason: In a reverse bias condition the current is
small but it is more sensitive to changes in incident
light intensity
12 Assertion: When two semiconductors of p and n type
are brought in contact, they form p-n junction which
act like a rectifier.
Reason: A rectifier is used to convent alternating
current into direct current
13 Assertion: The diffusion current in a p-n junction is
from the p-side to the n-side.
Reason: The diffusion current in a p-n junction is
greater than the drift current when the junction is in
forward biased.
14 Assertion: The drift current in a p-n junction is from
the n-side to the p-side.
Reason: It is due to free electrons only.
15 Assertion : A pure semiconductor has negative
temperature coefficient of resistance.
Reason : In a semiconductor on raising the
temperature, more charge carriers are released,
conductance increases and resistance decreases.
16 Assertion : If the temperature of a semiconductor is
increased then its resistance decreases.
Reason : The energy gap between conduction band
and valence band is very small.
17 Assertion : In semiconductors, thermal collisions are
respossible for taking a valence electron to the
conduction band.
Reason : The number of conduction electrons go on
increasing with time as thermal collisions
continuously take place.
18 Assertion : A p-type semiconductors is a positive
type crystal.
Reason : A p- type semiconductor is an uncharged
crystal.
240
19 Assertion : Silicon is preferred over germanium for
making semiconductor devices.
Reason : The energy gap in germanium is more than
the energy gap in silicon.
20 Assertion : Electron has higher mobility than hole in
a semiconductor.
Reason : The mass of electron is less than the mass
of the hole.
CASE STUDY BASED QUESTIONS
Case I
Consider a thin p-type silicon (p-Si) semiconductor wafer.
By adding precisely a small quantity of pentavelent
impurity, part of the p-Si wafer can be converted into n-Si.
There are several processes by which a semiconductor can
be formed. The wafer now contains p-region and n-region
and a metallurgical junction between p-, and n- region. Two
important processes occur during the formation of a p-n
junction: diffusion and drift. We know that in an n-type
semiconductor, the concentration of electrons (number of
electrons per unit volume) is more compared to the
concentration of holes. Similarly, in a p-type semiconductor,
the concentration of holes is more than the concentration of
electrons. During the formation of p-n junction, and due to
the concentration gradient across p-, and n- sides, holes
diffuse from p-side to n-side (p → n) and electrons diffuse
from n-side to p-side (n → p). This motion of charge carries
gives rise to diffusion current across the junction.
(i) How can a p-type semiconductor be converted into n-
type semiconductor?
a) adding pentavalent impurity
b) adding trivalent impurity
c) not possible
d) heavy doping
(ii) Which of the following is true about n type
semiconductor?
241
a) concentration of electrons is less than that of holes.
b) concentration of electrons is more than that of
holes.
c) concentration of electrons equal to that of holes.
d) None of these
(iii).Which of the following is true about p type
semiconductor?
a) concentration of electrons is less than that of holes.
b)concentration of electrons is more than that of
holes.
c)concentration of electrons equal to that of holes.
d)None of these
(iv).Which of the following is the reason about diffusion
current?
a) diffusion of holes from p to n
b)diffusion of electronss from n to p
c) both (a) and (b)
d) None of these
(v). What are the processes that occur during formation of
a p-n junction?
a) drift
b) diffusion
c) both (a) and (b)
d)None of these
Case II
A solar cell is basically a p-n junction which generates emf
when solar radiation falls on the p-n junction. It works on
the same principle (photovoltaic effect) as the photodiode,
except that no external bias is applied and the junction area
is kept much larger for solar radiation to be incident because
we are interested in more power.
p-Si wafer of about 300 µm is taken over which a thin layer
(~0.3 µm) of n-Si is grown on one-side by diffusion
process. The other side of p-Si is coated with a metal (back
contact). On the top of n-Si layer, metal finger electrode (or
242
metallic grid) is deposited. This acts as a front contact. The
metallic grid occupies only a very small fraction of the cell
area (<15%) so that light can be incident on the cell from the
top. The generation of emf by a solar cell, when light falls
on, it is due to the following three basic processes:
generation, separation and collection.
(i) Working principle of solar cell is same as:
a) Photodiode
b) Zener diode
c) LED
d) Half wave rectifier
(ii) Which type of external biasing is applied in Solar cell?
a) Forward
b) Reverse
c) both (a) and (b)
d) No external biasing is applied
(iii) Which of the following is true for a solar cell?
a) hv < E
b) hv>E
c) E = hv
d) None of these

(iv) What is the principle of Solar cell?


(a) Rectifier
(b)Photo Diode
(c) Photovoltaic Effect
(d) LED
(v). In a solar cell,
a) Junction area is large
b) Junction area is small
c) Junction area is negligible
d) None of these
243
Case III
It is a heavily doped p-n junction which under forward bias
emits spontaneous radiation. The diode is encapsulated with
a transparent cover so that emitted light can come out. When
the diode is forward biased, electrons are sent from n → p
(where they are minority carriers) and holes are sent from p
→ n (where they are minority carriers). At the junction
boundary the concentration of minority carriers increases
compared to the equilibrium concentration (i.e., when there
is no bias). Thus at the junction boundary on either side of
the junction, excess minority carriers are there which
recombine with majority carriers near the junction. On
recombination, the energy is released in the form of photons.
Photons with energy equal to or slightly less than the band
gap are emitted. When the forward current of the diode is
small, the intensity of light emitted is small. As the forward
current increases, intensity of light increases and reaches a
maximum. Further increase in the forward current results in
decrease of light intensity.
(i) Which special purpose diode is mentioned above?
a) Solar cell
b)LED
c) Photodiode
d) Zener Diode
(ii). Which of the following device is forward biased?
a) Solar cell
b)LED
c) Photodiode
d) Zener Diode
(iii) Energy is released in the form of
a) Electron
b) Proton
c) Photon
d) None of these

244
(iv). Which of the following is false?
a) As forward current increases, intensity of light
increases.
b) Increase in further forward current decreases the
intensity of light.
c) Photons with energy greater than band gap is emitted.
d) Photons with energy less than band gap is emitted.
(v ) What happens at the junction?
a) Excess minority carriers recombine with majority
majority carriers.
b)Excess majority carriers recombine with majority
majority carriers.
c)Excess minority carriers recombine with majority
minority carriers.
d) None of these
Case IV
A Photodiode is again a special purpose p-n junction diode
fabricated with a transparent window to allow light to fall on
the diode. It is operated under reverse bias. When the
photodiode is illuminated with light (photons) with energy
(hν) greater than the energy gap (E) of the semiconductor,
then electron-hole pairs are generated due to the absorption
of photons. The diode is fabricated such that the generation
of e-h pairs takes place in or near the depletion region of the
diode. Due to electric field of the junction, electrons and
holes are separated before they recombine. The direction of
the electric field is such that electrons reach n-side and holes
reach p-side. Electrons are collected on n-side and holes are
collected on p-side giving rise to an emf. When an external
load is connected, current flows. The magnitude of the
photocurrent depends on the intensity of incident light.
(i) Photo Diode is
a) forward biased
b) reverse biased
c) Not biased
245
d) none of these
(ii). Which of the following is true about photodiode?
a) E > hv
b) E = hv
c) E < hv
d) None of these
(iii) Magnitude of photocurrent depends on
a) Intensity of light
b) Biasing
c) Potential
d) None of these
(iv). Electrons and holes are separated before they
recombine by:
a) Diffusion current
b) Drift current
c) Electric field
d) Electric potential
(v). Direction of electric field is such that
a) electrons reach n- side
b) holes reach p- side
c) Both (a) and (b)
d) holes reach n side
Case V
A pure semiconductor germanium or silicon, free of every
impurity is called intrinsic semiconductor. At room
temperature, a pure semiconductor has very small number of
current carriers (electrons and holes) .Hence its conductivity
is low. When the impurity atoms of valance five or three are
doped in a pure semiconductor, we get respectively n- type
or p- type extrinsic semiconductor. In case of doped
semiconductor ne nh=ni2. Where ne and nh are the number
density of electron and hole charge carriers in a pure
semiconductor. The conductivity of extrinsic semiconductor
is much higher than that of intrinsic semiconductor.
(i). Which of the following statements is not true?
246
a. The resistance of intrinsic semiconductor decreases
with increase of temperature.
b. Doping pures Si with trivalent impurities gives p-
type semiconductors.
c. The majority charges in n- type semiconductors are
holes.
d. A p-n junction can act as semiconductor diode.

(ii). The impurity atoms with which pure Si should be doped


to make a p- type
semiconductor is
a. Phosphorus
b. Boron
c. Arsenic
d. Antimony
(iii). Holes are majority charge carriers in
a. Intrinsic semiconductors.
b. Ionic Solids
c. p- type semiconductors
d. Metals
(iv). At absolute zero, Si acts as
a. Non- metal
b. Metal
c. Insulator
d. None of these
Case VI
p-n junction is a semiconductor diode. It is obtained by
bringing p-type semiconductor in close contact with n- type
semiconductor. A hin layer is developed at the p- n junction
which is devoid of any charge carrier but has immobile ions.
It is called depletion layer. At the junction a potential barrier
appears, which does not allow the movement of majority
charge carriers across the junction in the absence of any
biasing of the junction. p-n junction offers low resistance

247
when forward biased and high resistance when reverse
biased.
(i) In the middle of depletion layer of reverse biased p- n
junction, the
a. Electric field is zero
b. Potential is zero
c. Potential is maximum
d. Electric field is maximum
(ii). The energy band gap is maximum in
a. Metals
b. Superconductors
c. Insulators
d. Semiconductors
(iii). The number of majority carriers crossing the junction
of diode depends primarily
on the
a. Concentration of doping impurities
b. Magnitude of potential barriers
c. Magnitude of the forward bias voltage
d. Rate of thermal generation of electron –hole pairs
(iv). Hole is
a. Antiparticle of electron
b. A vacancy created when an electron leaves
covalent bond
c. Absence of free electrons
d. An artificially created particle.
Case VII
It is a heavily doped p-n junction which under forward bias
emitsspontaneous radiation. The diode is encapsulated with
a transparent cover so that emitted light can come out.When
the diode is forward biased, electrons are sent from n → p
(where they are minority carriers) and holes are sent from p
→ n (where they are minority carriers). At the junction
boundary the concentration of minority carriers increases
compared to the equilibrium concentration (i.e., when there
248
is no bias). Thus at the junction boundary on either side of
the junction, excess minority carriers are there which
recombine with majority carriers near the junction. On
recombination, the energy is released in the form of photons.
Photons with energy equal to or slightly less than the band
gap are emitted. When the forward current of the diode is
small, the intensity of light emitted is small. As the forward
current increases, intensity of light increases and reaches a
maximum. Further increase in the forward current results in
decrease of light intensity.
(i). In the depletion region of a diode
a. There are no mobile charges
b. Equal number of holes and electrons exist, making
the region neutral.
c. Recombination of holes and electrons has taken
place.
d. Immobile charge ions exist.
(ii).When a p-n junction diode is reverse biased then
a. No Current flows
b. The depletion reason is increased
c. The depletion reason is reduced
d. Height of potential barrier is reduced
(iii). Diode is used as
a. Oscillator
b. Amplifier
c. Rectifier
d. Modulator
(iv).Which one statement is incorrect?
a. Diode is used as rectifier
b. Diode is used as half wave rectifier
c. Diode is used as Amplifier
d. Diode is used as full wave rectifier

Case VIII

249
A pure semiconductor in which no impurity of any sort has
been mixed, is called intrinsic semiconductor. Germanium
(Eg = 0.72 eV) and silicon (Eg = 1.1 eV) are intrinsic
semiconductors. In an intrinsic semiconductor the number of
free electrons in conduction band ne is exactly equal to the
number of holes nh in valence band. Thus, ne = nh = ni
where ni is called the number density of intrinsic carriers. At
0 K these behave as 100% insulators. But at any other
temperature they have thermally generated charge carriers
and thus behave as semiconductor. Conductivity of an
intrinsic semiconductor is s = e(ne me + nh mh), where ne is
free electron density, nh is the hole density and me and mh
are their respective mobilities. Electrical conductivity of
pure semiconductor is very small.
To prepare a n-type semiconductor a pentavalent impurity,
eg., P, As, Sb is used as a dopant with Si or Ge. Such an
impurity is called donor impurity because each dopant atom
provides one free electron. In n-type semiconductor ne >>
nh, i.e., electrons are majority charge carriers and the holes
are minority charge carriers such that ne × nh = ni 2. A n-
type semiconductor is electrically neutral and is not
negatively charged. To prepare a p-type semiconductor a
trivalent impurity, eg., B, Al, In, Ga, etc. is used as adopant
with Si or Ge. Such an impurity is called acceptor impurity
as each impurity atom wants to accept an electron from the
crystal lattice. Th us, eff ectively each dopant atom provides
a hole. In p-type semiconductor nh >> ne, ie, holes are
majority charge carriers and electrons minority charge
carriers such that nh × ne = ni 2 A p-type semiconductor is
electrically neutral and is not positively charged.
(i). In a semiconductor
(a) there are no free electrons at 0 K
(b) there are no free electrons at any temperature
(c) the number of free electrons increases with pressure

250
(d) the number of fre electrons is more than that in a
conductor
(ii) Let nh and ne be the number of holes and conduction
electrons in an extrinsic semiconductor. Then
(a) nh > ne (b) nh = ne
(c) nh < ne (d) none of these
(iii). A p-type semiconductor is
(a) positively charged
(b) negatively charged
(c) uncharged
(d) uncharged at 0K but charged at higher
temperatures
(iv). Electric conduction in a semiconductor takes place due
to
(a) electrons only
(b) holes only
(c) both electrons and holes
(d) neither electrons nor holes
(v). The impurity atoms with which pure silicon may be
doped to make it a p-type semiconductor are those of
(a) phosphorus
(b) boron
(c) antimony
(d) nitrogen

Case IX
In half-wave rectifier only one diode is used. In it no current
flow takes place and no output signal is obtained.
Even during one half cycle the output obtained is a mixture
of dc and ac.
Effective AC component of voltage
The ripple factor =1.21 or 121%
1.21or 121% Effective DC
In full-wave rectifier, two p-n junction diodes have been
joined in complimentary modes. In this
251
Rectifier, we obtain a continuous unidirectional current
through the load resistor RL.
(i) Barrier potential of a P-N junction diode does not
depend on
(a) doping density (b) diode design
(c) temperature (d) forward bias
(ii) In a half wave rectifier, the r.m.s. value of the
a.c. component of the wave is
(a) equal to d.c. value
(b) more than d.c. value
(c) less than d.c. value
(d) zero
Case X
A p-n junction is obtained by joining a small p-type crystal
with a n-type crystal without employing any other binding
material in between them. Whenever a p-n junction is
formed, electrons from n-region diff use through the
junction into p-region and the holes from p-region diff use
into n-region. As a result neutrality of both n and p-regions
is disturbed and a thin layer of immobile negative charged
ions appear near the junction in the p-crystal and a layer of
positive ions appear near the junction in n- crystal. Th is
layer containing immobile ions is called depletion layer. The
thickness of depletion layer is approximately of the order of
10–6 m.
The potential difference created across the p-n junction due
to diffusion of electrons and holes is called the potential
barrier Vb (or emfoffictitious battery). For germanium diode
barrier potential to 0.3 V but for Si diode its value is 0.7 V.
Th e barrier electric field developed due to it
(i) Silicon is doped with which of the following to
obtain P type semiconductor
a) Phosphorus b)
Gallium

252
c) Germanium d)
Bismuth
(ii) What happens to resistance of an intrinsic
semiconductor when heated
a) increases b)
remains constant
c) decreases d)
decreases linearly
(iii) In an unbiased p-n junction, holes diffuse from the p-
region to n-region because
(a) free electrons in the n-region attract them.
(b) they move across the junction by the potential
difference.
(c) hole concentration in p-region is more as
compared to n-region.
(d) All the above.
(iv) When a forward bias is applied to a p-n junction, it
(a) raises the potential barrier. (b)
reduces the majority carrier current to zero.
(c) lowers the potential barrier. (d)
None of the above.
(v) In a PN-junction diode
(a) The current in the reverse biased condition is
generally very small
(b) The current in the reverse biased condition is
small but the forward biased current is independent
of the bias voltage
(c) The reverse biased current is strongly dependent
on the applied bias voltage
(d) The forward biased current is very small in
comparison to reverse biased current μA ~

253
Answer Key (1 Mark)
1. The energy level diagram is shown below.

2. The characteristics are as shown.

3. N-type Extrinsic Semiconductor P-type Extrinsic


Semiconductor

4. The diagram is as shown.

5. The output frequency remains the same in a half-wave


rectifier, i.e. 60 Hz.
254
The output frequency becomes twice the input frequency
in the case of the full-wave rectifier, i.e. 120 Hz.
6. The ratio is one.
7. It is a thin layer between p and n sections of the p-n
junction which is devoid of free electrons and holes.
8. 1) Boron, aluminum, etc.
ii) Phosphorous, antimony, etc.
9. Reverse biasing.
10. Forward biasing.
11. The circuit symbols are as shown below.

12. It functions as a detector of optical signals.


13. Potential barrier decreases in forwarding bias.
14. Solar cell.
15. With the increase in the reverse bias, the depletion
layer increases.
16. If the reverse bias decreases, the width of the
depletion layer also decreases.
17. It is because in n-type the majority carriers are
electrons, whereas in p-type they are holes. Electrons
have greater mobility than holes.
18. Increases with an increase in temperature.
19. It is because it contains an equal number of electrons
and protons and is made by doping with a neutral
impurity.
255
20. Eg = hc/λm
21. In figure (i) D1&D2 are forward biased
In figure (ii) D1 is forward biased but D2 is reverse
biased due to which offers infinite resistance
22. No, because the voltmeter must have a resistance
very high compared to the junction resistance, the latter
being nearly infinite.
Answer Key (2 Marks)
1. Ref. from NCERT.
2. It is a p-n junction used to convert light into electrical
energy. In such a diode, one region either the p-type or
the n-type is made so thin that light falling on the diode is
not absorbed appreciably before reaching the junction.
The thin region in the solar cell is called the emitter and
the other is called the base. The magnitude of current
depends upon the intensity of light reaching the junction.
A solar cell can be used to charge storage batteries during
the daytime, which can be used during the night.
These are used as power supplies for satellites and space
vehicles.
3. The diode will conduct only when it is forward biased.
Therefore, till the input voltage is + 5 V, we will get an
output across R, accordingly the output waveform shown
in the figure.

4. (a) Forward biased.


(b) Reverse biased.
256
(c) Forward biased,
(d) Reverse biased.
5. The important considerations are
1. It should be heavily doped.
2. The diode should be encapsulated with a transparent
cover so that emitted light can come out.
The semiconductor used for the fabrication of visible LEDs
must at least have a bandgap of 1.8 eV.
6. The redrawn diagram showing the flow of appreciable
current is shown below.

Here diode D2 is forward biased, hence it conducts.


Therefore appreciable current will pass through it.
However, diode 0, is reverse biased, hence negligible
current will flow through it.
7. When a semiconductor is heated, its resistance decreases.
As a result, the total resistance of the circuit will
decrease. In order to maintain constant current flow, the
total resistance of the circuit must remain constant.
Hence, the external resistance has to be increased to
compensate for the decrease of resistance of the
semiconductor.
8. Material X is p-type and material Y is n-type.
(i) The junction is reverse biased.
(ii) For the V-l graph

257
The characteristics are as shown.

9. It is a half-wave rectifier, therefore only the positive


cycle will be rectified. Thus the output waveform is as
shown.

10. (i) Zener junction diode and solar cell.


(ii) Zener breakdown voltage
(iii) P-open circuit voltage.
Q-short circuit current
11. Reverse biasing will make the resistance high as it

will not allow the current to pass.


12. (i) Drift speed in B (n-type semiconductor) is higher.
Reason: Since the two bars A and B are connected in
258
series, the current through each is the same.
Now l = neAvd
Or
vd = 1/neA ⇒ vd ∝ 1/n (As l and A are same).
As n is much lower in semiconductors, drift velocity will be
more.
13. The width of the depletion region in a p-n junction
diode decreases when it is forward biased because the
majority of charge carriers flow towards the junction.
While it increases when the junction diode is reverse
biased because the majority of charge carriers move away
from the junction
14. Circuit diagram of illuminated photo diode in reverse
bias is shown below:

259
15. Circuit diagram of forward biased and reverse biased
p-n junction diode ref. NCERT
The width of depletion layer (a) decreases in forward
bias.
(b) increases in reverse bias.
(ii)(a)Differences between forward and reverse biases are
given below:

260
16.

261
17. Advantages – Semiconductor devices are very small
in size as compared to the vacuum tubes. It requires low
voltage for their operation
Disadvantage – Due to the rise in temperature and by
applying high voltage it can be damaged.
18. When photo diode is illuminated with light due to
breaking of covalent bonds, equal number of additional
electrons and holes comes into existence whereas
fractional change in minority charge carrier is much
higher than fractional change in majority charge carrier.
Since, the fractional change of minority carrier current is
measurable significantly in reverse bias than that of
forward bias. Therefore, photo diode are connected in
reverse bias.
19. A vacancy created in a covalent bond in a
semiconductor due to the release of electron is known as
hole in a semiconductor.
Characteristics of hole
(i) Hole is equivalent to a positive electronic charge.
(ii) Mobility of hole is less than that of an electron
20. A layer around the junction between p and n-sections
of a junction diode where charge carriers electrons and
holes are less in number is called depletion region. The
potential difference created across the junction due to the

262
diffusion of charge carriers across the junction is called
potential barrier.
21. B : reverse biased, C: forward biased
22. The photons of infrared radiation have smaller
energies,so they fall to excite the electrons in the valence
band. Hence infrared radiations pass through the
semiconductors as such; i.e. a semiconductor is
transparent to infrared radiation.
23. R= V/I= 2/70 x 103Ohms
24. Because the energy gap for Ge ( Eg = 0.7 ev) is
smaller than the energy gap for Si (Eg = 1.1ev ) .
Moreover, the germanium diode is much more open to
the Danger of high temperature affect than silicon at
high voltage.
25. If ne/nh=1.Hence A is intrinsic semiconductor. If
ne/nh<1 ,then B is P -type.
26. I4>I3>I2>I1
27. E=0.207ev As E<Eg hence cannot be detect.
28. The potential barrier of germanium junction diode is
0.3v and silicon is 0.7V, both are forward
biased.Therefore for conduction the minimum potential
difference across junction diode is 0.3V.Max.readingof
voltmeter connected across R=12-0.3=11.7V.
29. Hint: Conductor - no energy gap
Semi Conductor - It is of the order of 1 ev.
Insulator - 6 ev (or) more than 6 ev.
30. (i) n-type. semiconductor
ii ne .nh=ni 2=>nh=6x108x6x108= 4x104perm2
Answer key (3 Marks)
1. Potential barrier: The potential barrier is the fictitious
battery, which seems to be connected across the p-n

263
junction with its positive terminal in the n-region and the
negative terminal in the p-region.
Depletion region: The region around the junction, which
is devoid of any mobile charge carriers, is called the
depletion layer or region.
1. When the p-n junction is forward biased, there is a
decrease in the depletion region.
2. When the p-n junction is reverse biased, there is an
increase in the depletion region.
2. (i) A p-n junction is said to be forward-biased if its p-type
is connected to the positive terminal and its n-type is
connected to the negative terminal of a battery.
(ii) A p-n junction is said to be reverse-biased if its n-type
is connected to the positive terminal and its p-type is
connected to the negative terminal of a battery. The
diagrams are as shown.
Diagram ref. NCERT
3. The characteristics are as shown.

(i) This is because even a small voltage is sufficient to


sweep the minority carriers from one side of the junction
to the other side of the junction.
(ii) As the reverse bias voltage is increased, the electric field
at the junction becomes significant. When the reverse
bias voltage V = Vz critical voltage, then the electric field
strength is high enough to pull valence electrons from the
264
host atoms on the p-side which are accelerated to the n-
side. These electrons account for the high current
observed at the breakdown.
Zener diode operates under the reverse bias in the
breakdown region.
4. For energy bands
(i) The energy level diagram is shown below.

(ii) The diagram is shown as

In the energy band diagram of n-type Si semiconductor,


the donor energy level EA is slightly below the bottom Ec
of the conduction band and electrons from this level
move into the conduction band with a very small supply
of energy. At room temperature, most of the donor atoms
get ionized but very few (-10-12) atoms of Si get ionized.
So the conduction band will have most electrons coming
from the donor impurities.
Similarly, for p-type semiconductors, the acceptor energy
level EA is slightly above the top Ev of the valence band.
With the very small supply of energy, an electron from
the valence band can jump to the level EA and ionize the
265
acceptor negatively. Alternately, we can also say that
with a very small supply of energy, the hole from level
EA sinks down into the valence band. Electrons rise up
and holes fall down when they gain external energy.
5. It is because the reverse breakdown voltage of LED is
very low, i.e. nearly 5 V.
Because solar cells can work with any light whose photon
energy is more than the bandgap energy.
The heavy doping of p and n sides of the p-n junction
makes the depletion region very thin, hence for a small
reverse bias voltage, the electric field is very high.
6. It is easier to observe the change in the current with the
change in the light intensity if a reverse bias is applied.
Thus photodiode is used in the reverse bias mode even
when the current in the forward bias is more the energy
gap (Eg) of the semiconductor, then electron-hole pairs
are generated due to the absorption of photons.
Due to the electric field of the junction, electrons and
holes are separated before they recombine. The direction
of the electric field is such that electrons reach the n-side
and holes reach the p-side. Electrons are collected on the
n-side and holes are collected on the p-side giving rise to
an emf. When an external load is connected, current
flows.
Given λ = 400 nm,
Energy of photon
E
= hc/λ=6.63×10^−34×3×10^8/400×10^−9×1.6×10^−19 =
3.105 eV
Since the bandgap is lesser than this energy, therefore it
will be able to detect the wavelength.
7. The two processes are
(i) Diffusion and
(ii) Drift

266
Diffusion: The holes diffuse from the p-side to the n-side
and electrons diffuse from the n-side to the p-side.
Drift: The motion of charge carriers due to the applied
electric field which results in the drifting of holes along
the electric field and of electrons opposite to the electric
field.

The potential barrier is the fictitious battery that seems to


be connected across the junction with its positive end on
the n-type and the negative end on the p-type.
8. A Photodiode is again a special purpose p-n junction
diode fabricated with a transparent window to allow light
to fall on the diode. It is operated under reverse bias.
When the photodiode is illuminated with light (photons)
with energy (hv) greater than the energy gap (Eg) of the
semiconductor, then electron-hole pairs are generated due
to the absorption of photons. The diode is fabricated such
that the generation of e-h pairs takes place in or near the
depletion region of the diode.
9. The p-n junction diode, which emits spontaneous
radiation when forward biased, is the ―light-emitting
diode‖ or LED.
The visible tight is from 400 nm to 700 nm and the
corresponding energy is between 2.8 eV to 1.8 eV.
Therefore, the energy gap of the semiconductor to be
used in LED, in order to have the emitted radiation be in
the visible region, should be 1.8 eV. Phosphorous doped

267
gallium arsenide and gallium phosphide are two such
suitable semiconductors.
10. The diode used is Zener diode
The circuit diagram is as shown.

The Zener diode can be used as a voltage regulator in its


breakdown region. The Zener voltage remains constant
even when the current through the Zener diode changes.
11. The biasing of a light-emitting diode (LED), has
been shown below.

Two main advantages of LED over conventional


incandescent lamps are as follows:

1. Low operational voltage and less power consumption.


2. Fast action and no warm-up time required.

268
3. The bandwidth of emitted light is 100 A to 500 A or in
other words, it is nearly (but not exactly) monochromatic.
4. Long life and ruggedness.
5. Fast on-off switching capability.

12. It is fabricated with a transparent window to allow


light to fall on the diode. It is fabricated such that the
generation of e-h pairs takes place in or near the depletion
region of the diode.
When the photodiode is illuminated with light (photons)
with energy (hw) greater than the energy gap (Eg) of the
semiconductor, then electron-hole pairs are generated due
to the absorption of photons. The diode is fabricated such
that the generation of e-h pairs takes place in or near the
depletion region of the diode. Due to the electric field of
the junction, electrons and holes are separated before they
recombine.
The direction of the electric field is such that electrons reach
the n-side and holes reach the p-side. Electrons are
collected on the n-side and holes are collected on the p-
side giving rise to an emf. When an external load is
connected, current flows. The magnitude of the
photocurrent depends on the intensity of incident light
(photocurrent is proportional to incident light intensity).
The diagram is as shown.
It is easier to observe the change in the current with a change
in the light intensity if a reverse bias is applied. Thus
photodiode is used in the reverse bias mode even when

269
the current in the forward bias is more.

13. The circuit diagram is as shown.

The two ends S1 and S2 of a center-tapped secondary of a


transformer are connected to the P sections of the two
diodes D1 and D2 respectively. The n-sections of the two
diodes are joined together and their com¬mon junction is
connected to the central tap C of the secondary winding
through a load resistance RL. The input is applied across
the primary and the output is ob¬tained across the load
resistance RL. The arrows show the direction of the
current.
Assume that the end A of the secondary is positive during
the first half cycle of the supply voltage. This makes
diode D1 forward biased and diode D2 reverse biased.
Thus diode D1 conducts and an output is obtained across
the load RL.
270
During the second half cycle of the supply voltage, the
polarities of the secondary windings reverse. A becomes
negative and B becomes positive with respect to the
central terminal C. This makes diode D2 forward biased.
Hence it conducts and an output is obtained across RL.
The input-output waveforms are as shown.

14. Correct diagram and explanation from NCERT


15. Corrct explaination from NCERT
16. Ref. to NCERT for correct explaination.
17. Solar Cell: A solar cell is a junction diode that
converts solar energy into electrical energy. In a solar
cell, the n-region is very thin and transparent so that most
of the incident light reaches the junction. The thin region
is called the emitter and the other base. When light is
incident on it, it passes through the crystal onto the
junction. The electrons and holes are generated due to
light (with hv > Eg). The electrons are kicked to the n-side
and holes to the p-side due to the electric field of the
depletion region. Thus p-side becomes positive and the n-
side becomes negative giving rise to a photo-voltage.
271
Thus it behaves as a cell.

Si and GaAs are preferred for solar cell fabrication due to


the fact that their bandgap is ideal. Further, they have
high electrical conductivity and high optical absorption.
18.

1. Drift and
2. diffusion.
The n-type has an excess of electrons and the p-type has an
excess of holes. When a p-n junction has formed the
electrons from the n-type diffuse into the p-region and the
holes in the p-type diffuse into the n-region. These
diffusing electrons and holes combine near the junction.
Each combination eliminates an electron and a hole. This
results in the n-region near the junction becoming
positively charged by losing its electrons and the p-region
near the junction becoming negatively charged by losing
its holes.
This accumulation of electric charge of opposite polarities in
the two regions across the junction establishes a potential
difference between the two regions. This is called the
potential barrier or junction barrier.
The potential barrier developed across the junction
opposes the further diffusion of the charge carriers from p
272
to n and vice versa. As a result, a region develops on
either side of the junction where there is a depletion of
mobile charges and has only immobile charges. The
region around the junction which is devoid of any mobile
charge carriers is called the depletion layer or region.
Answer Key (MCQs)
S. NO. Correct option S. Correct option
NO.
1 A 16 B
2 B 17 B
3 C 18 D
4 C 19 C
5 C 20 B
6 D 21 B
7 C 22 B
8 C 23 C
9 C 24 B
10 C 25 D
11 B 26 C
12 C 27 B
13 C 28 B
14 C 29 D
15 B 30 D

Answer Key (ASSERTION REASON)


S. NO. Correct option S. NO. Correct option
1 C 11 A
2 C 12 B
3 A 13 B
4 A 14 A
5 C 15 A
6 B 16 A
273
7 D 17 C
8 B 18 D
9 D 19 C
10 A 20 A
Answer Key (case study based)
Case I Case II Case III Case IV
1 A 1 A 1 B 1 B
2 B 2 D 2 B 2 C
3 A 3 B 3 C 3 A
4 C 4 C 4 C 4 C
5 C 5 A 5 A 5 C
Case V Case VI Case VII Case VIII
1 C 1 C 1 D 1 A
2 B 2 C 2 B 2 D
3 C 3 D 3 C 3 C
4 C 4 B 4 C 4 C
- 5 B
Case IX Case X
1 B 1 B
2 B 2 C
3 C
4 C
- 5 A

274
कें द्रीय विद्यालय संगठन क्षेत्रीय कायाालय रायपुर
Kendriya Vidyalaya Sangathan Regional Office Raipur

PHYSICS

Class - XII
Question Bank Term- II 2021-22
कें द्रीय विद्यालय सगं ठन क्षेत्रीय कायाालय रायपरु
Kendriya Vidyalaya Sangathan Regional Office Raipur

MESSAGE FROM DUPUTY


COMMISSIONER

It gives me immense pleasure to bring out the study material for 2 nd Term in
different subject of Classes X and XII for Raipur Region. All of us know that in the
1st Term Examination questions were objective but in 2nd Term questions will be
subjective so once again to get our children acquainted and familiarized with the new
scheme of examination and types of questions, it is of utmost significance that an
extensive study material should be provided to our children. This question bank is
in complete consonance with CBSE Circular Number 51 and 53 issued in the month
of July 2021. It will help students to prepare themselves better for the examination.
Sound and deeper knowledge of the Units and Chapters is must for grasping the
concepts, understanding the questions. Study materials help in making suitable and
effective notes for quick revision just before the examination.

Due to the unprecedented circumstances of COVID-19 pandemic the


students and the teachers are getting very limited opportunity to interact face to face
in the classes. In such a situation the supervised and especially prepared value
points will help the students to develop their understanding and analytical skills
together. The students will be benefitted immensely after going through the question
bank and practice papers. The study materials will build a special bond and act as
connecting link between the teachers and the students as both can undertake a
guided and experiential learning simultaneously. It will help the students develop
the habit of exploring and analyzing the Creative & Critical Thinking Skills. The
new concepts introduced in the question pattern related to case study, reasoning
and ascertain will empower the students to take independent decision on different
situational problems. The different study materials are designed in such a manner
to help the students in their self-learning pace. It emphasizes the great pedagogical
dictum that ‘everything can be learnt but nothing can be taught’. The self-motivated
learning as well as supervised classes will together help them achieve the new
academic heights.

I would like to extend my sincere gratitude to all the principals and the teachers
who have relentlessly striven for completion of the project of preparing study
materials for all the subjects. Their enormous contribution in making this project
successful is praiseworthy.
Happy learning and best of luck!

Vinod Kumar
(Deputy Commissioner)
कें द्रीय विद्यालय सगं ठन क्षेत्रीय कायाालय रायपरु
Kendriya Vidyalaya Sangathan Regional Office Raipur

Our Patorn

Vinod Kumar
Deputy Commissioner
KVS RO Raipur

Smt.Biraja Mishra Sh.A.K. Mishra


Assistant Commissioner Assistant Commissioner
KVS RO Raipur KVS RO Raipur

Sh.BHOOP SINGH
PRINCIPAL, KV KANKER
CONTENT TEAM
UNIT NAME OF TEACHER
CHAPTER 8: ELECTROMAGNETIC WAVES SMT AMITA SINGH
CHAPTER 9: RAY OPTICS AND OPTICAL K V BILASPUR
INSTRUMENTS

CHAPTER 10: WAVE OPTICS SH SIMANCHAL PRADHAN


K V NO.1 RAIPUR (SHIFT-1)
CHAPTER 12: DUAL NATURE OF MATTER SH VINOD KUMAR
AND RADIATION K V NO.4 KORBA

CHAPTER 13: ATOMS SMT RUNA CHOUDHARY


CHAPTER 14: NUCLEI K V CISF BHILAI
CHAPTER 14: SEMICONDUCTOR-
ELECTRONICS-: MATERIALS DEVICES SH AMITAV ADHIKARI
AND SIMPLE CIRCUITS K V DONGARGARH

PREPARATION OF SAMPLE PAPER SH B R GAJPAL


K V NO.1 RAIPUR (SHIFT-1)
MS KAMALPREET
K V DHAMTARI
SH H S TRIPATHI
K V MAHASAMUND

REVIEW COMMITTEE SMT SUNITA KHIRBAT


K V BVMY BHILAI
SH N D SAHU
K V NO.1 RAIPUR SHIFT 2

1
ELECTROMAGNETIC WAVES
FORMULA
Equation for travelling electromagnetic waves along Z – axis
E = EX (t) = E0 sin (kz – ωt)
B = By (t) = B0 sin (kz – ωt)

Gamma X-rays UV Visible IR waves Micro Radio waves


rays rays Light waves
Wave length 10-14 to10-10 6x10-10 4x10-7 to 7x10-7 to 10-3 10-3 to 10-1 >0.1
range 10-13 to10-8 to 4x10-7 7x10-7
(m)

Frequency 1023 to 10181021 to 1016 1017 to 7x1014 to 1014 to 1011 1012 to 108 109 to 5x105
(Hz) 1014 4x1014
Production Radioactive Bombarding 1.High Electrons in Vibrations of 1.Klystrons Accelerated
decay of the high energy temperatu atoms of an atoms and 2.Magnetrons electrons in
nucleus electrons re bodies object moves molecules of 3.Gunn diodes conducting wires
with heavy 2.Sun from higher hot bodies
metal targets to lower
energy level

Detection 1.Photograp 1.Photograph 1.Photoce 1.Human eye 1.Bolo meter Point contact Receiver’s aerial
hic film ic film ll 2.Photograph 2.Thermopile diodes
2.Geiger 2.Geiger tube 2.Photogr ic film 3.Photographic
tube 3.Ionisat ion aphic 3.Photocell film
3.Ionisati on chamber film
chamber
Use In medicine 1.Medicine 1.Eye Helps to 1.Earth 1.RADAR 1.Radio
to kill the 2.Spectrscop surgery view the satellites system 2.Television
cancer cells y 3.industries 2.Kill objects 2.Remote 2.Speed guns broadcasting
germs in switches 3.Micro ovens 3.Communicat
water 3.Night vision ion system
purifiers cameras

2
QUESTIONS OF 2 MARKS
Q.1.The oscillating magnetic field in a plane electromagnetic wave is given by
By = 8 X 10-6 sin (2 x 1011 t + 300)] T
a. Calculate the wavelength of electromagnetic wave?
b. Write down the expression for the oscillating electric field.

HINTS. a. =3m, b. Ez = 2400 sin (2 x 1011 t + 300)]

Q.2. EM waves travel in a medium at the speed of 2 x 10 8 m/s. The relative permeability of the medium
is 1. Find the relative permittivity of that medium.

HINTS. v = 2 x 10 8 m/s , µr = 1 , c = 3 x 10 8 m/s Speed of EM wave in the medium v = 1 / √µε = c


/ √µr εr Or εr = c 2 / v 2 µr = (3 x 10 8 )2 / (2 x 10 8 )2 x 1 = 2.25

Q.3.Optical and radio telescopes are built on the ground while X- ray astronomy is possible only from
satellites orbiting the Earth. Why?

HINTS. The earth’s atmosphere is transparent to visible light and radio waves but absorbs X- rays.
Satellites orbiting the earth at a height of 36000 km, where atmosphere is very thin and X-rays are
not absorbed.

Q.4. The small ozone layer on top of the stratosphere is crucial for human survival. Why?

HINTS. Ozone layer absorbs ultraviolet radiation from the sun and prevent these radiations from
reaching the earth which causes cancer.

Q.5. Identify the following electromagnetic radiation as per the wavelength given below. Write one
application of each.
(a) 10-12m (b). 10-4m (c.) 106m

HINTS:- Identification:- (a.) gamma rays use- radiotherapy (b.) Infrared rays use – haze
photography ( c). long radio wave use in radio communication

Q.6.Give one uses of each of the following and arrange them in ascending order of frequency.
a.Microwave b. Infra-red wave c. Ultra violet radiation d. Gamma rays

3
QUESTIONS OF 3 MARKS

Q.7.Name the following constituent radiations of electromagnetic spectrum which-

(i) are used in satellite communication/in radar and geostationary satellite


(ii) are used for studying crystal structure of solids
(iii) are similar to the radiations emitted during decay of radioactive nuclei
(iv) are used for water purification/ are absorbed from sunlight by ozone layer

Q.8.In a plane electromagnetic wave, the electric field oscillates sinusoidaly at a frequency of 2 x 1010 Hz
and amplitude 48 V.

(a) What is the wavelength of the wave?


(b) What is the amplitude of the oscillating magnetic field?
(c) Show that the average energy density of the E field equals the average energy density of the B field.
[c = 3 x 108 m/s]
HINTS a) The wavelength is given by λ= c/ν =1.5×10-2 m
(b) Bo=Eo/c=1.6×10-7 T
(c) Energy density due to the electric field, EE=1/2ϵ0E2
Energy density due to the magnetic field, EB=1/2B2/μo on solving above equations, EE=EB
Q.9. What physical quantity is the same for X-rays of wavelength 10–10m, red light of wavelength 6800 Å
and radio waves of wavelength 500m? A plane electromagnetic wave travels in vacuum along z-direction.
What can you say about the directions of its electric and magnetic field vectors? If the frequency of the
wave is 30 MHz, what is its wavelength?
Ans. The wave speed is same for all radiation, λ = 10m
Q.10. Suppose that the electric field part of an electromagnetic wave in vacuum is E = {(3.1 N/C) cos
[(1.8 rad/m) y + (5.4 × 106 rad/s)t]} ˆi . (a) What is the direction of propagation? (b) What is the
wavelength λ? (c) What is the frequency ν? (d) What is the amplitude of the magnetic field part of the
wave? (e) Write an expression for the magnetic field part of the wave.
Ans. (a) Negative Y direction, (b) λ = 3.5m (c) ν = 86 MHz (d) 10.3 nT

4
CASE STUDY BASED
1. X- Rays

X-rays are a form of electromagnetic radiation, similar to visible light. Unlike light, however, x-rays have
higher energy and can pass through most objects, including the body. Medical x-rays are used to generate
images of tissues and structures inside the body

Q1. What is the most common method of preparation of X rays ?


a) magnetron valve b) vibration of atoms and molecules
c) bombardment of metal by high energy electrons d) radioactive decay of nucleus
Q2) which of the following set of instrument /equipment can detect X- rays
a) Photocells, photographic film b) Thermopiles, bolometer
c) Photographic film, Geiger tube d) Geiger tube, human eye
Q3) where do X rays fall on the electromagnetic spectrum?
a) Between UV region and infrared region b) Between gamma rays and UV region
c) Between infrared and microwaves d) Between microwaves and radio waves
Q4) what is the use of rays lying beyond X ray region in electromagnetic spectrum
a) used to kill microbes
b) used to detect heat loss in insulated systems
c) used in standard broadcast radio and television
d) used In oncology, to kill cancerous cells.
Q5) Which of the following has the lowest frequency
(a) microwaves (b)ultra-violet (c) X-rays (d) None
ANSWER:
Q1. c Q2. C Q3. b Q4. d Q.5.a

5
2. GAMMA RAYS IN TREATMENT OF CANCER

Gamma rays are used in radiotherapy to Treat cancer. They are used to spot tumors. they kill the living
cells and damage malignant tumor.

Q.1. What is the source of gamma rays?


a) radioactive decay of nucleus b) accelerated motion of charges in conducting wire
c) hot bodies and molecule d) klystron valve
Q.2. How is wavelength of gamma rays
a) low b) high c) infinite d) zero
Q.3. Choose the one with correct radiation order?
a) alpha>beta>gamma b) beta>alpha>gamma
c) gamma>beta>alpha d) gamma>alpha>beta
Q.4. What is other use of gamma rays?
a) used to change white topaz to blue topaz b) used in aircraft navigation
(c)used in kill microbes d) checking fractures of bone
Q.5.What is ratio of velocity of X rays and gamma rays in vaccum.
(a) 3:2 b) 2 :3 c) 1:1 d) none

ANSWER
1) a 2) a 3) c 4) a 5)c
3. Microwave oven:
The spectrum of electromagnetic radiation contains a part known as microwaves. These waves have
frequency and energy smaller than visible light and wavelength larger than it. What is the principle of a

6
microwave oven and how does it work ? Our objective is to cook food or warm it up. All food items such
as fruit, vegetables, meat, cereals, etc., contain water as a constituent. Now, what does it mean when we
say that a certain object has become warmer? When the temperature of a body rises, the energy of the
random motion of atoms and molecules increases and the molecules travel or vibrate or rotate with higher
energies. The frequency of rotation of water molecules is about 2.45 gigahertz (GHz). If water receives
microwaves of this frequency, its molecules absorb this radiation, which is equivalent to heating up water.
These molecules share this energy with neighbouring food molecules, heating up the food. One should
use porcelain vessels and non metal containers in a microwave oven because of the danger of getting a
shock from accumulated electric charges. Metals may also melt from heating. The porcelain container
remains unaffected and cool, because its large molecules vibrate and rotate with much smaller
frequencies, and thus cannot absorb microwaves. Hence, they do not get heaten up. Thus, the basic
principle of a microwave oven is to generate microwave radiation of appropriate frequency in the working
space of the oven where we keep food. This way energy is not wasted in heating up the vessel. In the
conventional heating method, the vessel on the burner gets heated first and then the food inside gets
heated because of transfer of energy from the vessel. In the microwave oven, on the other hand, energy
is directly delivered to water molecules which is shared by the entire food.
Q 1. As compared to visible light microwave has frequency and energy:
(a) more than visible light. (b) less than visible light.
(c) equal to visible light. (d) Frequency is less but energy is more
Q.2.When the temperature of a body rises:
(a) the energy of the random motion of atoms and molecules increases.
(b) the energy of the random motion of atoms and molecules decreases.
(c) the energy of the random motion of atoms and molecules remains same.
(d) the random motion of atoms and molecules becomes streamlined
Q.3. The frequency of rotation of water molecules is about:
(a) 2.45 MHz. (b) 2.45 kHz. (c) 2.45 GHz. (d) 2.45 THz
Q.4. Why should one use porcelain vessels and nonmetal containers in a microwave oven?
(a) Because it will get too much hot.
(b) Because it may crack due to high frequency.
(b) Because it will prevent the food items to become hot.
(d) Because of the danger of getting a shock from accumulated electric charges

7
Q. 5. In the microwave oven,
(a) energy is directly delivered to water molecules which is shared by the entire food.
(b) the vessel gets heated first, and then the food grains inside.
(c) the vessel gets heated first and then the water molecules collect heat from the body of the vessel.
(d) energy is directly delivered to the food grains.
ANS. 1.(b) 2.(a) 3.(c) 4.(d) 5.(a)

4. GREEN HOUSE EFFECT

The greenhouse effect is a natural process that warms the Earth's surface. When the Sun's energy reaches
the Earth's atmosphere, some of it is reflected back to space and the rest is absorbed and re-radiated by
greenhouse gases. The absorbed energy warms the atmosphere and the surface of the Earth

Q1. The one which is not considered as naturally occurring greenhouse gas is
(a) methane (b) CFCs
(c) carbon dioxide (d) nitrous oxide
Q2. Which of the following is not a use of infrared waves
a) Used in treatment for certain forms of cancer
b) in military and civilian applications include target acquisition, surveillance, night vision, homing, and
tracking.
c) to observe changing blood flow in the skin
d) In imaging cameras, used to detect heat loss in insulated systems
Q3. which of the following is the best method for production of infrared waves
a) bombardment of metal by high energy electrons
b) radioactive decay of nucleus
c) magnetron valve

8
d) vibration of atoms and molecules
Q4. Wavelength of infrared radiations is
(a) shorter (b) longer
(c) infinite (d) zero
Q.5. The radiowave was discovered by
(a) Marconi (b) Faraday
(c) Austurn (d) Joule
(ANSWER KEY) Q1. b Q2. a Q3. d Q4. b Q.5.a

5. ELECTROMAGNETIC (EM) SPECTRUM

The electromagnetic (EM) spectrum is the range of all types of EM radiation. Radiation is• energy that
travels and spreads out as it goes – the visible light that comes from a lamp in your house and the radio
waves that come from a radio station are two types of electromagnetic radiation. The other types of EM
radiation that make up the electromagnetic spectrum are microwaves, infrared light, ultraviolet rays, X-
rays and gamma rays.
Q1. The classification is roughly based on?
a) Wavelength and frequency of waves. b) Production and detection of waves.
c) The way of travelling of waves. d) Year discovered.
Q2. Which of the following is NOT an example of EM RAYS.
a) Radiotherapy(medicine). b) Checking fractures.
c) Sterilisation. d) Explosives.
Q3. Identify the pair having highest frequency and highest wavelength EM WAVES.
a) UV rays and X- rays b) Gamma rays and Microwaves.
c) Gamma rays and Radio waves. d) Radio waves and UV rays.
Q4. What physical quantity is the same for X rays of wavelength 10-10m, red light of wavelength 6800
Ao and radiowaves of wavelength 500 m?
a) Speed in vacuum (c) b) frequency (f)
c) Scattering d) Energy (e)
Q.5. What is the ratio of velocity of electromagnetic waves having frequency 1MHz and 10MHz in air?
(a) 1:1 (b)2:1 (c) 1:2 (d)1:10
ANSWER KEY

9
1. b) 2. d) 3. c) 4. a) 5. a)
---------------------------------------------------------------------------------------------------------

RAY OPTICS AND OPTICAL INSTRUMENTS

1. Refractive index

2. Apparent shift

3. Relation between critical angle and


refractive index .

4. Refraction at spherical surface

5. Lens formula

6. Lens makers formula

7. Magnification (m) = I/O = v/u


8. Power of lens

9. Combination of lens

10. Relation between i, e and δm

11. Prism formula

12. Magnifying power of simple (final image at D)


microscope m = 1 + D/f

10
(final image at infinity)
m = D/f

QUESTIONS OF 2 MARKS
Q.1. Define the angle of deviation. Write the relation between angle of incidence i, angle of prism
A and angle of minimum deviation δm for a glass prism.
HINTS. Definition, 2i = A + δm
Q.2. It is known that the refractive index, , of the material of a prism, depends on the wavelength
, , of the incident radiation as per the relation

where A and B are constants. Plot a graph showing the dependence of µon λ.
HINTS.

Q.3.Define the power and write its SI unit. How does power of lens vary when incident red light
is replaced by blue light?
HINTS: Def. and SI unit, Wavelength decreases hence power increases.
Q.4. Two thin lenses of power +6 D and – 2 D are in contact. What is the focal length of the
combination?
HINTS: Net power of lens combination P = P1 + P2 = + 6 D - 2 D = + 4 D
∴ Focal length, f = 1/P = ¼ m = 25 cm
Q.5. Under what condition does a convex lens of glass having certain refractive index, acts as a
plane glass sheet? Justify with mathematical calculation.
HINTS: When refractive index of lens is equal to refractive index of liquid.
Q.6.You are given following three lenses. Which two lens you will use to make objective and
eyepiece of an astronomical telescope and why ?
LENS POWER APERTURE
L1 3D 8cm

11
L2 6D 1cm
L3 10D 1cm
HINTS: L1 as objective.
L3 as eyepiece , Justification
Q.7. A concave lens of refractive index 1.5 is immersed in a medium of refractive index 1.65.
What is the nature of the lens? Justify with mathematical calculation.
HINTS: Since µ for lens. < µ for surrounding. It behaves like converging lens.
Q.8. A converging lens is kept coaxially in contact with a diverging lens , both the lenses being of
equal focal lengths .What is the focal length of the combination?
HINTS. Focal length = infinity
Q.9. State the condition for the phenomenon of total internal reflection to occur. Name the device
which work on the principle of total internal reflection.
HINTS. (a) Light travels from denser to rare medium.
(b) Angle of incidence greater than critical angle.

Q.10. Calculate the speed of light in a medium whose critical angle is 300
HINTS.

Therefore, speed of light is 1.5 X 108m/s


Q.11. A beam of light converges at a point P. Now a convex lens is placed in the path of the
convergent beam at 15cm from P. At what point does a beam converge if the convex lens has a
focal length 10cm?
HINT : 6cm
Q.12. An illuminated object and a screen are placed 90 cm apart. Determine the focal length and
nature of lens required to produce a clear image on the screen, twice the size of the object.
Hint: f = 20 cm (convex lens)
Q.13. An object is placed at the principal focus of a concave lens of focal length f. Where will its
image be formed? Draw the ray diagram.

12
Hint: Between optical centre and focus of lens: towards the side of the object.
Q.14. A prism of angle 600 gives a minimum deviation of 300 . What is the refractive index of the
material of prism?
Hint: Refractive index n = 1.41
Q.15. An equi-convex lens has refractive index 1.5. Write its focal length in terms of radius of
curvature R.
Hint: f = R
Q.16. Which of the following properties of light: velocity, wavelength and frequency, changes
during the phenomenon (i) reflection (ii) refraction. Justify your answer
Hint: (i) No change (ii) Velocity, wavelength change
Q.17. Draw a ray diagram of compound microscope. Write the expression for its magnifying
power.
SHORT AANSWER QUESTIONS (3m)
Q.1. For the same value of angle of incidence, the angle of refraction in three media is 15 0, 250
and 350respectively. In which medium the velocity of light will be minimum?
Ans.

Therefore, Velocity of light is minimum in medium A.


Q.2. Draw a ray diagram of an astronomical telescope in the normal adjustment position. State two
drawbacks of this type of telescope.
Ans. (i) Magnifying power m = -f0/fe . It does not change with increase of aperture of objective
lens, because focal length of a lens has no concern with the aperture of lens.

13
(ii) Drawbacks:
Lesser resolving power.
The image formed is inverted and faintes.
Q.3. What is the focal length of a combination of a convex lens of focal length 30cm and a concave
lens of focal length 20cm ? Is the system a converging or diverging lens? Ignore thickness of the
lenses.
Hint: f = -60cm
Q.4. Light from a point source in air falls on a convex spherical glass surface (µ = 1.5, radius of
curvature = 20 cm). The distance of light source from the glass surface is 100 cm. At what position
is the image formed?
Hint: v= +100cm. (The image is formed at a distance of 100cm from the glass surface in the
direction of incident light )
Q.5. Velocity of light in a liquid is 1.5 X 108 m/s and in air, it is 3 X 108 m/s. If a ray of light passes
from liquid into the air, calculate the value of critical angle.
Hint: Critical angle = 300
Q.6. Draw the ray diagram and derive the relation between object distance, image distance and
focal length, when object is placed at the center of curvature of a convex lens.
Hint: Diagram and formula 1/v - 1/u = 1/f
Q.7. (a) Write the necessary conditions for the phenomenon of total internal reflection to occur.
(b) Derive the relation between the refractive index and critical angle for a given pair of optical
media. How the critical angle change when red incident light is replaced by violet incident light?
Hint: Light travel from denser medium to rare medium. aµb = 1/sinC
Q.8. Three rays of light, red (R), green (G) and blue (B) are incident on the face AB of a right
angled prism as shown in the figure. The refractive indices of the material prism for red, green and

14
blue colours are 1.39, 1.44, and 1.47 respectively. Which one of the three rays will emerge out of
the prism? Give reason to support your answer.

Hint: Only red ray will emerge out of the prism. (µR = 1.39 < 1.414)
Q.12. Derive the lens maker formula and write the assumptions.
Q.13. A convex lens of focal length 20 cm and a concave lens of focal length 15 cm are kept 30 cm apart
with their principal axes coincident. When an object is placed 30 cm in front of the convex lens, calculate
the position of the final image formed by the combination. Would this result change if the object were
placed 30 cm in front of the concave lens? Give reason.
Hint: v = -30 cm
The result will not change on interchanging the lenses due to the
principle of reversibility of light.
Q.14. A point object ‘O’ is kept in a medium of refractive index
n1 in front of a convex spherical surface of radius of curvature R
which separates the second medium of refractive index n2 from the
first one, as shown in the figure. Draw the ray diagram showing
the image formation and deduce the relationship between the object distance and the image
distance in terms of n1 , n2 and R
Q.15. Why does a convex lens behave as a diverging lens when immersed in Carbon disulphide
(µ=1.6). Justify with calculation.
b) A ray of light incident on an equilateral glass prism propagates parallel to the base line of
the prism inside it. Find the angle of incidence of this ray. Given refractive index of a material of
glass prism is √3.
Q.16. (a) Draw the ray diagram of astronomical telescope in normal adjustment.
(b) convex lenses A and B of an astronomical telescope having focal
lengths 5cm and 20cm respectively are arranged as shown in the figure.

15
i) Which one of the two lenses you will select to use as the objective lens and why?
ii) What should be the change in the distance between the lenses to have the telescope in its
normal adjustment position?
iii) Calculate the magnifying power of the telescope in the normal adjustment position?
Q.17. What is an equivalent lens. Obtain an expression for the effective focal length of two thin lenses
placed in contact co-axially with each other.
Q.18. A lens forms a real image of an object. The distance of the object to the lens is 4 cm and the distance
of the image from the lens is v cm. The given graph shows the variation of v with u. (i) What is the nature
of the lens? (ii) Using the graph, find the focal length of this lens.

Hint: Lens is convex lens f = + 10 cm


Q.19. A small bulb is placed at the bottom of a tank containing water to depth of 80 cm. What is the area
of the surface of water through which light from the bulb can emerge out? Refractive index of water is
1.33 (Consider the bulb to be a point source ).
Hint: Area = 2.59 m2
Q.20. A ray PQ incident on the face AC of a prism ABC, as shown in the figure , emerges from
the face AB such that AQ = AR .

Draw the ray diagram showing the passage of the ray through the prism. If the angle of
prism is 600 and refractive index of the material of the prism is √3 determine the values of
angle of incidence and angle of deviation.

16
Hint: Correct diagram ( QR II BC ) minimum deviation position
<i = 600
δ= 600
Q.21. Draw a ray diagram of reflecting type telescope . State two advantage of this type telescope .
Write magnifying power formula .
Q.22. Draw a ray diagram of compound microscope when image is formed at least distance of
distinct vision . Derive magnifying power formula .How can the magnifying power of a microscope
be increased ?
Q.23. Draw ray diagram to show refraction of light through a glass prism. Draw a graph, show the angle
of deviation with that of the angle of incidence. For small angle prism, prove that δ = A (µ-1), where the
symbols have their usual meanings.
Q.24. A ray of light passing from air through an equilateral glass prism undergoes minimum deviation.
When the angle of incidence is ¾ of angle of prism. Calculate the speed of light in prism.
Hint : 2.1 x 108 m/s
Q.25. A prism is made of glass of unknown refractive index. A parallel beam of light is incident on a face
of the prism. The angle of minimum deviation is measured to be 40°. What is the refractive index of the
material of the prism? The refracting angle of the prism is 60°. If the prism is placed in water (refractive
index 1.33), predict the new angle of minimum deviation of a parallel beam of light
Hint: aµg = 1.532, D’w = 100
Q.26. A small telescope has an objective lens of focal length 144cm and an eyepiece of focal length 6.0cm.
What is the magnifying power of the telescope? What is the separation between the objective and the
eyepiece?
Hint: m= 24, separation = 150 cm
Q.27. A person with a normal near point (25 cm) using a compound microscope with objective of focal
length 8.0 mm and an eyepiece of focal length 2.5cm can bring an object placed at 9.0mm from the
objective in sharp focus. What is the separation between the two lenses? Calculate the magnifying power
of the microscope.
Hint: Separation 9.4cm, magnifying power = 88
Q.28. An illuminated object and a screen are placed 90cm apart. Determine the focal length and
nature of the lens required to produce a clear image on the screen, twice the size of the object.
Hint: f = 20cm, convex lens

17
Q.29. A bioconvex lens of refractive index of 1.5 Having focal length of 20cm is placed in a medium of
refractive index 1.65. Find its focal length. What should be the value of the refractive index of the medium
in which the lens should be placed so that it acts as a plain sheet of glass.
Hint: Focal length = -110 cm, refractive index = 1.5
Q.30. An object of size 3.0cm is placed 14cm in front of a concave lens of focal length 21cm. Describe
the image produced by the lens. What happens if the object is moved further away from the lens?
Hint: Image is virtual, erect and is formed at 8.4cm from the lens on the same side as the object, Height
of the image= 1.8cm, diminished, object is moved away from the lens, the virtual image moves towards
focus of the lens( but never beyond it) and progressively diminishes in size.

CASE STUDY QUESTION

CASE STUDY 1.Optical fibres: Now-a-days optical fibres are extensively used for transmitting audio
and video signals through long distances. Optical fibres too make use of the phenomenon of total internal
reflection. Optical fibres are fabricated with high quality composite glass/quartz fibres. Each fibre consists
of a core and cladding. The refractive index of the material of the core is higher than that of the cladding.
When a signal in the form of light is directed at one end of the fibre at a suitable angle, it undergoes
repeated total internal reflections along the length of the fibre and finally comes out at the other end. Since
light undergoes total internal reflection at each stage, there is no appreciable loss in the intensity of the
light signal. Optical fibres are fabricated such that light reflected at one side of inner surface strikes the
other at an angle larger than the critical angle. Even if the fibre is bent, light can easily travel along its
length. Thus, an optical fibre can be used to act as an optical pipe.

i) Which of the following statement is not true.


a) Optical fibres is based on the principle of total internal reflection.
b) The refractive index of the material of the core is less than that of the cladding.
c) an optical fibre can be used to act as an optical pipe.

18
d) there is no appreciable loss in the intensity of the light signal while propagating through an optical
fibre
ii) What is the condition for total internal reflection to occur?
a) angle of incidence must be equal to the critical angle.
b) angle of incidence must be less than the critical angle.
c) angle of incidence must be greater than the critical angle.
d) None of the above.
iii) Which of the following is not an application of total internal reflection?
a) Mirage b) Sparkling of diamond
c) Splitting of white light through a prism. d) Totally reflecting prism.
iv) Optical fibres are used extensively to transmit
a) Optical Signal b) current c) Sound waves d) None of the above
v) Mirage is found due to,
a.) Refraction of light b.) Total internal reflection
c.) Reflection of light d.) All
ANSWER
i) (b) ii) (c) iii) (c) iv) (a) v)(b)
CASE STUDY2. The total internal reflection of the light is used in polishing diamonds to create a
sparking brilliance. By polishing the diamond with specific cuts, it is adjusted the most of the light rays
approaching the surface are incident with an angle of incidence more than critical angle. Hence, they
suffer multiple reflections and ultimately come out of diamond from the top. This gives the diamond a
sparking brilliance

Q.1. T critical angle for a diamond is


a) 1.41 b) Same as glass c) 2.42 d) 1
Q.2. The basic reason for the extraordinary sparkle of suitably cut diamond is that

19
a) It has low refractive index
b) It has high transparency
c) It has high refractive index
d) It is very hard
Q.3. The extraordinary sparkling of diamond
a) Does not depend on its shape
b) Depends on its shape
c) Has no fixed reason
d) None
Q.4. A diamond is immersed in a liquid with a refractive index greater than water. Then the critical
angle for total internal reflection will
a) Increase b) Decrease
c) Depend on the nature of the liquid d) Remains the same
Q.5. OFC cables work on the principle of
a) Dispersion of light
b) Refraction of light
c) Total internal reflection
d) Interference of light
ANSWER: 1. (c) 2. (c ) 3.(b) 4.(a) 5.(c)

CA SE STUDY 3. TOTAL INTENAL REFLECTION

(i) What is refractive index of a medium (in terms of speed of light)


a) Speed of light in medium/speed of light in vacuum
b) Speed of light in vacuum/speed of light in medium speed of light in vacuum

20
c) Speed of light in medium
d) None of the above.
(ii) In the above diagram, calculate the speed of light in the liquid of unknown refractive index.
a) 1.2 × 108 m/𝑠 b) 1.4 × 108 m/𝑠
c) 1.6 × 108 m/𝑠 d) 1.8 × 108 m/𝑠
(iii) What is refractive index of a medium (in terms of real and apparent depth).
a) Real depth/ App depth b) App/ Real depth
c) App Real x depth d) Real + App depth
(iv) What is the relation between refractive index and critical angle for a medium.
a) n = 1/sin ic b) n = sin ic
c) 1 = n/ sin ic d) None of the above
(v) What is the value of angle of incidence for ray of light travelling from a medium of refractive index
√2 into the medium of refractive index 1 so that it just grazes along the surface of separation.
a) 00 (b)450 (c)300 (d)900
ANSWER: i) (b) ii) (d) iii) (a) iv) (a) v) (b)

CASE STUDY 4. Compound microscope


A compound microscope consist of two lenses. A lens of short aperture and short focal length facing the
object is called the object lens and another lens of short focal length but large aperture is called the eye
lens. Magnifying power is defined as the ration of angle subtended by the final image at the eye to the
angle subtended by the object is seen directly, when both are placed at least distance of distinct vision

1. An objective lens consist of


(i). Short aperture and short focal length (ii).large aperture and large focal length
(iii).Short aperture and large focal length (iv).large aperture and short focal length
2. An eyepiece consist of

21
(i).short aperture and short focal length (ii).large aperture and large focal length
(iii).short aperture and large focal length (iv).large aperture and short focal length
3.Formula of magnifying power
𝛽 𝛼
(i).𝑀 = 𝛼 (ii).𝑀 = 𝛽
𝛼 𝛽
(iii).𝑀 = 1 + 𝛽 (iv).𝑀 = 1 + 𝛼
4. A compound microscope with an objective of 1.0 cm, focal length and eyepiece 2.0 cm . Focal
length of a tube is 20 cm. Calculate the magnifying power of the microscope
(i).270 (ii).27 (iii).140 (iv).14
5. Final image formed by compound microscope
(i).inverted (ii). Erect
(iii). virtual (iv).highly diminished
ANSWER. 1 (i). 2 (iv). 3. (i). 4. (iii) 5. (iv)
CASE STUDY 6.Dispersion
Dispersion If a beam of white light is made to fall on one face of prism the light emerging from the
other face of the prism consist of seven colours violet, indigo, blue, green , yellow, orange, red . The
phenomena of splitting of white light into its constituent colours is called dispersion of light .

1. Which one of the following colours will suffer greatest dispersion


(i).violet (ii).indigo (iii).blue (iv).red
2. The critical angle between an equilateral prism and air is 45. If the incident ray is perpendicular to
refracting surface then
(i).it is reflected totally from the second surface and emerges perpendicular from the third surface.
(ii).it gets reflected from second and third surface and emerges from the first surface.
(iii).it keeps reflecting from all the three side of the prism and never emerges out.
(iv).after deviation, it gets refracted from the second surface.
3. Which colour is taken as the mean colour(ie- mean refractive index for a material)
(i). yellow (ii).red (iii).violet (iv).green

22
4. A prism with a refracting angle of 60 gives angle of minimum deviation 53°,51°,52° for blue, red,
yellow light respectively . What is the dispersive power of the material of the prism
(i).385 (ii)0.385 (iii).0.0385 (iv).38.5
5. The refractive angle of a prism for a monochromatic light is 60° and refractive index is √2. For
minimum deviation the angle of incidence will be
(i).60° (ii).45° (iii).30° (iv).75°

ANSWER. 1 (i). 2 (ii) 3 (i). 4(iii) 5(iii).


CASE STUDY 7. Lens maker's formula.
The lens maker’s formula relates the focal length of a lens to the refractive index of its material and the
radii of curvature of its two surfaces. This formula is used to manufacture a lens of particular focal length
from the glass of a given refractive index. For this reason, it is called the lens maker’s formula.

1.For a plano-convex lens of radius of curvature 10 cm the focal length is 30 cm . If the refractive index
of the material of the lens is
(i).2.0 (ii).1.33 (iii).1.66 (iv).1.5
2.An image is formed on the screen by a convex lens when upper half part of lens is covered with black
paper then
(i).half image is formed (ii).full image is formed
(iii)intensity of image is enhanced (iv).all of the above
3.A convex lens of focal length 20 cm is placed in contact with a diverging lens of unknown focal
length. The lens combination acts as a converging lens and has a focal length of 30 cm . What is the
focal length of diverging lens
(i). -90 cm (ii). -60 cm
(iii). -30 cm (iv).-10 cm

23
4. The focal length of a lens, made up of glass, is 5 cm in air. What would be the focal length of the
same lens in water? The refractive indices of glass and water are 3/2 and 4/3 respectively. (i).12cm
(ii). -`12 cm (iii). 16 cm (iv). -16cm
5. Two thin lenses of focal length 60 and -20 cm in contact have a resultant focal length of
(i).-30 (ii). +15 (iii).-15 (iv).+30

ANSWER. 1 (ii). 2 (ii) 3 (ii) 4 (iii) 5 (i)

----------------------------------------------------------------------------------------------------------------

WAVE OPTICS
I.Short Answer types Questions (2 Marks each)
1. Maximum intensity in YDSE is I0. Find the intensity at a point on the screen where.
(a) the phase difference between the two interfering beams is 𝜋/3
(b) the path difference between them is 𝜆/4

2. Two waves of equal frequencies have their amplitudes in the ratio of 3:5 . They are superimposed on
each other. Calculate the ratio of maximum and minimum intensities of the resultant wave.

3. Draw a diagram to show the refraction of a plane wave front incident on a convex lens and hence draw
the refracted wave front.

4. Why light ways do not diffracted around buildings, while radio waves diffract easily?

5. In double-slit experiment using light of wavelength 600 nm, the angular width of a fringe formed on a
distant screen is 0.1º.What is the spacing between the two slits?

6. For what distance is ray optics a good approximation when the aperture is 3 mm wide and the
wavelength is 500 nm?

7. The 6563 Å Hα line emitted by hydrogen in a star is found to be red shifted by 15 Å. Estimate the speed
with which the star is receding from the Earth.

8. (a) In a single slit diffraction experiment, a slit of width ‘d’ is illuminated by red light of wavelength
650 nm. For what value of‘d’ will

24
(i) the first minimum fall at an angle of diffraction of 30°, and
(ii) the first maximum fall at an angle of diffraction of 30°?

9. In a single slit diffraction experiment, the width of the slit is reduced to half its original width. How
would this affect the size and intensity of the central maximum?

10. How does the fringe width of interference fringes change, when the whole apparatus of Young’s
experiment is kept in a liquid of refractive index 4/3?

--------------------------------------------------------------------------------------------------------

II.Short Answer types Questions (3 Marks each)

1. Define the term wave front? Using Huygens’s construction draw a figure showing the propagation of a
plane wave reflecting at the interface of the two media. Show that the angle of incidence is equal to the
angle of reflection.

2. Define the term ‘wave front’. Draw the wave front and corresponding rays in the case of a (i) diverging
spherical wave (ii) plane wave. Using Huygens’s construction of a wave front, explain the refraction of a
plane wave front at a plane surface and hence deduce Snell’s law.

3. In Young’s double-slit experiment a monochromatic light of wavelength λ, is used. The intensity of


light at a point on the screen where path difference is λ is estimated as K units. What is the intensity of
light at a point where path difference is λ /3?

4. Draw fringe patterns of Interference and Diffractions and write two difference between them.

5. State one feature by which the phenomenon of interference can be distinguished from that of diffraction.
A parallel beam of light of wavelength 600 nm is incident normally on a slit of width ‘a’. If the distance
between the slits and the screen is 0.8 m and the distance of 2nd order maximum from the centre of the
screen is. 15 mm, calculate the width of the slit.

6. What is interference of light? Write two essential conditions for sustained interference pattern to be
produced on the screen. Draw a graph showing the variation of intensity versus the position on the screen
in Young’s experiment when

7.Explain graphically the intensity of fringe pattern of single and double slit interference and write two
differences between them.

25
8.What do you mean by Fresnel distance? Estimate the distance for which ray optics is good
approximation for an aperture of 4 mm and wavelength 400 nm.
9. In a single slit diffraction experiment first minimum for 𝜆1 660 nm coincides with first maxima for
wavelength 𝜆2 . Calculate the value of 𝜆2
𝑰𝒎𝒂𝒙 𝐴𝑚𝑎𝑥 𝐴 𝐼
10. If in interference = α, Find out (𝑎) , (𝑏) 𝐴1 and(𝑐) 𝐼1
𝑰𝒎𝒊𝒏 𝐴𝑚𝑖𝑛 2 2

III.Case Study based Questions (5Marks)


1. Wavefront is a locus of points which vibrate in same phase. A ray of light is perpendicular to the wave
front. According to Huygens principle, each point of the wave front is the source of a secondary
disturbance and the wavelets connecting from these points spread out in all directions with the speed of
wave. The figure shows a surface XY separating two transparent media, medium-I and medium-2. The
lines ab and cd represent wave fronts of a light wave travelling in medium- 1 and incident on XY. The
lines ef and gh represent wave fronts of the light wave in medium -2 after refraction.

(i) Light travels as a


(a) parallel beam in each medium (b) convergent beam in each medium
(d) divergent beam in one medium and convergent beam in the
(c) divergent beam in each medium
other medium.
(ii) The phases of the light wave at c, d, e and f are ∅𝑐 , ∅𝑑 , ∅𝑒 𝑎𝑛𝑑 ∅𝑓 respectively. It is given that ∅𝑐 ≠
∅𝑓
((a) ∅𝑐 can not be equal to ∅𝑑 (b) ∅𝑑 can be equal to ∅𝑒
(c) (∅𝑑 − ∅𝑓 ) is equal to (∅𝑐 − ∅𝑒 ) (d) (∅𝑑 − ∅𝑐 ) is not equal to(∅𝑓 − ∅𝑒 )
(iii) Wave front is the locus of all points, where the particles of the medium vibrate with the same
(a) phase (b) amplitude (c) frequency (d) period
(iv) A point source that emits waves uniformly in all directions, produces wavefronts that are
(a) spherical (b) elliptical (c) cylindrical (d) planar
(v) What are the types of wavefronts ?

26
(a) Spherical (b) Cylindrical (c) Plane (d) All of these
2) Jimmy and Johnny were both creating a series of circular waves by jiggling their legs in water. The
waves form a pattern similar to the diagram as shown. Their friend, Anita, advised Jimmy and Johnny not
to play with water for a long time. She then observed beautiful patterns of ripples which became very
colourful. When her friend Latha poured an oil drop on it. Latha, a 12th standard girl, had explained the
cause for colorful ripple patterns to Anita earlier.

(i) Name the phenomenon involved in the activity


(a) Reflection (b) Refraction (c) Interference (d) Diffraction
(ii) A surface over which an optical wave has a constant phase is called.
(a) Wave (b) Wave front (c) Elasticity (d) None of these
(iii) Which of the following is correct for light diverging from a point source?
(a) The intensity decreases in proportion for the distance squared.
(b) The wave front is parabolic.
(c) The intensity at the wavelength does depend of the distance.
(d) None of these.
(iv) The phenomena which is not explained by Huygens’s construction of wave front
(a) reflection (b) diffraction (c) refraction (d) origin of spectra
(v) Huygens’s concept of secondary wave
(a) allows us to find the focal length of a thick lens
(b) is a geometrical method to find a wave front
(c) is used to determine the velocity of light
(d) is used to explain polarization

27
3) Geeta was watching her favorite TV programme KBC. Suddenly the picture started shaking on the TV
Screen. She asked her elder brother to check the dish antenna. Her brother found nothing wrong with the
antenna. A little later, Geeta again noticed the same problem on the TV Screen. At the same time she
heard the sound of a low flying aircraft passing over their house. She asked her brother again. His brother
being a Physics student explained the cause of shaking the picture on the TV Screen when aircraft passes
over head.

(i) Why does the picture started shaking when a low lying aircraft passes overhead
a) Due to Interference b) Due to reflection
c) Due to refraction d) Due to polarization
(ii) Which of the following does not show any interference pattern?
a) Soap bubble b) Excessively thin film c) A thick film d) Wedge Shaped film
(iii) The main principle used in Interference is _____________
a) Heisenberg’s Uncertainty Principle b) Superposition Principle
c) Quantum Mechanics d) Fermi Principle
(iv) When two waves of same amplitude add constructively, the intensity becomes
a) Double b) Half c) Four Times d) One-Fourth
(v) The shape of the fringes observed in interference is ____________
a) Straight b) Circular c) Hyperbolic d) Elliptical
4) Rohan observed the thin film such as soap bubble or a thin layer of oil on water show beautiful colours
when illuminated by white light. He felt happy and surprised to see that. He went to his Physics teacher
to understand the reason behind it. The teacher explained him that a thin film of oil spread over water
shows interference of light due to interference between the light waves reflected by the lower and upper
surface of the thin film. On understanding the phenomenon, Rohan than gave an example of thin film of
kerosene oil which is spread over water to prevent malaria and dengue.

28
(i) If instead of monochromatic light white light is used for interference of light, what would be the
change in the observation?
a) The pattern will not be visible
b) The shape of the pattern will change from hyperbolic to circular
c) Colored fringes will be observed with a white bright fringe at the center
d) The bright and dark fringes will change position
(ii) Zero order fringe can be identified using ____________
a) White light b) Yellow light c) Achromatic light d) Monochromatic light
(iii) The interference pattern of soap bubble changes continuously.
a) True b) False c) Neither a nor b d) Both a and b
(iv)Which of the following does not show any interference pattern?
a) Soap bubble b) Excessively thin film c) A thick film d) Wedge Shaped film
v) A thin sheet of refractive index 1.5 and thickness 1 cm is placed in the path of light. What is the path
difference observed?
a) 0.003 m b) 0.004 m c) 0.005 m d) 0.006 m
5) Ram and Rahim were returning home from the cricket field, on their way they found a new 500 rupee
note on the road. Rahim advised Ram to handover the money to the cashier of the charity home they did
so and the cashier checked to see whether the currency was genuine or fake. He appreciated the boys and
showed them how to check the currency. The number 500 at the centre of the note appears green when
looked straight and blue when tilted at an angle. The cashier also explained that the colour shift on tilting
is due to constructive interference of blue light produced by the variation of thickness of chemical layers
specially added in the printing ink.

29
(i) colour shift on tilting is due to
(a) Constructive interference of blue light produced by the variation of thickness of chemical layers
specially added in the printing ink.
(b) Destructive interference of blue light produced by the variation of thickness of chemical layers
specially added in the printing ink.
(c) Diffraction of blue light produced by the variation of thickness of chemical layers specially added in
the printing ink.
(d) None of these
(ii) Is it necessary that the amplitude be constant over a given wave front?
(a)Yes (b)No (c) Both a and b (d)Neither a nor b
(iii) Can two wave-fronts cross one another? Give reason.
(a)Yes (b)No (c) Both a and b (d)Neither a nor b
(iv) When a wave undergoes reflection at a denser medium, what happens to its phase?
(a) π/2 radian (b)π/4 radian (c) π radian (d)π/ 6 radian
(v) If a wave undergoes refraction, what will be the phase change?
(a) π/2 radian (b)π/4 radian (c) π radian (d)zero
6) CD reflecting rainbow colours:
Almost all of you have seen a rainbow formation on rainy days. Well, rainbow is formed because water
droplets in the atmosphere separate white light into different colours of the rainbow. Compact Disc (CD)
also resembles the same kind of colours when viewed from different angles. Recorded data on CD is stored
in microscopic pits of different lengths which carries information in the CD. These pits are placed in a
row of the same width and at equal distance. This forms a diffraction grating on the CD mirror surface.

30
(i) Formation of rainbow colours on the CD is due to
(a) Reflection (b) Refraction (c) Diffraction (d) None of these
(ii) The recorded data on the discs behaves as
(a) Diffraction grating (b) Rainbow
(c) Drops (d) Colours
(iii) Bending of light at the corners of the door is an example of
(a) Reflection (b) Refraction
(c) Interference (d) Diffraction
(iv) Which of the following is an example of diffraction?
(a) Holograms (b) Sun appears red during sunset
(c) From the shadow of an object (d) All of these
(v) The intensity of light from the central maxima goes on ……In Diffraction pattern.
(a) Increasing (b) Decreasing
(c) Both a and b (d) Neither a nor b
Answer Key

Solution of Case Study Questions of Wave Optics


1) (i) a (ii) c (iii) c (iv) a (v) c 2) (i) c (ii) b (iii) a (iv) d (v) b
3) (i) a (ii) b (iii) b (iv) c (v) c 4) (i) c (ii) a (iii) a (iv) b (v) c
5) (i) a (ii) a (iii) b (iv) c (v) d 6) (i) c (ii) (a) (iii) (d) (iv) (d) (v)
DUAL NATURE OF MATTER AND RADIATION
Important formulae
ℎ𝑐
Energy of a photon, E = hν = 𝜆
1242𝑒𝑉−𝑛𝑚
If λ is in nm and energy of photon is in eV, then𝐸 = 𝜆 (𝑖𝑛 𝑛𝑚)

31
ℎ𝑐
Work function w = hν0 = 𝜆0

Momentum of photon p = 𝜆

Cut-off potential (V0) then KEmax = eV0


1 2
m𝑣𝑚𝑎𝑥 = eV0
2

Einstein’s equation for photoelectric effecthν = KEmax + W0



V0 = 𝑒 (ν – ν0)
ℎ 𝑐 𝑐
V0 = ( − )
𝑒 𝜆 𝜆0
ℎ ℎ
de-Broglie matter wave equation 𝜆 = =
𝑝 𝑚𝑣

1 2𝑞𝑉
mv2 = qV or 𝑣 = √
2 𝑚

Momentum p = mv = √2𝑞𝑉𝑚
ℎ ℎ
The wave length associated with moving charge is given by λ = 𝑝 =
√2𝑞𝑉𝑚

If accelerate charge is electron then λ = , where me = mass of electron.
√2𝑞𝑉𝑚𝑒
12.27
𝜆= 𝐴̇ , For electron beam
√𝑉

(2 Marks Questions)
1
1.The two lines marked A and B in the given figure show a plot of de-Broglie wavelength λ versus ,
√𝑉

where V is the accelerating potential for two nuclei 21𝐻 𝑎𝑛𝑑 31𝐻.(i)What does the slope of the lines
represent? (ii) Identify which of the lines corresponded to these nuclei.

2.Draw suitable graphs to show the variation of photoelectric current with collector plate potential for (i) a
fixed frequency but different intensities I1>I2>I3. (ii) a fixed intensity but different frequencies ν1>ν2> ν3.

32
3.Figure shows variation of stopping potential (Vo) with the frequency (ν) for two photo sensitive materials
M1 and M2. (i) Why is the slope same for both lines? (ii) For which material will the emitted electron have
greater kinetic energy for the incident radiation of the same frequency? Justify your answer.

4.An electron is accelerated through a potential difference of 100 V. What is the de-Broglie wavelength
associated with it? To which part of the electromagnetic spectrum does this value of wavelength
correspond?
5.An α-particle and a proton are accelerated from rest by the same potential. Find the ratio of their de-
Broglie wavelengths.
(3 Marks Questions)
6.Define the terms cut-off voltage and threshold frequency in relation to the phenomenon of photoelectric
effect. Using Einstein’s photoelectric equation show how the cut-off voltage and threshold frequency for
a given photosensitive material can be determine with the help of a suitable graph.
7. The following graph shows the variation of stopping potential Vo with the frequency ν of the incident
radiation for two photosensitive metals X and Y (i) Which of the metals has larger threshold wavelength?
Give reason. (ii) Explain giving reason which metal gives out electrons having larger kinetic energy. For
the same wavelength of the incident radiation. (iii) If the distance between the light source and metal X is
halved how will the kinetic energy of electrons emitted from it change? Give reason.

8. Write two characteristic features observed in photoelectric effect which supports the photon pictures of
electromagnetic radiation. Draw a graph between the frequency of incident radiation (ν) and the maximum
kinetic energy of the electrons emitted from the surface of a photosensitive material. State clearly how
this graph can be used to determine (i) Planck’s constant and (ii) work function of the material?

33
9. An electron and a photon each have a wavelength 10-9 m. Find (i) Their momenta (ii) The energy of the
photon and (iii) The kinetic energy of electron.
10. Draw a plot showing the variation of photoelectric current with collector plate potential for two different
frequencies ν2>ν1 of incident radiation having the same intensity. In which case will the stopping potential
be higher? Justify your answer.

CASE BASED QUESTIONS (5 Marks each)


11.The photoelectric emission is possible only if the incident light is in the form of packets of energy, each
having a definite value, more than the work function of the metal. This shows that light is not of wave
nature but of particle nature. It is due to this reason that photoelectric emission was accounted by quantum
theory of light.
Q1. Packet of energy are called
(a)electron (b) quanta (c)frequency (d)neutron
Q2. One quantum of radiation is called
(a) meter (b) meson (c) photon (d)quark
Q3. Energy associated with each photon
(a) hc (b) mc (c) hv (d)hk
Q4. Which of the following waves can produce photo electric effect
(a). UV radiation (b). Infrared radiation (c). Radio waves (d) .Microwaves
Q5. Work function of alkali metals is
(a) less than zero (b) just equal to other metals
(c) greater than other metals (d) quite less than other metals
11. Lenard observed that when ultraviolet radiations were allowed to fall on the emitter plate of an evacuated
glass tube, enclosing two electrodes (metal plates), current started flowing in the circuit connecting the
plates. As soon as the ultraviolet radiations were stopped, the current flow also stopped. These
observations proved that it was ultraviolet radiations, falling on the emitter plate, that ejected some
charged particles from the emitter and the positive plate attracted them.
Q1. Alkali metals like Li, Na, K and Cs show photo electric effect with visible light but metals
like Zn, Cd and Mg respond to ultraviolet light. Why?
a) Frequency of visible light is more than that for ultraviolet light
b) Frequency of visible light is less than that for ultraviolet light
c) Frequency of visible light is same for ultraviolet light

34
d) Stopping potential for visible light is more than that for ultraviolet light
Q2. Why do we not observe the phenomenon of photoelectric effect with non-metals?
a) For non metals the work function is high
b) Work function is low
c) Work function can’t be calculated
d) For non metals, threshold frequency is low
Q3. What is the effect of increase in intensity on photoelectric current?
a) Photoelectric current increases b) Decreases
c) No change d) Varies with the square of intensity
Q4. Name one factor on which the stopping potential depends
a) Work function b) Frequency
c) Current d) Energy of photon
Q5. How does the maximum K.E of the electrons emitted vary with the work function of metal?
a) It doesn’t depend on work function b)It decreases as the work function increases
c) It increases as the work function increases d) It’s value is doubled with the work function
12. The concept of ‘wave nature of matter’ was postulated by de Broglie in 1924. It was confirmed
experimentally by Davisson and Germer a few years after its postulation. Therefore, the realization was
that ‘wave nature’ and ‘particle nature’ can be viewed as the ‘two sides of a coin’. Both matter and
radiation can exhibit either of these ‘natures’, depending on the experimental situation. The phenomena
of photoelectric effect and the concept of ‘matter waves’, have been put to very useful and interesting
practical applications. We are aware of photocells, automatic doors at shops and malls, automatic light
switches that turn on the lights as soon as the intensity drops.
Q1. Who confirmed experimentally the wave nature of electron?
a) De-broglie b) Davisson & Germer c) Einstein d) None of these
Q2. A proton and an electron have same kinetic energy. Which one has greater de-Broglie wavelength ?
a) Electron b) Proton c) Same d) Can’t be calculated
Q3. An electron is accelerated through a potential difference of 100 volts. What is the de-Broglie
wavelength associated with it?
a) 1.227 A ̊ b) 12.27 A ̊ c) 122.7 A ̊ d) 1227 A ̊
Q4. The de-broglie wavelength, associated with a proton and neutron are found to be equal. Which of
the two has a higher value of K.E?

35
a) Proton b) Neutron c)Same for both d)Can’t be calculated
Q5. An electron is accelerated through a potential difference of 300 volt. What is its energy in eV?
a) 30 eV b) 300 eV c)10 eV d) 0.3eV

HINTS
(2 Marks Questions)

1. 𝜆=
√2𝑚𝑞𝑉

(a) The slope of line represents
√2𝑚𝑞

(b) The lighter mass i.e. 21𝐻 is represents by line of greater slope.
2. Two graphs
3. (a) Slope of stopping potential with frequency of incident radiation gives the value of plank’s
constant
(b) The intercept of graph on stopping potential gives the value of stopping potential which is
higher for M2.
12.27
4. 𝜆 = 𝐴̇, 𝜆 = 1.227 The wavelength belong to the X- Ray part of electromagnetic radiation.
√𝑉
ℎ 𝜆 1
5. 𝜆 = , 𝜆𝛼 = 2√2
√2𝑚𝑞𝑉 𝑝

(3 Marks Questions)
6. Definition of Cut off voltage and Threshold frequency

Einstein’s equation 𝑉𝑜 = (𝜗 − 𝜗𝑜) and graph
𝑒
𝑐
7. 𝜆 = , (𝑎)𝜆𝑥 > 𝜆𝑦 , (b) KEmax for metal X is greater than Y, (c) No effect.
𝑣

8. Correct answer and graph.


ℎ ℎ𝑐 𝑝2
9. (i) = , P = 6.63x10-25 m (ii) 𝐸 = = 1243 eV (iii) 𝐸 = = 1.52 eV.
𝑝 𝜆 2𝑚

10. Graph and correct answer, It is clear that for higher frequency cut off potential is higher.
(CASE BASED 5 Marks Questions)
11. Q1.(b) Q2.(c) Q3.(c) Q4.(a) Q5.(d)
12. Q1.(b) Q2.(a) Q3.(a) Q4.(b) Q5.(b)
13. Q1.(b) Q2.(a) Q3.(a) Q4.(a) Q5.(b)

36
Self Assessment
2 Marks Questions
1.Draw suitable graphs to show the variation of photoelectric current with collector plate potential for (i) a
fixed frequency but different intensities I1>I2>I3. (ii) a fixed intensity but different frequencies ν1>ν2> ν3.
2.Figure shows variation of stopping potential (Vo) with the frequency (ν) for two photo sensitive materials
M1 and M2. (i) Why is the slope same for both lines? (ii) For which material will the emitted electron
have greater kinetic energy for the incident radiation of the same frequency? Justify your answer.

3. Define the terms cut-off voltage and threshold frequency in relation to the phenomenon of photoelectric
effect. Using Einstein’s photoelectric equation show how the cut-off voltage and threshold frequency for
a given photosensitive material can be determine with the help of a suitable graph.
4. Draw a plot showing the variation of photoelectric current with collector plate potential for two different
frequencies ν2>ν1 of incident radiation having the same intensity. In which case will the stopping potential
be higher? Justify your answer.
3 Marks Questions
5. What do you mean by photo electric emission? State the laws of photo electric emission.
In a plot of photo electric current versus anode potential, how does
i. The saturation current varies with anode potential for incident radiation of different frequencies of
incident radiation but same intensity?
ii. The stopping potential varies for incident radiations of different intensities but same frequency?
iii. Photo electric current vary for different intensities but same frequency of incident radiations?
6. The work function for cesium is 2.14 eV. Find (i) Threshold wavelength for cesium, (ii) The frequency
of radiation incident on cesium if stopping potential is 0.60 V.
CASE BASED QUESTION
7. Studies, that followed, showed that different metals emit electrons when irradiated by different
electromagnetic radiations. For example, alkali metals (sodium, cesium, potassium) emit electrons with

37
X–rays ultraviolet light and also with visible light, except red and orange light. Heavy metals, like zinc,
cadmium, magnesium, emit electrons only when ultraviolet radiations fall on.
A. Why do we not observe the phenomenon of photoelectric effect with non-metals?
i) For nonmetals the work function is high ii) Work function is low
iii) Work function can’t be calculated iv) For non-metals, threshold frequency is low
B. The practical application of the phenomenon of photoelectric effect and the concept of ‘matter waves’
is
i) Photocells ii)Automatic doors at shops and malls
iii) automatic light switches iv)All of them
C. What is the effect of increase in intensity on photoelectric current?
i) Photoelectric current increases ii) Decreases
iii) No change iv)Varies with the square of intensity
D. Name one factor on which the stopping potential depends
i) Work function ii)Frequency
iii) Current iv)Energy of photon
E. Alkali metals like Li, Na, K and Cs show photo electric effect with visible light but metals like Zn,
Cd and Mg respond to ultraviolet light. Why?
i) Frequency of visible light is more than that for ultraviolet light
ii) Frequency of visible light is less than that for ultraviolet light
iii) Frequency of visible light is same for ultraviolet light
iv) Stopping potential for visible light is more than that for ultraviolet light

CHAPTER12: ATOMS AND CHAPTER13: NUCLEI

Section A (2 MARKS EACH)

1. Determine the value of the de-Broglie wave length associated with electron orbiting in the ground
state of hydrogen atom ?
2. What is the wavelength of the radiation emitted when the electron in the hydrogen atom jumps
from n =α to n=1?
3. Compare the radii of two nuclei with mass number 1 and 27 respectively?

38
4. What is the kinetic energy and potential energy of electron in hydrogen atom in the second excited
state?
5. Hydrogen atom is excited from ground state to another state with principal quantum number equal
to 4 then how many spectral lines are emitted in the emission spectra?
6. If the ionization potential of the hydrogen atom is 13.6 e V. How much is the energy required to
remove an electron from second orbit (𝑛 = 2)of the hydrogen atom?
7. A beam of alpha particles of velocity 2.1x107m/s is scattered by a gold foil (Z=79). Find the
distance of closest approach of the Alpha particle to the gold nucleus. The value of the charge/mass
of Alpha particle is 4.8x107Ckg–1.
8. A 12.5eV electron beam is used to excite a gaseous hydrogen atom at room temperature.
Determine the wavelength and the corresponding series of the lines emitted.
9. Consider two different hydrogen atoms. The electron in each atom is in an excited state. Is it
possible for electron to have different energies but same orbital angular momentum according to
the Bohr model? Justify your answer.
10. Show mathematically how Bohr's postulate of quantization of orbital angular momentum in
hydrogen atom is explained by the de-Broglie's hypothesis.
Section B (03 MARKS EACH)
1. Use Bohr's postulates to derive the expression for the potential and the kinetic energy of the electron
moving in then 𝑛𝑡ℎ orbit of the hydrogen atom. How is the total energy of the electron expressed in
terms of its kinetic and potential energy?
2. The wave length of second line of Balmer series in the hydrogen spectrum is 4861 Å . Calculate the
wavelength of the first line.
3. The energy of an electron in an excited hydrogen atom is -3.4eV. Calculate the angular momentum
of the electron according to the Bohr's theory.
4. (a).Define neutron multiplication factor and critical size for a fissionable material.
(b)If 200 MeV energy is released in the fission of a single nucleus of 238𝑈92 , how many fission
must occur to produce a power of 1KW?
5. (a)Draw a plot showing the variation of potential energy of a pair of nucleons as a function of their
separation. Mark the regions where the nuclear force is
(i) attractive and (ii) repulsive.
(b)State two characteristic property of nuclear force.

39
6. What is wavelength of the spectral line emitted when an electron make a transaction from energy
level -0.85 eV to -3.4 eV?
7. The energy level of a hypothetical atom is shown below. Which of the following transition will result
in the emission of photon of wavelength 275nm? Which of these transition will result in the emission
of radiation of (1) maximum and (2) minimum energy?

8. Derive an expression for the frequency of radiation emitted when an hydrogen atom de-excites from
level n to level (n-1). Also show that for large values of n, this frequency equals to classical frequency
of revolution of a electron.
9. How long can an electric lamp of 100 W be kept glowing by fusion of 2 kg of deuterium? Take the
fusion reaction as
12𝐻 + 12𝐻 → 23𝐻𝑒 + 𝑛 + 3.27 𝑀𝑒𝑉
10. (a)State the postulates of Bohr’s model of hydrogen atom.
(b)Show that the radius of the orbit in hydrogen atom varies as 𝑛2 , when n is the principle quantum
number of the atom.
SECTION C (CASE STUDY)

1. HYDROGEN ATOM SPECTRUM


According to the third postulates of Bohr's model, when an atom makes a transition from the higher
energy state with quantum number ni to the lower energy state with quantum number nf (n f ‹ n i) , the
difference of the energy is carried away by the photon of frequency such that
hνif =Eni-Enf

40
Since both n i and nf are integers, this immediately shows that in transitions between the different
atomic levels, light is radiated in various discrete frequencies. For hydrogen atom spectrum, the
Balmer formula corresponds to n f= 2 and n i =3, 4, 5 etc. This result of the Bohr's model suggested
the presence of other series spectra for hydrogen atom-those
corresponding to the transitions resulting from nf= 1 and n
i=2, 3 etc ; and nf=3 and n i= 4 ,5etc.and so on. Such series
were identified in the course of spectroscopic investigations
and are known as Lyman, Balmer, Paschen, Brackett and
Pfund-series. The electronic transitions correspondingto this
series are shown in the figure. The various lines in the atomic
spectra are produced when electrons jump from higher
energy state to a lower energy state and photons are emitted.
These spectral lines are called emission lines. But when an atom absorbs a photon that has precisely
the same energy needed by the electron in the lower energy state to make a transition to the higher
than the state, the process is called absorption.
Thus, a photon with a continuous range of frequencies pass through a rarefied gas and then analysed
with a spectrometer, a series of the spectral absorption lines appears in a continuous spectrum .The
dark lines indicate the frequency that has been observed by the atoms of the gas.
This explanation of the hydrogen atom spectrum provided by the Bohr's model was a brilliant
achievement which greatly stimulated progress towards the modern Quantum theory.
(i) The total energy of an electron in an atom in an orbit is-3.4eV.Its kinetic and potential Energies
are respectively
(a) 3.4 eV, 3.4 eV (b) -3.4 eV ,-3.4 eV

(c) -3.4 eV ,-6.8 eV (d)3.4eV, -6.8eV


(ii).Given the value of Rydberg constant is107m-1,the wave number of the last line of the Balmer series
in hydrogen spectrum will be
(a)0.5x107 (b)0.25x107
(c)2.5x107 (d)0.025x104
(iii). The ratio of wavelength of last line of Balmer series and the last line of Lyman series
(a)0.5 (b)2 (c)1 (d)4
(iv). The wave length of Balmer series lies in

41
(a) Ultraviolet region (b)Infrared region
(c) far infra-red region (d) visible region
(v) In the empirical formula for the observed wavelength (λ) for hydrogen is 1/λ=R(1/42-1/n2)
Where n is integral values higher than 4 then it represents series.
(a) Balmer series (b)Brackett series
(c) Pfund series (d) Lyman series
II. RUTHERFORD ATOMIC MODEL
A radio active source emitting alpha particles was enclosed within a protective lead shield.
The radiation was focused into a narrow beam after passing through a slit in a lead screen. A thin section
of gold foil was placed in front of the slit and a screen coated with zinc sulfide to render it fluorescent
served as a counter to detect alpha particles. As each alpha particle struck the fluorescent screen, it
produced a burst of light called a scintillation, which was visible through a viewing microscope attached
to the back of the screen. The screen itself was movable, allowing to determine whether or not any alpha
particles were being deflected by the gold foil.

(i) The particles which were deflected backwards in Rutherford’s experiment were hit upon by
(a) Nucleus (b)Empty space (c)Electrons (d)Protons
(ii) According to the Rutherford atomic model, the whole atom is
(a) Positively Charged (b)Negatively Charged
(c)Neutral (d)None of the above
(iii)Rutherford in his atomic model could not explain the behavior of which of the following
(a) Proton (b)Neutron (c)Electron (d)Neutrino

42
(iv) Electron revolves around the nucleus in orbits which have
(a) variable energy (b)fixed energy
(c) infinite energy (d)zero energy
(v) According to Rutherford, most of the space occupied by the atom is
(a) Filled (b)partially filled
(c) empty (d) none of above
III.NUCLEAR REACTOR
Apsara is the oldest of India's research reactors. The reactor was designed by the Bhabha Atomic Research
Center (BARC) and built with assistance from the United Kingdom. A nuclear reactor, formerly known
as an atomic pile, is a device used to initiate and control a self- sustained nuclear chain reaction. Nuclear
reactors are used at nuclear power plants for electricity generation and in nuclear marine propulsion.
Heat from nuclear fission is passed to a working fluid (water or gas), which in turn runs through steam
turbines.

(i) The splitting of a nucleus into smaller nuclei is


(a) Fusion
(b) Fission
(c) Half-life
(d) gamma –radiation
(ii) Name the moderator used in the nuclear reactor?
(a) Plutonium
(b) Thorium
(c) Graphite

43
(d) Berilium
(iii)What is the beneficial aspect of nuclear fission?
(a) The ability to absorb energy
(b) The ability to produce more energy than nuclear fusion
(c) The ability to release tremendous amounts of energy
(d) There are no beneficial aspects of nuclear fission
(iv)The energy we get in nuclear reaction comes from
(a)Energy we put into the reactor (b)The mass of the fuel
(c)Water (d)The sun
(v) Which isotope of Uranium has the capacity to sustain the chain reaction?
(a) U-230 (b) U-235 (c) U-245 (d) U-225

IV.NUCLEAR DENSITY
The Nucleus of an atom consists of a tightly packed arrangement of protons and neutrons. These are the
two heavy particles in an atom and hence 99.9% of the mass is concentrated in the nucleus. Of the two,
the protons possess a net positive charge and hence the nucleus of an atom is positively charged on the
whole and the negatively charged electrons revolve around the central nucleus. Since the mass
concentration at the nucleus of an atom is immense the nuclear forces holding the protons and the neutrons
together are also large.
(i) The nuclide 92U238 has all the following except
(a)92 protons (b)146 neutrons (c)238 nucleons (d) 0 electrons

(ii) The density of a nucleus is of the order of:


(a) 1015 kg m-3 (b) 1018 kg m-3
(c) 1017 kg m-3 (d) 1016 kg m-3
(iii)Nuclear force is:
(a) strong, short range and charge independent force
(b) charge independent, attractive and long range force
(c) strong, charge dependent and short range attractive force
(d) long range, change dependent and attractive force
(iv) The mass no. of a nucleus is M and its atomic no. is Z. The number of neutrons in the nucleus is
:

44
(a) M – Z (b)M (c)Z (d)M + Z
(v) The atomic mass number is equivalent to which of the following?
(a) The number of neutrons in the atom.
(b) The number of protons in the atom.
(c) The number of nucleons in the atom
(d) The number of 𝛼 −particles in the atom.
V.BOHR’S MODEL
The Bohr model is a big part of Physics history. Neils Bohr proposed his model in 1913. It states that
electrons orbit the nucleus at set distances. The model was an expansion on the Rutherford model
overcame. Neils Bohr’s model was based on his observations of the atomic emissions spectrum of the
hydrogen atom. His findings said that the electron can move to a higher-energy orbit by gaining an amount
of energy equal to the difference in energy between the higher-energy orbit and the initial lower-energy
orbit. But time and research has proven and changed the Bohr model; Making this model one of the most
famous models in Physics history.
(i) Rutherford’s model of the atom concentrated on the nucleus while Bohr’s model focused on the:
(a) Electrons (b)Protons (c)Quarks (d)Neutrons
(ii) When an electron jumps from outer orbit to inner orbit, energy is
(a) Absorbed (b)Released (c)No change (d)Remains constant
(iii)The orbits in which electrons move according to Bohr are
(a) Elliptical (b) Cylindrical (c) Circular (d)Oval
(iv) In Bohr’s model of the atom, the energy level closest to the nucleus would be the:
(a) Valence energy level (b)Lowest energy level
(c)Average energy level (d) Highest energy level
(v) The radii of stationary orbits is proportional to
(a) n (b) n2 (c) n-1 (d)n-2
PRACTICE PAPER
SECTION A (02 MARK each)
1. A nucleus with mass number A is equal to 240 and B.E/ A = 7.6 eV breaks into two fragments
each of A=120 with B.E/A = 8.5 eV. Calculate the released energy.
SECTION B ( 03 MARK each)
2. The value of ground the value of ground state energy of hydrogen atom is -13.6 eV

45
(a) Find the energy required to remove an electron from ground state to the first excited state of the
atom.
(b) Determine (i)the kinetic energy, and (ii) the orbital radius in the first excited state of the atom.
SECTION C
CASE STUDY I
NUCLEAR FORCE
Neutrons and protons are identical particle in the sense that their masses are nearly the same and the force,
called nuclear force, does into distinguish them. Nuclear force is the strongest force. Stability of nucleus
is determined by the neutron Proton ratio or mass defect or packing fraction. Shape of nucleus is calculated
by quadruple moment and the spin of nucleus depends on even or odd mass number. Volume of nucleus
depends on the mass number. Whole mass of the atom (nearly 99% )is centered at the nucleus.
1. The correct statement about the nuclear force is are
(a) Charge independent
(b) Short range force
(c) Non conservative force
(d) all of these
2. The range of nuclear force is the order of
(a) 2 x10 -10 m (b) 1.5 x10-20 m (c) 1.2 x10 – 4 m (d) 1.4 x10 – 15 m
3. A force between two protons is same as the force between proton and neutron. The nature of the
force is
(a) electrical force (b)weak nuclear force
(c) gravitational force (d)strong nuclear force
4. All the nucleons in an atom are held by
(a) Nuclear forces (b) Vander Waal’s forces
(c) Tensor forces (d)Coulomb forces
5. Nuclear force is:
(a) strong, short range and charge independent force
(b)charge independent, attractive and long range force
(c) strong, charge dependent and short range attractive force
(d)long range, change dependent and attractive force

46
Case study (II)
The emission spectrum of a chemical element or chemical compound is the spectrum of frequencies of
electromagnetic radiation emitted by an atom’s electrons when they are returned to a lower energy state.
Each element’s emission spectrum is unique, and therefore spectroscopy can be used to identify elements
present in matter of unknown composition. Similarly, the emission spectra of molecules can be used in
chemical analysis of substances. The emission spectrum of atomic hydrogen is divided into a number of
spectral series, with wavelengths given by the Rydberg formula.
(i) When an electron jumps in n=1 orbit, the series of spectral lines obtained is called:
(a) Balmer Series (b)Pfund Series (c)Brackett Series (d)Lyman Series
(ii) The wave numbers decreases from
(a) Lyman to Pfund series (b)Pfund series to Lyman series
(c) Balmer series to Brackett series (d)None of these
(iii)Which of the series lies in Visible region of electromagnetic spectrum?
(a) Lyman (b)Balmer (c)Paschen (d)Brackett
(iv) Which of the series lies in U.V region of electromagnetic spectrum?
(a) Lyman (b)Balmer (c)Paschen (d)Brackett
(v) The value of maximum wavelength emitted in Lyman series is
(a) 1/R (b) 4/3R (c) 4/R (d)3/4R

SOLUTION
Section –A (2 MARKS EACH)
Solution-1 :
In ground state, the kinetic energy of the electron is
13.6
𝐾 = −𝐸 = + 𝑒𝑉 = 13.6 × 1.6 × 10−19𝐽 = 2.18 × 10−19𝐽
12

ℎ ℎ
de-Broglie wavelength 𝜆 = 𝑝 =
√2𝑚𝐾

6.63 × 10−34
𝜆=
√2 × 9.1 × 10−31 × 2.18 × 10−19
𝜆 = 0.33 × 10−9 𝑚 = 0.33𝑛𝑚
Solution-2 :

47
Energy released when electron jumps from 𝑛 = 𝛼 𝑡𝑜 𝑛 = 1
1 1
𝐸 = −13.6 [ − ( 2 )] 𝑒𝑉 = −13.6 𝑒𝑉
𝛼 1
ℎ𝑐 ℎ𝑐
𝐸= ⇒𝜆= = 91.17 𝑛𝑚
𝜆 𝐸
Solution-3 :
1
𝑅1 1 3 1
=( ) =
𝑅2 27 3
Solution-4 :
For second excited state, 𝑛 = 3
−13.6 −13.6
𝐸𝑛 = = 𝑒𝑉 = −1.5 𝑒𝑉
32 9
𝐾 ∙ 𝐸 = −𝐸𝑛 = 1.5 𝑒𝑉
𝑃 ∙ 𝐸 = −2(𝐾 ∙ 𝐸) = −2(1.5) = −3.0 𝑒𝑉
Solution-5 :
𝑛(𝑛−1)
Total no.of spectral lines emitted = 𝑁 = 2

4(4 − 1)
𝑁= =6
2
Solution-6 :
Energy of electron in ground state 𝐸 = −13.6 𝑒𝑉
−13.6 −13.6
Energy of electron in 𝑛 = 2 state = =
22 4

Energy required to remove an electron from 𝑛 = 2 orbit = 3.4 𝑒𝑉


Solution-7 :
At the distance of closest approach
Electrostatic potential energy = initial kinetic energy of Alpha particle. K Ze.2e/ro= 1/2 (mv2)
ro= 2KZe.2e/v2m= 2.5x10-14m
Solution-8 :
(i) En α 1/n2
(ii) The energy level are -13.6 eV , -3.4 eV , -1.5 eV
Therefore 12.5 eV electron beam can excite the electron upto n = 3 level only.
(iii) Energy values , of the emitted photons, of the three possible lines are 3 →1 : (-1.5 +
13.6) eV = 12.1 eV

48
2→1: ( - 3.4 +13.6 )eV = 10.2 eV
3→2 : (- 1.5 + 3.4) eV = 1.9 eV
The corresponding wavelengths are are 102nm , 122 nm and 653 nm
Solution-9 :
No, because according to Bohr’s model
−13.6
𝐸𝑛 = and electron having different energies belong to different levels having different values
𝑛2
𝑛ℎ
of n. So, their angular momentum will be different, as 𝐿 = 𝑚𝑣𝑟 = 2𝜋

Solution-10 :
According to de- Broglie hypothesis λ = h/mv
According to de de- Broglie condition for stationary orbit, the stationary orbits are those
which contains complete de - Broglie wavelength 2π r= n λ

2π r =nh/mv
mvr= nh/2π

Section –B (3 MARKS EACH)


Solution-1:
According to Bohr's postulate, in an hydrogen atom, a single electron revolves around the nucleus of
charge + e. For an electron moving with a uniform speed in a circular orbit on given radius, the
centripetal force is provided by the Coulomb force of attraction between the electron and the nucleus.

𝒎𝒗𝟐 𝒁𝒆𝟐
=𝒌
𝒓 𝒓𝟐
1 𝑍𝑒 2 𝒁𝒆×−𝒆 −𝒌𝒁𝒆𝟐
𝐾. 𝐸 = 2 𝑚𝑣 2 = 𝑘 Potential Energy = 𝑷𝑬 = 𝒌 =
2𝑟 𝒓 𝒓

49
−𝒌𝒁𝒆𝟐 𝒁𝒆𝟐
Total Energy = 𝑻𝑬 = +𝒌
𝒓 𝟐𝒓

−𝒌𝒁𝒆𝟐
𝑬=
𝟐𝒓
K.E = -E and P.E = - 2 K.E

Solution-2 :
1/λ1 = R(1/22 – 1/32 ) =( 5/36 ) R
1/λ2 = R(1/22 – 1/42 ) =( 3/16 ) R
λ1 / λ2 = 27/20
λ1 = (27/20)x4861 = 6562 Ao
Solution-3:
En = -13.6/n2 , Given E n = -3.4 Ev ,

-3.4 = -13.6/n2 N2 = 4 , n=2


Angular momentum ,mvr= nh/2π = 2.1x10 -34 Js
Solution-4 :
(a) The neutron multiplication factor of fissionable mass is defined as the ratio of the number of
neutrons present in the beginning of a particular generation to the number of neutrons present in the
beginning of the previous generation .Multiplication factor k gives a measure of the growth rate of
the neutron in a fissionable mass.
Critical size : The size of the fissionable material for which the multiplication factor k=1 , is called
the critical size and the corresponding mass is called the Critical Mass
(b) Let the number of fissions per second be n.
Energy released per second = nx200MeV = nx200X 1.6X10-19 J Energy required per second = 1000J
nx200X 1.6X10-19 J = 1000
n = 3.125x1013
Solution-5 :
(a)

50
(b) Properties of nuclear force:
1. Nuclear forces are the strongest attractive force known in nature.
2. They are short range force.

Solution-6 :

13.6
𝐸𝑛 = − 𝑒𝑉
𝑛2
13.6
−0.85 𝑒𝑉 == − 2 𝑒𝑉 ⇒ 𝑛𝐴 = 4
𝑛𝐵
13.6
−3.4 𝑒𝑉 == − 𝑒𝑉 ⇒ 𝑛𝐵 = 2
𝑛𝐵2
Therefore electron transaction from energy level
1 1 1
−0.85 𝑒𝑉 𝑡𝑜 − 3.4 𝑒𝑉 = = 𝑅 ( 2 − 2)
𝜆 𝑛𝐵 𝑛𝐴
1 1 1
= 1.097 × 107 ( 2 − 2 )
𝜆 2 4
𝜆 = 4862Å
Solution-7:

Given 𝜆 = 275 𝑛𝑚
ℎ𝑐
𝐸=
𝜆
ℎ𝑐 6.63 × 10−34 × 3 × 108
𝐸= = = 4.5 𝑒𝑉
𝜆 275 × 10−9 × 1.6 × 10−19
This corresponds to transition B as from the figure
ℎ𝑐 ℎ𝑐
(1) ∆𝐸 = ⇒𝜆= ,
𝜆 ∆𝐸
For maximum wavelength should be minimum.
Minimum energy corresponds to transition A.
(2) For minimum wavelength should be maximum.
Maximum energy corresponds to transition D.

51
Solution-8 :
Frequency of revolution of electron = f = 1/T = v/2πr = 6.57x1015 m/s
From Bohr’s theory, the frequency f = 2π2mk2Z2e4/h3 (1/n12 – 1/n2 2)
Given
f = 2π2mk2Z2e4/h3 (1/(n-1)2 – 1/n2) 2

f = 2π mk Z e (2n-1)/h (n-1) n
2 2 2 4 3 2 2

For large value of n , 2n-1 = 2n , n-1=n and for hydrogen atom Z=1 f =( 2π2mk2e4)/h3(2n/n4) ==
4π2mk2e4/n3h3
Velocity of electron in nth orbit = nh/2πmr Radius of nth orbit = n 2h2/4πmke2
The orbital frequency of electron in the nth orbit v/2πr = 4π2mk2e4/n3h3
Hence for large value of n , the classical frequency of revolution of electron in the nth orbit is same
as that obtained from Bohr’s theory.
Solution-9 :
No. of atoms present in 2𝑔 of deuterium= 6 × 1023
No. of atoms present in 2 𝑘𝑔 of deuterium= 6 × 1026
Energy released in fusion of 2𝑔 deuterium atoms= 3.27 𝑀𝑒𝑉
3.24
Energy released in fusion of 2 𝑘𝑔 deuterium atoms= × 6 × 10 × 1026 𝑀𝑒𝑉
2

= 9.8 × 1026 𝑀𝑒𝑉 = 15.696 × 1013 𝐽


Energy consumed by bulb per second = 100 𝐽
15.696×1013 𝑆
Time for which bulb will glow = × 4.9 × 104 years.
100

Solution-10 :
(a) Bohr gave following three postulates for hydrogen atom:
1. An electron in an atom could revolve in stable orbits without the emission of radiant energy.
Each atom has certain definite stable states in which it can exist, and each possible state has definite
total energy. These are called the stationary states of the atom.
2. An electron revolves round the nucleus in for orbit to be stationary (or non-radiating), the angular
momentum of the electron must be an integer multiple of h where it is the Planck’s constant. Thus,
Ln = mvnrn = nh/2π
3. Whenever an electron shifts from one of its specified non-radiating orbit to another orbit, it emits/
absorbs a photon whose energy is equal to the energy difference between the initial and final states .

52
Thus, Ef – Ei = hc/λ
(b) We know that when an electron revolves in a stable orbit, the centripetal force is provide
electrostatic force of attraction acting on it due to a proton present in certain specified circular orbits
in which it does not radiate energy. The centripetal force required for uniform circular motion in
stationary orbit is provided by electrostatic force of attraction. Thus,
𝒎𝒗𝟐 𝒁𝒆𝟐 𝒁𝒆𝟐
=𝒌 or 𝒗𝟐 = 𝒌 𝒎𝒓
𝒓 𝒓𝟐

and from Bohr’s quantization condition ,


mvn rn= nh/2π or
vn = nh/2πmrn….(2)
𝒁𝒆𝟐 𝒏𝟐 𝒉 𝟐
Squaring (2) and equating it with (i) , we get (nh/2πmr n )2 = 𝒌 or 𝒓 = 𝟒𝝅𝟐 𝒎 𝒌 𝒁𝒆𝟐
𝒎𝒓
𝒉𝟐
In stable orbit of hydrogen atom n = 1, and Z=1 then the Bohr’s radius is 𝒓𝟎 = 𝟒𝝅𝟐 𝒎 𝒌 𝒆𝟐

Section –C(CASE BASED)


CASE BASED OPTION
(I) HYDROGEN ATOM SPECTRUM 1.d 2.b 3.d 4.d 5.b
(II) RUTHERFORD ATOMIC MODEL 1.a 2.c 3.d 4.b 5.c
(III)NUCLEAR REACTOR 1.b 2.c 3.b 4.b 5.b
(IV)NUCLEAR DENSITY 1.c 2.c 3.a 4.a 5.c
(V)BOHR’S MODEL 1.a 2.b 3.c 4.d 5.b

SEMICONDUCTOR ELECTRONICS
MIND MAP

53
SHORT ANSWER TYPE QUESTIONS(2 MARKS)
1. The V-I characteristics of a silicon diode is shown in the figure. Calculate the resistance of the diode at
(i) I = 10 mA and V = -10 V

2. In the following diagrams, write which of the diodes are forward biased and which are reverse biased ?

54
(a) (b)
3. A p-n photo diode is fabricated from a semiconductor with a band gap 2.8eV. Can it detect a wavelength
of 6000nm?
4. An intrinsic semiconductor has 5x1028 atoms per metre2 with ni =1.5 x 106 m3,If it is doped with
pentavalent impurity of concentration 1 ppm, calculate the number of electrons holes in the sample.
5. Three photo diodes D1,D2, D3 are made of semiconductors having bas gaps of 2.5eV,2eV and 3eV
respectively .Which one will be able to detect the light of wavelength 6000A0?
6. Write two points of difference between intrinsic and extrinsic semiconductors.
7. Find the maximum wavelength of electromagnetic radiation which can create a hole-electron pair in
Ge. The band gap in germanium is 0.65e V.
8.A p-n photodiode is fabricated from a semiconductor with band gap of 2.8 e V. Can it detect a
wavelength of 6000 nm?
9. What is dynamic resistance of a diode? When the voltage drop across a p-n junction diode is increased
from 0.70V to 0.071V, the change in the diode current is 10mA.What is the dynamic resistance of the
diode?
10. Pure silicon at 300K has equal electron & hole concentrations of 1.5 x 1016/m3.Doping by indium
increases the hole concentration to 4.5 x 1022/m3.Calculate the new electron concentration in the doped
solution.
SHORT ANSWER TYPE QUESTIONS (3 MARKS)
1. Explain, with the help of a circuit diagram, the working of a p-n junction diode as a half wave rectifier.
2. A semiconductor has equal electron and hole concentration of 4.2 x 108m-3 . On doping with a certain
impurity, electron concentration increases to 6 x 10 12 m-3.
(i) Identify the new semiconductor obtained after doping .(ii) calculate the new hole concentration .

55
3. Define the terms potential barrier and depletion region for a p – n junction diode. State how the
thickness of depletion region will change when the p – n junction diode is ( i ) forward biased ( ii ) reverse
biased.
4. On the basis of energy band diagrams distinguish between metals, insulators and semiconductors.
5. (i) Explain briefly the process of emission of light by an LED. (ii) Which semiconductors are
preferred to make LED and why?
(iii) Give two advantages of using LEDs over conventional in candescent lamps.
6. Draw energy band diagrams of n-type and p-type semiconductors. Also write two differences between
n-type and p-type semiconductors.
7. Explain, with the help of a circuit diagram, the working of a p-n junction diode as a full-wave rectifier.
8.Draw the circuit to forward bias a diode (The supply is 3V and 100mA battery).If the diode is made of
silicon and knee voltage is 0.7V, and a current of 20mA passes through the diode, find the wattage of the
resistor and the diode.
9. (a) Explain reverse biasing of a P-N junction diode with the help of a diagram.
(b) Draw V- I characteristics of a p-n junction diode. Answer the following questions giving reasons:
(i)Why is the current under reverse bias almost independent of the applied potential up to a critical
voltage?
(ii)Why does the reverse current show a sudden increase at the critical voltage?
10. With the help of a labeled diagram, explain the working of full wave rectifier. Draw the input and
output waveforms.
CASE BASED QUESTIONS (5 MARKS)
Read the Case Study given below and answer the questions that follow:
Consider a thin p-type silicon (p-Si) semiconductor wafer. By adding precisely, a small quantity of
pentavalent impurity, part of the p-Si wafer can be converted into n-Si. There are several processes by
which a semiconductor can be formed. The wafer contains p-region and n-region and a metallurgical
junction between p-, and n- region. Two important processes occur during the formation of a p-n
junction: diffusion and drift. We know that in an n-type semiconductor, the concentration of electrons
(number of electrons per unit volume) is more compared to the concentration of holes. Similarly,
in a p-type semiconductor, the concentration of holes is more than the concentration of electrons.
During the formation of p-n junction, and due to the concentration gradient across p-, and n- sides, holes
diffuse from p-side to n-side (p → n) and electrons diffuse from n-side to p-side (n → p). This motion
of charge carries gives rise to diffusion current across the junction.

56
I.How can a p-type semiconductor be converted into n- type semiconductor?
a) adding pentavalent impurity b) adding trivalent impurity
c)not possible d) heavy doping
II. Which of the following is true about n type semiconductor?
a) concentration of electrons is less than that of holes.
b) concentration of electrons is more than that of holes.
c)concentration of electrons equal to that of holes.
d) None of these
III. Which of the following is true about p type semiconductor?
a) concentration of electrons is less than that of holes.
b) concentration of electrons is more than that of holes.
c)concentration of electrons equal to that of holes.
d)None of these
IV. Which of the following is the reason about diffusion current?
a) diffusion of holes from p to n b) diffusion of electrons from n to p
c) both (a) and (b) d) None of these
V. What are the processes that occur during formation of a p-n junction?
a) drift b) diffusion c) both (a) and (b) d)None of these .
ANSWER/HINT:
SHORT ANSWER TYPE QUESTIONS(2 MARKS)-
1. (i) R = ΔV/ΔI = (0.8- 0.7)/ (20-10)x 10-3 =10 Ω
(ii) R= V/I= 10/10-6 = 107Ω
2.(a)Reverse biased(b) Forward biased
3. Wavelength of incident photon ,
λ = 6000nm =6x 10-6m
energy of incident photon E =hc/λ =0.207eV
as E< Eg p-n junction can not detect the radiation.
4. nh = ni2/ ne=4.5 x 109m-3
5. E =hc /λ = (6.6 x 10-34 ) x( 3 x 108) / (6x10-7) x 1.6 x 10-19) =2.06eV
Incident radiation can be detected by a photo diode if energy of incident radiation photon is greater than
the band gap. This is true for D2 ,therefore only D2 will detect these radiation.
7.Eg = 0.65e V =0.65 X 1.6 X 10-19 J

57
Eg = hc /λmax . λmax= hc/ Eg=1.9 x 10-6 m.
8. E =hc/λ =3.313 X 10-20 J
9. rd = Δv/ΔI =0.01V /10X10-3 A =1Ω
10. ne = ni2 / nh = 5 x 109 m-3.
SHORT ANSWER TYPE QUESTIONS (3 MARKS)-
2. (i) since electron concentration increases, so the majority charge carriers in the doped semiconductor
are electrons hence new semiconductor is n-type .
(ii) nh = ni2/ ne =2.94 x 104m-3
8. Wattage of R =voltage drop across R x current =2.3 x 20 x 10-3 =0.046W
Wattage of diode = Voltage drop across diode x current
=0.7 x 20 x 10-3 = 0.014 W
CASE BASED QUESTIONS-
I.Ans : a II.Ans:b III.Ans :a IV.Ans. c V.Ans: c

TEST YOURSELF
Q.1. I n a p-n- junction diode the forward bias resistance is low as compared to the reverse bias
resistance. Give reason.
Q.2.Can a slab of p-type semiconductor be physically joined to another n-type semiconductor slab to
form p-n junction ? Justify your answer.
Q.3.What is an ideal junction diode? How does the width of the depletion region of p-n junction diode
vary, If the reverse bias applied to it decreases?
Q.4. Describe briefly using the necessary circuit diagram, the three basic processes which takes place to
generate the emf in a solar cell when light falls on it. Draw the I-V characteristics of a solar cell.
Q.5. Show the biasing of a photo-diode with the help of a circuit diagram. Draw graphs to show variations
in reverse bias currents for different illumination intensities.
Q.6.A semiconductor has equal electron and hole concentration of 2 x 108 m-3.On doping with a certain
impurity, the hole concentration increases to 4 x 1010 m-3 (i) What type of semiconductor is obtained on
doping?(ii) How does the energy gap vary with doping?
Q.7. A full wave rectifier has two diodes, the internal resistance of each diode may be assumed constant
at 25 Ω . The transformer rms secondary voltage from centre tap to each end of the secondary is 50V and
the load resistance is 975 Ω . Find the (i) mean load current(ii) rms value of load current.

58
Q.8. (i) Explain briefly the process of emission of light by an LED. (ii) Which semiconductors
are preferred to make LED and why?
(iii) Give two advantages of using LEDs over conventional in candescent lamps.
Q.9.(i) With the help of circuit diagrams distinguish between forward biasing and reverse biasing of a
pn junction diode.
(ii) Draw V-I characteristics of a p-n junction diode in (a) forward bias, (b) reverse bias.
(iii) Describe briefly the following terms: (a) “minority carrier injection” in forward bias, (b)
“breakdown voltage” in reverse bias.
Q10. Read the Case Study given below and answer the following questions:
A Photodiode is again a special purpose p-n junction diode fabricated with a transparent window to
allow light to fall on the diode. It is operated under reverse bias. When the photodiode is illuminated
with light (photons) with energy (hν) greater than the energy gap (E) of the semiconductor, then electron-
hole pairs are generated due to the absorption of photons. The diode is fabricated such that the generation
of e-h pairs takes place in or near the depletion region of the diode. Due to electric field of the junction,
electrons and holes are separated before they recombine. The direction of the electric field is such that
electrons reach n-side and holes reach p-side. Electrons are collected on n-side and holes are collected
on p-side giving rise to an emf. When an external load is connected, current flows. The magnitude of
the photocurrent depends on the intensity of incident light.
I. Photo Diode is operated in
a) forward biased b) reverse biased c) Not biased
II. Which of the following is true about photodiode?
a) E > hv b) E = hv c) E < hv d) None of these.
III. Magnitude of photocurrent depends on
a) Intensity of light b) Biasing c) Potential d) None of these
IV. Electrons and holes are separated before they recombine by:
a) Diffusion current b) Drift current c) Electric field d) Electric potential.
V. Direction of electric field is such that
a) electrons reach n- side b) holes reach p- side
c) Both (a) and (b) d) holes reach n side

59
SAMPLE QUESTION PAPER -1
CLASS XII
PHYSICS THEORY
TERM II SESSION 2021 – 22
BLUE PRINT

NAME OF CHAPTERS SA-I (2 SA-II CASE STUDY TOTAL


MARKS) (3MARKS) (5 MARKS)
CH-8 ELECTROMAGNETIC WAVES 1(3) 17
CH-9 RAY OPTICS AND OPTICAL 1(3) 1(5)
INSTRUMENTS 1(3) OR
CH-10 WAVE OPTICS 2(3)
1(3) OR
CH-11 DUAL NATURE OF RADIATION AND 1(2) 1(3) 12
MATTER
CH-12 ATOMS 1(3)
CH-13 NUCLEI 1(3)

CH-14 SEMICONDUCTOR 2(2) 1(3) 7


1(2) OR
3(2) 8(3) 1(5) 12(35)

QUESTION PAPER

TIME : 2 Hours MM : 35
General Instructions:
(i) There are 12 questions in all. All questions are compulsory.
(ii) This question paper has three sections: Section A, Section B and Section C.
(iii) Section A contains three questions of two marks each, Section B contains eight questions
of three marks each, Section C contains one case study-based question of five marks.
(iv) There is no overall choice. However, an internal choice has been provided in one question
of two marks and two questions of three marks. You have to attempt only one of the
choices in such questions.
(v) You may use log tables if necessary but use of calculator is not allowed.
SECTION A
Q.1 Two lines A and B in the plot given below show the variation of De Broglie wavelength 2
λ versus 1/ . Where V is the accelerating potential difference for two particles carrying

60
the same charge .Which one represents a particles of smaller mass.
Q.2 Name the optoelectronic device used for detecting optical signals and mention the 2
biasing in which it is operated. Draw its I-V characteristics.
OR
Three photo diodes D1, D2 and D3 are made of semiconductors having band gaps of 2.5 2
eV, 2 eV and 3 eV respectively. Which of them will not be able to detect light of
wavelength 600 nm?
Q.3 Write two characteristic features to distinguish between n-type and p-type 2
semiconductors.
SECTION-B
Q.4 3
Draw a graph between the frequency of incident radiation (ν) and the maximum kinetic
energyof the electrons emitted from the surface of two photosensitive materials A & B.
State clearly how this graph can be used to determine (i) Planck’s constant and (ii) work
function of the material.

Q.5 (a) How is the size of a nucleus experimentally determined? Write the relation between 1
the radius and mass number of the nucleus.

(b)Distinguish between nuclear fission and fusion. Show how in both these processes 2
energy isreleased.
Calculate the energy release in MeV in the deuterium-tritium fusion reaction :

Q.6 1
(a) Write two important limitations of Rutherford model which could not explain the
observedfeatures of atomic spectra. How were these explained in Bohr’s model of
hydrogen atom? Use the Rydberg formula to calculate the wavelength of the Hα line.
R=1.1x10-7m-1
(b) Using Bohr’s postulates, obtain the expression for the radius of the nth orbit in 2
hydrogen atom.
Q.7 (a) Name the EM waves which are produced during radioactive decay of a nucleus. 3
Write their frequency range.

61
(b) Welders wear special glass goggles while working. Why? Explain.
(c) Why are infrared waves often called as heat waves? Give their one application.
Q.8 Draw a ray diagram for formation of image of a point object by a thin double convex 3
lens having radii of curvature R1 and R2. Hence, derive lens maker’s formula for a double
convex lens. State the assumptions made and sign convention used.
OR
Using the data given below, state as to which of the given lenses will you prefer to use 3
as an eye piece & an objective to design compound microscope.
Lens Power Aperture
A 20 D 0.02 m
B 10 D 0.02 m
C 10 D 0.05 m
D 1.0 D 0.1 m
Q.9. (a) Use Huygens’ principle to show how a plane wave front propagates from a denser to 2
rarer medium. Hence, verify Snell’s law of refraction.
(b) In a single slit diffraction experiment , a slit of width d is illuminated by red light of 1
wavelength 650 nm .For what value of ‘d’ will First minimum fall at angle of
diffraction of 30°
Q.10. Draw a labelled diagram of a full -wave rectifier circuit. State its working principle. 3
Show the input-output wave forms.
Q.11 (a) In Young’s double slit experiment, describe briefly how bright and dark fringes are 3
obtained on the screen kept in front of a double slit. Hence obtain the expression for the
fringe width.
(b) The ratio of the intensities at minima to the maxima in the Young’s double slit
experiment is 9: 25. Find the ratio of the widths of the two slits.
OR
In young’s double slit experiment, deduce the condition for (a) constructive and 3
destructive interference at a point on the screen. Draw a graph showing variation of
intensity in the interference pattern against position ‘x’ on the screen.
. SECTION-C
Q.12 Case Study based questions 1x5=5

Sparking Brilliance of Diamond:


62
The total internal reflection of the light is used in polishing diamonds to create a
sparking brilliance. By polishing the diamond with specific cuts, it is adjusted the most
of the light rays approaching the surface are incident with an angle of incidence more
than critical angle. Hence,they suffer multiple reflections and ultimately come out of
diamond from the top. This gives thediamond a sparking brilliance.
(i) Light cannot easily escape a diamond without multiple internal reflections. This is
because:
(a) Its critical angle with reference to air is too large
(b) Its critical angle with reference to air is too small
(c) The diamond is transparent
(d) Rays always enter at angle greater than critical angle
(ii) The critical angle for a diamond is 24.4°. Then its refractive index is
(a) 2.42 (b) 0.413 (c) 1 (d) 1.413
(iii) The basic reason for the extraordinary sparkle of suitably cut diamond is that
(a) It has low refractive index (b) It has high transparency
(c) It has high refractive index (d) It is very hard

(iv) A diamond is immersed in a liquid with a refractive index greater than water. Then
the criticalangle for total internal reflection will
(a) will depend on the nature of the liquid (b) decrease
(c) remains the same (d) increase

(v) The following diagram shows same diamond cut in two different shapes.

63
The brilliance of diamond in the second diamond will be:
(a) less than the first (b) greater than first
(c) same as first (d) will depend on the intensity of light

MARKING SCHEME
Q ANSWER MARKS
NO.
A1 1+1=2

A2 Photodiode is used for detecting optical signals. ½+1/2+1=2


It is operated in reverse biasing.
I-V Characteristics:
A2-OR Energy of incident light photon ½+1/2+1=2
E =hv=hc/ λ
2 .06eV
For the incident radiation to be detected by the photodiode, energy of incident
radiation photon should be greater than the band gap. This is true only for D2.
Therefore, only D2 will detect this radiation.
A3 n-type Semiconductor p-type Semiconductor 1/2X4=2
(i) It is formed by doping pentavalent impurities.
It is doped with trivalent impurities.
(ii) The electrons are majority carriers and holes are minority carriers (ne >> nh).
The holes are majority carriers and electrons are minority carriers (nh >> ne).
64
SECTION-B
A4 1+1+1=3

determine (i) Planck’s constant and (ii) work function


A5 (a) By Alpha scattering R=R0A1/3 ½+1/2=1
(b) Mass of reactance =m1H2 + m 1H3= 2.014102+3.016049=5.030151 amu 1/2x4=2
Mass of product =m2He4 + m 0n1= 4.002603+1.008665=5.011268 amu
Q=(5.030151-5.011268) 931MeV=17.58MeV
A6 (a) two important limitations of Rutherford model 1/2X2=1
(b) expression for the radius 2
A7 (a) EM waves : γ-rays 1+1+1=3
Range : 1019 Hz to 1023 Hz
(b) This is because the special glass goggles protect the eyes from large amount of
UV radiations
produced by welding arcs.
(c) Infrared waves are called heat waves because water molecules present in the
materials
readily absorb the infrared rays and get heated up.
Application: They are used in green houses to warm the plants.
A8 Draw a ray diagram for formation of image derive lens maker’s formula 1+2=3
A8 OR 1+1+1=3

And fo <<fe
A → objective and
B → Eyepiece

65
A9 2+1=3

(a)

(b)D sin θ = nλ
d = sin 30 = 1× 650 × 10-9
d = 1300 nm.
A10 Diagram, working & waveform 1+1+1=3
A11(a) Expression for the fringe width. 2
(b) Intensity of light (using classical theory) is given as 1
I ∝ (Width of the slit) ∝ (Amplitude)2
I1:I2=16:1
A11- condition for (a) constructive and destructive interference And interference pattern 1+1+1=3
OR
SECTION -C 5x1=5
A12
(i) (b) Its critical angle with reference to air is too small 1
(ii) (a) 2.42 1
(iii) (c) It has high refractive index 1
(iv) (d) increase 1
(v) (a)less than the first 1

66
SAMPLE PAPER II
BLUE PRINT
CHAPTERS SA-I(2) SA-II(3) CBS(5) TOTAL(35)
ELECTROMAGNETIC WAVES 1 3
RAY OPTICS AND OPTICAL 2 1 11
INSTRUMENTS
WAVE OPTICS 1+1(OR) 3
DUAL NATURE OF RADIATION OR(1) 1 3
AND MATTER
ATOMS 1 1 5
NUCLEI 1 3
SEMI CONDUCTOR 2 1 7
ELECTRONICS

QUESTION PAPER II
SECTION A
Q1. Explain, with the help of a circuit diagram, the working of a p-n junction diode as a half-wave
rectifier.
Q2. Find the ratio of energies of photons produced due to transition of an electron of hydrogen atom
from its
• Second permitted energy level to the first permitted level and
• The highest permitted energy level to the first permitted level.
OR
Draw a plot showing the variation of photoelectric current with collector plate potential for two
different frequencies, v1 > v2, of incident radiation having the same intensity. In which case will the
stopping potential be higher? Justify your answer.
Q3. Distinguish between a metal and an insulator on the basis of energy band diagram.
SECTION B
Q4. Show that the radius of the orbit in hydrogen atom varies as n2, where n is the principal quantum
number of the atom.
Q5. (i) Why is a photo diode operated in reverse bias mode?
(ii)For what purpose is a photo diode used?
(iii)Draw its I-V characteristics for different intensities of illumination.

67
Q6. (a) In a typical nuclear reaction, e.g.

although number of nucleons is conserved, yet energy is released. How? Explain.


(b) Show that nuclear density in a given nucleus is independent of mass number A.
Q7. How is a wavefront defined? Using Huygen’s construction draw a figure showing the
propagation of a plane wave refracting at a plane surface separating two media. Hence verify Snell’s
law of refraction.
Q8. Draw a ray diagram to show refraction of a ray of monochromatic light passing through a glass
prism. Deduce the expression for the refractive index of glass in terms of angle of prism and angle of
minimum deviation.
OR
Draw a labeled ray diagram of a refracting telescope. Define its magnifying power and write the
expression for it. Write two important limitations of a refracting telescope over a reflecting type
telescope.
Q9. (i) How does one explain the emission of electrons from a photosensitive surface with the help
of Einstein’s photoelectric equation?
(ii) The work function of the following metals is given : Na = 2.75 eV, K = 2.3 eV, Mo = 4.17 eV
and Ni=5.15 eV. Which of these metals will not cause photoelectric emission for radiation of
wavelength 3300 A from a laser source placed 1 m away from these metals? What happens if the
laser source is brought nearer and placed 50 cm away?
Q10. A convex lens made up of glass of refractive index 1.5 is dipped, in turn,
(i) a medium of refractive index 1.6,
(ii) a medium of refractive index 1.3.
(a) Will it behave as a converging or a diverging lens in the two cases?
(b) How will its focal length change in the two media?
Q11. (i) Name the EM waves which is suitable for radar systems used in aircraft navigation. Write
the range of frequency of these waves.
(ii) If the earth did not have atmosphere, would its average surface temperature be higher or lower
than what it is now? Explain.
(iii) An EM wave exerts pressure on the surface on which it is incident. Justify.
OR
(i)State the importance of coherent sources in the phenomenon of interference.
(ii) In Young’s double slit experiment to produce interference pattern, obtain the conditions for
68
constructive and destructive interference. Hence deduce the expression for the fringe width.
(iii) How does the fringe width get affected, if the entire experimental apparatus of Young is immersed
in water?

Q12. CASE STUDY: Total Internal Refraction


Total internal reflection is the phenomenon of reflection of light into denser medium at the interface
of denser medium with a rarer medium. For this phenomenon to occur necessary condition is that
light must travel from denser to rarer and angle of incidence in denser medium must be greater than
critical angle (C) for the pair of media in contact. Critical angle depends on nature of medium and
wavelength of light. We can show that n=1/sinC
(i)Critical angle for glass air interface, where n of glass is 3/2,is
(a) 41.80 (b) 60° (c) 30° (d) 15°
(ii) Critical angle for water air interface is 48.6°. What is the refractive index of water?
(a) 1 (b)3/2 (c)4/3 (d)3/4
(iii) Critical angle for air water interface for violet colour is 490. Its value for red colour would be
(a) 49° (b) 50° (c) 480 (d) cannot say
(iv) Which of the following is not due to total internal reflection?
(a) Working of optical fibre. (b) Difference between apparent and real depth of a
pond.
(c) Mirage on hot summer days. (d) Brilliance of diamond.
(v) Critical angle of glass is Ɵ1, and that of water is Ɵ2.The critical angle for water and glass surface
would be (ng = 3/2, nw=4/3).
(a) less than Ɵ2 (b) between Ɵ1 and Ɵ2
(c) greater than Ɵ2 (d) less than Ɵ1
MARKING SCHEME
Ans1. AC voltage to be rectified is connected to the primary coil of a step-down transformer.
Secondary coil is connected to the diode through resistors RL, across which output is
obtained.

69
During positive half cycle of the input AC, the p-n junction is forward biased. Thus, the
resistance in p-n junction becomes low and current flows. Hence, we get output in the load.
During negative half cycle of the input AC, the p-n junction is reverse biased. Thus, the
resistance of p-n junction is high and current does not flow. Hence, no output in the load.
So, for complete cycle of AC, current flows through the load resistance in the same
direction.

Ans 2

Ratio of energies of photons=10.2/13.6=3/4=3:4


OR
Stopping potential is directly proportional to the frequency of incident radiation. The stopping
potential is more negative for higher frequencies of incident radiation. Therefore, stopping
potential is higher in v1.

Ans 3 For metals, the valence band is completely filled and the conduction band can have two
possibilities either it is partially filled with an extremely small energy gap between the
valence and conduction bands or it is empty, with two bands overlapping each other as shown
below:

for insulator, the energy gap between the conduction and valence bands are very large, also
the conduction band is practically empty, as shown below:
Ans 4 When an electron moves around hydrogen nucleus, the electrostatic force between electron
and hydrogen nucleus provides necessary centripetal force.
70
Also we know from Bohr’s postulate,

Ans 5 (i) Photo diode is connected in reverse bias and feeble reverse current flows due to thermally
generated electron-hole pair, known as dark current. When light of suitable frequency(v)
such that hv>Eg where Eg is the band gap is incident on diode, additional electron hole
pair generated and current flows in the circuit
(ii) Main use of photo diode In demodulation of optical signal and detection of optical
signal.
(iii) Circuit diagram of illuminated photo diode in reverse bias is shown below:

Intensity of incident light.


Ans 6 (a)In all types of nuclear reactions, the law of conservation of number of nucleons is
followed. But during the reaction, the mass of the final product is found to be slightly less
than the sum of the masses of the reactant components. This difference in mass of a
nucleus and its constituents is called mass defect. So, as per mass energy relation E =
(∆M)c2, energy is released. In the given reaction the sum of the masses of two deutrons is
more than the mass of helium and neutron. Energy equivalent of mass defect is released.

71
Ans 7 (i) Wavefront : Wavefront is defined as the continuous locus of all such particles of the
medium which are vibrating in the same phase at any instant.

(ii) We take a plane wavefront AB incident at a plane surface XY. We use secondary
wavelets starting at different times. We get refracted wavefront only when the time taken
by light to travel along different rays from one wavefront to another is same. We take
any arbitrary ray starting from point ‘P’ on incident wavefront to refracted wavefront at
point ‘O’. Let total time be ‘t’.

As time should be independent of the ray to be considered


The coefficient of AO in the above equation should be zero

Where ’µ2 is called refractive index of medium 2 w.r.t. medium 1. This is Snell’s law of
refraction.
Ans 8 Ray diagram : The minimum deviation Dm, the refracted ray inside the prism becomes parallel
to its base, we have

72
OR

Magnifying power. It is defined as the ratio of angle (β) subtended by the final image on the
eye to the angle (α) subtended by object on eye.

Limitations of refracting telescope over a a reflecting type telescope :


1. It suffers from chromatic aberration due to refraction and hence the image obtained
is multi coloured and blurred.
2. As a lens of large apparatus can’t be manufactured easily, its light gathering power
is low and hence can’t be used to see faint stars
Ans 9

73
Because the work function of Mo and Ni is more than the energy of the incident photons; so
photoelectric emission will not take place from these two metals Mo and Ni. When the laser
source is brought nearer and placed 50 cm away, the kinetic energy of photo-electrons will
not change, only photoelectric current will change.
Ans 10

Let fair be the focal length of the lens in air


According to lens maker formula :

As the sign of fB is opposite to that of fair, the lens will behave as a diverging lens.
(b) When lens is dipped in medium B :

74
As the sign of fB is same as that of fair, the lens will behave as a converging lens.
Ans 11

OR
(i) (i) Importance of coherent source : Coherent sources are necessary to produce sustained
interference pattern. Otherwise the phase difference between the two interfering waves
will change rapidly and the interference pattern will be lost. .
(ii) For constructive interference : We will have constructive interference resulting in a
bright fringe when path difference is equal to nλ.

Since the separation between the centres of two consecutive bright fringes is called fringe
width. It is denoted by β

Hence all bright and dark fringes are of equal width.

Then fringe width becomes 1/μ times the original fringe width i.e., it will decrease in water.
Ans 12 (i) (a), (ii) (c) (iii) (c) (iv)(b) (v) (c)
75
SAMPLE PAPER III
BLUE PRINT

NAME OF CHAPTERS SA-I SA-II LA TOTAL


(2MARKS) (3MARKS) (5 MARKS)
CH-8 ELECTROMAGNETIC WAVES 1 17

CH-9 RAY OPTICS AND OPTICAL 2


INSTRUMENTS
CH-10 WAVE OPTICS 1 1

CH-11 DUAL NATURE OF RADIATION 1 1 11


AND MATTER
CH-12 ATOMS 1
CH-13 NUCLEI 1

CH-14 SEMICONDUCTOR 2 1 7

QUESTION PAPER

General Instructions:
(i)All questions are compulsory. There are 12 questions in the question paper.
(ii)The question paper have three sections: Section A, Section B, Section C
(iii)Section A contain three questions each of 2 mark. Section B contains 8 question of 3 marks each.
Section C contains one case study based question of 5 marks.
(iv)There is no overall choice in the question paper. However, an internal choice has been provided
in one question of two mark and two questions of three marks. You have to attempt only one of the
choices in such questions.
(v)You may use log table if necessary but use of Calculator is not allowed.
SECTION A
Q1.The graph shows the variation of stopping potential with frequency of incident radiation for two
photosensitive metals A and B. Which one of the two has higher value of work-function? Justify your
answer.

76
OR
An electron and a proton are moving with equal speed. Which one has greater de- Broglie wavelength.
Justify your answer.
Q2. Draw a labelled diagram of a full -wave rectifier circuit. Show the input-output wave forms.
Q3. What happens to the width of depletion layer of a p-n junction when it is (I) Forward Biased and
(II) Reverse Biased

SECTION B
Q4- Name the electromagnetic waves which

(i) maintain the earth’s warmth and


(ii) are used in aircraft navigation.
(iii) Used in water purifier.
Q.5 (a) State one feature by which the phenomenon of interference can be distinguish from that of
diffraction.
(b)A parallel beam of light of wavelength 600 nm is incident normally on a slit of width ‘a’. if the
distance between the slits and screen is 0.8m and the distance of 2nd order maximum from the centre
of screen is 1.5mm, calculate the width of the slit.
OR
Draw a schematic ray diagram of reflecting telescope showing how rays coming from distantobjects
are received at the eye-piece. Write its two important advantages over refracting telescope.
Q6. The given graph shows the variation of photo electric current (I) with the applied voltage (V) for
two different materials and for two different intensities of the incident radiations. Identify and explain
using Einstein‘s photoelectric equation for the pair of curves that correspond to (i) different materials
but same intensity of incident radiation (ii) different intensities but samematerials.
Q7. The ground state energy of hydrogen atom is -13.6 eV

77
(i) What is the potential energy of an electron in the 3rd excited state?

(ii) If the electron jumps to the ground state from the 3rd excited state, calculate the
wavelengthof the photon emitted.

Q8. Figure shows a ray of light passing through a prism. If the refracted ray QR is parallel to the base
BC, show that

(i) r1=r2=A/2
(ii) angle of minimum deviation Dm=2i-A

Q9. A convex lens made up of glass of refractive index 1.5 is dipped, in turn,
(i) a medium of refractive index 1.6,
(ii) a medium of refractive index 1.3.
(a) Will it behave as a converging or a diverging lens in the two cases?
(b) How will its focal length change in the two media?

Q10. (a) How is the size of a nucleus experimentally determined? Write the relation between the radius
and mass number of the nucleus. Show that the density of nucleus is independent of itsmass number.

Q11.(a) In the following diagrams, write which of the diodes are forward biased and which are reverse
biased?

78
(b) A photo diode is fabricated from a semiconductor with a band gap 2.8eV. Can it detect a
wavelength of 6000nm?

SECTION C
CASE STUDY BASED QUESTIONS

Wave front is a locus of points which vibrate in same phase. A ray of light is perpendicular to the wave
front. According to Huygens principle, each point of the wave front is the source of a secondary
disturbance and the wavelets connecting from these points spread out in all directions with the speed
of wave. The figure shows a surface XY separating two transparent media, medium-I and medium-2.
The lines ab and cd represent wave fronts of a light wave travelling in medium- 1and incident on XY.
The lines ef and gh represent wave fronts of the light wave in medium -2 afterrefraction.

(i) Light travels as a


(a) parallel beam in each medium
(b) convergent beam in each medium
(c) divergent beam in each medium
(d) divergent beam in one medium and convergent beam in the other medium
(ii)The phases of the light wave at c, d, e and f are ∅𝑐, ∅ , ∅𝑒𝑎𝑛𝑑 ∅𝑓 respectively. It is giventhat
∅𝑐 ≠ ∅𝑓
((a) ∅𝑐 cannot be equal to ∅𝑑 (b) ∅𝑑can be equal to ∅𝑒
(c) (∅𝑑 − ∅𝑓) is equal to (∅𝑐 − ∅𝑒) (d) (∅𝑑 − ∅𝑐) is not equal to(∅𝑓 − ∅𝑒)
(iii) Wave front is the locus of all points, where the particles of the medium vibrate with the
79
same……………..
(a) phase (b) amplitude (c) frequency (d) period
(iv) A point source that emits waves uniformly in all directions, produces wave fronts that are

(a) spherical (b) elliptical (c) cylindrical (d) planar


(v) What are the types of wave fronts?

(a) Spherical (b) Cylindrical (c) Plane (d) All of these

**************************

80

You might also like